Как ΡƒΠΌΠ½ΠΎΠΆΠΈΡ‚ΡŒ Π΄Ρ€ΠΎΠ±ΡŒ Π½Π° ΠΎΡ‚Ρ€ΠΈΡ†Π°Ρ‚Π΅Π»ΡŒΠ½ΡƒΡŽ Π΄Ρ€ΠΎΠ±ΡŒ: ΠžΡ‚Ρ€ΠΈΡ†Π°Ρ‚Π΅Π»ΡŒΠ½Ρ‹Π΅ Π΄Ρ€ΠΎΠ±ΠΈ. ДСйствия с ΠΎΡ‚Ρ€ΠΈΡ†Π°Ρ‚Π΅Π»ΡŒΠ½Ρ‹ΠΌΠΈ дробями

Π‘ΠΎΠ΄Π΅Ρ€ΠΆΠ°Π½ΠΈΠ΅

ΠžΡ‚Ρ€ΠΈΡ†Π°Ρ‚Π΅Π»ΡŒΠ½Ρ‹Π΅ Π΄Ρ€ΠΎΠ±ΠΈ. ДСйствия с ΠΎΡ‚Ρ€ΠΈΡ†Π°Ρ‚Π΅Π»ΡŒΠ½Ρ‹ΠΌΠΈ дробями

  • Π‘Π»ΠΎΠΆΠ΅Π½ΠΈΠ΅ ΠΈ Π²Ρ‹Ρ‡ΠΈΡ‚Π°Π½ΠΈΠ΅
  • Π£ΠΌΠ½ΠΎΠΆΠ΅Π½ΠΈΠ΅ ΠΈ Π΄Π΅Π»Π΅Π½ΠΈΠ΅

ΠžΡ‚Ρ€ΠΈΡ†Π°Ρ‚Π΅Π»ΡŒΠ½Ρ‹Π΅ Π΄Ρ€ΠΎΠ±ΠΈ β€” это Π΄Ρ€ΠΎΠ±ΠΈ, Ρ‡ΠΈΡΠ»ΠΈΡ‚Π΅Π»ΡŒ ΠΈΠ»ΠΈ Π·Π½Π°ΠΌΠ΅Π½Π°Ρ‚Π΅Π»ΡŒ ΠΊΠΎΡ‚ΠΎΡ€Ρ‹Ρ… являСтся ΠΎΡ‚Ρ€ΠΈΡ†Π°Ρ‚Π΅Π»ΡŒΠ½Ρ‹ΠΌ числом.

ΠžΡ‚Ρ€ΠΈΡ†Π°Ρ‚Π΅Π»ΡŒΠ½Ρ‹Π΅ Π΄Ρ€ΠΎΠ±ΠΈ ΠΌΠΎΠ³ΡƒΡ‚ Π±Ρ‹Ρ‚ΡŒ записаны ΠΏΠΎ-Ρ€Π°Π·Π½ΠΎΠΌΡƒ. НапримСр, рассмотрим Π΄Π²Π° частных:

-2 : 7 Β Β  ΠΈ Β Β  2 : (-7),

ΠΊΠ°ΠΆΠ΄ΠΎΠ΅ ΠΈΠ· Π½ΠΈΡ… Ρ€Π°Π²Π½ΠΎ ΠΎΡ‚Ρ€ΠΈΡ†Π°Ρ‚Π΅Π»ΡŒΠ½ΠΎΠΌΡƒ числу

КаТдоС ΠΈΠ· Π΄Π°Π½Π½Ρ‹Ρ… частных ΠΌΠΎΠΆΠ½ΠΎ Π·Π°ΠΏΠΈΡΠ°Ρ‚ΡŒ Π² Π²ΠΈΠ΄Π΅ Π΄Ρ€ΠΎΠ±ΠΈ, Π² ΠΊΠΎΡ‚ΠΎΡ€ΠΎΠΉ дробная Ρ‡Π΅Ρ€Ρ‚Π° Π·Π°ΠΌΠ΅Π½ΠΈΡ‚ Π·Π½Π°ΠΊ дСлСния:

-2 : 7Β =Β -2Β Β Β  ΠΈ Β Β Β 2 : (-7)Β =Β 2Β .
7-7

Π‘Π»Π΅Π΄ΠΎΠ²Π°Ρ‚Π΅Π»ΡŒΠ½ΠΎ, ΠΏΡ€ΠΈ записи ΠΎΡ‚Ρ€ΠΈΡ†Π°Ρ‚Π΅Π»ΡŒΠ½Ρ‹Ρ… Π΄Ρ€ΠΎΠ±Π΅ΠΉ Π·Π½Π°ΠΊ минус ΠΌΠΎΠΆΠ½ΠΎ ΡΡ‚Π°Π²ΠΈΡ‚ΡŒ ΠΏΠ΅Ρ€Π΅Π΄ Π΄Ρ€ΠΎΠ±ΡŒΡŽ, ΠΏΠ΅Ρ€Π΅Π΄ числитСлСм ΠΈΠ»ΠΈ ΠΏΠ΅Ρ€Π΅Π΄ Π·Π½Π°ΠΌΠ΅Π½Π°Ρ‚Π΅Π»Π΅ΠΌ:

2Β =Β -2Β =Β 2Β .
77-7

Π‘Π»ΠΎΠΆΠ΅Π½ΠΈΠ΅ ΠΈ Π²Ρ‹Ρ‡ΠΈΡ‚Π°Π½ΠΈΠ΅

Π§Ρ‚ΠΎΠ±Ρ‹ ΡΠ»ΠΎΠΆΠΈΡ‚ΡŒ Π΄Π²Π΅ ΠΎΡ‚Ρ€ΠΈΡ†Π°Ρ‚Π΅Π»ΡŒΠ½Ρ‹Π΅ Π΄Ρ€ΠΎΠ±ΠΈ, Π½Π°Π΄ΠΎ сначала привСсти ΠΈΡ… ΠΊ ΠΎΠ±Ρ‰Π΅ΠΌΡƒ Π·Π½Π°ΠΌΠ΅Π½Π°Ρ‚Π΅Π»ΡŽ, Π° Π·Π°Ρ‚Π΅ΠΌ ΡΠ»ΠΎΠΆΠΈΡ‚ΡŒ числитСли ΠΏΠΎ ΠΏΡ€Π°Π²ΠΈΠ»Π°ΠΌ слоТСния Ρ€Π°Ρ†ΠΈΠΎΠ½Π°Π»ΡŒΠ½Ρ‹Ρ… чисСл.

ΠŸΡ€ΠΈΠΌΠ΅Ρ€.

2Β +Β  (-1)Β .
54

ΠŸΡ€ΠΈΠ²Π΅Π΄Ρ‘ΠΌ Π΄Ρ€ΠΎΠ±ΠΈ ΠΊ ΠΎΠ±Ρ‰Π΅ΠΌΡƒ Π·Π½Π°ΠΌΠ΅Π½Π°Ρ‚Π΅Π»ΡŽ:

2Β +Β  (-1) Β =Β -8Β +Β -5Β .
542020

Π’Π΅ΠΏΠ΅Ρ€ΡŒ слоТим числитСли Π΄Ρ€ΠΎΠ±Π΅ΠΉ ΠΏΠΎ ΠΏΡ€Π°Π²ΠΈΠ»Π°ΠΌ слоТСния Ρ€Π°Ρ†ΠΈΠΎΠ½Π°Π»ΡŒΠ½Ρ‹Ρ… чисСл:

-8Β +Β -5Β =Β -8Β +Β (-5)Β =Β -13Β =Β 13Β .
2020202020

Π’Π°ΠΊΠΈΠΌ ΠΎΠ±Ρ€Π°Π·ΠΎΠΌ:

2Β +Β  (-1) Β =Β -8Β +Β -5Β =
542020

=Β -8 + (-5)Β =Β -13Β =Β 13Β .
202020

Для вычислСния разности Π΄Π²ΡƒΡ… ΠΎΡ‚Ρ€ΠΈΡ†Π°Ρ‚Π΅Π»ΡŒΠ½Ρ‹Ρ… Π΄Ρ€ΠΎΠ±Π΅ΠΉ ΠΌΠΎΠΆΠ½ΠΎ Π²Ρ‹Ρ‡ΠΈΡ‚Π°Π½ΠΈΠ΅ Π·Π°ΠΌΠ΅Π½ΠΈΡ‚ΡŒ слоТСниСм, взяв ΡƒΠΌΠ΅Π½ΡŒΡˆΠ°Π΅ΠΌΠΎΠ΅ со свои Π·Π½Π°ΠΊΠΎΠΌ, Π° Π²Ρ‹Ρ‡ΠΈΡ‚Π°Π΅ΠΌΠΎΠ΅ с ΠΏΡ€ΠΎΡ‚ΠΈΠ²ΠΎΠΏΠΎΠ»ΠΎΠΆΠ½Ρ‹ΠΌ.

ΠŸΡ€ΠΈΠΌΠ΅Ρ€.

5Β —Β (-11) Β =Β 5Β +Β (+11) Β =
12121212

=Β 5Β +Β 11Β =Β -5 + 11Β =Β 6Β .
12121212

Π‘Π»ΠΎΠΆΠ΅Π½ΠΈΠ΅ ΠΈ Π²Ρ‹Ρ‡ΠΈΡ‚Π°Π½ΠΈΠ΅ ΠΎΡ‚Ρ€ΠΈΡ†Π°Ρ‚Π΅Π»ΡŒΠ½Ρ‹Ρ… Π΄Ρ€ΠΎΠ±Π΅ΠΉ производится ΠΏΠΎ ΠΏΡ€Π°Π²ΠΈΠ»Π°ΠΌ слоТСния ΠΎΠ±Ρ‹ΠΊΠ½ΠΎΠ²Π΅Π½Π½Ρ‹Ρ… Π΄Ρ€ΠΎΠ±Π΅ΠΉ, Ρ‚ΠΎ Π΅ΡΡ‚ΡŒ сначала ΠΈΠ΄Ρ‘Ρ‚ ΠΏΡ€ΠΈΠ²Π΅Π΄Π΅Π½ΠΈΠ΅ ΠΊ ΠΎΠ±Ρ‰Π΅ΠΌΡƒ Π·Π½Π°ΠΌΠ΅Π½Π°Ρ‚Π΅Π»ΡŽ, Ссли это Π½ΡƒΠΆΠ½ΠΎ, Π° Π·Π°Ρ‚Π΅ΠΌ производятся вычислСния.

Π£ΠΌΠ½ΠΎΠΆΠ΅Π½ΠΈΠ΅ ΠΈ Π΄Π΅Π»Π΅Π½ΠΈΠ΅

Π§Ρ‚ΠΎΠ±Ρ‹ Π½Π°ΠΉΡ‚ΠΈ ΠΏΡ€ΠΎΠΈΠ·Π²Π΅Π΄Π΅Π½ΠΈΠ΅ Π΄Π²ΡƒΡ… ΠΎΡ‚Ρ€ΠΈΡ†Π°Ρ‚Π΅Π»ΡŒΠ½Ρ‹Ρ… Π΄Ρ€ΠΎΠ±Π΅ΠΉ, Π½Π°Π΄ΠΎ Π·Π½Π°ΠΊΠΈ минус пСрСнСсти ΠΈΠ»ΠΈ Π² числитСли, ΠΈΠ»ΠΈ Π² Π·Π½Π°ΠΌΠ΅Π½Π°Ρ‚Π΅Π»ΠΈ, Π° Π·Π°Ρ‚Π΅ΠΌ ΠΏΠ΅Ρ€Π΅ΠΌΠ½ΠΎΠΆΠΈΡ‚ΡŒ Π΄Ρ€ΠΎΠ±ΠΈ ΠΏΠΎ ΠΏΡ€Π°Π²ΠΈΠ»Ρƒ умноТСния Π΄Ρ€ΠΎΠ±Π΅ΠΉ.

ΠŸΡ€ΠΈΠΌΠ΅Ρ€.

2Β Β·Β (-4)Β Β =Β -2Β Β·Β -4Β =Β -2Β Β·Β (-4)Β =Β 8Β .
35353 Β· 515

Π’Π°ΠΊ ΠΊΠ°ΠΊ ΠΏΡ€ΠΈ ΡƒΠΌΠ½ΠΎΠΆΠ΅Π½ΠΈΠΈ Π΄Π²ΡƒΡ… ΠΎΡ‚Ρ€ΠΈΡ†Π°Ρ‚Π΅Π»ΡŒΠ½Ρ‹Ρ… чисСл Ρ€Π΅Π·ΡƒΠ»ΡŒΡ‚Π°Ρ‚ Π±ΡƒΠ΄Π΅Ρ‚ ΠΏΠΎΠ»ΠΎΠΆΠΈΡ‚Π΅Π»ΡŒΠ½Ρ‹ΠΌ, Ρ‚ΠΎ Π΄Π°Π½Π½Ρ‹ΠΉ ΠΏΡ€ΠΈΠΌΠ΅Ρ€ ΠΌΠΎΠΆΠ½ΠΎ Ρ€Π΅ΡˆΠΈΡ‚ΡŒ сразу, отбросив ΠΎΠ±Π° минуса:

2Β Β·Β (-4) Β =Β 2Β Β·Β 4Β =Β 2 Β· 4Β =Β 8Β .
35353 Β· 515

ΠŸΡ€ΠΈ ΡƒΠΌΠ½ΠΎΠΆΠ΅Π½ΠΈΠΈ ΠΎΡ‚Ρ€ΠΈΡ†Π°Ρ‚Π΅Π»ΡŒΠ½ΠΎΠΉ Π΄Ρ€ΠΎΠ±ΠΈ Π½Π° ΠΏΠΎΠ»ΠΎΠΆΠΈΡ‚Π΅Π»ΡŒΠ½ΡƒΡŽ Ρ€Π΅Π·ΡƒΠ»ΡŒΡ‚Π°Ρ‚ Π±ΡƒΠ΄Π΅Ρ‚ ΠΎΡ‚Ρ€ΠΈΡ†Π°Ρ‚Π΅Π»ΡŒΠ½Ρ‹ΠΌ.

ΠŸΡ€ΠΈΠΌΠ΅Ρ€.

2Β Β·Β 4Β =Β 2 Β· 4Β =Β 8Β .
353 Β· 515

К ΠΎΡ‚Ρ€ΠΈΡ†Π°Ρ‚Π΅Π»ΡŒΠ½Ρ‹ΠΌ дробям ΠΌΠΎΠΆΠ½ΠΎ ΠΏΡ€ΠΈΠΌΠ΅Π½ΡΡ‚ΡŒ Π»ΡŽΠ±Ρ‹Π΅ Π·Π°ΠΊΠΎΠ½Ρ‹ умноТСния. ΠŸΠΎΡΡ‚ΠΎΠΌΡƒ ΠΏΡ€Π΅Π΄Ρ‹Π΄ΡƒΡ‰ΠΈΠΉ ΠΏΡ€ΠΈΠΌΠ΅Ρ€ ΠΌΠΎΠΆΠ½ΠΎ ΠΏΠ΅Ρ€Π΅ΠΏΠΈΡΠ°Ρ‚ΡŒ Ρ‚Π°ΠΊ:

4Β Β·Β  (-2) Β =Β 4 Β· 2Β =Β 8Β .
535 Β· 315

Π’ΠΎ Π΅ΡΡ‚ΡŒ ΠΏΡ€ΠΈ ΡƒΠΌΠ½ΠΎΠΆΠ΅Π½ΠΈΠΈ ΠΏΠΎΠ»ΠΎΠΆΠΈΡ‚Π΅Π»ΡŒΠ½ΠΎΠΉ Π΄Ρ€ΠΎΠ±ΠΈ Π½Π° ΠΎΡ‚Ρ€ΠΈΡ†Π°Ρ‚Π΅Π»ΡŒΠ½ΡƒΡŽ Ρ€Π΅Π·ΡƒΠ»ΡŒΡ‚Π°Ρ‚ Π±ΡƒΠ΄Π΅Ρ‚ ΠΎΡ‚Ρ€ΠΈΡ†Π°Ρ‚Π΅Π»ΡŒΠ½Ρ‹ΠΌ.

Π§Ρ‚ΠΎΠ±Ρ‹ Π½Π°ΠΉΡ‚ΠΈ частноС Π΄Π²ΡƒΡ… ΠΎΡ‚Ρ€ΠΈΡ†Π°Ρ‚Π΅Π»ΡŒΠ½Ρ‹Ρ… Π΄Ρ€ΠΎΠ±Π΅ΠΉ, Π½Π°Π΄ΠΎ Π·Π½Π°ΠΊΠΈ минус пСрСнСсти ΠΈΠ»ΠΈ Π² числитСли, ΠΈΠ»ΠΈ Π² Π·Π½Π°ΠΌΠ΅Π½Π°Ρ‚Π΅Π»ΠΈ, Π° Π·Π°Ρ‚Π΅ΠΌ произвСсти вычислСния.

ΠŸΡ€ΠΈΠΌΠ΅Ρ€.

2Β :Β (-4) Β =Β -2Β :Β -4Β =
3535

=Β -2 Β· 5Β =Β -10Β =Β 10Β .
3 Β· (-4)-1212

Π—Π½Π°ΠΊ Ρ€Π΅Π·ΡƒΠ»ΡŒΡ‚Π°Ρ‚Π° умноТСния ΠΈΠ»ΠΈ дСлСния ΠΎΡ‚Ρ€ΠΈΡ†Π°Ρ‚Π΅Π»ΡŒΠ½Ρ‹Ρ… Π΄Ρ€ΠΎΠ±Π΅ΠΉ ΠΌΠΎΠΆΠ½ΠΎ ΡƒΠ·Π½Π°Ρ‚ΡŒ ΠΏΠΎ ΠΏΡ€Π°Π²ΠΈΠ»Π°ΠΌ Π·Π½Π°ΠΊΠΎΠ² Ρ†Π΅Π»Ρ‹Ρ… чисСл.

Π£ΠΌΠ½ΠΎΠΆΠ΅Π½ΠΈΠ΅ ΠΏΠΎΠ»ΠΎΠΆΠΈΡ‚Π΅Π»ΡŒΠ½Ρ‹Ρ… ΠΈ ΠΎΡ‚Ρ€ΠΈΡ†Π°Ρ‚Π΅Π»ΡŒΠ½Ρ‹Ρ… чисСл β€” Π ΠžΠ‘Π’ΠžΠ’Π‘ΠšΠ˜Π™ ЦЕНВР ПОМОЩИ Π”Π•Π’Π―Πœ β„– 7

Π‘ΠΎΠ΄Π΅Ρ€ΠΆΠ°Π½ΠΈΠ΅

ΠžΡ‚Ρ€ΠΈΡ†Π°Ρ‚Π΅Π»ΡŒΠ½Ρ‹Π΅ Π΄Ρ€ΠΎΠ±ΠΈ, понятиС ΠΈ ΠΏΡ€Π°Π²ΠΈΠ»Π°.

Π’ этой Ρ‚Π΅ΠΌΠ΅ Ρ€Π°Π·Π±Π΅Ρ€Π΅ΠΌ Π½ΠΎΠ²ΠΎΠ΅ понятиС β€œΠžΡ‚Ρ€ΠΈΡ†Π°Ρ‚Π΅Π»ΡŒΠ½Ρ‹Π΅ дроби”. Π”Ρ€ΠΎΠ±ΠΈ, ΠΊΠ°ΠΊ ΠΈ Π»ΡŽΠ±Ρ‹Π΅ числа ΠΌΠΎΠ³ΡƒΡ‚ Π±Ρ‹Ρ‚ΡŒ ΠΏΠΎΠ»ΠΎΠΆΠΈΡ‚Π΅Π»ΡŒΠ½Ρ‹ΠΌΠΈ ΠΈ ΠΎΡ‚Ρ€ΠΈΡ†Π°Ρ‚Π΅Π»ΡŒΠ½Ρ‹ΠΌΠΈ.

ΠžΡ‚Ρ€ΠΈΡ†Π°Ρ‚Π΅Π»ΡŒΠ½Ρ‹Π΅ Π΄Ρ€ΠΎΠ±ΠΈ понятиС ΠΈ смысл. ΠŸΡ€ΠΈΠΌΠ΅Ρ€Ρ‹.

Π Π°Π½Π΅Π΅ ΠΌΡ‹ ΠΈΠ·ΡƒΡ‡ΠΈΠ»ΠΈ Ρ‚Π΅ΠΌΡƒ ΠΎΠ±Ρ‹ΠΊΠ½ΠΎΠ²Π΅Π½Π½Ρ‹Π΅ Π΄Ρ€ΠΎΠ±ΠΈ.Β  ΠžΡ‚Ρ€ΠΈΡ†Π°Ρ‚Π΅Π»ΡŒΠ½Ρ‹Π΅ Π΄Ρ€ΠΎΠ±ΠΈ ΠΎΡ‚Π»ΠΈΡ‡Π°ΡŽΡ‚ΡΡ ΠΎΡ‚ ΠΎΠ±Ρ‹ΠΊΠ½ΠΎΠ²Π΅Π½Π½Ρ‹Ρ… Π΄Ρ€ΠΎΠ±Π΅ΠΉ лишь Π·Π½Π°ΠΊΠΎΠΌ. ΠžΠ±Ρ‹ΠΊΠ½ΠΎΠ²Π΅Π½Π½Ρ‹Π΅ Π΄Ρ€ΠΎΠ±ΠΈ ΠΈΠΌΠ΅ΡŽΡ‚ Π·Π½Π°ΠΊ β€œ+”. НапримСр:

\(\frac{1}{2}; \frac{3}{5}; \frac{7}{10}; \frac{8}{8}; \frac{9}{5}; \frac{3}{1}\)

ВсС эти Π΄Ρ€ΠΎΠ±ΠΈ ΠΌΠΎΠΆΠ½ΠΎ Π·Π°ΠΏΠΈΡΠ°Ρ‚ΡŒ со Π·Π½Π°ΠΊΠΎΠΌ плюс ΠΈ смысл Π΄Ρ€ΠΎΠ±Π΅ΠΉ Π½Π΅ измСнится.

\(\frac{1}{2}= +\frac{1}{2}; \frac{3}{5}= +\frac{3}{5}; \frac{7}{10}= +\frac{7}{10}; \frac{8}{8}= +\frac{8}{8}; \frac{9}{5}= +\frac{9}{5}; \frac{3}{1}= +\frac{3}{1}\)

Если ΠΏΠ΅Ρ€Π΅Π΄ Π΄Ρ€ΠΎΠ±ΡŒΡŽ ΠΏΠΎΡΡ‚Π°Π²ΠΈΡ‚ΡŒ Π·Π½Π°ΠΊ β€œβ€“β€, Ρ‚ΠΎ Π΄Ρ€ΠΎΠ±ΡŒ станСт ΠΎΡ‚Ρ€ΠΈΡ†Π°Ρ‚Π΅Π»ΡŒΠ½ΠΎΠΉ. НапримСр ΠΏΠ΅Ρ€Π΅Π΄ Π΄Ρ€ΠΎΠ±ΡŒΡŽ \(\frac{1}{2}\) поставим Π·Π½Π°ΠΊ минус, ΠΏΠΎΠ»ΡƒΡ‡ΠΈΠΌ \(-\frac{1}{2}\)Π”Ρ€ΠΎΠ±ΠΈ Π²ΠΈΠ΄Π° \(-\frac{1}{2}; -\frac{3}{5}; -\frac{7}{10}; -\frac{8}{8}; -\frac{9}{5}; -\frac{3}{1}\)Β Π½Π°Π·Ρ‹Π²Π°ΡŽΡ‚ΡΡ ΠΎΡ‚Ρ€ΠΈΡ†Π°Ρ‚Π΅Π»ΡŒΠ½Ρ‹ΠΌΠΈ дробями.

ΠŸΡ€ΠΎΡ‚ΠΈΠ²ΠΎΠΏΠΎΠ»ΠΎΠΆΠ½Ρ‹Π΅ Π΄Ρ€ΠΎΠ±ΠΈ, ΠΏΡ€Π°Π²ΠΈΠ»Π°.

Π”Ρ€ΠΎΠ±ΠΈΒ \(\frac{1}{2}\) ΠΈΒ \(-\frac{1}{2}\) Π½Π°Π·Ρ‹Π²Π°ΡŽΡ‚ΡΡ

ΠΏΡ€ΠΎΡ‚ΠΈΠ²ΠΎΠΏΠΎΠ»ΠΎΠΆΠ½Ρ‹ΠΌΠΈ дробями. Π”Ρ€ΠΎΠ±ΠΈ ΠΈΠ»ΠΈ числа, ΠΊΠΎΡ‚ΠΎΡ€Ρ‹Π΅ ΠΎΡ‚Π»ΠΈΡ‡Π°ΡŽΡ‚ΡΡ Ρ‚ΠΎΠ»ΡŒΠΊΠΎ Π·Π½Π°ΠΊΠΎΠΌ Π½Π°Π·Ρ‹Π²Π°ΡŽΡ‚ΡΡ ΠΏΡ€ΠΎΡ‚ΠΈΠ²ΠΎΠΏΠΎΠ»ΠΎΠΆΠ½Ρ‹ΠΌΠΈ дробями ΠΈΠ»ΠΈ числами.

Π’Ρ‹Π²ΠΎΠ΄: Ссли ΠΏΠ΅Ρ€Π΅Π΄ Π΄Ρ€ΠΎΠ±ΡŒΡŽ ΠΏΠΎΡΡ‚Π°Π²ΠΈΡ‚ΡŒ Π·Π½Π°ΠΊ β€œ+”, Ρ‚ΠΎ Π΄Ρ€ΠΎΠ±ΡŒ смысл Π΄Ρ€ΠΎΠ±ΠΈ Π½Π΅ измСнится. Если ΠΏΠΎΡΡ‚Π°Π²ΠΈΡ‚ΡŒ ΠΏΠ΅Ρ€Π΅Π΄ Π΄Ρ€ΠΎΠ±ΡŒΡŽ Π·Π½Π°ΠΊ β€œβ€“β€, Ρ‚ΠΎ ΠΏΠΎΠ»ΡƒΡ‡ΠΈΠΌ ΠΏΡ€ΠΎΡ‚ΠΈΠ²ΠΎΠΏΠΎΠ»ΠΎΠΆΠ½ΡƒΡŽ Π΄Ρ€ΠΎΠ±ΡŒ Π΄Π°Π½Π½ΠΎΠΉ Π΄Ρ€ΠΎΠ±ΠΈ.

НС всСгда Π·Π½Π°ΠΊ минус ΠΏΠΈΡˆΠ΅Ρ‚ΡΡ ΠΏΠ΅Ρ€Π΅Π΄ Π΄Ρ€ΠΎΠ±ΡŒΡŽ, ΠΈΠ½ΠΎΠ³Π΄Π° минус Π·Π°ΠΏΠΈΡΡ‹Π²Π°ΡŽΡ‚ Π² числитСли ΠΈΠ»ΠΈ Π·Π½Π°ΠΌΠ΅Π½Π°Ρ‚Π΅Π»Π΅. Рассмотрим ΠΏΡ€ΠΈΠΌΠ΅Ρ€:

\(-\frac{7}{10}=\frac{-7}{10}=\frac{7}{-10}\)

ΠžΡ‚Ρ€ΠΈΡ†Π°Ρ‚Π΅Π»ΡŒΠ½Ρ‹Π΅ Π΄Ρ€ΠΎΠ±ΠΈ ΠΈ Π½ΡƒΠ»ΡŒ.

ΠΡƒΠ»ΡŒ являСтся ΠΈΡΠΊΠ»ΡŽΡ‡Π΅Π½ΠΈΠ΅ΠΌ, Π½ΡƒΠ»ΡŒ – ΠΏΡ€ΠΎΡ‚ΠΈΠ²ΠΎΠΏΠΎΠ»ΠΎΠΆΠ΅Π½ самому сСбС.Β 

\(0=+\frac{0}{n}=-\frac{0}{n}\)

Вопросы ΠΏΠΎ Ρ‚Π΅ΠΌΠ΅ β€œΠžΡ‚Ρ€ΠΈΡ†Π°Ρ‚Π΅Π»ΡŒΠ½Ρ‹Π΅ дроби”:
НазовитС Ρ‚Ρ€ΠΈ ΠΎΡ‚Ρ€ΠΈΡ†Π°Ρ‚Π΅Π»ΡŒΠ½Ρ‹Π΅ Π΄Ρ€ΠΎΠ±ΠΈ?
ΠžΡ‚Π²Π΅Ρ‚:Β \(-\frac{1}{3}; -\frac{4}{4}; -\frac{7}{3}; \)

ΠŸΡ€ΠΈΠ²Π΅Π΄ΠΈΡ‚Π΅ ΠΏΡ€ΠΈΠΌΠ΅Ρ€ ΠΏΡ€ΠΎΡ‚ΠΈΠ²ΠΎΠΏΠΎΠ»ΠΎΠΆΠ½Ρ‹Ρ… чисСл?
ΠžΡ‚Π²Π΅Ρ‚:Β \(-\frac{8}{5}\) ΠΈΒ \(\frac{8}{5}\)

НазовитС ΠΊΠ°ΠΊΠΎΠΌΡƒ числу ΠΏΡ€ΠΎΡ‚ΠΈΠ²ΠΎΠΏΠΎΠ»ΠΎΠΆΠ½ΠΎ число Π½ΡƒΠ»ΡŒ?


ΠžΡ‚Π²Π΅Ρ‚: Π½ΡƒΠ»ΡŒ ΠΏΡ€ΠΎΡ‚ΠΈΠ²ΠΎΠΏΠΎΠ»ΠΎΠΆΠ΅Π½ сам сСбС.

ΠšΠ°ΠΊΠΎΠΌΡƒ числу ΠΏΡ€ΠΎΡ‚ΠΈΠ²ΠΎΠΏΠΎΠ»ΠΎΠΆΠ½ΠΎ ΠΏΠΎΠ»ΠΎΠΆΠΈΡ‚Π΅Π»ΡŒΠ½ΠΎΠ΅ число?
ΠžΡ‚Π²Π΅Ρ‚: ΠΏΠΎΠ»ΠΎΠΆΠΈΡ‚Π΅Π»ΡŒΠ½ΠΎΠ΅ число ΠΏΡ€ΠΎΡ‚ΠΈΠ²ΠΎΠΏΠΎΠ»ΠΎΠΆΠ½ΠΎ Π΄Π°Π½Π½ΠΎΠΌΡƒ ΠΎΡ‚Ρ€ΠΈΡ†Π°Ρ‚Π΅Π»ΡŒΠ½ΠΎΠΌΡƒ числу.

ΠžΡ‚Ρ€ΠΈΡ†Π°Ρ‚Π΅Π»ΡŒΠ½Π°Ρ Π΄Ρ€ΠΎΠ±ΡŒ ΠΏΡ€ΠΎΡ‚ΠΈΠ²ΠΎΠΏΠΎΠ»ΠΎΠΆΠ½Π° ΠΊΠ°ΠΊΠΎΠΉ Π΄Ρ€ΠΎΠ±ΠΈ?
ΠžΡ‚Π²Π΅Ρ‚: ΠΎΡ‚Ρ€ΠΈΡ†Π°Ρ‚Π΅Π»ΡŒΠ½Π°Ρ Π΄Ρ€ΠΎΠ±ΡŒ ΠΏΡ€ΠΎΡ‚ΠΈΠ²ΠΎΠΏΠΎΠ»ΠΎΠΆΠ½Π° Π΄Π°Π½Π½ΠΎΠΉ ΠΏΠΎΠ»ΠΎΠΆΠΈΡ‚Π΅Π»ΡŒΠ½ΠΎΠΉ Π΄Ρ€ΠΎΠ±ΠΈ.

ΠŸΡ€ΠΈΠΌΠ΅Ρ€:
ЯвляСтся Π»ΠΈ Π΄Ρ€ΠΎΠ±ΡŒ ΠΏΠΎΠ»ΠΎΠΆΠΈΡ‚Π΅Π»ΡŒΠ½ΠΎΠΉ ΠΈΠ»ΠΈ ΠΎΡ‚Ρ€ΠΈΡ†Π°Ρ‚Π΅Π»ΡŒΠ½ΠΎΠΉ: \(\frac{1}{5}; -\frac{3}{7}; \frac{4}{1}; \frac{5}{5}; -\frac{9}{4}; -\frac{2}{3}; -\frac{0}{6}.\)

РСшСниС:
ΠžΡ‚Ρ€ΠΈΡ†Π°Ρ‚Π΅Π»ΡŒΠ½Ρ‹Π΅ Π΄Ρ€ΠΎΠ±ΠΈΒ \(-\frac{3}{7}; -\frac{9}{4}; -\frac{2}{3}\)
ΠŸΠΎΠ»ΠΎΠΆΠΈΡ‚Π΅Π»ΡŒΠ½Ρ‹Π΅ Π΄Ρ€ΠΎΠ±ΠΈΒ \(\frac{1}{5}; \frac{4}{1}; \frac{5}{5}\)
ЯвляСтся Π½ΠΈ ΠΏΠΎΠ»ΠΎΠΆΠΈΡ‚Π΅Π»ΡŒΠ½ΠΎΠΉ, Π½ΠΈ ΠΎΡ‚Ρ€ΠΈΡ†Π°Ρ‚Π΅Π»ΡŒΠ½ΠΎΠΉ Π΄Ρ€ΠΎΠ±ΡŒΡŽΒ \(-\frac{0}{6}.\)

Как ΠΏΡ€Π°Π²ΠΈΠ»ΡŒΠ½ΠΎ ΡƒΠΌΠ½ΠΎΠΆΠ°Ρ‚ΡŒ ΠΎΡ‚Ρ€ΠΈΡ†Π°Ρ‚Π΅Π»ΡŒΠ½Ρ‹Π΅ числа?

ΠžΡΠ½ΠΎΠ²Π½Ρ‹Π΅ опрСдСлСния

Вспомним, ΠΊΠ°ΠΊ ΠΎΡ‚Π»ΠΈΡ‡ΠΈΡ‚ΡŒ ΠΏΠΎΠ»ΠΎΠΆΠΈΡ‚Π΅Π»ΡŒΠ½ΠΎΠ΅ число ΠΎΡ‚ ΠΎΡ‚Ρ€ΠΈΡ†Π°Ρ‚Π΅Π»ΡŒΠ½ΠΎΠ³ΠΎ, Ρ‡Ρ‚ΠΎ Ρ‚Π°ΠΊΠΎΠ΅ ΡƒΠΌΠ½ΠΎΠΆΠ΅Π½ΠΈΠ΅ ΠΈ ΠΊΠ°ΠΊΠΈΠ΅ Ρƒ Π½Π΅Π³ΠΎ свойства.

НачнСм с Ρ‚ΠΎΠ³ΠΎ, Ρ‡Ρ‚ΠΎ ΠΏΡ€ΠΎΠ²Π΅Π΄Π΅ΠΌ ΠΏΡ€ΡΠΌΡƒΡŽ ΠΈ ΠΎΡ‚ΠΌΠ΅Ρ‚ΠΈΠΌ Π½Π° Π½Π΅ΠΉ Π½Π°Ρ‡Π°Π»ΠΎ отсчСта β€” Ρ‚ΠΎΡ‡ΠΊΡƒ Π½ΡƒΠ»ΡŒ (0). А Ρ‚Π΅ΠΏΠ΅Ρ€ΡŒ ΡƒΠΊΠ°ΠΆΠ΅ΠΌ Π½Π°ΠΏΡ€Π°Π²Π»Π΅Π½ΠΈΠ΅ двиТСния ΠΏΠΎ прямой Π²ΠΏΡ€Π°Π²ΠΎ ΠΎΡ‚ Π½Π°Ρ‡Π°Π»Π° ΠΊΠΎΠΎΡ€Π΄ΠΈΠ½Π°Ρ‚. Π’ этом Π½Π°ΠΌ ΠΏΠΎΠΌΠΎΠΆΠ΅Ρ‚ красивая стрСлка:

Π”Π²Π° Π³Π»Π°Π²Π½Ρ‹Ρ… опрСдСлСния:

ΠŸΠΎΠ»ΠΎΠΆΠΈΡ‚Π΅Π»ΡŒΠ½Ρ‹Π΅ числа β€” это Ρ‚ΠΎΡ‡ΠΊΠΈ ΠΊΠΎΠΎΡ€Π΄ΠΈΠ½Π°Ρ‚Π½ΠΎΠΉ прямой, ΠΊΠΎΡ‚ΠΎΡ€Ρ‹Π΅ Π»Π΅ΠΆΠ°Ρ‚ ΠΏΡ€Π°Π²Π΅Π΅ Π½Π°Ρ‡Π°Π»Π° отсчСта (нуля). Иногда рядом с Π½ΠΈΠΌΠΈ ставят Π·Π½Π°ΠΊ плюс β€” Β«+Β», Π½ΠΎ Ρ‡Π°Ρ‰Π΅ всСго ΠΏΠΎΠ»ΠΎΠΆΠΈΡ‚Π΅Π»ΡŒΠ½Ρ‹Π΅ числа Π½ΠΈΠΊΠ°ΠΊ Π½Π΅ ΠΎΠ±ΠΎΠ·Π½Π°Ρ‡Π°ΡŽΡ‚. Π’ΠΎ Π΅ΡΡ‚ΡŒ Β«+1Β» ΠΈ Β«1Β» β€” это ΠΎΠ΄Π½ΠΎ ΠΈ Ρ‚ΠΎΠΆΠ΅ число.

Π—Π°ΠΏΠΎΠΌΠΈΠ½Π°Π΅ΠΌ!

ΠŸΠΎΠ»ΠΎΠΆΠΈΡ‚Π΅Π»ΡŒΠ½Ρ‹Π΅ числа β€” это Ρ‚Π΅, Ρ‡Ρ‚ΠΎ большС нуля, Π° ΠΎΡ‚Ρ€ΠΈΡ†Π°Ρ‚Π΅Π»ΡŒΠ½Ρ‹Π΅ β€” мСньшиС.

ΠžΡ‚Ρ€ΠΈΡ†Π°Ρ‚Π΅Π»ΡŒΠ½Ρ‹Π΅ числа β€” это Ρ‚ΠΎΡ‡ΠΊΠΈ ΠΊΠΎΠΎΡ€Π΄ΠΈΠ½Π°Ρ‚Π½ΠΎΠΉ прямой, ΠΊΠΎΡ‚ΠΎΡ€Ρ‹Π΅ Π»Π΅ΠΆΠ°Ρ‚ Π»Π΅Π²Π΅Π΅ Π½Π°Ρ‡Π°Π»Π° отсчСта (нуля). Π˜Ρ… всСгда ΠΎΠ±ΠΎΠ·Π½Π°Ρ‡Π°ΡŽΡ‚ Π·Π½Π°ΠΊΠΎΠΌ минус β€” Β«-Β».

ΠΡƒΠ»ΡŒ (0) β€” Π½ΠΈ ΠΏΠΎΠ»ΠΎΠΆΠΈΡ‚Π΅Π»ΡŒΠ½ΠΎΠ΅, Π½ΠΈ ΠΎΡ‚Ρ€ΠΈΡ†Π°Ρ‚Π΅Π»ΡŒΠ½ΠΎΠ΅ число. Π’ΠΎΡ‚ это Π΅ΠΌΡƒ ΠΏΠΎΠ²Π΅Π·Π»ΠΎ!

Π§ΠΈΡΠ»ΠΎΠ²ΡƒΡŽ ось ΠΌΠΎΠΆΠ½ΠΎ Ρ€Π°ΡΠΏΠΎΠ»ΠΎΠΆΠΈΡ‚ΡŒ ΠΊΠ°ΠΊ Π³ΠΎΡ€ΠΈΠ·ΠΎΠ½Ρ‚Π°Π»ΡŒΠ½ΠΎ (стрСлка Π²Π²Π΅Ρ€Ρ…), Ρ‚Π°ΠΊ ΠΈ Π²Π΅Ρ€Ρ‚ΠΈΠΊΠ°Π»ΡŒΠ½ΠΎ (стрСлка Π²ΠΏΡ€Π°Π²ΠΎ).

Если стрСлка Π½Π°ΠΏΡ€Π°Π²Π»Π΅Π½Π° Π²Π²Π΅Ρ€Ρ…, Ρ‚ΠΎ Π² Π²Π΅Ρ€Ρ…Π½Π΅ΠΉ части ΠΎΡ‚ Π½Π°Ρ‡Π°Π»Π° отсчСта всСгда располоТСны ΠΏΠΎΠ»ΠΎΠΆΠΈΡ‚Π΅Π»ΡŒΠ½Ρ‹Π΅ числа, Π° Π² Π½ΠΈΠΆΠ½Π΅ΠΉ β€” ΠΎΡ‚Ρ€ΠΈΡ†Π°Ρ‚Π΅Π»ΡŒΠ½Ρ‹Π΅. Π‘ΠΌΠΎΡ‚Ρ€ΠΈΡ‚Π΅:

ΠŸΡ€ΡΠΌΠ°Ρ, Π½Π° ΠΊΠΎΡ‚ΠΎΡ€ΠΎΠΉ ΠΎΡ‚ΠΌΠ΅Ρ‡Π΅Π½Π° Π½Π°Ρ‡Π°Π»ΡŒΠ½Π°Ρ Ρ‚ΠΎΡ‡ΠΊΠ°, ΠΏΠΎΠ»ΠΎΠΆΠΈΡ‚Π΅Π»ΡŒΠ½ΠΎΠ΅ Π½Π°ΠΏΡ€Π°Π²Π»Π΅Π½ΠΈΠ΅ ΠΈ Π΅Π΄ΠΈΠ½ΠΈΡ‡Π½Ρ‹ΠΉ ΠΎΡ‚Ρ€Π΅Π·ΠΎΠΊ, называСтся ΠΊΠΎΠΎΡ€Π΄ΠΈΠ½Π°Ρ‚Π½ΠΎΠΉ ΠΈΠ»ΠΈ числовой осью.

Π£ΠΌΠ½ΠΎΠΆΠ΅Π½ΠΈΠ΅ β€” арифмСтичСскоС дСйствиС Π² ΠΊΠΎΡ‚ΠΎΡ€ΠΎΠΌ ΡƒΡ‡Π°ΡΡ‚Π²ΡƒΡŽΡ‚ Π΄Π²Π° Π°Ρ€Π³ΡƒΠΌΠ΅Π½Ρ‚Π°. Один ΠΌΠ½ΠΎΠΆΠΈΠΌΡ‹ΠΉ, Π²Ρ‚ΠΎΡ€ΠΎΠΉ ΠΌΠ½ΠΎΠΆΠΈΡ‚Π΅Π»ΡŒ. Π Π΅Π·ΡƒΠ»ΡŒΡ‚Π°Ρ‚ ΠΈΡ… умноТСния называСтся ΠΏΡ€ΠΎΠΈΠ·Π²Π΅Π΄Π΅Π½ΠΈΠ΅ΠΌ.

Бвойства умноТСния

  1. ΠžΡ‚ пСрСстановки ΠΌΠ½ΠΎΠΆΠΈΡ‚Π΅Π»Π΅ΠΉ мСстами ΠΏΡ€ΠΎΠΈΠ·Π²Π΅Π΄Π΅Π½ΠΈΠ΅ Π½Π΅ мСняСтся.
    a * b = b * a
  2. Π Π΅Π·ΡƒΠ»ΡŒΡ‚Π°Ρ‚ произвСдСния Ρ‚Ρ€Ρ‘Ρ… ΠΈ Π±ΠΎΠ»Π΅Π΅ ΠΌΠ½ΠΎΠΆΠΈΡ‚Π΅Π»Π΅ΠΉ Π½Π΅ измСнится, Ссли Π»ΡŽΠ±ΡƒΡŽ Π³Ρ€ΡƒΠΏΠΏΡƒ Π·Π°ΠΌΠ΅Π½ΠΈΡ‚ΡŒ ΠΏΡ€ΠΎΠΈΠ·Π²Π΅Π΄Π΅Π½ΠΈΠ΅ΠΌ.
    a * b * c = (a * b) * c = a * (b * c)

Π’Ρ‹Ρ‡ΠΈΡΠ»ΡΡ‚ΡŒ ΠΌΠΎΠΆΠ½ΠΎ Π² ΡƒΠΌΠ΅, ΠΏΡ€ΠΈ ΠΏΠΎΠΌΠΎΡ‰ΠΈ Ρ‚Π°Π±Π»ΠΈΡ†Ρ‹ умноТСния ΠΈΠ»ΠΈ Π² столбик. ΠŸΡ€ΠΎΠ΄Π²ΠΈΠ½ΡƒΡ‚Ρ‹Π΅ школьники ΠΌΠΎΠ³ΡƒΡ‚ ΠΈΡΠΏΠΎΠ»ΡŒΠ·ΠΎΠ²Π°Ρ‚ΡŒ ΠΎΠ½Π»Π°ΠΉΠ½-ΠΊΠ°Π»ΡŒΠΊΡƒΠ»ΡΡ‚ΠΎΡ€.Β 

ΠŸΡ€Π°Π²ΠΈΠ»ΠΎ умноТСния ΠΎΡ‚Ρ€ΠΈΡ†Π°Ρ‚Π΅Π»ΡŒΠ½Ρ‹Ρ… чисСл: Ρ‡Ρ‚ΠΎΠ±Ρ‹ ΡƒΠΌΠ½ΠΎΠΆΠΈΡ‚ΡŒ Π΄Π²Π° ΠΎΡ‚Ρ€ΠΈΡ†Π°Ρ‚Π΅Π»ΡŒΠ½Ρ‹Ρ… числа, Π½ΡƒΠΆΠ½ΠΎ ΠΏΠ΅Ρ€Π΅ΠΌΠ½ΠΎΠΆΠΈΡ‚ΡŒ ΠΈΡ… ΠΌΠΎΠ΄ΡƒΠ»ΠΈ. Π­Ρ‚ΠΎ Π·Π½Π°Ρ‡ΠΈΡ‚, Ρ‡Ρ‚ΠΎ для Π»ΡŽΠ±Ρ‹Ρ… ΠΎΡ‚Ρ€ΠΈΡ†Π°Ρ‚Π΅Π»ΡŒΠ½Ρ‹Ρ… чисСл -a, -b Π²Π΅Ρ€Π½ΠΎ равСнство:

А Π²ΠΎΡ‚ ΠΊΠ°ΠΊ ΡƒΠΌΠ½ΠΎΠΆΠΈΡ‚ΡŒ Π΄Π²Π° числа с Ρ€Π°Π·Π½Ρ‹ΠΌΠΈ Π·Π½Π°ΠΊΠ°ΠΌΠΈ:

  • ΠΏΠ΅Ρ€Π΅ΠΌΠ½ΠΎΠΆΠΈΡ‚ΡŒ ΠΌΠΎΠ΄ΡƒΠ»ΠΈ этих чисСл
  • ΠΏΠ΅Ρ€Π΅Π΄ ΠΏΠΎΠ»ΡƒΡ‡Π΅Π½Π½Ρ‹ΠΌ числом ΠΏΠΎΡΡ‚Π°Π²ΠΈΡ‚ΡŒ Π·Π½Π°ΠΊ минус

А Ρ‚Π΅ΠΏΠ΅Ρ€ΡŒ упростим ΠΏΡ€Π°Π²ΠΈΠ»Π°. Π‘Ρ„ΠΎΡ€ΠΌΡƒΠ»ΠΈΡ€ΡƒΠ΅ΠΌ ΠΈΡ… Π² Π»Π΅Π³ΠΊΠΎΠΉ Ρ„ΠΎΡ€ΠΌΠ΅ с ΠΌΠΈΠ½ΠΈΠΌΡƒΠΌΠΎΠΌ слов, Ρ‡Ρ‚ΠΎΠ±Ρ‹ ΠΏΡ€ΠΎΡ‰Π΅ Π·Π°ΠΏΠΎΠΌΠ½ΠΈΡ‚ΡŒ:

  • Β«β€”Β» β€” ΠΏΡ€ΠΈ ΡƒΠΌΠ½ΠΎΠΆΠ΅Π½ΠΈΠΈ минус Π½Π° минус ΠΎΡ‚Π²Π΅Ρ‚ Π±ΡƒΠ΄Π΅Ρ‚ ΠΏΠΎΠ»ΠΎΠΆΠΈΡ‚Π΅Π»ΡŒΠ½Ρ‹ΠΌ
    ΠΈΠ»ΠΈ минус Π½Π° минус Π΄Π°Π΅Ρ‚ плюс
  • Β«-+Β» β€” ΠΏΡ€ΠΈ ΡƒΠΌΠ½ΠΎΠΆΠ΅Π½ΠΈΠΈ минуса Π½Π° плюс ΠΎΡ‚Π²Π΅Ρ‚ Π±ΡƒΠ΄Π΅Ρ‚ ΠΎΡ‚Ρ€ΠΈΡ†Π°Ρ‚Π΅Π»ΡŒΠ½Ρ‹ΠΌ
    ΠΈΠ»ΠΈ минус Π½Π° плюс Π΄Π°Π΅Ρ‚ минус
  • Β«+-Β» β€” ΠΏΡ€ΠΈ ΡƒΠΌΠ½ΠΎΠΆΠ΅Π½ΠΈΠΈ плюса Π½Π° минус ΠΎΡ‚Π²Π΅Ρ‚ Π±ΡƒΠ΄Π΅Ρ‚ ΠΎΡ‚Ρ€ΠΈΡ†Π°Ρ‚Π΅Π»ΡŒΠ½Ρ‹ΠΌ
    ΠΈΠ»ΠΈ плюс Π½Π° минус Π΄Π°Π΅Ρ‚ минус
  • Β«++Β» β€” ΠΏΡ€ΠΈ ΡƒΠΌΠ½ΠΎΠΆΠ΅Π½ΠΈΠΈ плюса Π½Π° плюс ΠΎΡ‚Π²Π΅Ρ‚ Π±ΡƒΠ΄Π΅Ρ‚ ΠΏΠΎΠ»ΠΎΠΆΠΈΡ‚Π΅Π»ΡŒΠ½Ρ‹ΠΌ
    ΠΈΠ»ΠΈ плюс Π½Π° плюс Π΄Π°Π΅Ρ‚ плюс.

ΠŸΡ€ΠΈΠΌΠ΅Ρ€Ρ‹ умноТСния ΠΎΡ‚Ρ€ΠΈΡ†Π°Ρ‚Π΅Π»ΡŒΠ½Ρ‹Ρ… чисСл

ΠŸΡ€ΠΈΠΌΠ΅Ρ€ 1. Π’Ρ‹Ρ‡ΠΈΡΠ»ΠΈΡ‚ΡŒ: (-2)βˆ—(-2) ΠΈ (-3)βˆ—(-7)

Как Ρ€Π΅ΡˆΠ°Π΅ΠΌ:

Вспомним ΠΏΡ€Π°Π²ΠΈΠ»ΠΎ: ΠΎΡ‚Ρ€ΠΈΡ†Π°Ρ‚Π΅Π»ΡŒΠ½ΠΎΠ΅ число ΡƒΠΌΠ½ΠΎΠΆΠΈΡ‚ΡŒ Π½Π° ΠΎΡ‚Ρ€ΠΈΡ†Π°Ρ‚Π΅Π»ΡŒΠ½ΠΎΠ΅ β€” получаСтся ΠΎΡ‚Π²Π΅Ρ‚ со Π·Π½Π°ΠΊΠΎΠΌ плюс. Π‘Ρ‡ΠΈΡ‚Π°Π΅ΠΌ:

Β 

  1. (-2)βˆ—(-2) = 4

  2. (-3)βˆ—(-7) = 21

ΠžΡ‚Π²Π΅Ρ‚: 4; 21.

ΠŸΡ€ΠΈΠΌΠ΅Ρ€ 2. Π’Ρ‹Ρ‡ΠΈΡΠ»ΠΈΡ‚ΡŒ: (-11)βˆ—11 ΠΈ (-20)βˆ—2

Как Ρ€Π΅ΡˆΠ°Π΅ΠΌ:

Вспомним ΠΏΡ€Π°Π²ΠΈΠ»ΠΎ: ΠΎΡ‚Ρ€ΠΈΡ†Π°Ρ‚Π΅Π»ΡŒΠ½ΠΎΠ΅ число ΡƒΠΌΠ½ΠΎΠΆΠΈΡ‚ΡŒ Π½Π° ΠΏΠΎΠ»ΠΎΠΆΠΈΡ‚Π΅Π»ΡŒΠ½ΠΎΠ΅ β€” получаСтся ΠΎΡ‚Π²Π΅Ρ‚ со Π·Π½Π°ΠΊΠΎΠΌ минус. Π‘Ρ‡ΠΈΡ‚Π°Π΅ΠΌ:

Β 

  1. -11 * 11 = -121

  2. (-20) * 2 = -40

Β ΠžΡ‚Π²Π΅Ρ‚: -121; -40.

ΠŸΡ€ΠΈΠΌΠ΅Ρ€ 3. Π’Ρ‹Ρ‡ΠΈΡΠ»ΠΈΡ‚ΡŒ ΠΏΡ€ΠΎΠΈΠ·Π²Π΅Π΄Π΅Π½ΠΈΠ΅: 5βˆ—(-5) ΠΈ 12βˆ—(-8)

Как Ρ€Π΅ΡˆΠ°Π΅ΠΌ:

Вспомним ΠΏΡ€Π°Π²ΠΈΠ»ΠΎ: ΡƒΠΌΠ½ΠΎΠΆΠ΅Π½ΠΈΠ΅ ΠΏΠΎΠ»ΠΎΠΆΠΈΡ‚Π΅Π»ΡŒΠ½ΠΎΠ³ΠΎ Π½Π° ΠΎΡ‚Ρ€ΠΈΡ†Π°Ρ‚Π΅Π»ΡŒΠ½ΠΎΠ΅ число Π΄Π°Π΅Ρ‚ ΠΎΡ‚Ρ€ΠΈΡ†Π°Ρ‚Π΅Π»ΡŒΠ½Ρ‹ΠΉ Ρ€Π΅Π·ΡƒΠ»ΡŒΡ‚Π°Ρ‚. Π‘Ρ‡ΠΈΡ‚Π°Π΅ΠΌ:

Β 

  1. 5 βˆ— (-5)= -25

  2. 12 βˆ— (-8)= -96

ΠžΡ‚Π²Π΅Ρ‚: -25; -96.

ΠŸΡ€ΠΈΠΌΠ΅Ρ€ 4. Π’Ρ‹Ρ‡ΠΈΡΠ»ΠΈΡ‚ΡŒ ΠΏΡ€ΠΎΠΈΠ·Π²Π΅Π΄Π΅Π½ΠΈΠ΅: (-0,125 ) * (-6)

Как Ρ€Π΅ΡˆΠ°Π΅ΠΌ:

Β 

  1. Π˜ΡΠΏΠΎΠ»ΡŒΠ·ΡƒΠ΅ΠΌ ΠΏΡ€Π°Π²ΠΈΠ»ΠΎ умноТСния ΠΎΡ‚Ρ€ΠΈΡ†Π°Ρ‚Π΅Π»ΡŒΠ½Ρ‹Ρ… чисСл:
    (-0,125 ) * (-6) = 0,125 * 6.

  2. Π’Ρ‹ΠΏΠΎΠ»Π½ΠΈΠΌ ΡƒΠΌΠ½ΠΎΠΆΠ΅Π½ΠΈΠ΅ дСсятичной Π΄Ρ€ΠΎΠ±ΠΈ Π½Π° Π½Π°Ρ‚ΡƒΡ€Π°Π»ΡŒΠ½ΠΎΠ΅ число столбиком:

ΠžΡ‚Π²Π΅Ρ‚: 0,75.

Π Π°Π·Π²ΠΈΠ²Π°ΠΉΡ‚Π΅ матСматичСскоС ΠΌΡ‹ΡˆΠ»Π΅Π½ΠΈΠ΅ Π΄Π΅Ρ‚Π΅ΠΉ Π½Π° Π½Π°ΡˆΠΈΡ… ΡƒΡ€ΠΎΠΊΠ°Ρ… ΠΌΠ°Ρ‚Π΅ΠΌΠ°Ρ‚ΠΈΠΊΠΈ вмСстС с Π΅Π½ΠΎΡ‚ΠΎΠΌ Максом ΠΈ Π΅Π³ΠΎ Π΄Ρ€ΡƒΠ·ΡŒΡΠΌΠΈ. ΠœΡ‹ ΠΏΠΎΠ΄ΠΎΠ±Ρ€Π°Π»ΠΈ для вашСго Ρ€Π΅Π±Π΅Π½ΠΊΠ° тысячи ΡƒΠ²Π»Π΅ΠΊΠ°Ρ‚Π΅Π»ΡŒΠ½Ρ‹Ρ… Π·Π°Π΄Π°Π½ΠΈΠΉ β€” ΠΎΡ‚ простых логичСских Π·Π°Π³Π°Π΄ΠΎΠΊ Π΄ΠΎ Ρ…ΠΈΡ‚Ρ€Ρ‹Ρ… Π³ΠΎΠ»ΠΎΠ²ΠΎΠ»ΠΎΠΌΠΎΠΊ, Π½Π°Π΄ ΠΊΠΎΡ‚ΠΎΡ€Ρ‹ΠΌΠΈ интСрСсно ΠΏΠΎΠ΄ΡƒΠΌΠ°Ρ‚ΡŒ. ВсС это ΠΏΠΎΠΌΠΎΠΆΠ΅Ρ‚ Π»Π΅Π³Ρ‡Π΅ ΠΈ быстрСС ΡΠΏΡ€Π°Π²ΠΈΡ‚ΡŒΡΡ со школьной ΠΌΠ°Ρ‚Π΅ΠΌΠ°Ρ‚ΠΈΠΊΠΎΠΉ.

ΠŸΡ€ΠΈΡ…ΠΎΠ΄ΠΈΡ‚Π΅ Π½Π° бСсплатный Π²Π²ΠΎΠ΄Π½Ρ‹ΠΉ ΡƒΡ€ΠΎΠΊ вмСстС с Ρ€Π΅Π±Π΅Π½ΠΊΠΎΠΌ: познакомимся, ΠΏΠΎΡ€Π΅ΡˆΠ°Π΅ΠΌ Π·Π°Π΄Π°Ρ‡ΠΊΠΈ ΠΈ Π²Π΄ΠΎΡ…Π½ΠΎΠ²ΠΈΠΌ Π½Π° ΡƒΡ‡Π΅Π±Ρƒ!

ΠΏΡ€Π°Π²ΠΈΠ»ΠΎ, ΠΏΡ€ΠΈΠΌΠ΅Ρ€Ρ‹, ΡƒΠΌΠ½ΠΎΠΆΠ΅Π½ΠΈΠ΅ ΠΎΡ‚Ρ€ΠΈΡ†Π°Ρ‚Π΅Π»ΡŒΠ½Ρ‹Ρ… чисСл Π½Π° ΠΏΠΎΠ»ΠΎΠΆΠΈΡ‚Π΅Π»ΡŒΠ½Ρ‹Π΅

Π’ Π΄Π°Π½Π½ΠΎΠΉ ΡΡ‚Π°Ρ‚ΡŒΠ΅ сформулируСм ΠΏΡ€Π°Π²ΠΈΠ»ΠΎ умноТСния ΠΎΡ‚Ρ€ΠΈΡ†Π°Ρ‚Π΅Π»ΡŒΠ½Ρ‹Ρ… чисСл ΠΈ Π΄Π°Π΄ΠΈΠΌ Π΅ΠΌΡƒ объяснСниС. Π‘ΡƒΠ΄Π΅Ρ‚ ΠΏΠΎΠ΄Ρ€ΠΎΠ±Π½ΠΎ рассмотрСн процСсс умноТСния ΠΎΡ‚Ρ€ΠΈΡ†Π°Ρ‚Π΅Π»ΡŒΠ½Ρ‹Ρ… чисСл. На ΠΏΡ€ΠΈΠΌΠ΅Ρ€Π°Ρ… ΠΏΠΎΠΊΠ°Π·Π°Π½Ρ‹ всС Π²ΠΎΠ·ΠΌΠΎΠΆΠ½Ρ‹Π΅ случаи.

Π£ΠΌΠ½ΠΎΠΆΠ΅Π½ΠΈΠ΅ ΠΎΡ‚Ρ€ΠΈΡ†Π°Ρ‚Π΅Π»ΡŒΠ½Ρ‹Ρ… чисСл

ΠžΠΏΡ€Π΅Π΄Π΅Π»Π΅Π½ΠΈΠ΅ 1

ΠŸΡ€Π°Π²ΠΈΠ»ΠΎ умноТСния ΠΎΡ‚Ρ€ΠΈΡ†Π°Ρ‚Π΅Π»ΡŒΠ½Ρ‹Ρ… чисСл Π·Π°ΠΊΠ»ΡŽΡ‡Π°Π΅Ρ‚ΡΡ Π² Ρ‚ΠΎΠΌ, Ρ‡Ρ‚ΠΎ для Ρ‚ΠΎΠ³ΠΎ, Ρ‡Ρ‚ΠΎΠ±Ρ‹ ΡƒΠΌΠ½ΠΎΠΆΠΈΡ‚ΡŒ Π΄Π²Π° ΠΎΡ‚Ρ€ΠΈΡ†Π°Ρ‚Π΅Π»ΡŒΠ½Ρ‹Ρ… числа, Π½Π΅ΠΎΠ±Ρ…ΠΎΠ΄ΠΈΠΌΠΎ ΠΏΠ΅Ρ€Π΅ΠΌΠ½ΠΎΠΆΠΈΡ‚ΡŒ ΠΈΡ… ΠΌΠΎΠ΄ΡƒΠ»ΠΈ. Π”Π°Π½Π½ΠΎΠ΅ ΠΏΡ€Π°Π²ΠΈΠ»ΠΎ записываСтся Ρ‚Π°ΠΊ: для Π»ΡŽΠ±Ρ‹Ρ… ΠΎΡ‚Ρ€ΠΈΡ†Π°Ρ‚Π΅Π»ΡŒΠ½Ρ‹Ρ… чисСл –a,Β -b Π΄Π°Π½Π½ΠΎΠ΅ равСнство считаСтся Π²Π΅Ρ€Π½Ρ‹ΠΌ.

(-Π°)Β·(-b)=aΒ·b.

Π’Ρ‹ΡˆΠ΅ ΠΏΡ€ΠΈΠ²Π΅Π΄Π΅Π½ΠΎ ΠΏΡ€Π°Π²ΠΈΠ»ΠΎ умноТСния Π΄Π²ΡƒΡ… ΠΎΡ‚Ρ€ΠΈΡ†Π°Ρ‚Π΅Π»ΡŒΠ½Ρ‹Ρ… чисСл. Π˜ΡΡ…ΠΎΠ΄Ρ ΠΈΠ· Π½Π΅Π³ΠΎ, Π΄ΠΎΠΊΠ°ΠΆΠ΅ΠΌ Π²Ρ‹Ρ€Π°ΠΆΠ΅Π½ΠΈΠ΅: (-Π°)Β·(-b)=aΒ·b. Π‘Ρ‚Π°Ρ‚ΡŒΡ ΡƒΠΌΠ½ΠΎΠΆΠ΅Π½ΠΈΠ΅ чисСл с Ρ€Π°Π·Π½Ρ‹ΠΌΠΈ Π·Π½Π°ΠΊΠ°ΠΌΠΈ рассказываСт ΠΎ Ρ‚ΠΎΠΌ, Ρ‡Ρ‚ΠΎ равСнств Π°Β·(-b)=-aΒ·b справСдливоС, ΠΊΠ°ΠΊ ΠΈ (-Π°)Β·b=-aΒ·b. Π­Ρ‚ΠΎ слСдуСт ΠΈΠ· свойства ΠΏΡ€ΠΎΡ‚ΠΈΠ²ΠΎΠΏΠΎΠ»ΠΎΠΆΠ½Ρ‹Ρ… чисСл, благодаря ΠΊΠΎΡ‚ΠΎΡ€ΠΎΠΌΡƒ равСнства Π·Π°ΠΏΠΈΡˆΡƒΡ‚ΡΡ ΡΠ»Π΅Π΄ΡƒΡŽΡ‰ΠΈΠΌ ΠΎΠ±Ρ€Π°Π·ΠΎΠΌ:

(-a)Β·(-b)=(-aΒ·(-b))=-(-(aΒ·b))=Β aΒ·b.

Π’ΡƒΡ‚ явно Π²ΠΈΠ΄Π½ΠΎ Π΄ΠΎΠΊΠ°Π·Π°Ρ‚Π΅Π»ΡŒΡΡ‚Π²ΠΎ ΠΏΡ€Π°Π²ΠΈΠ»Π° умноТСния ΠΎΡ‚Ρ€ΠΈΡ†Π°Ρ‚Π΅Π»ΡŒΠ½Ρ‹Ρ… чисСл. Π˜ΡΡ…ΠΎΠ΄Ρ ΠΈΠ· ΠΏΡ€ΠΈΠΌΠ΅Ρ€ΠΎΠ² явно, Ρ‡Ρ‚ΠΎ ΠΏΡ€ΠΎΠΈΠ·Π²Π΅Π΄Π΅Π½ΠΈΠ΅ Π΄Π²ΡƒΡ… ΠΎΡ‚Ρ€ΠΈΡ†Π°Ρ‚Π΅Π»ΡŒΠ½Ρ‹Ρ… чисСл – ΠΏΠΎΠ»ΠΎΠΆΠΈΡ‚Π΅Π»ΡŒΠ½ΠΎΠ΅ число. ΠŸΡ€ΠΈ ΠΏΠ΅Ρ€Π΅ΠΌΠ½ΠΎΠΆΠ΅Π½ΠΈΠΈ ΠΌΠΎΠ΄ΡƒΠ»Π΅ΠΉ чисСл Ρ€Π΅Π·ΡƒΠ»ΡŒΡ‚Π°Ρ‚ всСгда ΠΏΠΎΠ»ΠΎΠΆΠΈΡ‚Π΅Π»ΡŒΠ½ΠΎΠ΅ число.

Π”Π°Π½Π½ΠΎΠ΅ ΠΏΡ€Π°Π²ΠΈΠ»ΠΎ ΠΏΡ€ΠΈΠΌΠ΅Π½ΠΈΠΌΠΎ для умноТСния Π΄Π΅ΠΉΡΡ‚Π²ΠΈΡ‚Π΅Π»ΡŒΠ½Ρ‹Ρ… чисСл, Ρ€Π°Ρ†ΠΈΠΎΠ½Π°Π»ΡŒΠ½Ρ‹Ρ… чисСл, Ρ†Π΅Π»Ρ‹Ρ… чисСл.

ΠŸΡ€ΠΈΠΌΠ΅Ρ€Ρ‹ умноТСния ΠΎΡ‚Ρ€ΠΈΡ†Π°Ρ‚Π΅Π»ΡŒΠ½Ρ‹Ρ… чисСл

Π’Π΅ΠΏΠ΅Ρ€ΡŒ рассмотрим ΠΏΠΎΠ΄Ρ€ΠΎΠ±Π½ΠΎ ΠΏΡ€ΠΈΠΌΠ΅Ρ€Ρ‹ умноТСния Π΄Π²ΡƒΡ… ΠΎΡ‚Ρ€ΠΈΡ†Π°Ρ‚Π΅Π»ΡŒΠ½Ρ‹Ρ… чисСл. ΠŸΡ€ΠΈ вычислСнии Π½Π΅ΠΎΠ±Ρ…ΠΎΠ΄ΠΈΠΌΠΎ ΠΏΠΎΠ»ΡŒΠ·ΠΎΠ²Π°Ρ‚ΡŒΡΡ ΠΏΡ€Π°Π²ΠΈΠ»ΠΎΠΌ, написанным Π²Ρ‹ΡˆΠ΅.

ΠŸΡ€ΠΈΠΌΠ΅Ρ€ 1

ΠŸΡ€ΠΎΠΈΠ·Π²Π΅ΡΡ‚ΠΈ ΡƒΠΌΠ½ΠΎΠΆΠ΅Π½ΠΈΠ΅ чисСл -3 ΠΈ -5.

РСшСниС.

По ΠΌΠΎΠ΄ΡƒΠ»ΡŽ ΡƒΠΌΠ½ΠΎΠΆΠ°Π΅ΠΌΡ‹Π΅ Π΄Π°Π½Π½Ρ‹Π΅ Π΄Π²Π° числа Ρ€Π°Π²Π½Ρ‹ ΠΏΠΎΠ»ΠΎΠΆΠΈΡ‚Π΅Π»ΡŒΠ½Ρ‹ΠΌ числам 3 ΠΈ 5. Π˜Ρ… ΠΏΡ€ΠΎΠΈΠ·Π²Π΅Π΄Π΅Π½ΠΈΠ΅ Π΄Π°Π΅Ρ‚Β  Π² Ρ€Π΅Π·ΡƒΠ»ΡŒΡ‚Π°Ρ‚Π΅ 15. ΠžΡ‚ΡΡŽΠ΄Π° слСдуСт, Ρ‡Ρ‚ΠΎ ΠΏΡ€ΠΎΠΈΠ·Π²Π΅Π΄Π΅Π½ΠΈΠ΅ Π·Π°Π΄Π°Π½Π½Ρ‹Ρ… чисСл Ρ€Π°Π²Π½ΠΎ 15

Π—Π°ΠΏΠΈΡˆΠ΅ΠΌ ΠΊΡ€Π°Ρ‚ΠΊΠΎ само ΡƒΠΌΠ½ΠΎΠΆΠ΅Π½ΠΈΠ΅ ΠΎΡ‚Ρ€ΠΈΡ†Π°Ρ‚Π΅Π»ΡŒΠ½Ρ‹Ρ… чисСл:

(-3)Β·(-5)=3Β·5=15

ΠžΡ‚Π²Π΅Ρ‚: (-3)Β·(-5)=15.

ΠŸΡ€ΠΈ ΡƒΠΌΠ½ΠΎΠΆΠ΅Π½ΠΈΠΈ ΠΎΡ‚Ρ€ΠΈΡ†Π°Ρ‚Π΅Π»ΡŒΠ½Ρ‹Ρ… Ρ€Π°Ρ†ΠΈΠΎΠ½Π°Π»ΡŒΠ½Ρ‹Ρ… чисСл, ΠΏΡ€ΠΈΠΌΠ΅Π½ΠΈΠ² Ρ€Π°Π·ΠΎΠ±Ρ€Π°Π½Π½ΠΎΠ΅ ΠΏΡ€Π°Π²ΠΈΠ»ΠΎ, ΠΌΠΎΠΆΠ½ΠΎ ΠΌΠΎΠ±ΠΈΠ»ΠΈΠ·ΠΎΠ²Π°Ρ‚ΡŒΡΡ ΠΊ ΡƒΠΌΠ½ΠΎΠΆΠ΅Π½ΠΈΡŽ Π΄Ρ€ΠΎΠ±Π΅ΠΉ, ΡƒΠΌΠ½ΠΎΠΆΠ΅Π½ΠΈΡŽ ΡΠΌΠ΅ΡˆΠ°Π½Π½Ρ‹Ρ… чисСл, ΡƒΠΌΠ½ΠΎΠΆΠ΅Π½ΠΈΡŽ дСсятичных Π΄Ρ€ΠΎΠ±Π΅ΠΉ.

НуТна ΠΏΠΎΠΌΠΎΡ‰ΡŒ прСподаватСля?

Опиши Π·Π°Π΄Π°Π½ΠΈΠ΅Β β€” и наши экспСрты Ρ‚Π΅Π±Π΅ ΠΏΠΎΠΌΠΎΠ³ΡƒΡ‚!

ΠžΠΏΠΈΡΠ°Ρ‚ΡŒ Π·Π°Π΄Π°Π½ΠΈΠ΅ ΠŸΡ€ΠΈΠΌΠ΅Ρ€ 2

Π’Ρ‹Ρ‡ΠΈΡΠ»ΠΈΡ‚ΡŒ ΠΏΡ€ΠΎΠΈΠ·Π²Π΅Π΄Π΅Π½ΠΈΠ΅ (-0,125)Β·(-6).

РСшСниС.

Π˜ΡΠΏΠΎΠ»ΡŒΠ·ΡƒΡ ΠΏΡ€Π°Π²ΠΈΠ»ΠΎ умноТСния ΠΎΡ‚Ρ€ΠΈΡ†Π°Ρ‚Π΅Π»ΡŒΠ½Ρ‹Ρ… чисСл, ΠΏΠΎΠ»ΡƒΡ‡ΠΈΠΌ, Ρ‡Ρ‚ΠΎ Β (βˆ’0,125)Β·(βˆ’6)=0,125Β·6. Для получСния Ρ€Π΅Π·ΡƒΠ»ΡŒΡ‚Π°Ρ‚Π° Π½Π΅ΠΎΠ±Ρ…ΠΎΠ΄ΠΈΠΌΠΎ Π²Ρ‹ΠΏΠΎΠ»Π½ΠΈΡ‚ΡŒ ΡƒΠΌΠ½ΠΎΠΆΠ΅Π½ΠΈΠ΅ дСсятичной Π΄Ρ€ΠΎΠ±ΠΈ Π½Π° Π½Π°Ρ‚ΡƒΡ€Π°Π»ΡŒΠ½ΠΎΠ΅ число столбиков. Π­Ρ‚ΠΎ выглядит Ρ‚Π°ΠΊ:

ΠŸΠΎΠ»ΡƒΡ‡ΠΈΠ»ΠΈ, Ρ‡Ρ‚ΠΎ Π²Ρ‹Ρ€Π°ΠΆΠ΅Π½ΠΈΠ΅ ΠΏΡ€ΠΈΠΌΠ΅Ρ‚ Π²ΠΈΠ΄ (βˆ’0,125)Β·(βˆ’6)=0,125Β·6=0,75.

ΠžΡ‚Π²Π΅Ρ‚: Β (βˆ’0,125)Β·(βˆ’6)=0,75.

Π’ случаС, ΠΊΠΎΠ³Π΄Π° ΠΌΠ½ΠΎΠΆΠΈΡ‚Π΅Π»ΠΈ – ΠΈΡ€Ρ€Π°Ρ†ΠΈΠΎΠ½Π°Π»ΡŒΠ½Ρ‹Π΅ числа, Ρ‚ΠΎΠ³Π΄Π° ΠΈΡ… ΠΏΡ€ΠΎΠΈΠ·Π²Π΅Π΄Π΅Π½ΠΈΠ΅ ΠΌΠΎΠΆΠ΅Ρ‚ Π±Ρ‹Ρ‚ΡŒ записано Π² Π²ΠΈΠ΄Π΅ числового выраТСния. Π—Π½Π°Ρ‡Π΅Π½ΠΈΠ΅ вычисляСтся Ρ‚ΠΎΠ»ΡŒΠΊΠΎ ΠΏΠΎ нСобходимости.

ΠŸΡ€ΠΈΠΌΠ΅Ρ€ 3

НСобходимо произвСсти ΡƒΠΌΠ½ΠΎΠΆΠ΅Π½ΠΈΠ΅ ΠΎΡ‚Ρ€ΠΈΡ†Π°Ρ‚Π΅Π»ΡŒΠ½ΠΎΠ³ΠΎ -2Β Π½Π° Π½Π΅ΠΎΡ‚Ρ€ΠΈΡ†Π°Ρ‚Π΅Π»ΡŒΠ½ΠΎΠ΅ log5Β 13.

РСшСниС

Находим ΠΌΠΎΠ΄ΡƒΠ»ΠΈ Π·Π°Π΄Π°Π½Π½Ρ‹Ρ… чисСл:

-2=2Β ΠΈ log513=-log5Β 3=log5Β 3.

БлСдуя ΠΈΠ· ΠΏΡ€Π°Π²ΠΈΠ» умноТСния ΠΎΡ‚Ρ€ΠΈΡ†Π°Ρ‚Π΅Π»ΡŒΠ½Ρ‹Ρ… чисСл, ΠΏΠΎΠ»ΡƒΡ‡ΠΈΠΌ Ρ€Π΅Π·ΡƒΠ»ΡŒΡ‚Π°Ρ‚ -2Β·log5Β 13=-2Β·log5Β 3=2Β·log5Β 3. Π­Ρ‚ΠΎ Π²Ρ‹Ρ€Π°ΠΆΠ΅Π½ΠΈΠ΅ ΠΈ являСтся ΠΎΡ‚Π²Π΅Ρ‚ΠΎΠΌ.

ΠžΡ‚Π²Π΅Ρ‚: Β -2Β·log5Β 13=-2Β·log5Β 3=2Β·log5Β 3.

Для продолТСния изучСния Ρ‚Π΅ΠΌΡ‹ Π½Π΅ΠΎΠ±Ρ…ΠΎΠ΄ΠΈΠΌΠΎ ΠΏΠΎΠ²Ρ‚ΠΎΡ€ΠΈΡ‚ΡŒ Ρ€Π°Π·Π΄Π΅Π» ΡƒΠΌΠ½ΠΎΠΆΠ΅Π½ΠΈΠ΅ Π΄Π΅ΠΉΡΡ‚Π²ΠΈΡ‚Π΅Π»ΡŒΠ½Ρ‹Ρ… чисСл.

Π£ΠΌΠ½ΠΎΠΆΠ΅Π½ΠΈΠ΅ ΠΈ Π΄Π΅Π»Π΅Π½ΠΈΠ΅ Ρ€Π°Ρ†ΠΈΠΎΠ½Π°Π»ΡŒΠ½Ρ‹Ρ… чисСл

1. Π£Ρ‚Π²Π΅Ρ€ΠΆΠ΄Π΅Π½ΠΈΠ΅ (ΡƒΠΌΠ½ΠΎΠΆΠ΅Π½ΠΈΠ΅) 1 Π²ΠΈΠ΄ β€” Ρ€Π΅Ρ†Π΅ΠΏΡ‚ΠΈΠ²Π½Ρ‹ΠΉ Π»Ρ‘Π³ΠΊΠΎΠ΅ 1 Π‘. НСобходимо Π²Ρ‹Π±Ρ€Π°Ρ‚ΡŒ Π²Π΅Ρ€Π½ΠΎΠ΅/Π»ΠΎΠΆΠ½ΠΎΠ΅ ΡƒΡ‚Π²Π΅Ρ€ΠΆΠ΄Π΅Π½ΠΈΠ΅ (ΡƒΠΌΠ½ΠΎΠΆΠ΅Π½ΠΈΠ΅).
2. Π£Ρ‚Π²Π΅Ρ€ΠΆΠ΄Π΅Π½ΠΈΠ΅ (частноС) 1 Π²ΠΈΠ΄ β€” Ρ€Π΅Ρ†Π΅ΠΏΡ‚ΠΈΠ²Π½Ρ‹ΠΉ Π»Ρ‘Π³ΠΊΠΎΠ΅ 1 Π‘. НСобходимо Π²Ρ‹Π±Ρ€Π°Ρ‚ΡŒ Π²Π΅Ρ€Π½ΠΎΠ΅/Π»ΠΎΠΆΠ½ΠΎΠ΅ ΡƒΡ‚Π²Π΅Ρ€ΠΆΠ΄Π΅Π½ΠΈΠ΅ (частноС).
3. Π£ΠΌΠ½ΠΎΠΆΠ΅Π½ΠΈΠ΅ Π½Π° Π΅Π΄ΠΈΠ½ΠΈΡ†Ρƒ 1 Π²ΠΈΠ΄ β€” Ρ€Π΅Ρ†Π΅ΠΏΡ‚ΠΈΠ²Π½Ρ‹ΠΉ Π»Ρ‘Π³ΠΊΠΎΠ΅ 2 Π‘. Π£ΠΌΠ½ΠΎΠΆΠ΅Π½ΠΈΠ΅ Π½Π° Π΅Π΄ΠΈΠ½ΠΈΡ†Ρƒ.
4. ΠŸΡ€ΠΎΠΈΠ·Π²Π΅Π΄Π΅Π½ΠΈΠ΅ Ρ†Π΅Π»Ρ‹Ρ… чисСл (Π΄ΠΎ 10) 1 Π²ΠΈΠ΄ β€” Ρ€Π΅Ρ†Π΅ΠΏΡ‚ΠΈΠ²Π½Ρ‹ΠΉ Π»Ρ‘Π³ΠΊΠΎΠ΅ 1 Π‘. ΠŸΡ€ΠΎΠΈΠ·Π²Π΅Π΄Π΅Π½ΠΈΠ΅ Ρ†Π΅Π»Ρ‹Ρ… чисСл.
5. ΠŸΡ€ΠΎΠΈΠ·Π²Π΅Π΄Π΅Π½ΠΈΠ΅ Ρ€Π°Ρ†ΠΈΠΎΠ½Π°Π»ΡŒΠ½Ρ‹Ρ… чисСл (дСсятичныС Π΄Ρ€ΠΎΠ±ΠΈ) 2 Π²ΠΈΠ΄ β€” интСрпрСтация срСднСС 1 Π‘. ДСсятичная Π΄Ρ€ΠΎΠ±ΡŒ ΠΈ Ρ†Π΅Π»ΠΎΠ΅ число.
6. Π£ΠΌΠ½ΠΎΠΆΠ΅Π½ΠΈΠ΅ ΠΎΠ±Ρ‹ΠΊΠ½ΠΎΠ²Π΅Π½Π½ΠΎΠΉ Π΄Ρ€ΠΎΠ±ΠΈ Π½Π° ΠΏΠΎΠ»ΠΎΠΆΠΈΡ‚Π΅Π»ΡŒΠ½ΠΎΠ΅ ΠΈΠ»ΠΈ ΠΎΡ‚Ρ€ΠΈΡ†Π°Ρ‚Π΅Π»ΡŒΠ½ΠΎΠ΅ Ρ†Π΅Π»ΠΎΠ΅ число 2 Π²ΠΈΠ΄ β€” интСрпрСтация
срСднСС
2 Π‘. Один ΠΈΠ»ΠΈ ΠΎΠ±Π° мноТитСля ΠΎΡ‚Ρ€ΠΈΡ†Π°Ρ‚Π΅Π»ΡŒΠ½Ρ‹Π΅.
7. ΠŸΡ€ΠΎΠΈΠ·Π²Π΅Π΄Π΅Π½ΠΈΠ΅ Π΄Π²ΡƒΡ… дСсятичных Π΄Ρ€ΠΎΠ±Π΅ΠΉ с Ρ€Π°Π·Π½Ρ‹ΠΌΠΈ Π·Π½Π°ΠΊΠ°ΠΌΠΈ 2 Π²ΠΈΠ΄ β€” интСрпрСтация срСднСС 1 Π‘. ΠŸΡ€ΠΎΠΈΠ·Π²Π΅Π΄Π΅Π½ΠΈΠ΅ Π΄Π²ΡƒΡ… дСсятичных Π΄Ρ€ΠΎΠ±Π΅ΠΉ с Ρ€Π°Π·Π½Ρ‹ΠΌΠΈ Π·Π½Π°ΠΊΠ°ΠΌΠΈ.
8. ΠŸΡ€ΠΎΠΈΠ·Π²Π΅Π΄Π΅Π½ΠΈΠ΅ Ρ†Π΅Π»ΠΎΠ³ΠΎ числа ΠΈ смСшанного (ΠΎΡ‚Ρ€ΠΈΡ†Π°Ρ‚Π΅Π»ΡŒΠ½Ρ‹Π΅ числа) 2 Π²ΠΈΠ΄ β€” интСрпрСтация срСднСС 3 Π‘. ΠŸΡ€ΠΎΠΈΠ·Π²Π΅Π΄Π΅Π½ΠΈΠ΅ Ρ†Π΅Π»ΠΎΠ³ΠΎ числа ΠΈ смСшанного.
9. ΠŸΡ€ΠΎΠΈΠ·Π²Π΅Π΄Π΅Π½ΠΈΠ΅ дСсятичной Π΄Ρ€ΠΎΠ±ΠΈ ΠΈ ΠΎΠ±Ρ‹ΠΊΠ½ΠΎΠ²Π΅Π½Π½ΠΎΠΉ (ΠΎΡ‚Ρ€ΠΈΡ†Π°Ρ‚Π΅Π»ΡŒΠ½Ρ‹Π΅ числа) 2 Π²ΠΈΠ΄ β€” интСрпрСтация срСднСС 3 Π‘. ΠŸΡ€ΠΎΠΈΠ·Π²Π΅Π΄Π΅Π½ΠΈΠ΅ дСсятичной Π΄Ρ€ΠΎΠ±ΠΈ ΠΈ ΠΎΠ±Ρ‹ΠΊΠ½ΠΎΠ²Π΅Π½Π½ΠΎΠΉ.
10. ΠŸΡ€ΠΎΠΈΠ·Π²Π΅Π΄Π΅Π½ΠΈΠ΅ дСсятичной Π΄Ρ€ΠΎΠ±ΠΈ ΠΈ смСшанного числа (ΠΎΡ‚Ρ€ΠΈΡ†Π°Ρ‚Π΅Π»ΡŒΠ½Ρ‹Π΅) 2 Π²ΠΈΠ΄ β€” интСрпрСтация срСднСС 3 Π‘. ΠŸΡ€ΠΎΠΈΠ·Π²Π΅Π΄Π΅Π½ΠΈΠ΅ дСсятичной Π΄Ρ€ΠΎΠ±ΠΈ ΠΈ смСшанного числа.
11. ЧастноС чисСл с Ρ€Π°Π·Π½Ρ‹ΠΌΠΈ Π·Π½Π°ΠΊΠ°ΠΌΠΈ 2 Π²ΠΈΠ΄ β€” интСрпрСтация
срСднСС
1 Π‘. Π¦Π΅Π»Ρ‹Π΅ числа.
12. Π”Π΅Π»Π΅Π½ΠΈΠ΅ ΠΎΠ±Ρ‹ΠΊΠ½ΠΎΠ²Π΅Π½Π½ΠΎΠΉ Π΄Ρ€ΠΎΠ±ΠΈ Π½Π° ΠΏΠΎΠ»ΠΎΠΆΠΈΡ‚Π΅Π»ΡŒΠ½ΠΎΠ΅ ΠΈΠ»ΠΈ ΠΎΡ‚Ρ€ΠΈΡ†Π°Ρ‚Π΅Π»ΡŒΠ½ΠΎΠ΅ Ρ†Π΅Π»ΠΎΠ΅ число 2 Π²ΠΈΠ΄ β€” интСрпрСтация срСднСС 3 Π‘. Π”Π΅Π»Π΅Π½ΠΈΠ΅ ΠΎΠ±Ρ‹ΠΊΠ½ΠΎΠ²Π΅Π½Π½ΠΎΠΉ Π΄Ρ€ΠΎΠ±ΠΈ Π½Π° ΠΏΠΎΠ»ΠΎΠΆΠΈΡ‚Π΅Π»ΡŒΠ½ΠΎΠ΅ ΠΈΠ»ΠΈ ΠΎΡ‚Ρ€ΠΈΡ†Π°Ρ‚Π΅Π»ΡŒΠ½ΠΎΠ΅ Ρ†Π΅Π»ΠΎΠ΅ число.
13. Π”Π΅Π»Π΅Π½ΠΈΠ΅ Ρ†Π΅Π»ΠΎΠ³ΠΎ числа Π½Π° ΠΏΡ€Π°Π²ΠΈΠ»ΡŒΠ½ΡƒΡŽ Π΄Ρ€ΠΎΠ±ΡŒ 2 Π²ΠΈΠ΄ β€” интСрпрСтация срСднСС 2 Π‘. Одно число ΠΈΠ»ΠΈ ΠΎΠ±Π° числа ΠΎΡ‚Ρ€ΠΈΡ†Π°Ρ‚Π΅Π»ΡŒΠ½Ρ‹Π΅. Π Π΅Π·ΡƒΠ»ΡŒΡ‚Π°Ρ‚ β€” Ρ†Π΅Π»ΠΎΠ΅ число.
14. Π”Π΅Π»Π΅Π½ΠΈΠ΅ смСшанного числа Π½Π° ΠΏΠΎΠ»ΠΎΠΆΠΈΡ‚Π΅Π»ΡŒΠ½ΠΎΠ΅ ΠΈΠ»ΠΈ ΠΎΡ‚Ρ€ΠΈΡ†Π°Ρ‚Π΅Π»ΡŒΠ½ΠΎΠ΅ Ρ†Π΅Π»ΠΎΠ΅ число 2 Π²ΠΈΠ΄ β€” интСрпрСтация срСднСС 3 Π‘. Π”Π΅Π»Π΅Π½ΠΈΠ΅ Ρ†Π΅Π»ΠΎΠΉ части ΠΈ Π΄Ρ€ΠΎΠ±Π½ΠΎΠΉ ΠΎΡ‚Π΄Π΅Π»ΡŒΠ½ΠΎ.
15. Π”Π΅Π»Π΅Π½ΠΈΠ΅ дСсятичной Π΄Ρ€ΠΎΠ±ΠΈ / Ρ†Π΅Π»ΠΎΠ³ΠΎ числа Π½Π° ΠΎΠ±Ρ‹ΠΊΠ½ΠΎΠ²Π΅Π½Π½ΡƒΡŽ Π΄Ρ€ΠΎΠ±ΡŒ (Ρ€Π°Π·Π½Ρ‹Π΅ Π·Π½Π°ΠΊΠΈ) 2 Π²ΠΈΠ΄ β€” интСрпрСтация срСднСС 3 Π‘. Π”Π΅Π»Π΅Π½ΠΈΠ΅ дСсятичной Π΄Ρ€ΠΎΠ±ΠΈ / Ρ†Π΅Π»ΠΎΠ³ΠΎ числа Π½Π° ΠΎΠ±Ρ‹ΠΊΠ½ΠΎΠ²Π΅Π½Π½ΡƒΡŽ Π΄Ρ€ΠΎΠ±ΡŒ.
16. Π”Π΅Π»Π΅Π½ΠΈΠ΅ дСсятичных Π΄Ρ€ΠΎΠ±Π΅ΠΉ (Ρ€Π°Π·Π½Ρ‹Π΅ Π·Π½Π°ΠΊΠΈ) 2 Π²ΠΈΠ΄ β€” интСрпрСтация срСднСС 2 Π‘.
Π”Π΅Π»Π΅Π½ΠΈΠ΅ дСсятичных Π΄Ρ€ΠΎΠ±Π΅ΠΉ.
17. Π”Π΅Π»Π΅Π½ΠΈΠ΅ дСсятичной Π΄Ρ€ΠΎΠ±ΠΈ Π½Π° ΠΎΠ±Ρ‹ΠΊΠ½ΠΎΠ²Π΅Π½Π½ΡƒΡŽ (Ρ€Π°Π·Π½Ρ‹Π΅ Π·Π½Π°ΠΊΠΈ) 2 Π²ΠΈΠ΄ β€” интСрпрСтация срСднСС 3 Π‘. Π”Π΅Π»Π΅Π½ΠΈΠ΅ дСсятичной Π΄Ρ€ΠΎΠ±ΠΈ Π½Π° ΠΎΠ±Ρ‹ΠΊΠ½ΠΎΠ²Π΅Π½Π½ΡƒΡŽ.
18. Π”Π΅Π»Π΅Π½ΠΈΠ΅ Π΄Π²ΡƒΡ… ΠΎΡ‚Ρ€ΠΈΡ†Π°Ρ‚Π΅Π»ΡŒΠ½Ρ‹Ρ… дСсятичных Π΄Ρ€ΠΎΠ±Π΅ΠΉ 2 Π²ΠΈΠ΄ β€” интСрпрСтация срСднСС 2 Π‘. Π”Π΅Π»Π΅Π½ΠΈΠ΅ Π΄Π²ΡƒΡ… ΠΎΡ‚Ρ€ΠΈΡ†Π°Ρ‚Π΅Π»ΡŒΠ½Ρ‹Ρ… дСсятичных Π΄Ρ€ΠΎΠ±Π΅ΠΉ.
19. ЧастноС Π΄Π²ΡƒΡ… ΠΎΡ‚Ρ€ΠΈΡ†Π°Ρ‚Π΅Π»ΡŒΠ½Ρ‹Ρ… ΡΠΌΠ΅ΡˆΠ°Π½Π½Ρ‹Ρ… чисСл 2 Π²ΠΈΠ΄ β€” интСрпрСтация срСднСС 3 Π‘. ЧастноС Π΄Π²ΡƒΡ… ΠΎΡ‚Ρ€ΠΈΡ†Π°Ρ‚Π΅Π»ΡŒΠ½Ρ‹Ρ… ΡΠΌΠ΅ΡˆΠ°Π½Π½Ρ‹Ρ… чисСл.
20. ΠŸΡ€ΠΎΠΈΠ·Π²Π΅Π΄Π΅Π½ΠΈΠ΅ 2 Π²ΠΈΠ΄ β€” интСрпрСтация срСднСС 1 Π‘. ΠžΠΏΡ€Π΅Π΄Π΅Π»Π΅Π½ΠΈΠ΅ Π·Π½Π°ΠΊΠ° произвСдСния.
21. ΠŸΡ€ΠΎΠΈΠ·Π²Π΅Π΄Π΅Π½ΠΈΠ΅ Π½Π΅ΡΠΊΠΎΠ»ΡŒΠΊΠΈΡ… Ρ†Π΅Π»Ρ‹Ρ… чисСл с Ρ€Π°Π·Π½Ρ‹ΠΌΠΈ Π·Π½Π°ΠΊΠ°ΠΌΠΈ 2 Π²ΠΈΠ΄ β€” интСрпрСтация срСднСС 2 Π‘. ΠŸΡ€ΠΎΠΈΠ·Π²Π΅Π΄Π΅Π½ΠΈΠ΅ Π½Π΅ΡΠΊΠΎΠ»ΡŒΠΊΠΈΡ… Ρ†Π΅Π»Ρ‹Ρ… чисСл с Ρ€Π°Π·Π½Ρ‹ΠΌΠΈ Π·Π½Π°ΠΊΠ°ΠΌΠΈ.
22. ΠŸΡ€ΠΎΠΈΠ·Π²Π΅Π΄Π΅Π½ΠΈΠ΅ Ρ‚Ρ€Ρ‘Ρ… ΠΎΠ±Ρ‹ΠΊΠ½ΠΎΠ²Π΅Π½Π½Ρ‹Ρ… Π΄Ρ€ΠΎΠ±Π΅ΠΉ, ΠΊΠ°ΠΊ ΠΌΠΈΠ½ΠΈΠΌΡƒΠΌ ΠΎΠ΄Π½Π° Π΄Ρ€ΠΎΠ±ΡŒ β€” ΠΎΡ‚Ρ€ΠΈΡ†Π°Ρ‚Π΅Π»ΡŒΠ½Π°Ρ 2 Π²ΠΈΠ΄ β€” интСрпрСтация срСднСС 3 Π‘. Π‘ΠΎΠΊΡ€Π°Ρ‰Π΅Π½ΠΈΠ΅, ΠΏΡ€ΠΈΠ²Π΅Π΄Π΅Π½ΠΈΠ΅ ΠΊ ΠΏΡ€Π°Π²ΠΈΠ»ΡŒΠ½ΠΎΠΉ Π΄Ρ€ΠΎΠ±ΠΈ.
23. Π”Π΅Π»Π΅Π½ΠΈΠ΅ Π½Π° Π΄Ρ€ΠΎΠ±ΡŒ ΠΈ ΡƒΠΌΠ½ΠΎΠΆΠ΅Π½ΠΈΠ΅ Π½Π° Π΄Ρ€ΠΎΠ±ΡŒ, ΠΊΠ°ΠΊ ΠΌΠΈΠ½ΠΈΠΌΡƒΠΌ ΠΎΠ΄Π½Π° Π΄Ρ€ΠΎΠ±ΡŒ β€” ΠΎΡ‚Ρ€ΠΈΡ†Π°Ρ‚Π΅Π»ΡŒΠ½Π°Ρ 2 Π²ΠΈΠ΄ β€” интСрпрСтация срСднСС 3 Π‘. ДСйствия с трСмя дробями. Π”Π΅Π»Π΅Π½ΠΈΠ΅ ΠΈ ΡƒΠΌΠ½ΠΎΠΆΠ΅Π½ΠΈΠ΅.
24. Π£Ρ€Π°Π²Π½Π΅Π½ΠΈΠ΅ (дСсятичныС Π΄Ρ€ΠΎΠ±ΠΈ ΠΈ Ρ†Π΅Π»Ρ‹Π΅ числа) 2 Π²ΠΈΠ΄ β€” интСрпрСтация слоТноС 3 Π‘. РСшСниС уравнСния.
25. Π£Ρ€Π°Π²Π½Π΅Π½ΠΈΠ΅ с ΠΌΠΎΠ΄ΡƒΠ»Π΅ΠΌ 2 Π²ΠΈΠ΄ β€” интСрпрСтация слоТноС 6 Π‘. РСшСниС уравнСния.
26. НСизвСстный Ρ‡Π»Π΅Π½ ΠΏΡ€ΠΎΠΏΠΎΡ€Ρ†ΠΈΠΈ 2 Π²ΠΈΠ΄ β€” интСрпрСтация слоТноС 4 Π‘. ΠŸΡ€ΠΈΠΌΠ΅Π½Π΅Π½ΠΈΠ΅ основного свойства ΠΏΡ€ΠΎΠΏΠΎΡ€Ρ†ΠΈΠΈ.

Π‘Π»ΠΎΠΆΠ΅Π½ΠΈΠ΅ ΠΈ Π²Ρ‹Ρ‡ΠΈΡ‚Π°Π½ΠΈΠ΅ Ρ€Π°Ρ†ΠΈΠΎΠ½Π°Π»ΡŒΠ½Ρ‹Ρ… чисСл

Π’ Π΄Π°Π½Π½ΠΎΠΌ ΡƒΡ€ΠΎΠΊΠ΅ рассматриваСтся слоТСниС ΠΈ Π²Ρ‹Ρ‡ΠΈΡ‚Π°Π½ΠΈΠ΅ Ρ€Π°Ρ†ΠΈΠΎΠ½Π°Π»ΡŒΠ½Ρ‹Ρ… чисСл. Π’Π΅ΠΌΠ° относится ΠΊ ΠΊΠ°Ρ‚Π΅Π³ΠΎΡ€ΠΈΠΈ слоТных. Π—Π΄Π΅ΡΡŒ Π½Π΅ΠΎΠ±Ρ…ΠΎΠ΄ΠΈΠΌΠΎ ΠΈΡΠΏΠΎΠ»ΡŒΠ·ΠΎΠ²Π°Ρ‚ΡŒ вСсь арсСнал ΠΏΠΎΠ»ΡƒΡ‡Π΅Π½Π½Ρ‹Ρ… Ρ€Π°Π½Π΅Π΅ Π·Π½Π°Π½ΠΈΠΉ.

ΠŸΡ€Π°Π²ΠΈΠ»Π° слоТСния ΠΈ вычитания Ρ†Π΅Π»Ρ‹Ρ… чисСл справСдливы ΠΈ для Ρ€Π°Ρ†ΠΈΠΎΠ½Π°Π»ΡŒΠ½Ρ‹Ρ… чисСл. Напомним, Ρ‡Ρ‚ΠΎ Ρ€Π°Ρ†ΠΈΠΎΠ½Π°Π»ΡŒΠ½Ρ‹ΠΌΠΈ Π½Π°Π·Ρ‹Π²Π°ΡŽΡ‚ числа, ΠΊΠΎΡ‚ΠΎΡ€Ρ‹Π΅ ΠΌΠΎΠ³ΡƒΡ‚ Π±Ρ‹Ρ‚ΡŒ прСдставлСны Π² Π²ΠΈΠ΄Π΅ Π΄Ρ€ΠΎΠ±ΠΈΒ  ,Β  Π³Π΄Π΅ a – это Ρ‡ΠΈΡΠ»ΠΈΡ‚Π΅Π»ΡŒ Π΄Ρ€ΠΎΠ±ΠΈ, b – Π·Π½Π°ΠΌΠ΅Π½Π°Ρ‚Π΅Π»ΡŒ Π΄Ρ€ΠΎΠ±ΠΈ. ΠŸΡ€ΠΈ этом,Β b Π½Π΅ Π΄ΠΎΠ»ΠΆΠ½ΠΎ Π±Ρ‹Ρ‚ΡŒ Π½ΡƒΠ»Ρ‘ΠΌ.

Π’ Π΄Π°Π½Π½ΠΎΠΌΒ ΡƒΡ€ΠΎΠΊΠ΅ Π΄Ρ€ΠΎΠ±ΠΈ ΠΈ ΡΠΌΠ΅ΡˆΠ°Π½Π½Ρ‹Π΅ числа ΠΌΡ‹ всё Ρ‡Π°Ρ‰Π΅ Π±ΡƒΠ΄Π΅ΠΌ Π½Π°Π·Ρ‹Π²Π°Ρ‚ΡŒ ΠΎΠ΄Π½ΠΈΠΌ ΠΎΠ±Ρ‰ΠΈΠΌ словосочСтаниСм — Ρ€Π°Ρ†ΠΈΠΎΠ½Π°Π»ΡŒΠ½Ρ‹Π΅ числа.

Β 

ΠŸΡ€ΠΈΠΌΠ΅Ρ€ 1. Найти Π·Π½Π°Ρ‡Π΅Π½ΠΈΠ΅ выраТСния:Β 

Π—Π°ΠΊΠ»ΡŽΡ‡ΠΈΠΌΒ ΠΊΠ°ΠΆΠ΄ΠΎΠ΅ Ρ€Π°Ρ†ΠΈΠΎΠ½Π°Π»ΡŒΠ½ΠΎΠ΅ число Π² скобки вмСстС со своими Π·Π½Π°ΠΊΠ°ΠΌΠΈ. Π£Ρ‡ΠΈΡ‚Ρ‹Π²Π°Π΅ΠΌ, Ρ‡Ρ‚ΠΎ плюс ΠΊΠΎΡ‚ΠΎΡ€Ρ‹ΠΉ Π΄Π°Π½ Π² Π²Ρ‹Ρ€Π°ΠΆΠ΅Π½ΠΈΠΈ, являСтся Π·Π½Π°ΠΊΠΎΠΌ ΠΎΠΏΠ΅Ρ€Π°Ρ†ΠΈΠΈ ΠΈ Π½Π΅ относится ΠΊ Π΄Ρ€ΠΎΠ±ΠΈ . Π£ этой Π΄Ρ€ΠΎΠ±ΠΈ свой Π·Π½Π°ΠΊ плюса, ΠΊΠΎΡ‚ΠΎΡ€Ρ‹ΠΉ Π½Π΅Π²ΠΈΠ΄ΠΈΠΌ ΠΏΠΎ ΠΏΡ€ΠΈΡ‡ΠΈΠ½Π΅ Ρ‚ΠΎΠ³ΠΎ, Ρ‡Ρ‚ΠΎ Π΅Π³ΠΎ Π½Π΅ Π·Π°ΠΏΠΈΡΡ‹Π²Π°ΡŽΡ‚. Но ΠΌΡ‹ запишСм Π΅Π³ΠΎ для наглядности:

Π­Ρ‚ΠΎ слоТСниС Ρ€Π°Ρ†ΠΈΠΎΠ½Π°Π»ΡŒΠ½Ρ‹Ρ… чисСл с Ρ€Π°Π·Π½Ρ‹ΠΌΠΈ Π·Π½Π°ΠΊΠ°ΠΌΠΈ. Π§Ρ‚ΠΎΠ±Ρ‹ ΡΠ»ΠΎΠΆΠΈΡ‚ΡŒ Ρ€Π°Ρ†ΠΈΠΎΠ½Π°Π»ΡŒΠ½Ρ‹Π΅ числа с Ρ€Π°Π·Π½Ρ‹ΠΌΠΈ Π·Π½Π°ΠΊΠ°ΠΌΠΈ, Π½ΡƒΠΆΠ½ΠΎ ΠΈΠ· большСго модуля Π²Ρ‹Ρ‡Π΅ΡΡ‚ΡŒ мСньший ΠΌΠΎΠ΄ΡƒΠ»ΡŒ, ΠΈ ΠΏΠ΅Ρ€Π΅Π΄ ΠΏΠΎΠ»ΡƒΡ‡Π΅Π½Π½Ρ‹ΠΌ ΠΎΡ‚Π²Π΅Ρ‚ΠΎΠΌ ΠΏΠΎΡΡ‚Π°Π²ΠΈΡ‚ΡŒ Π·Π½Π°ΠΊ Ρ‚ΠΎΠ³ΠΎ Ρ€Π°Ρ†ΠΈΠΎΠ½Π°Π»ΡŒΠ½ΠΎΠ³ΠΎ числа, ΠΌΠΎΠ΄ΡƒΠ»ΡŒ ΠΊΠΎΡ‚ΠΎΡ€ΠΎΠ³ΠΎ большС. А Ρ‡Ρ‚ΠΎΠ±Ρ‹ ΠΏΠΎΠ½ΡΡ‚ΡŒ ΠΊΠ°ΠΊΠΎΠΉ ΠΌΠΎΠ΄ΡƒΠ»ΡŒ большС, Π° ΠΊΠ°ΠΊΠΎΠΉ мСньшС, Π½ΡƒΠΆΠ½ΠΎ ΡΡƒΠΌΠ΅Ρ‚ΡŒ ΡΡ€Π°Π²Π½ΠΈΡ‚ΡŒ ΠΌΠΎΠ΄ΡƒΠ»ΠΈ этих Π΄Ρ€ΠΎΠ±Π΅ΠΉ Π΄ΠΎ ΠΈΡ… вычислСния:

ΠœΠΎΠ΄ΡƒΠ»ΡŒ Ρ€Π°Ρ†ΠΈΠΎΠ½Π°Π»ΡŒΠ½ΠΎΠ³ΠΎ числа   большС, Ρ‡Π΅ΠΌ ΠΌΠΎΠ΄ΡƒΠ»ΡŒ Ρ€Π°Ρ†ΠΈΠΎΠ½Π°Π»ΡŒΠ½ΠΎΠ³ΠΎ числа . ΠŸΠΎΡΡ‚ΠΎΠΌΡƒ ΠΌΡ‹ ΠΈΠ· Β Π²Ρ‹Ρ‡Π»ΠΈ . ΠŸΠΎΠ»ΡƒΡ‡ΠΈΠ»ΠΈ ΠΎΡ‚Π²Π΅Ρ‚Β . Π—Π°Ρ‚Π΅ΠΌ сократив эту Π΄Ρ€ΠΎΠ±ΡŒ Π½Π° 2, ΠΏΠΎΠ»ΡƒΡ‡ΠΈΠ»ΠΈ ΠΎΠΊΠΎΠ½Ρ‡Π°Ρ‚Π΅Π»ΡŒΠ½Ρ‹ΠΉ ΠΎΡ‚Π²Π΅Ρ‚ .

НСкоторыС ΠΏΡ€ΠΈΠΌΠΈΡ‚ΠΈΠ²Π½Ρ‹Π΅ дСйствия, Ρ‚Π°ΠΊΠΈΠ΅ ΠΊΠ°ΠΊ Π·Π°ΠΊΠ»ΡŽΡ‡Π΅Π½ΠΈΠ΅ чисСл Π² скобки ΠΈ проставлСниС ΠΌΠΎΠ΄ΡƒΠ»Π΅ΠΉ, ΠΌΠΎΠΆΠ½ΠΎ ΠΏΡ€ΠΎΠΏΡƒΡΡ‚ΠΈΡ‚ΡŒ. Π”Π°Π½Π½Ρ‹ΠΉ ΠΏΡ€ΠΈΠΌΠ΅Ρ€ Π²ΠΏΠΎΠ»Π½Π΅ ΠΌΠΎΠΆΠ½ΠΎ Π·Π°ΠΏΠΈΡΠ°Ρ‚ΡŒ ΠΏΠΎΠΊΠΎΡ€ΠΎΡ‡Π΅:


ΠŸΡ€ΠΈΠΌΠ΅Ρ€ 2. Найти Π·Π½Π°Ρ‡Π΅Π½ΠΈΠ΅ выраТСния:Β 

Π—Π°ΠΊΠ»ΡŽΡ‡ΠΈΠΌΒ ΠΊΠ°ΠΆΠ΄ΠΎΠ΅ Ρ€Π°Ρ†ΠΈΠΎΠ½Π°Π»ΡŒΠ½ΠΎΠ΅ число Π² скобки вмСстС со своими Π·Π½Π°ΠΊΠ°ΠΌΠΈ. Π£Ρ‡ΠΈΡ‚Ρ‹Π²Π°Π΅ΠΌ, Ρ‡Ρ‚ΠΎ минус, стоящий ΠΌΠ΅ΠΆΠ΄Ρƒ Ρ€Π°Ρ†ΠΈΠΎΠ½Π°Π»ΡŒΠ½Ρ‹ΠΌΠΈ числами  и  являСтся Π·Π½Π°ΠΊΠΎΠΌ ΠΎΠΏΠ΅Ρ€Π°Ρ†ΠΈΠΈ ΠΈ Π½Π΅ относится ΠΊ Π΄Ρ€ΠΎΠ±ΠΈ . Π£ этой Π΄Ρ€ΠΎΠ±ΠΈ свой Π·Π½Π°ΠΊ плюса, ΠΊΠΎΡ‚ΠΎΡ€Ρ‹ΠΉ Π½Π΅Π²ΠΈΠ΄ΠΈΠΌ ΠΏΠΎ ΠΏΡ€ΠΈΡ‡ΠΈΠ½Π΅ Ρ‚ΠΎΠ³ΠΎ, Ρ‡Ρ‚ΠΎ Π΅Π³ΠΎ Π½Π΅ Π·Π°ΠΏΠΈΡΡ‹Π²Π°ΡŽΡ‚. Но ΠΌΡ‹ запишСм Π΅Π³ΠΎ для наглядности:

Π—Π°ΠΌΠ΅Π½ΠΈΠΌ Π²Ρ‹Ρ‡ΠΈΡ‚Π°Π½ΠΈΠ΅ слоТСниСм. Напомним, Ρ‡Ρ‚ΠΎ для этого нуТно ΠΊ ΡƒΠΌΠ΅Π½ΡŒΡˆΠ°Π΅ΠΌΠΎΠΌΡƒ ΠΏΡ€ΠΈΠ±Π°Π²ΠΈΡ‚ΡŒ число, ΠΏΡ€ΠΎΡ‚ΠΈΠ²ΠΎΠΏΠΎΠ»ΠΎΠΆΠ½ΠΎΠ΅ Π²Ρ‹Ρ‡ΠΈΡ‚Π°Π΅ΠΌΠΎΠΌΡƒ:

ΠŸΠΎΠ»ΡƒΡ‡ΠΈΠ»ΠΈ слоТСниС ΠΎΡ‚Ρ€ΠΈΡ†Π°Ρ‚Π΅Π»ΡŒΠ½Ρ‹Ρ… Ρ€Π°Ρ†ΠΈΠΎΠ½Π°Π»ΡŒΠ½Ρ‹Ρ… чисСл. Π§Ρ‚ΠΎΠ±Ρ‹ ΡΠ»ΠΎΠΆΠΈΡ‚ΡŒ ΠΎΡ‚Ρ€ΠΈΡ†Π°Ρ‚Π΅Π»ΡŒΠ½Ρ‹Π΅ Ρ€Π°Ρ†ΠΈΠΎΠ½Π°Π»ΡŒΠ½Ρ‹Π΅ числа, Π½ΡƒΠΆΠ½ΠΎ ΡΠ»ΠΎΠΆΠΈΡ‚ΡŒ ΠΈΡ… ΠΌΠΎΠ΄ΡƒΠ»ΠΈ ΠΈ ΠΏΠ΅Ρ€Π΅Π΄ ΠΏΠΎΠ»ΡƒΡ‡Π΅Π½Π½Ρ‹ΠΌ ΠΎΡ‚Π²Π΅Ρ‚ΠΎΠΌ ΠΏΠΎΡΡ‚Π°Π²ΠΈΡ‚ΡŒ минус:

Π—Π°ΠΏΠΈΡˆΠ΅ΠΌ Ρ€Π΅ΡˆΠ΅Π½ΠΈΠ΅ Π΄Π°Π½Π½ΠΎΠ³ΠΎ ΠΏΡ€ΠΈΠΌΠ΅Ρ€Π° ΠΏΠΎΠΊΠΎΡ€ΠΎΡ‡Π΅:

ΠŸΡ€ΠΈΠΌΠ΅Ρ‡Π°Π½ΠΈΠ΅. Π—Π°ΠΊΠ»ΡŽΡ‡Π°Ρ‚ΡŒ Π² скобки ΠΊΠ°ΠΆΠ΄ΠΎΠ΅ Ρ€Π°Ρ†ΠΈΠΎΠ½Π°Π»ΡŒΠ½ΠΎΠ΅ число вовсС Π½Π΅ΠΎΠ±ΡΠ·Π°Ρ‚Π΅Π»ΡŒΠ½ΠΎ. ДСлаСтся это для удобства, Ρ‡Ρ‚ΠΎΠ±Ρ‹ Ρ…ΠΎΡ€ΠΎΡˆΠΎ Π²ΠΈΠ΄Π΅Ρ‚ΡŒ ΠΊΠ°ΠΊΠΈΠ΅ Π·Π½Π°ΠΊΠΈ ΠΈΠΌΠ΅ΡŽΡ‚ Ρ€Π°Ρ†ΠΈΠΎΠ½Π°Π»ΡŒΠ½Ρ‹Π΅ числа.


ΠŸΡ€ΠΈΠΌΠ΅Ρ€ 3. Найти Π·Π½Π°Ρ‡Π΅Π½ΠΈΠ΅ выраТСния:Β 

Π’ этом Π²Ρ‹Ρ€Π°ΠΆΠ΅Π½ΠΈΠΈ Ρƒ Π΄Ρ€ΠΎΠ±Π΅ΠΉ Ρ€Π°Π·Π½Ρ‹Π΅ Π·Π½Π°ΠΌΠ΅Π½Π°Ρ‚Π΅Π»ΠΈ. Π§Ρ‚ΠΎΠ±Ρ‹ ΠΎΠ±Π»Π΅Π³Ρ‡ΠΈΡ‚ΡŒ сСбС Π·Π°Π΄Π°Ρ‡Ρƒ, ΠΏΡ€ΠΈΠ²Π΅Π΄Ρ‘ΠΌ эти Π΄Ρ€ΠΎΠ±ΠΈ ΠΊ ΠΎΠ±Ρ‰Π΅ΠΌΡƒ Π·Π½Π°ΠΌΠ΅Π½Π°Ρ‚Π΅Π»ΡŽ. НС Π±ΡƒΠ΄Π΅ΠΌ ΠΏΠΎΠ΄Ρ€ΠΎΠ±Π½ΠΎ ΠΎΡΡ‚Π°Π½Π°Π²Π»ΠΈΠ²Π°Ρ‚ΡŒΡΡ Π½Π° Ρ‚ΠΎΠΌ ΠΊΠ°ΠΊ это ΡΠ΄Π΅Π»Π°Ρ‚ΡŒ. Если испытываСтС с этим затруднСния, ΠΎΠ±ΡΠ·Π°Ρ‚Π΅Π»ΡŒΠ½ΠΎ ΠΏΠΎΠ²Ρ‚ΠΎΡ€ΠΈΡ‚Π΅ ΡƒΡ€ΠΎΠΊ дСйствия с дробями.

ПослС привСдСния Π΄Ρ€ΠΎΠ±Π΅ΠΉ ΠΊ ΠΎΠ±Ρ‰Π΅ΠΌΡƒ Π·Π½Π°ΠΌΠ΅Π½Π°Ρ‚Π΅Π»ΡŽ Π²Ρ‹Ρ€Π°ΠΆΠ΅Π½ΠΈΠ΅ ΠΏΡ€ΠΈΠΌΠ΅Ρ‚ ΡΠ»Π΅Π΄ΡƒΡŽΡ‰ΠΈΠΉ Π²ΠΈΠ΄:

Π—Π°ΠΊΠ»ΡŽΡ‡ΠΈΠΌ ΠΊΠ°ΠΆΠ΄ΠΎΠ΅ Ρ€Π°Ρ†ΠΈΠΎΠ½Π°Π»ΡŒΠ½ΠΎΠ΅ число Π² скобки вмСстС своими Π·Π½Π°ΠΊΠ°ΠΌΠΈ:

Π­Ρ‚ΠΎ слоТСниС Ρ€Π°Ρ†ΠΈΠΎΠ½Π°Π»ΡŒΠ½Ρ‹Ρ… чисСл с Ρ€Π°Π·Π½Ρ‹ΠΌΠΈ Π·Π½Π°ΠΊΠ°ΠΌΠΈ. Π’Ρ‹Ρ‡ΠΈΡ‚Π°Π΅ΠΌ ΠΈΠ· большСго модуля мСньший ΠΌΠΎΠ΄ΡƒΠ»ΡŒ, ΠΈ ΠΏΠ΅Ρ€Π΅Π΄ ΠΏΠΎΠ»ΡƒΡ‡Π΅Π½Π½Ρ‹ΠΌ ΠΎΡ‚Π²Π΅Ρ‚ΠΎΠΌ ставим Π·Π½Π°ΠΊ Ρ‚ΠΎΠ³ΠΎ Ρ€Π°Ρ†ΠΈΠΎΠ½Π°Π»ΡŒΠ½ΠΎΠ³ΠΎ числа, ΠΌΠΎΠ΄ΡƒΠ»ΡŒ ΠΊΠΎΡ‚ΠΎΡ€ΠΎΠ³ΠΎ большС:

Π—Π°ΠΏΠΈΡˆΠ΅ΠΌ Ρ€Π΅ΡˆΠ΅Π½ΠΈΠ΅ Π΄Π°Π½Π½ΠΎΠ³ΠΎ ΠΏΡ€ΠΈΠΌΠ΅Ρ€Π° ΠΏΠΎΠΊΠΎΡ€ΠΎΡ‡Π΅:


ΠŸΡ€ΠΈΠΌΠ΅Ρ€ 4. Найти Π·Π½Π°Ρ‡Π΅Π½ΠΈΠ΅ выраТСния Β 

Π—Π°ΠΊΠ»ΡŽΡ‡ΠΈΠΌ ΠΊΠ°ΠΆΠ΄ΠΎΠ΅ Ρ€Π°Ρ†ΠΈΠΎΠ½Π°Π»ΡŒΠ½ΠΎΠ΅ число в скобки вмСстС со своими Π·Π½Π°ΠΊΠ°ΠΌΠΈ:

Вычислим Π΄Π°Π½Π½ΠΎΠ΅ Π²Ρ‹Ρ€Π°ΠΆΠ΅Π½ΠΈΠ΅ Π² ΡΠ»Π΅Π΄ΡƒΡŽΡ‰Π΅ΠΌ порядкС: слóТим Ρ€Π°Ρ†ΠΈΠΎΠ½Π°Π»ΡŒΠ½Ρ‹Π΅ числа  и , Π·Π°Ρ‚Π΅ΠΌ ΠΈΠ· ΠΏΠΎΠ»ΡƒΡ‡Π΅Π½Π½ΠΎΠ³ΠΎ Ρ€Π΅Π·ΡƒΠ»ΡŒΡ‚Π°Ρ‚Π° Π²Ρ‹Ρ‡Ρ‚Π΅ΠΌ Ρ€Π°Ρ†ΠΈΠΎΠ½Π°Π»ΡŒΠ½ΠΎΠ΅ число .Β 

ΠŸΠ΅Ρ€Π²ΠΎΠ΅ дСйствиС:

Π’Ρ‚ΠΎΡ€ΠΎΠ΅ дСйствиС:

Π’Π°ΠΊΠΈΠΌ ΠΎΠ±Ρ€Π°Π·ΠΎΠΌ, Π·Π½Π°Ρ‡Π΅Π½ΠΈΠ΅ выраТСния  равно 


ΠŸΡ€ΠΈΠΌΠ΅Ρ€ 5. Найти Π·Π½Π°Ρ‡Π΅Π½ΠΈΠ΅ выраТСния:Β 

ΠŸΡ€Π΅Π΄ΡΡ‚Π°Π²ΠΈΠΌ Ρ†Π΅Π»ΠΎΠ΅ Ρ‡ΠΈΡΠ»ΠΎΒ βˆ’1 Π² Π²ΠΈΠ΄Π΅ Π΄Ρ€ΠΎΠ±ΠΈ , Π° смСшанноС число  пСрСвСдём Π² Π½Π΅ΠΏΡ€Π°Π²ΠΈΠ»ΡŒΠ½ΡƒΡŽ Π΄Ρ€ΠΎΠ±ΡŒ:

ΠŸΡ€ΠΈΠ²Π΅Π΄Ρ‘ΠΌ Π΄Π°Π½Π½Ρ‹Π΅ Π΄Ρ€ΠΎΠ±ΠΈ ΠΊ ΠΎΠ±Ρ‰Π΅ΠΌΡƒ Π·Π½Π°ΠΌΠ΅Π½Π°Ρ‚Π΅Π»ΡŽ. ПослС ΠΈΡ… привСдСния ΠΊ ΠΎΠ±Ρ‰Π΅ΠΌΡƒ Π·Π½Π°ΠΌΠ΅Π½Π°Ρ‚Π΅Π»ΡŽ, ΠΎΠ½ΠΈ ΠΏΡ€ΠΈΠΌΡƒΡ‚ ΡΠ»Π΅Π΄ΡƒΡŽΡ‰ΠΈΠΉ Π²ΠΈΠ΄:

Π—Π°ΠΊΠ»ΡŽΡ‡ΠΈΠΌ ΠΊΠ°ΠΆΠ΄ΠΎΠ΅ Ρ€Π°Ρ†ΠΈΠΎΠ½Π°Π»ΡŒΠ½ΠΎΠ΅ число Π² скобки вмСстС со своими Π·Π½Π°ΠΊΠ°ΠΌΠΈ:

ΠŸΠΎΠ»ΡƒΡ‡ΠΈΠ»ΠΈ слоТСниС Ρ€Π°Ρ†ΠΈΠΎΠ½Π°Π»ΡŒΠ½Ρ‹Ρ… чисСл с Ρ€Π°Π·Π½Ρ‹ΠΌΠΈ Π·Π½Π°ΠΊΠ°ΠΌΠΈ. Π’Ρ‹Ρ‡ΠΈΡ‚Π°Π΅ΠΌ ΠΈΠ· большСго модуля мСньший ΠΌΠΎΠ΄ΡƒΠ»ΡŒ, ΠΈ ΠΏΠ΅Ρ€Π΅Π΄ ΠΏΠΎΠ»ΡƒΡ‡Π΅Π½Π½Ρ‹ΠΌ ΠΎΡ‚Π²Π΅Ρ‚ΠΎΠΌ ставим Π·Π½Π°ΠΊ Ρ‚ΠΎΠ³ΠΎ Ρ€Π°Ρ†ΠΈΠΎΠ½Π°Π»ΡŒΠ½ΠΎΠ³ΠΎ числа, ΠΌΠΎΠ΄ΡƒΠ»ΡŒ ΠΊΠΎΡ‚ΠΎΡ€ΠΎΠ³ΠΎ большС:

ΠŸΠΎΠ»ΡƒΡ‡ΠΈΠ»ΠΈ ΠΎΡ‚Π²Π΅Ρ‚Β .

Π•ΡΡ‚ΡŒ ΠΈ Π²Ρ‚ΠΎΡ€ΠΎΠΉ способ Ρ€Π΅ΡˆΠ΅Π½ΠΈΡ. Он Π·Π°ΠΊΠ»ΡŽΡ‡Π°Π΅Ρ‚ΡΡ Π² Ρ‚ΠΎΠΌ, Ρ‡Ρ‚ΠΎΠ±Ρ‹ ΡΠ»ΠΎΠΆΠΈΡ‚ΡŒ ΠΎΡ‚Π΄Π΅Π»ΡŒΠ½ΠΎ Ρ†Π΅Π»Ρ‹Π΅ части.

Π˜Ρ‚Π°ΠΊ, вСрнёмся ΠΊ ΠΈΠ·Π½Π°Ρ‡Π°Π»ΡŒΠ½ΠΎΠΌΡƒ Π²Ρ‹Ρ€Π°ΠΆΠ΅Π½ΠΈΡŽ:

Π—Π°ΠΊΠ»ΡŽΡ‡ΠΈΠΌ ΠΊΠ°ΠΆΠ΄ΠΎΠ΅ число Π² скобки. Для этого смСшанноС число  Π²Ρ€Π΅ΠΌΠ΅Π½Π½ΠΎ Ρ€Π°Π·Π²Π΅Ρ€Π½Ρ‘ΠΌ:

Вычислим Ρ†Π΅Π»Ρ‹Π΅ части:

(βˆ’1)Β +Β (+2)Β =Β 1

Π’ Π³Π»Π°Π²Π½ΠΎΠΌ Π²Ρ‹Ρ€Π°ΠΆΠ΅Π½ΠΈΠΈ вмСсто (βˆ’1)Β +Β (+2) запишСм ΠΏΠΎΠ»ΡƒΡ‡Π΅Π½Π½ΡƒΡŽ Π΅Π΄ΠΈΠ½ΠΈΡ†Ρƒ:

ΠŸΠΎΠ»ΡƒΡ‡Π΅Π½Π½ΠΎΠ΅ выраТСниС  свСрнём. Для этого запишСм Π΅Π΄ΠΈΠ½ΠΈΡ†Ρƒ ΠΈ Π΄Ρ€ΠΎΠ±ΡŒ вмСстС:

Π—Π°ΠΏΠΈΡˆΠ΅ΠΌ Ρ€Π΅ΡˆΠ΅Π½ΠΈΠ΅ этим способом ΠΏΠΎΠΊΠΎΡ€ΠΎΡ‡Π΅:


ΠŸΡ€ΠΈΠΌΠ΅Ρ€ 6. Найти Π·Π½Π°Ρ‡Π΅Π½ΠΈΠ΅ выраТСния

ΠŸΠ΅Ρ€Π΅Π²Π΅Π΄Ρ‘ΠΌ смСшанноС число  в Π½Π΅ΠΏΡ€Π°Π²ΠΈΠ»ΡŒΠ½ΡƒΡŽ Π΄Ρ€ΠΎΠ±ΡŒ. ΠžΡΡ‚Π°Π»ΡŒΠ½ΡƒΡŽ Ρ‡Π°ΡΡ‚ΡŒ ΠΏΠ΅Ρ€Π΅ΠΏΠΈΡˆΠ΅ΠΌ Π±Π΅Π· измСнСния:

Π—Π°ΠΊΠ»ΡŽΡ‡ΠΈΠΌ ΠΊΠ°ΠΆΠ΄ΠΎΠ΅ Ρ€Π°Ρ†ΠΈΠΎΠ½Π°Π»ΡŒΠ½ΠΎΠ΅ число Π² скобки вмСстС со своими Π·Π½Π°ΠΊΠ°ΠΌΠΈ:

Π—Π°ΠΌΠ΅Π½ΠΈΠΌ Π²Ρ‹Ρ‡ΠΈΡ‚Π°Π½ΠΈΠ΅ слоТСниСм:

ΠŸΠΎΠ»ΡƒΡ‡ΠΈΠ»ΠΈ слоТСниС ΠΎΡ‚Ρ€ΠΈΡ†Π°Ρ‚Π΅Π»ΡŒΠ½Ρ‹Ρ… Ρ€Π°Ρ†ΠΈΠΎΠ½Π°Π»ΡŒΠ½Ρ‹Ρ… чисСл. Π‘Π»Γ³ΠΆΠΈΠΌ ΠΌΠΎΠ΄ΡƒΠ»ΠΈ этих чисСл ΠΈ ΠΏΠ΅Ρ€Π΅Π΄ ΠΏΠΎΠ»ΡƒΡ‡Π΅Π½Π½Ρ‹ΠΌ ΠΎΡ‚Π²Π΅Ρ‚ΠΎΠΌ поставим минус:

Π—Π°ΠΏΠΈΡˆΠ΅ΠΌ Ρ€Π΅ΡˆΠ΅Π½ΠΈΠ΅ Π΄Π°Π½Π½ΠΎΠ³ΠΎ ΠΏΡ€ΠΈΠΌΠ΅Ρ€Π° ΠΏΠΎΠΊΠΎΡ€ΠΎΡ‡Π΅:


ΠŸΡ€ΠΈΠΌΠ΅Ρ€ 7. Найти Π·Π½Π°Ρ‡Π΅Π½ΠΈΠ΅ Π²Ρ‹Ρ€Π°ΠΆΠ΅Π½ΠΈΠ΅

ΠŸΡ€Π΅Π΄ΡΡ‚Π°Π²ΠΈΠΌ Ρ†Π΅Π»ΠΎΠ΅ Ρ‡ΠΈΡΠ»ΠΎΒ βˆ’5 Π² Π²ΠΈΠ΄Π΅ Π΄Ρ€ΠΎΠ±ΠΈΒ , Π° смСшанноС число  пСрСвСдём Π² Π½Π΅ΠΏΡ€Π°Π²ΠΈΠ»ΡŒΠ½ΡƒΡŽ Π΄Ρ€ΠΎΠ±ΡŒ:

ΠŸΡ€ΠΈΠ²Π΅Π΄Ρ‘ΠΌ Π΄Π°Π½Π½Ρ‹Π΅ Π΄Ρ€ΠΎΠ±ΠΈ ΠΊ ΠΎΠ±Ρ‰Π΅ΠΌΡƒ Π·Π½Π°ΠΌΠ΅Π½Π°Ρ‚Π΅Π»ΡŽ. ПослС ΠΈΡ… привСдСния ΠΊ ΠΎΠ±Ρ‰Π΅ΠΌΡƒ Π·Π½Π°ΠΌΠ΅Π½Π°Ρ‚Π΅Π»ΡŽ, ΠΎΠ½ΠΈ ΠΏΡ€ΠΈΠΌΡƒΡ‚ ΡΠ»Π΅Π΄ΡƒΡŽΡ‰ΠΈΠΉ Π²ΠΈΠ΄:

Π—Π°ΠΊΠ»ΡŽΡ‡ΠΈΠΌ ΠΊΠ°ΠΆΠ΄ΠΎΠ΅ Ρ€Π°Ρ†ΠΈΠΎΠ½Π°Π»ΡŒΠ½ΠΎΠ΅ число Π² скобки вмСстС со своими Π·Π½Π°ΠΊΠ°ΠΌΠΈ:

Π—Π°ΠΌΠ΅Π½ΠΈΠΌ Π²Ρ‹Ρ‡ΠΈΡ‚Π°Π½ΠΈΠ΅ слоТСниСм:

ΠŸΠΎΠ»ΡƒΡ‡ΠΈΠ»ΠΈ слоТСниС ΠΎΡ‚Ρ€ΠΈΡ†Π°Ρ‚Π΅Π»ΡŒΠ½Ρ‹Ρ… Ρ€Π°Ρ†ΠΈΠΎΠ½Π°Π»ΡŒΠ½Ρ‹Ρ… чисСл. Π‘Π»Γ³ΠΆΠΈΠΌ ΠΌΠΎΠ΄ΡƒΠ»ΠΈ этих чисСл ΠΈ ΠΏΠ΅Ρ€Π΅Π΄ ΠΏΠΎΠ»ΡƒΡ‡Π΅Π½Π½Ρ‹ΠΌ ΠΎΡ‚Π²Π΅Ρ‚ΠΎΠΌ поставим минус:

Π’Π°ΠΊΠΈΠΌ ΠΎΠ±Ρ€Π°Π·ΠΎΠΌ, Π·Π½Π°Ρ‡Π΅Π½ΠΈΠ΅ выраТСния Β  Ρ€Π°Π²Π½ΠΎ .

РСшим Π΄Π°Π½Π½Ρ‹ΠΉ ΠΏΡ€ΠΈΠΌΠ΅Ρ€ Π²Ρ‚ΠΎΡ€Ρ‹ΠΌ способом. ВСрнСмся ΠΊ ΠΈΠ·Π½Π°Ρ‡Π°Π»ΡŒΠ½ΠΎΠΌΡƒ Π²Ρ‹Ρ€Π°ΠΆΠ΅Π½ΠΈΡŽ:

Π—Π°ΠΏΠΈΡˆΠ΅ΠΌ смСшанноС число Π² Ρ€Π°Π·Π²Ρ‘Ρ€Π½ΡƒΡ‚ΠΎΠΌ Π²ΠΈΠ΄Π΅. ΠžΡΡ‚Π°Π»ΡŒΠ½ΠΎΠ΅ ΠΏΠ΅Ρ€Π΅ΠΏΠΈΡˆΠ΅ΠΌ Π±Π΅Π· ΠΈΠ·ΠΌΠ΅Π½Π΅Π½ΠΈΠΉ:

Π—Π°ΠΊΠ»ΡŽΡ‡ΠΈΠΌΒ ΠΊΠ°ΠΆΠ΄ΠΎΠ΅ Ρ€Π°Ρ†ΠΈΠΎΠ½Π°Π»ΡŒΠ½ΠΎΠ΅ число Π² скобки вмСстС своими Π·Π½Π°ΠΊΠ°ΠΌΠΈ:

Π—Π°ΠΌΠ΅Π½ΠΈΠΌ Π²Ρ‹Ρ‡ΠΈΡ‚Π°Π½ΠΈΠ΅ слоТСниСм Ρ‚Π°ΠΌ, Π³Π΄Π΅ это ΠΌΠΎΠΆΠ½ΠΎ:

Вычислим Ρ†Π΅Π»Ρ‹Π΅ части:

Π’ Π³Π»Π°Π²Π½ΠΎΠΌ Π²Ρ‹Ρ€Π°ΠΆΠ΅Π½ΠΈΠΈ вмСсто запишСм ΠΏΠΎΠ»ΡƒΡ‡Π΅Π½Π½ΠΎΠ΅ число βˆ’7

Π’Ρ‹Ρ€Π°ΠΆΠ΅Π½ΠΈΠ΅ Β  являСтся Ρ€Π°Π·Π²Ρ‘Ρ€Π½ΡƒΡ‚ΠΎΠΉ Ρ„ΠΎΡ€ΠΌΠΎΠΉ записи смСшанного числа .Β  Π—Π°ΠΏΠΈΡˆΠ΅ΠΌ число βˆ’7 ΠΈ Π΄Ρ€ΠΎΠ±ΡŒ  вмСстС, образуя ΠΎΠΊΠΎΠ½Ρ‡Π°Ρ‚Π΅Π»ΡŒΠ½Ρ‹ΠΉ ΠΎΡ‚Π²Π΅Ρ‚:

Π—Π°ΠΏΠΈΡˆΠ΅ΠΌ это Ρ€Π΅ΡˆΠ΅Π½ΠΈΠ΅ ΠΏΠΎΠΊΠΎΡ€ΠΎΡ‡Π΅:


ΠŸΡ€ΠΈΠΌΠ΅Ρ€ 8. Найти Π·Π½Π°Ρ‡Π΅Π½ΠΈΠ΅ выраТСния

ΠŸΠ΅Ρ€Π΅Π²Π΅Π΄Ρ‘ΠΌ ΡΠΌΠ΅ΡˆΠ°Π½Π½Ρ‹Π΅ числа Π² Π½Π΅ΠΏΡ€Π°Π²ΠΈΠ»ΡŒΠ½Ρ‹Π΅ Π΄Ρ€ΠΎΠ±ΠΈ:

Π—Π°ΠΊΠ»ΡŽΡ‡ΠΈΠΌΒ ΠΊΠ°ΠΆΠ΄ΠΎΠ΅ Ρ€Π°Ρ†ΠΈΠΎΠ½Π°Π»ΡŒΠ½ΠΎΠ΅ число Π² скобки вмСстС своими Π·Π½Π°ΠΊΠ°ΠΌΠΈ:

Π—Π°ΠΌΠ΅Π½ΠΈΠΌ Π²Ρ‹Ρ‡ΠΈΡ‚Π°Π½ΠΈΠ΅ слоТСниСм:

ΠŸΠΎΠ»ΡƒΡ‡ΠΈΠ»ΠΈ слоТСниС ΠΎΡ‚Ρ€ΠΈΡ†Π°Ρ‚Π΅Π»ΡŒΠ½Ρ‹Ρ… Ρ€Π°Ρ†ΠΈΠΎΠ½Π°Π»ΡŒΠ½Ρ‹Ρ… чисСл. Π‘Π»Γ³ΠΆΠΈΠΌ ΠΌΠΎΠ΄ΡƒΠ»ΠΈ этих чисСл ΠΈ ΠΏΠ΅Ρ€Π΅Π΄ ΠΏΠΎΠ»ΡƒΡ‡Π΅Π½Π½Ρ‹ΠΌ ΠΎΡ‚Π²Π΅Ρ‚ΠΎΠΌ поставим минус:

Π’Π°ΠΊΠΈΠΌ ΠΎΠ±Ρ€Π°Π·ΠΎΠΌ, Π·Π½Π°Ρ‡Π΅Π½ΠΈΠ΅ выраТСния Β  Ρ€Π°Π²Π½ΠΎΒ 

Π”Π°Π½Π½Ρ‹ΠΉ ΠΏΡ€ΠΈΠΌΠ΅Ρ€ ΠΌΠΎΠΆΠ½ΠΎ Ρ€Π΅ΡˆΠΈΡ‚ΡŒ ΠΈ Π²Ρ‚ΠΎΡ€Ρ‹ΠΌ способом. Он Π·Π°ΠΊΠ»ΡŽΡ‡Π°Π΅Ρ‚ΡΡ Π² Ρ‚ΠΎΠΌ, Ρ‡Ρ‚ΠΎΠ±Ρ‹ ΡΠ»ΠΎΠΆΠΈΡ‚ΡŒ Ρ†Π΅Π»Ρ‹Π΅ ΠΈ Π΄Ρ€ΠΎΠ±Π½Ρ‹Π΅ части ΠΏΠΎ ΠΎΡ‚Π΄Π΅Π»ΡŒΠ½ΠΎΡΡ‚ΠΈ. ВСрнёмся ΠΊ ΠΈΠ·Π½Π°Ρ‡Π°Π»ΡŒΠ½ΠΎΠΌΡƒ Π²Ρ‹Ρ€Π°ΠΆΠ΅Π½ΠΈΡŽ:

Π—Π°ΠΊΠ»ΡŽΡ‡ΠΈΠΌ ΠΊΠ°ΠΆΠ΄ΠΎΠ΅ Ρ€Π°Ρ†ΠΈΠΎΠ½Π°Π»ΡŒΠ½ΠΎΠ΅ число Π² скобки вмСстС со своими Π·Π½Π°ΠΊΠ°ΠΌΠΈ:

Π—Π°ΠΌΠ΅Π½ΠΈΠΌ Π²Ρ‹Ρ‡ΠΈΡ‚Π°Π½ΠΈΠ΅ слоТСниСм:

ΠŸΠΎΠ»ΡƒΡ‡ΠΈΠ»ΠΈ слоТСниС ΠΎΡ‚Ρ€ΠΈΡ†Π°Ρ‚Π΅Π»ΡŒΠ½Ρ‹Ρ… Ρ€Π°Ρ†ΠΈΠΎΠ½Π°Π»ΡŒΠ½Ρ‹Ρ… чисСл. Π‘Π»Γ³ΠΆΠΈΠΌ ΠΌΠΎΠ΄ΡƒΠ»ΠΈ этих чисСл ΠΈ ΠΏΠ΅Ρ€Π΅Π΄ ΠΏΠΎΠ»ΡƒΡ‡Π΅Π½Π½Ρ‹ΠΌ ΠΎΡ‚Π²Π΅Ρ‚ΠΎΠΌ поставим минус. Но Π² этот Ρ€Π°Π· слóТим ΠΏΠΎ ΠΎΡ‚Π΄Π΅Π»ΡŒΠ½ΠΎΡΡ‚ΠΈ Ρ†Π΅Π»Ρ‹Π΅ части (βˆ’1 ΠΈ βˆ’2), ΠΈ Π΄Ρ€ΠΎΠ±Π½Ρ‹Π΅Β Β ΠΈΒ 

Π—Π°ΠΏΠΈΡˆΠ΅ΠΌ это Ρ€Π΅ΡˆΠ΅Π½ΠΈΠ΅ ΠΏΠΎΠΊΠΎΡ€ΠΎΡ‡Π΅:


ΠŸΡ€ΠΈΠΌΠ΅Ρ€ 9. Найти выраТСния выраТСния

ΠŸΠ΅Ρ€Π΅Π²Π΅Π΄Ρ‘ΠΌ ΡΠΌΠ΅ΡˆΠ°Π½Π½Ρ‹Π΅ числа Π² Π½Π΅ΠΏΡ€Π°Π²ΠΈΠ»ΡŒΠ½Ρ‹Π΅ Π΄Ρ€ΠΎΠ±ΠΈ:

Π—Π°ΠΊΠ»ΡŽΡ‡ΠΈΠΌ Ρ€Π°Ρ†ΠΈΠΎΠ½Π°Π»ΡŒΠ½ΠΎΠ΅ число  в скобки вмСстС своим Π·Π½Π°ΠΊΠΎΠΌ. Π Π°Ρ†ΠΈΠΎΠ½Π°Π»ΡŒΠ½ΠΎΠ΅ число  в скобки Π·Π°ΠΊΠ»ΡŽΡ‡Π°Ρ‚ΡŒ Π½Π΅ Π½ΡƒΠΆΠ½ΠΎ, ΠΏΠΎΡΠΊΠΎΠ»ΡŒΠΊΡƒ ΠΎΠ½ΠΎ ΡƒΠΆΠ΅ Π² скобках:

ΠŸΡ€ΠΈΠ²Π΅Π΄Ρ‘ΠΌ Π΄Π°Π½Π½Ρ‹Π΅ Π΄Ρ€ΠΎΠ±ΠΈ Π² ΠΎΠ±Ρ‰Π΅ΠΌΡƒ Π·Π½Π°ΠΌΠ΅Π½Π°Ρ‚Π΅Π»ΡŽ. ПослС ΠΈΡ… привСдСния ΠΊ ΠΎΠ±Ρ‰Π΅ΠΌΡƒ Π·Π½Π°ΠΌΠ΅Π½Π°Ρ‚Π΅Π»ΡŽ, ΠΎΠ½ΠΈ ΠΏΡ€ΠΈΠΌΡƒΡ‚ ΡΠ»Π΅Π΄ΡƒΡŽΡ‰ΠΈΠΉ Π²ΠΈΠ΄:

ΠŸΠΎΠ»ΡƒΡ‡ΠΈΠ»ΠΈ слоТСниС ΠΎΡ‚Ρ€ΠΈΡ†Π°Ρ‚Π΅Π»ΡŒΠ½Ρ‹Ρ… Ρ€Π°Ρ†ΠΈΠΎΠ½Π°Π»ΡŒΠ½Ρ‹Ρ… чисСл. Π‘Π»Γ³ΠΆΠΈΠΌ ΠΌΠΎΠ΄ΡƒΠ»ΠΈ этих чисСл ΠΈ ΠΏΠ΅Ρ€Π΅Π΄ ΠΏΠΎΠ»ΡƒΡ‡Π΅Π½Π½Ρ‹ΠΌ ΠΎΡ‚Π²Π΅Ρ‚ΠΎΠΌ поставим минус:

Π’Π°ΠΊΠΈΠΌ ΠΎΠ±Ρ€Π°Π·ΠΎΠΌ, Π·Π½Π°Ρ‡Π΅Π½ΠΈΠ΅ выраТСния  равно 

Π’Π΅ΠΏΠ΅Ρ€ΡŒ ΠΏΠΎΠΏΡ€ΠΎΠ±ΡƒΠ΅ΠΌ Ρ€Π΅ΡˆΠΈΡ‚ΡŒ этот ΠΆΠ΅ ΠΏΡ€ΠΈΠΌΠ΅Ρ€ Π²Ρ‚ΠΎΡ€Ρ‹ΠΌ способом, Π° ΠΈΠΌΠ΅Π½Π½ΠΎ слоТСниСм Ρ†Π΅Π»Ρ‹Ρ… ΠΈ Π΄Ρ€ΠΎΠ±Π½Ρ‹Ρ… частСй ΠΏΠΎ ΠΎΡ‚Π΄Π΅Π»ΡŒΠ½ΠΎΡΡ‚ΠΈ.

Π’ этот Ρ€Π°Π·, Π² цСлях получСния ΠΊΠΎΡ€ΠΎΡ‚ΠΊΠΎΠ³ΠΎ Ρ€Π΅ΡˆΠ΅Π½ΠΈΡ, ΠΏΠΎΠΏΡ€ΠΎΠ±ΡƒΠ΅ΠΌ ΠΏΡ€ΠΎΠΏΡƒΡΡ‚ΠΈΡ‚ΡŒ Π½Π΅ΠΊΠΎΡ‚ΠΎΡ€Ρ‹Π΅ дСйствия, Ρ‚Π°ΠΊΠΈΠ΅ ΠΊΠ°ΠΊ: запись смСшанного числа Π² Ρ€Π°Π·Π²Ρ‘Ρ€Π½ΡƒΡ‚ΠΎΠΌ Π²ΠΈΠ΄Π΅ ΠΈ Π·Π°ΠΌΠ΅Π½Π° вычитания слоТСниСм:

ΠžΠ±Ρ€Π°Ρ‚ΠΈΡ‚Π΅ Π²Π½ΠΈΠΌΠ°Π½ΠΈΠ΅, Ρ‡Ρ‚ΠΎ Π΄Ρ€ΠΎΠ±Π½Ρ‹Π΅ части Π±Ρ‹Π»ΠΈ ΠΏΡ€ΠΈΠ²Π΅Π΄Π΅Π½Ρ‹ ΠΊ ΠΎΠ±Ρ‰Π΅ΠΌΡƒ Π·Π½Π°ΠΌΠ΅Π½Π°Ρ‚Π΅Π»ΡŽ.


ΠŸΡ€ΠΈΠΌΠ΅Ρ€ 10. Найти Π·Π½Π°Ρ‡Π΅Π½ΠΈΠ΅ выраТСния

Π—Π°ΠΌΠ΅Π½ΠΈΠΌ Π²Ρ‹Ρ‡ΠΈΡ‚Π°Π½ΠΈΠ΅ слоТСниСм:

Π’ ΠΏΠΎΠ»ΡƒΡ‡ΠΈΠ²ΡˆΠ΅ΠΌΡΡ Π²Ρ‹Ρ€Π°ΠΆΠ΅Π½ΠΈΠΈ Π½Π΅Ρ‚ ΠΎΡ‚Ρ€ΠΈΡ†Π°Ρ‚Π΅Π»ΡŒΠ½Ρ‹Ρ… чисСл, ΠΊΠΎΡ‚ΠΎΡ€Ρ‹Π΅ ΡΠ²Π»ΡΡŽΡ‚ΡΡ основной ΠΏΡ€ΠΈΡ‡ΠΈΠ½ΠΎΠΉ допущСния ошибок. А ΠΏΠΎΡΠΊΠΎΠ»ΡŒΠΊΡƒ Π½Π΅Ρ‚ ΠΎΡ‚Ρ€ΠΈΡ†Π°Ρ‚Π΅Π»ΡŒΠ½Ρ‹Ρ… чисСл, ΠΌΡ‹ ΠΌΠΎΠΆΠ΅ΠΌ ΡƒΠ±Ρ€Π°Ρ‚ΡŒ плюс ΠΏΠ΅Ρ€Π΅Π΄ Π²Ρ‹Ρ‡ΠΈΡ‚Π°Π΅ΠΌΡ‹ΠΌ, Π° Ρ‚Π°ΠΊΠΆΠ΅ ΡƒΠ±Ρ€Π°Ρ‚ΡŒ скобки:

ΠŸΠΎΠ»ΡƒΡ‡ΠΈΠ»ΠΎΡΡŒ ΠΏΡ€ΠΎΡΡ‚Π΅ΠΉΡˆΠ΅Π΅ Π²Ρ‹Ρ€Π°ΠΆΠ΅Π½ΠΈΠ΅, ΠΊΠΎΡ‚ΠΎΡ€ΠΎΠ΅ вычисляСтся Π»Π΅Π³ΠΊΠΎ. Вычислим Π΅Π³ΠΎ Π»ΡŽΠ±Ρ‹ΠΌ ΡƒΠ΄ΠΎΠ±Π½Ρ‹ΠΌ для нас способом:


ΠŸΡ€ΠΈΠΌΠ΅Ρ€ 11. Найти Π·Π½Π°Ρ‡Π΅Π½ΠΈΠ΅ выраТСния

Π­Ρ‚ΠΎ слоТСниС Ρ€Π°Ρ†ΠΈΠΎΠ½Π°Π»ΡŒΠ½Ρ‹Ρ… чисСл с Ρ€Π°Π·Π½Ρ‹ΠΌΠΈ Π·Π½Π°ΠΊΠ°ΠΌΠΈ. Π’Ρ‹Ρ‡Ρ‚Π΅ΠΌ ΠΈΠ· большСго модуля мСньший ΠΌΠΎΠ΄ΡƒΠ»ΡŒ, ΠΈ ΠΏΠ΅Ρ€Π΅Π΄ ΠΏΠΎΠ»ΡƒΡ‡Π΅Π½Π½Ρ‹ΠΌΠΈ ΠΎΡ‚Π²Π΅Ρ‚ΠΎΠΌ поставим Π·Π½Π°ΠΊ Ρ‚ΠΎΠ³ΠΎ Ρ€Π°Ρ†ΠΈΠΎΠ½Π°Π»ΡŒΠ½ΠΎΠ³ΠΎ числа, ΠΌΠΎΠ΄ΡƒΠ»ΡŒ ΠΊΠΎΡ‚ΠΎΡ€ΠΎΠ³ΠΎ большС:


ΠŸΡ€ΠΈΠΌΠ΅Ρ€ 12. Найти Π·Π½Π°Ρ‡Π΅Π½ΠΈΠ΅ выраТСния

Π’Ρ‹Ρ€Π°ΠΆΠ΅Π½ΠΈΠ΅ состоит ΠΈΠ· Π½Π΅ΡΠΊΠΎΠ»ΡŒΠΊΠΈΡ… Ρ€Π°Ρ†ΠΈΠΎΠ½Π°Π»ΡŒΠ½Ρ‹Ρ… чисСл. Богласно порядку дСйствий, Π² ΠΏΠ΅Ρ€Π²ΡƒΡŽ ΠΎΡ‡Π΅Ρ€Π΅Π΄ΡŒ Π½Π΅ΠΎΠ±Ρ…ΠΎΠ΄ΠΈΠΌΠΎ Π²Ρ‹ΠΏΠΎΠ»Π½ΠΈΡ‚ΡŒ дСйствия Π² скобках.

Π‘Π½Π°Ρ‡Π°Π»Π° вычислим Π²Ρ‹Ρ€Π°ΠΆΠ΅Π½ΠΈΠ΅ , Π·Π°Ρ‚Π΅ΠΌ Π²Ρ‹Ρ€Π°ΠΆΠ΅Π½ΠΈΠ΅ ΠŸΠΎΠ»ΡƒΡ‡Π΅Π½Π½Ρ‹Π΅ Ρ€Π΅Π·ΡƒΠ»ΡŒΡ‚Π°Ρ‚Ρ‹ слóТим .

ΠŸΠ΅Ρ€Π²ΠΎΠ΅ дСйствиС:

Π’Ρ‚ΠΎΡ€ΠΎΠ΅ дСйствиС:

Π’Ρ€Π΅Ρ‚ΡŒΠ΅ дСйствиС:

ΠžΡ‚Π²Π΅Ρ‚: Π·Π½Π°Ρ‡Π΅Π½ΠΈΠ΅ выраТСния Β Ρ€Π°Π²Π½ΠΎΒ 


ΠŸΡ€ΠΈΠΌΠ΅Ρ€ 13. Найти Π·Π½Π°Ρ‡Π΅Π½ΠΈΠ΅ выраТСния

ΠŸΠ΅Ρ€Π΅Π²Π΅Π΄Ρ‘ΠΌ ΡΠΌΠ΅ΡˆΠ°Π½Π½Ρ‹Π΅ числа Π² Π½Π΅ΠΏΡ€Π°Π²ΠΈΠ»ΡŒΠ½Ρ‹Π΅ Π΄Ρ€ΠΎΠ±ΠΈ:

Π—Π°ΠΊΠ»ΡŽΡ‡ΠΈΠΌ Ρ€Π°Ρ†ΠΈΠΎΠ½Π°Π»ΡŒΠ½ΠΎΠ΅ число  Π² скобки вмСстС со своим Π·Π½Π°ΠΊΠΎΠΌ. Π Π°Ρ†ΠΈΠΎΠ½Π°Π»ΡŒΠ½ΠΎΠ΅ Ρ‡ΠΈΡΠ»ΠΎΒ Β Π·Π°ΠΊΠ»ΡŽΡ‡Π°Ρ‚ΡŒ Π² скобки Π½Π΅ Π½ΡƒΠΆΠ½ΠΎ, ΠΏΠΎΡΠΊΠΎΠ»ΡŒΠΊΡƒ ΠΎΠ½ΠΎ ΡƒΠΆΠ΅ Π² скобках:

ΠŸΡ€ΠΈΠ²Π΅Π΄Ρ‘ΠΌ Π΄Π°Π½Π½Ρ‹Π΅ Π΄Ρ€ΠΎΠ±ΠΈ Π² ΠΎΠ±Ρ‰Π΅ΠΌΡƒ Π·Π½Π°ΠΌΠ΅Π½Π°Ρ‚Π΅Π»ΡŽ. ПослС ΠΈΡ… привСдСния ΠΊ ΠΎΠ±Ρ‰Π΅ΠΌΡƒ Π·Π½Π°ΠΌΠ΅Π½Π°Ρ‚Π΅Π»ΡŽ, ΠΎΠ½ΠΈ ΠΏΡ€ΠΈΠΌΡƒΡ‚ ΡΠ»Π΅Π΄ΡƒΡŽΡ‰ΠΈΠΉ Π²ΠΈΠ΄:

Π—Π°ΠΌΠ΅Π½ΠΈΠΌ Π²Ρ‹Ρ‡ΠΈΡ‚Π°Π½ΠΈΠ΅ слоТСниСм:

ΠŸΠΎΠ»ΡƒΡ‡ΠΈΠ»ΠΈ слоТСниС Ρ€Π°Ρ†ΠΈΠΎΠ½Π°Π»ΡŒΠ½Ρ‹Ρ… чисСл с Ρ€Π°Π·Π½Ρ‹ΠΌΠΈ Π·Π½Π°ΠΊΠ°ΠΌΠΈ. Π’Ρ‹Ρ‡Ρ‚Π΅ΠΌ ΠΈΠ· большСго модуля мСньший ΠΌΠΎΠ΄ΡƒΠ»ΡŒ, ΠΈ ΠΏΠ΅Ρ€Π΅Π΄ ΠΏΠΎΠ»ΡƒΡ‡Π΅Π½Π½Ρ‹ΠΌΠΈ ΠΎΡ‚Π²Π΅Ρ‚ΠΎΠΌ поставим Π·Π½Π°ΠΊ Ρ‚ΠΎΠ³ΠΎ Ρ€Π°Ρ†ΠΈΠΎΠ½Π°Π»ΡŒΠ½ΠΎΠ³ΠΎ числа, ΠΌΠΎΠ΄ΡƒΠ»ΡŒ ΠΊΠΎΡ‚ΠΎΡ€ΠΎΠ³ΠΎ большС:

Π’Π°ΠΊΠΈΠΌ ΠΎΠ±Ρ€Π°Π·ΠΎΠΌ, Π·Π½Π°Ρ‡Π΅Π½ΠΈΠ΅ выраТСния Ρ€Π°Π²Π½ΠΎ


Рассмотрим слоТСниС ΠΈ Π²Ρ‹Ρ‡ΠΈΡ‚Π°Π½ΠΈΠ΅ дСсятичных Π΄Ρ€ΠΎΠ±Π΅ΠΉ, ΠΊΠΎΡ‚ΠΎΡ€Ρ‹Π΅ Ρ‚ΠΎΠΆΠ΅ относятся ΠΊ Ρ€Π°Ρ†ΠΈΠΎΠ½Π°Π»ΡŒΠ½Ρ‹ΠΌ числам ΠΈ ΠΊΠΎΡ‚ΠΎΡ€Ρ‹Π΅ ΠΌΠΎΠ³ΡƒΡ‚ Π±Ρ‹Ρ‚ΡŒ ΠΊΠ°ΠΊ ΠΏΠΎΠ»ΠΎΠΆΠΈΡ‚Π΅Π»ΡŒΠ½Ρ‹ΠΌΠΈ, Ρ‚Π°ΠΊ ΠΈ ΠΎΡ‚Ρ€ΠΈΡ†Π°Ρ‚Π΅Π»ΡŒΠ½Ρ‹ΠΌΠΈ.

ΠŸΡ€ΠΈΠΌΠ΅Ρ€ 14. Найти Π·Π½Π°Ρ‡Π΅Π½ΠΈΠ΅ выраТСния βˆ’3,2 + 4,3

Π—Π°ΠΊΠ»ΡŽΡ‡ΠΈΠΌΒ ΠΊΠ°ΠΆΠ΄ΠΎΠ΅ Ρ€Π°Ρ†ΠΈΠΎΠ½Π°Π»ΡŒΠ½ΠΎΠ΅ число Π² скобки вмСстС со своими Π·Π½Π°ΠΊΠ°ΠΌΠΈ. Π£Ρ‡ΠΈΡ‚Ρ‹Π²Π°Π΅ΠΌ, Ρ‡Ρ‚ΠΎ плюс ΠΊΠΎΡ‚ΠΎΡ€Ρ‹ΠΉ Π΄Π°Π½ Π² Π²Ρ‹Ρ€Π°ΠΆΠ΅Π½ΠΈΠΈ, являСтся Π·Π½Π°ΠΊΠΎΠΌ ΠΎΠΏΠ΅Ρ€Π°Ρ†ΠΈΠΈ ΠΈ Π½Π΅ относится ΠΊ дСсятичной Π΄Ρ€ΠΎΠ±ΠΈ 4,3. Π£ этой дСсятичной Π΄Ρ€ΠΎΠ±ΠΈ свой Π·Π½Π°ΠΊ плюса, ΠΊΠΎΡ‚ΠΎΡ€Ρ‹ΠΉ Π½Π΅Π²ΠΈΠ΄ΠΈΠΌ ΠΏΠΎ ΠΏΡ€ΠΈΡ‡ΠΈΠ½Π΅ Ρ‚ΠΎΠ³ΠΎ, Ρ‡Ρ‚ΠΎ Π΅Π³ΠΎ Π½Π΅ Π·Π°ΠΏΠΈΡΡ‹Π²Π°ΡŽΡ‚. Но ΠΌΡ‹Β Π΅Π³ΠΎ запишСм для наглядности:

(βˆ’3,2) + (+4,3)

Π­Ρ‚ΠΎ слоТСниС Ρ€Π°Ρ†ΠΈΠΎΠ½Π°Π»ΡŒΠ½Ρ‹Ρ… чисСл с Ρ€Π°Π·Π½Ρ‹ΠΌΠΈ Π·Π½Π°ΠΊΠ°ΠΌΠΈ. Π§Ρ‚ΠΎΠ±Ρ‹ ΡΠ»ΠΎΠΆΠΈΡ‚ΡŒ Ρ€Π°Ρ†ΠΈΠΎΠ½Π°Π»ΡŒΠ½Ρ‹Π΅ числа с Ρ€Π°Π·Π½Ρ‹ΠΌΠΈ Π·Π½Π°ΠΊΠ°ΠΌΠΈ, Π½ΡƒΠΆΠ½ΠΎ ΠΈΠ· большСго модуля Π²Ρ‹Ρ‡Π΅ΡΡ‚ΡŒ мСньший ΠΌΠΎΠ΄ΡƒΠ»ΡŒ, ΠΈ ΠΏΠ΅Ρ€Π΅Π΄ ΠΏΠΎΠ»ΡƒΡ‡Π΅Π½Π½Ρ‹ΠΌ ΠΎΡ‚Π²Π΅Ρ‚ΠΎΠΌ ΠΏΠΎΡΡ‚Π°Π²ΠΈΡ‚ΡŒ Π·Π½Π°ΠΊ Ρ‚ΠΎΠ³ΠΎ Ρ€Π°Ρ†ΠΈΠΎΠ½Π°Π»ΡŒΠ½ΠΎΠ³ΠΎ числа, ΠΌΠΎΠ΄ΡƒΠ»ΡŒ ΠΊΠΎΡ‚ΠΎΡ€ΠΎΠ³ΠΎ большС. А Ρ‡Ρ‚ΠΎΠ±Ρ‹ ΠΏΠΎΠ½ΡΡ‚ΡŒ ΠΊΠ°ΠΊΠΎΠΉ ΠΌΠΎΠ΄ΡƒΠ»ΡŒ большС, Π° ΠΊΠ°ΠΊΠΎΠΉ мСньшС, Π½ΡƒΠΆΠ½ΠΎ ΡΡƒΠΌΠ΅Ρ‚ΡŒ ΡΡ€Π°Π²Π½ΠΈΡ‚ΡŒ ΠΌΠΎΠ΄ΡƒΠ»ΠΈ этих дСсятичных Π΄Ρ€ΠΎΠ±Π΅ΠΉ Π΄ΠΎ ΠΈΡ… вычислСния:

(βˆ’3,2) + (+4,3) = |+4,3|Β βˆ’ |βˆ’3,2| = 1,1

ΠœΠΎΠ΄ΡƒΠ»ΡŒ числа 4,3 большС, Ρ‡Π΅ΠΌ ΠΌΠΎΠ΄ΡƒΠ»ΡŒ числа βˆ’3,2 поэтому ΠΌΡ‹ ΠΈΠ· 4,3 Π²Ρ‹Ρ‡Π»ΠΈ 3,2. ΠŸΠΎΠ»ΡƒΡ‡ΠΈΠ»ΠΈ ΠΎΡ‚Π²Π΅Ρ‚ 1,1. ΠžΡ‚Π²Π΅Ρ‚ ΠΏΠΎΠ»ΠΎΠΆΠΈΡ‚Π΅Π»Π΅Π½, ΠΏΠΎΡΠΊΠΎΠ»ΡŒΠΊΡƒ ΠΏΠ΅Ρ€Π΅Π΄ ΠΎΡ‚Π²Π΅Ρ‚ΠΎΠΌ Π΄ΠΎΠ»ΠΆΠ΅Π½ ΡΡ‚ΠΎΡΡ‚ΡŒ Π·Π½Π°ΠΊ Ρ‚ΠΎΠ³ΠΎ Ρ€Π°Ρ†ΠΈΠΎΠ½Π°Π»ΡŒΠ½ΠΎΠ³ΠΎ числа, ΠΌΠΎΠ΄ΡƒΠ»ΡŒ ΠΊΠΎΡ‚ΠΎΡ€ΠΎΠ³ΠΎ большС. А ΠΌΠΎΠ΄ΡƒΠ»ΡŒ числа 4,3 большС, Ρ‡Π΅ΠΌ ΠΌΠΎΠ΄ΡƒΠ»ΡŒ Ρ‡ΠΈΡΠ»Π°Β βˆ’3,2

Π’Π°ΠΊΠΈΠΌ ΠΎΠ±Ρ€Π°Π·ΠΎΠΌ, Π·Π½Π°Ρ‡Π΅Π½ΠΈΠ΅ выраТСния βˆ’3,2 + (+4,3) Ρ€Π°Π²Π½ΠΎ 1,1

Π­Ρ‚ΠΎΡ‚ ΠΏΡ€ΠΈΠΌΠ΅Ρ€ ΠΌΠΎΠΆΠ½ΠΎ Π·Π°ΠΏΠΈΡΠ°Ρ‚ΡŒ ΠΏΠΎΠΊΠΎΡ€ΠΎΡ‡Π΅:

βˆ’3,2 + (+4,3) = 1,1


ΠŸΡ€ΠΈΠΌΠ΅Ρ€ 15. Найти Π·Π½Π°Ρ‡Π΅Π½ΠΈΠ΅ выраТСния 3,5 + (βˆ’8,3)

Π­Ρ‚ΠΎ слоТСниС Ρ€Π°Ρ†ΠΈΠΎΠ½Π°Π»ΡŒΠ½Ρ‹Ρ… чисСл с Ρ€Π°Π·Π½Ρ‹ΠΌΠΈ Π·Π½Π°ΠΊΠ°ΠΌΠΈ. Как ΠΈ Π² ΠΏΡ€ΠΎΡˆΠ»ΠΎΠΌ ΠΏΡ€ΠΈΠΌΠ΅Ρ€Π΅ ΠΈΠ· большСго модуля Π²Ρ‹Ρ‡ΠΈΡ‚Π°Π΅ΠΌ мСньший ΠΈ ΠΏΠ΅Ρ€Π΅Π΄ ΠΎΡ‚Π²Π΅Ρ‚ΠΎΠΌ ставим Π·Π½Π°ΠΊ Ρ‚ΠΎΠ³ΠΎ Ρ€Π°Ρ†ΠΈΠΎΠ½Π°Π»ΡŒΠ½ΠΎΠ³ΠΎ числа, ΠΌΠΎΠ΄ΡƒΠ»ΡŒ ΠΊΠΎΡ‚ΠΎΡ€ΠΎΠ³ΠΎ большС:

3,5 + (βˆ’8,3) = βˆ’(|βˆ’8,3| βˆ’ |3,5|) = βˆ’(8,3 βˆ’ 3,5) = βˆ’(4,8) = βˆ’4,8

Π’Π°ΠΊΠΈΠΌ ΠΎΠ±Ρ€Π°Π·ΠΎΠΌ, Π·Π½Π°Ρ‡Π΅Π½ΠΈΠ΅ выраТСния 3,5 + (βˆ’8,3) Ρ€Π°Π²Π½ΠΎ βˆ’4,8

Π­Ρ‚ΠΎΡ‚ ΠΏΡ€ΠΈΠΌΠ΅Ρ€ ΠΌΠΎΠΆΠ½ΠΎ Π·Π°ΠΏΠΈΡΠ°Ρ‚ΡŒ ΠΏΠΎΠΊΠΎΡ€ΠΎΡ‡Π΅:

Β 3,5 + (βˆ’8,3) = βˆ’4,8


ΠŸΡ€ΠΈΠΌΠ΅Ρ€ 16. Найти Π·Π½Π°Ρ‡Π΅Π½ΠΈΠ΅ выраТСния βˆ’7,2 + (βˆ’3,11)

Π­Ρ‚ΠΎ слоТСниС ΠΎΡ‚Ρ€ΠΈΡ†Π°Ρ‚Π΅Π»ΡŒΠ½Ρ‹Ρ… Ρ€Π°Ρ†ΠΈΠΎΠ½Π°Π»ΡŒΠ½Ρ‹Ρ… чисСл. Π§Ρ‚ΠΎΠ±Ρ‹ ΡΠ»ΠΎΠΆΠΈΡ‚ΡŒ ΠΎΡ‚Ρ€ΠΈΡ†Π°Ρ‚Π΅Π»ΡŒΠ½Ρ‹Π΅ Ρ€Π°Ρ†ΠΈΠΎΠ½Π°Π»ΡŒΠ½Ρ‹Π΅ числа, Π½ΡƒΠΆΠ½ΠΎ ΡΠ»ΠΎΠΆΠΈΡ‚ΡŒ ΠΈΡ… ΠΌΠΎΠ΄ΡƒΠ»ΠΈ ΠΈ ΠΏΠ΅Ρ€Π΅Π΄ ΠΏΠΎΠ»ΡƒΡ‡Π΅Π½Π½Ρ‹ΠΌ ΠΎΡ‚Π²Π΅Ρ‚ΠΎΠΌ ΠΏΠΎΡΡ‚Π°Π²ΠΈΡ‚ΡŒ минус.

Π—Π°ΠΏΠΈΡΡŒ с модулями ΠΌΠΎΠΆΠ½ΠΎ ΠΏΡ€ΠΎΠΏΡƒΡΡ‚ΠΈΡ‚ΡŒ, Ρ‡Ρ‚ΠΎΠ±Ρ‹ Π½Π΅ Π·Π°Π³Ρ€ΠΎΠΌΠΎΠΆΠ΄Π°Ρ‚ΡŒ Π²Ρ‹Ρ€Π°ΠΆΠ΅Π½ΠΈΠ΅:

βˆ’7,2 + (βˆ’3,11) =Β βˆ’7,20 + (βˆ’3,11) = βˆ’(7,20 + 3,11) = βˆ’(10,31) = βˆ’10,31

Π’Π°ΠΊΠΈΠΌ ΠΎΠ±Ρ€Π°Π·ΠΎΠΌ, Π·Π½Π°Ρ‡Π΅Π½ΠΈΠ΅ выраТСния βˆ’7,2 + (βˆ’3,11) Ρ€Π°Π²Π½ΠΎ βˆ’10,31

Π­Ρ‚ΠΎΡ‚ ΠΏΡ€ΠΈΠΌΠ΅Ρ€ ΠΌΠΎΠΆΠ½ΠΎ Π·Π°ΠΏΠΈΡΠ°Ρ‚ΡŒ ΠΏΠΎΠΊΠΎΡ€ΠΎΡ‡Π΅:

βˆ’7,2 + (βˆ’3,11) = βˆ’10,31


ΠŸΡ€ΠΈΠΌΠ΅Ρ€ 17. Найти Π·Π½Π°Ρ‡Π΅Π½ΠΈΠ΅ выраТСния βˆ’0,48 + (βˆ’2,7)

Π­Ρ‚ΠΎ слоТСниС ΠΎΡ‚Ρ€ΠΈΡ†Π°Ρ‚Π΅Π»ΡŒΠ½Ρ‹Ρ… Ρ€Π°Ρ†ΠΈΠΎΠ½Π°Π»ΡŒΠ½Ρ‹Ρ… чисСл. Π‘Π»Γ³ΠΆΠΈΠΌ ΠΈΡ… ΠΌΠΎΠ΄ΡƒΠ»ΠΈ ΠΈ ΠΏΠ΅Ρ€Π΅Π΄ ΠΏΠΎΠ»ΡƒΡ‡Π΅Π½Π½Ρ‹ΠΌ ΠΎΡ‚Π²Π΅Ρ‚ΠΎΠΌ поставим минус. Π—Π°ΠΏΠΈΡΡŒ с модулями ΠΌΠΎΠΆΠ½ΠΎ ΠΏΡ€ΠΎΠΏΡƒΡΡ‚ΠΈΡ‚ΡŒ, Ρ‡Ρ‚ΠΎΠ±Ρ‹ Π½Π΅ Π·Π°Π³Ρ€ΠΎΠΌΠΎΠΆΠ΄Π°Ρ‚ΡŒ Π²Ρ‹Ρ€Π°ΠΆΠ΅Π½ΠΈΠ΅:

βˆ’0,48 + (βˆ’2,7) = (βˆ’0,48) + (βˆ’2,70) = βˆ’(0,48 + 2,70) = βˆ’(3,18) = βˆ’3,18


ΠŸΡ€ΠΈΠΌΠ΅Ρ€ 18. Найти Π·Π½Π°Ρ‡Π΅Π½ΠΈΠ΅ выраТСния βˆ’4,9 βˆ’ 5,9

Π—Π°ΠΊΠ»ΡŽΡ‡ΠΈΠΌΒ ΠΊΠ°ΠΆΠ΄ΠΎΠ΅ Ρ€Π°Ρ†ΠΈΠΎΠ½Π°Π»ΡŒΠ½ΠΎΠ΅ число Π² скобки вмСстС со своими Π·Π½Π°ΠΊΠ°ΠΌΠΈ. Π£Ρ‡ΠΈΡ‚Ρ‹Π²Π°Π΅ΠΌ, Ρ‡Ρ‚ΠΎ минус ΠΊΠΎΡ‚ΠΎΡ€Ρ‹ΠΉ располагаСтся ΠΌΠ΅ΠΆΠ΄Ρƒ Ρ€Π°Ρ†ΠΈΠΎΠ½Π°Π»ΡŒΠ½Ρ‹ΠΌΠΈ числами βˆ’4,9 ΠΈ 5,9 являСтся Π·Π½Π°ΠΊΠΎΠΌ ΠΎΠΏΠ΅Ρ€Π°Ρ†ΠΈΠΈ ΠΈ Π½Π΅ относится ΠΊ числу 5,9. Π£ этого Ρ€Π°Ρ†ΠΈΠΎΠ½Π°Π»ΡŒΠ½ΠΎΠ³ΠΎ числа свой Π·Π½Π°ΠΊ плюса, ΠΊΠΎΡ‚ΠΎΡ€Ρ‹ΠΉ Π½Π΅Π²ΠΈΠ΄ΠΈΠΌ ΠΏΠΎ ΠΏΡ€ΠΈΡ‡ΠΈΠ½Π΅ Ρ‚ΠΎΠ³ΠΎ, Ρ‡Ρ‚ΠΎ ΠΎΠ½ Π½Π΅ записываСтся. Но ΠΌΡ‹ запишСм Π΅Π³ΠΎ для наглядности:

(βˆ’4,9) βˆ’ (+5,9)

Π—Π°ΠΌΠ΅Π½ΠΈΠΌ Π²Ρ‹Ρ‡ΠΈΡ‚Π°Π½ΠΈΠ΅ слоТСниСм:

(βˆ’4,9) + (βˆ’5,9)

ΠŸΠΎΠ»ΡƒΡ‡ΠΈΠ»ΠΈΒ  слоТСниС ΠΎΡ‚Ρ€ΠΈΡ†Π°Ρ‚Π΅Π»ΡŒΠ½Ρ‹Ρ… Ρ€Π°Ρ†ΠΈΠΎΠ½Π°Π»ΡŒΠ½Ρ‹Ρ… чисСл. Π‘Π»Γ³ΠΆΠΈΠΌ ΠΈΡ… ΠΌΠΎΠ΄ΡƒΠ»ΠΈ ΠΈ ΠΏΠ΅Ρ€Π΅Π΄ ΠΏΠΎΠ»ΡƒΡ‡Π΅Π½Π½Ρ‹ΠΌ ΠΎΡ‚Π²Π΅Ρ‚ΠΎΠΌ поставим минус:

(βˆ’4,9) + (βˆ’5,9) = βˆ’(4,9 + 5,9) = βˆ’(10,8) = βˆ’10,8

Π’Π°ΠΊΠΈΠΌ ΠΎΠ±Ρ€Π°Π·ΠΎΠΌ, Π·Π½Π°Ρ‡Π΅Π½ΠΈΠ΅ выраТСния βˆ’4,9 βˆ’ 5,9 Ρ€Π°Π²Π½ΠΎ βˆ’10,8

Π—Π°ΠΏΠΈΡˆΠ΅ΠΌ Ρ€Π΅ΡˆΠ΅Π½ΠΈΠ΅ этого ΠΏΡ€ΠΈΠΌΠ΅Ρ€Π° ΠΏΠΎΠΊΠΎΡ€ΠΎΡ‡Π΅:

βˆ’4,9 βˆ’ 5,9 = βˆ’10,8


ΠŸΡ€ΠΈΠΌΠ΅Ρ€ 19. Найти Π·Π½Π°Ρ‡Π΅Π½ΠΈΠ΅ выраТСния 7 βˆ’ 9,3

Π—Π°ΠΊΠ»ΡŽΡ‡ΠΈΠΌ Π² скобки ΠΊΠ°ΠΆΠ΄ΠΎΠ΅ число вмСстС со своими Π·Π½Π°ΠΊΠ°ΠΌΠΈ

(+7) βˆ’ (+9,3)

Π—Π°ΠΌΠ΅Π½ΠΈΠΌ Π²Ρ‹Ρ‡ΠΈΡ‚Π°Π½ΠΈΠ΅ слоТСниСм

(+7) + (βˆ’9,3)

ΠŸΠΎΠ»ΡƒΡ‡ΠΈΠ»ΠΈ слоТСниС Ρ€Π°Ρ†ΠΈΠΎΠ½Π°Π»ΡŒΠ½Ρ‹Ρ… чисСл с Ρ€Π°Π·Π½Ρ‹ΠΌΠΈ Π·Π½Π°ΠΊΠ°ΠΌΠΈ. Π’Ρ‹Ρ‡Ρ‚Π΅ΠΌ ΠΈΠ· большСго модуля мСньший ΠΌΠΎΠ΄ΡƒΠ»ΡŒ, ΠΈ ΠΏΠ΅Ρ€Π΅Π΄ ΠΎΡ‚Π²Π΅Ρ‚ΠΎΠΌ поставим Π·Π½Π°ΠΊ Ρ‚ΠΎΠ³ΠΎ числа, ΠΌΠΎΠ΄ΡƒΠ»ΡŒ ΠΊΠΎΡ‚ΠΎΡ€ΠΎΠ³ΠΎ большС:

(+7) + (βˆ’9,3) = βˆ’(9,3 βˆ’ 7) = βˆ’(2,3) = βˆ’2,3

Π’Π°ΠΊΠΈΠΌ ΠΎΠ±Ρ€Π°Π·ΠΎΠΌ, Π·Π½Π°Ρ‡Π΅Π½ΠΈΠ΅ выраТСния 7 βˆ’ 9,3 Ρ€Π°Π²Π½ΠΎ βˆ’2,3

Π—Π°ΠΏΠΈΡˆΠ΅ΠΌ Ρ€Π΅ΡˆΠ΅Π½ΠΈΠ΅ этого ΠΏΡ€ΠΈΠΌΠ΅Ρ€Π° ΠΏΠΎΠΊΠΎΡ€ΠΎΡ‡Π΅:

7 βˆ’ 9,3 = βˆ’2,3


ΠŸΡ€ΠΈΠΌΠ΅Ρ€ 20. Найти Π·Π½Π°Ρ‡Π΅Π½ΠΈΠ΅ выраТСния βˆ’0,25 βˆ’ (βˆ’1,2)

Π—Π°ΠΌΠ΅Π½ΠΈΠΌ Π²Ρ‹Ρ‡ΠΈΡ‚Π°Π½ΠΈΠ΅ слоТСниСм:

βˆ’0,25 + (+1,2)

ΠŸΠΎΠ»ΡƒΡ‡ΠΈΠ»ΠΈ слоТСниС Ρ€Π°Ρ†ΠΈΠΎΠ½Π°Π»ΡŒΠ½Ρ‹Ρ… чисСл с Ρ€Π°Π·Π½Ρ‹ΠΌΠΈ Π·Π½Π°ΠΊΠ°ΠΌΠΈ. Π’Ρ‹Ρ‡Ρ‚Π΅ΠΌ ΠΈΠ· большСго модуля мСньший ΠΌΠΎΠ΄ΡƒΠ»ΡŒ, ΠΈ ΠΏΠ΅Ρ€Π΅Π΄ ΠΎΡ‚Π²Π΅Ρ‚ΠΎΠΌ поставим Π·Π½Π°ΠΊ Ρ‚ΠΎΠ³ΠΎ числа, ΠΌΠΎΠ΄ΡƒΠ»ΡŒ ΠΊΠΎΡ‚ΠΎΡ€ΠΎΠ³ΠΎ большС:

βˆ’0,25 + (+1,2) = 1,2 βˆ’ 0,25 = 0,95

Π—Π°ΠΏΠΈΡˆΠ΅ΠΌ Ρ€Π΅ΡˆΠ΅Π½ΠΈΠ΅ этого ΠΏΡ€ΠΈΠΌΠ΅Ρ€Π° ΠΏΠΎΠΊΠΎΡ€ΠΎΡ‡Π΅:

βˆ’0,25 βˆ’ (βˆ’1,2) = 0,95


ΠŸΡ€ΠΈΠΌΠ΅Ρ€ 21. Найти Π·Π½Π°Ρ‡Π΅Π½ΠΈΠ΅ выраТСния βˆ’3,5 + (4,1 βˆ’ 7,1)

Π’Ρ‹ΠΏΠΎΠ»Π½ΠΈΠΌ дСйствия Π² скобках, Π·Π°Ρ‚Π΅ΠΌ слóТим ΠΏΠΎΠ»ΡƒΡ‡Π΅Π½Π½Ρ‹ΠΉ ΠΎΡ‚Π²Π΅Ρ‚ с числом βˆ’3,5

ΠŸΠ΅Ρ€Π²ΠΎΠ΅ дСйствиС:

4,1 βˆ’ 7,1 = (+4,1) βˆ’ (+7,1) = (+4,1) + (βˆ’7,1) = βˆ’(7,1 βˆ’ 4,1) = βˆ’(3,0) = βˆ’3,0

Π’Ρ‚ΠΎΡ€ΠΎΠ΅ дСйствиС:

βˆ’3,5 + (βˆ’3,0) = βˆ’(3,5 + 3,0) = βˆ’(6,5) = βˆ’6,5

ΠžΡ‚Π²Π΅Ρ‚: Π·Π½Π°Ρ‡Π΅Π½ΠΈΠ΅ выраТСния βˆ’3,5 + (4,1 βˆ’ 7,1) Ρ€Π°Π²Π½ΠΎ βˆ’6,5.


ΠŸΡ€ΠΈΠΌΠ΅Ρ€ 22. Найти Π·Π½Π°Ρ‡Π΅Π½ΠΈΠ΅ выраТСния (3,5 βˆ’ 2,9) βˆ’ (3,7 βˆ’ 9,1)

Π’Ρ‹ΠΏΠΎΠ»Π½ΠΈΠΌ дСйствия Π² скобках. Π—Π°Ρ‚Π΅ΠΌ ΠΈΠ· числа, ΠΊΠΎΡ‚ΠΎΡ€ΠΎΠ΅ ΠΏΠΎΠ»ΡƒΡ‡ΠΈΠ»ΠΎΡΡŒ Π² Ρ€Π΅Π·ΡƒΠ»ΡŒΡ‚Π°Ρ‚Π΅ выполнСния ΠΏΠ΅Ρ€Π²Ρ‹Ρ… скобок, Π²Ρ‹Ρ‡Ρ‚Π΅ΠΌ число, ΠΊΠΎΡ‚ΠΎΡ€ΠΎΠ΅ ΠΏΠΎΠ»ΡƒΡ‡ΠΈΠ»ΠΎΡΡŒ Π² Ρ€Π΅Π·ΡƒΠ»ΡŒΡ‚Π°Ρ‚Π΅ выполнСния Π²Ρ‚ΠΎΡ€Ρ‹Ρ… скобок:

ΠŸΠ΅Ρ€Π²ΠΎΠ΅ дСйствиС:

3,5 βˆ’ 2,9 = (+3,5) βˆ’ (+2,9) = (+3,5) + (βˆ’2,9) = 3,5 βˆ’ 2,9 = 0,6

Π’Ρ‚ΠΎΡ€ΠΎΠ΅ дСйствиС:

3,7 βˆ’ 9,1 = (+3,7) βˆ’ (+9,1) = (+3,7) + (βˆ’9,1) = βˆ’(9,1 βˆ’ 3,7) = βˆ’(5,4) = βˆ’5,4

Π’Ρ€Π΅Ρ‚ΡŒΠ΅ дСйствиС

0,6 βˆ’ (βˆ’5,4) = (+0,6) + (+5,4) = 0,6 + 5,4 = 6,0 = 6

ΠžΡ‚Π²Π΅Ρ‚: Π·Π½Π°Ρ‡Π΅Π½ΠΈΠ΅ выраТСния (3,5 βˆ’ 2,9) βˆ’ (3,7 βˆ’ 9,1) Ρ€Π°Π²Π½ΠΎ 6.


ΠŸΡ€ΠΈΠΌΠ΅Ρ€ 23. Найти Π·Π½Π°Ρ‡Π΅Π½ΠΈΠ΅ выраТСния βˆ’3,8 + 17,15 βˆ’ 6,2 βˆ’ 6,15

Π—Π°ΠΊΠ»ΡŽΡ‡ΠΈΠΌ Π² скобки ΠΊΠ°ΠΆΠ΄ΠΎΠ΅ Ρ€Π°Ρ†ΠΈΠΎΠ½Π°Π»ΡŒΠ½ΠΎΠ΅ число вмСстС со своими Π·Π½Π°ΠΊΠ°ΠΌΠΈ

(βˆ’3,8) + (+17,15) βˆ’ (+6,2) βˆ’ (+6,15)

Π—Π°ΠΌΠ΅Π½ΠΈΠΌ Π²Ρ‹Ρ‡ΠΈΡ‚Π°Π½ΠΈΠ΅ слоТСниСм Ρ‚Π°ΠΌ, Π³Π΄Π΅ это ΠΌΠΎΠΆΠ½ΠΎ:

(βˆ’3,8) + (+17,15) + (βˆ’6,2) + (βˆ’6,15)

Π’Ρ‹Ρ€Π°ΠΆΠ΅Π½ΠΈΠ΅ состоит ΠΈΠ· Π½Π΅ΡΠΊΠΎΠ»ΡŒΠΊΠΈΡ… слагаСмых. Богласно ΡΠΎΡ‡Π΅Ρ‚Π°Ρ‚Π΅Π»ΡŒΠ½ΠΎΠΌΡƒ Π·Π°ΠΊΠΎΠ½Ρƒ слоТСния, Ссли Π²Ρ‹Ρ€Π°ΠΆΠ΅Π½ΠΈΠ΅ состоит ΠΈΠ· Π½Π΅ΡΠΊΠΎΠ»ΡŒΠΊΠΈΡ… слагаСмых, Ρ‚ΠΎ сумма Π½Π΅ Π±ΡƒΠ΄Π΅Ρ‚ Π·Π°Π²ΠΈΡΠ΅Ρ‚ΡŒ ΠΎΡ‚ порядка дСйствий. Π­Ρ‚ΠΎ Π·Π½Π°Ρ‡ΠΈΡ‚, Ρ‡Ρ‚ΠΎ слагаСмыС ΠΌΠΎΠΆΠ½ΠΎ ΡΠΊΠ»Π°Π΄Ρ‹Π²Π°Ρ‚ΡŒ Π² любом порядкС.

НС Π±ΡƒΠ΄Π΅ΠΌ ΠΈΠ·ΠΎΠ±Ρ€Π΅Ρ‚Π°Ρ‚ΡŒ вСлосипСд, Π° слóТим всС слагаСмыС слСва Π½Π°ΠΏΡ€Π°Π²ΠΎ Π² порядкС ΠΈΡ… слСдования:

ΠŸΠ΅Ρ€Π²ΠΎΠ΅ дСйствиС:

(βˆ’3,8) + (+17,15) = 17,15 βˆ’ 3,80 = 13,35

Π’Ρ‚ΠΎΡ€ΠΎΠ΅ дСйствиС:

13,35 + (βˆ’6,2) = 13,35 βˆ’ βˆ’6,20 = 7,15

Π’Ρ€Π΅Ρ‚ΡŒΠ΅ дСйствиС:

7,15 + (βˆ’6,15) = 7,15 βˆ’ 6,15 = 1,00 = 1

ΠžΡ‚Π²Π΅Ρ‚: Π·Π½Π°Ρ‡Π΅Π½ΠΈΠ΅ выраТСния βˆ’3,8 + 17,15 βˆ’ 6,2 βˆ’ 6,15 Ρ€Π°Π²Π½ΠΎ 1.


ΠŸΡ€ΠΈΠΌΠ΅Ρ€ 24. Найти Π·Π½Π°Ρ‡Π΅Π½ΠΈΠ΅ выраТСния

ΠŸΠ΅Ρ€Π΅Π²Π΅Π΄Ρ‘ΠΌ Π΄Π΅ΡΡΡ‚ΠΈΡ‡Π½ΡƒΡŽ Π΄Ρ€ΠΎΠ±ΡŒΒ βˆ’1,8 Π² смСшанноС число. ΠžΡΡ‚Π°Π»ΡŒΠ½ΠΎΠ΅ ΠΏΠ΅Ρ€Π΅ΠΏΠΈΡˆΠ΅ΠΌ Π±Π΅Π· измСнСния:

Π”Π°Π»Π΅Π΅ вычисляСм Π΄Π°Π½Π½ΠΎΠ΅ Π²Ρ‹Ρ€Π°ΠΆΠ΅Π½ΠΈΠ΅, примСняя Ρ€Π°Π½Π΅Π΅ ΠΈΠ·ΡƒΡ‡Π΅Π½Π½Ρ‹Π΅ ΠΏΡ€Π°Π²ΠΈΠ»Π°:


ΠŸΡ€ΠΈΠΌΠ΅Ρ€ 25. Найти Π·Π½Π°Ρ‡Π΅Π½ΠΈΠ΅ выраТСния

Π—Π°ΠΌΠ΅Π½ΠΈΠΌ Π²Ρ‹Ρ‡ΠΈΡ‚Π°Π½ΠΈΠ΅ слоТСниСм. ΠŸΠΎΠΏΡƒΡ‚Π½ΠΎ ΠΏΠ΅Ρ€Π΅Π²Π΅Π΄Ρ‘ΠΌ Π΄Π΅ΡΡΡ‚ΠΈΡ‡Π½ΡƒΡŽ Π΄Ρ€ΠΎΠ±ΡŒ (βˆ’4,4) Π² Π½Π΅ΠΏΡ€Π°Π²ΠΈΠ»ΡŒΠ½ΡƒΡŽ Π΄Ρ€ΠΎΠ±ΡŒ

Π’ ΠΏΠΎΠ»ΡƒΡ‡ΠΈΠ²ΡˆΠ΅ΠΌΡΡ Π²Ρ‹Ρ€Π°ΠΆΠ΅Π½ΠΈΠΈ Π½Π΅Ρ‚ ΠΎΡ‚Ρ€ΠΈΡ†Π°Ρ‚Π΅Π»ΡŒΠ½Ρ‹Ρ… чисСл. А ΠΏΠΎΡΠΊΠΎΠ»ΡŒΠΊΡƒ Π½Π΅Ρ‚ ΠΎΡ‚Ρ€ΠΈΡ†Π°Ρ‚Π΅Π»ΡŒΠ½Ρ‹Ρ… чисСл, ΠΌΡ‹ ΠΌΠΎΠΆΠ΅ΠΌ ΡƒΠ±Ρ€Π°Ρ‚ΡŒ плюс ΠΏΠ΅Ρ€Π΅Π΄ Π²Ρ‚ΠΎΡ€Ρ‹ΠΌ числом, ΠΈ ΡƒΠ±Ρ€Π°Ρ‚ΡŒ скобки. Π’ΠΎΠ³Π΄Π° ΠΏΠΎΠ»ΡƒΡ‡ΠΈΠΌ простоС Π²Ρ‹Ρ€Π°ΠΆΠ΅Π½ΠΈΠ΅ Π½Π° слоТСниС, ΠΊΠΎΡ‚ΠΎΡ€ΠΎΠ΅ Ρ€Π΅ΡˆΠ°Π΅Ρ‚ΡΡ Π»Π΅Π³ΠΊΠΎ


ΠŸΡ€ΠΈΠΌΠ΅Ρ€ 26. Найти Π·Π½Π°Ρ‡Π΅Π½ΠΈΠ΅ выраТСния

ΠŸΠ΅Ρ€Π΅Π²Π΅Π΄Ρ‘ΠΌ смСшанноС число Π² Π½Π΅ΠΏΡ€Π°Π²ΠΈΠ»ΡŒΠ½ΡƒΡŽ Π΄Ρ€ΠΎΠ±ΡŒ, Π° Π΄Π΅ΡΡΡ‚ΠΈΡ‡Π½ΡƒΡŽ Π΄Ρ€ΠΎΠ±ΡŒ βˆ’0,85 Π² ΠΎΠ±Ρ‹ΠΊΠ½ΠΎΠ²Π΅Π½Π½ΡƒΡŽ Π΄Ρ€ΠΎΠ±ΡŒ. ΠŸΠΎΠ»ΡƒΡ‡ΠΈΠΌ ΡΠ»Π΅Π΄ΡƒΡŽΡ‰Π΅Π΅ Π²Ρ‹Ρ€Π°ΠΆΠ΅Π½ΠΈΠ΅:

ΠŸΠΎΠ»ΡƒΡ‡ΠΈΠ»ΠΈΒ  слоТСниС ΠΎΡ‚Ρ€ΠΈΡ†Π°Ρ‚Π΅Π»ΡŒΠ½Ρ‹Ρ… Ρ€Π°Ρ†ΠΈΠΎΠ½Π°Π»ΡŒΠ½Ρ‹Ρ… чисСл. Π‘Π»Γ³ΠΆΠΈΠΌ ΠΈΡ… ΠΌΠΎΠ΄ΡƒΠ»ΠΈ ΠΈ ΠΏΠ΅Ρ€Π΅Π΄ ΠΏΠΎΠ»ΡƒΡ‡Π΅Π½Π½Ρ‹ΠΌ ΠΎΡ‚Π²Π΅Ρ‚ΠΎΠΌ поставим минус:


ΠŸΡ€ΠΈΠΌΠ΅Ρ€ 27. Найти Π·Π½Π°Ρ‡Π΅Π½ΠΈΠ΅ выраТСния

ΠŸΠ΅Ρ€Π΅Π²Π΅Π΄Ρ‘ΠΌ ΠΎΠ±Π΅ Π΄Ρ€ΠΎΠ±ΠΈ Π² Π½Π΅ΠΏΡ€Π°Π²ΠΈΠ»ΡŒΠ½Ρ‹Π΅ Π΄Ρ€ΠΎΠ±ΠΈ. Π§Ρ‚ΠΎΠ±Ρ‹ пСрСвСсти Π΄Π΅ΡΡΡ‚ΠΈΡ‡Π½ΡƒΡŽ Π΄Ρ€ΠΎΠ±ΡŒ 2,05 Π² Π½Π΅ΠΏΡ€Π°Π²ΠΈΠ»ΡŒΠ½ΡƒΡŽ Π΄Ρ€ΠΎΠ±ΡŒ, ΠΌΠΎΠΆΠ½ΠΎ пСрСвСсти Π΅Π΅ сначала Π² смСшанноС число, Π° Π·Π°Ρ‚Π΅ΠΌ Π² Π½Π΅ΠΏΡ€Π°Π²ΠΈΠ»ΡŒΠ½ΡƒΡŽ Π΄Ρ€ΠΎΠ±ΡŒ:

ПослС ΠΏΠ΅Ρ€Π΅Π²ΠΎΠ΄Π° ΠΎΠ±Π΅ΠΈΡ… Π΄Ρ€ΠΎΠ±Π΅ΠΉ Π² Π½Π΅ΠΏΡ€Π°Π²ΠΈΠ»ΡŒΠ½Ρ‹Π΅ Π΄Ρ€ΠΎΠ±ΠΈ, ΠΏΠΎΠ»ΡƒΡ‡ΠΈΠΌ ΡΠ»Π΅Π΄ΡƒΡŽΡ‰Π΅Π΅ Π²Ρ‹Ρ€Π°ΠΆΠ΅Π½ΠΈΠ΅:

ΠŸΠΎΠ»ΡƒΡ‡ΠΈΠ»ΠΈ слоТСниС Ρ€Π°Ρ†ΠΈΠΎΠ½Π°Π»ΡŒΠ½Ρ‹Ρ… чисСл с Ρ€Π°Π·Π½Ρ‹ΠΌΠΈ Π·Π½Π°ΠΊΠ°ΠΌΠΈ. Π’Ρ‹Ρ‡Ρ‚Π΅ΠΌ ΠΈΠ· большСго модуля мСньший ΠΌΠΎΠ΄ΡƒΠ»ΡŒ ΠΈ ΠΏΠ΅Ρ€Π΅Π΄ ΠΏΠΎΠ»ΡƒΡ‡Π΅Π½Π½Ρ‹ΠΌ ΠΎΡ‚Π²Π΅Ρ‚ΠΎΠΌ поставим Π·Π½Π°ΠΊ Ρ‚ΠΎΠ³ΠΎ числа, ΠΌΠΎΠ΄ΡƒΠ»ΡŒ ΠΊΠΎΡ‚ΠΎΡ€ΠΎΠ³ΠΎ большС:


ΠŸΡ€ΠΈΠΌΠ΅Ρ€ 28. Найти Π·Π½Π°Ρ‡Π΅Π½ΠΈΠ΅ выраТСния 

Π—Π°ΠΌΠ΅Π½ΠΈΠΌ Π²Ρ‹Ρ‡ΠΈΡ‚Π°Π½ΠΈΠ΅ слоТСниСм. Π”Π°Π»Π΅Π΅ ΠΏΠ΅Ρ€Π΅Π²Π΅Π΄Ρ‘ΠΌ Π΄Π΅ΡΡΡ‚ΠΈΡ‡Π½ΡƒΡŽ Π΄Ρ€ΠΎΠ±ΡŒ Π² ΠΎΠ±Ρ‹ΠΊΠ½ΠΎΠ²Π΅Π½Π½ΡƒΡŽ Π΄Ρ€ΠΎΠ±ΡŒ. Π—Π°Ρ‚Π΅ΠΌ вычислим ΠΏΠΎΠ»ΡƒΡ‡ΠΈΠ²ΡˆΠ΅Π΅ΡΡ Π²Ρ‹Ρ€Π°ΠΆΠ΅Π½ΠΈΠ΅, примСняя Ρ€Π°Π½Π΅Π΅ ΠΈΠ·ΡƒΡ‡Π΅Π½Π½Ρ‹Π΅ ΠΏΡ€Π°Π²ΠΈΠ»Π°:


ΠŸΡ€ΠΈΠΌΠ΅Ρ€ 29. Найти Π·Π½Π°Ρ‡Π΅Π½ΠΈΠ΅ выраТСния

ΠŸΠ΅Ρ€Π΅Π²Π΅Π΄Ρ‘ΠΌ дСсятичныС Π΄Ρ€ΠΎΠ±ΠΈ βˆ’0,25 ΠΈΒ βˆ’1,25 Π² ΠΎΠ±Ρ‹ΠΊΠ½ΠΎΠ²Π΅Π½Π½Ρ‹Π΅ Π΄Ρ€ΠΎΠ±ΠΈ, ΠΎΡΡ‚Π°Π»ΡŒΠ½ΠΎΠ΅ ΠΏΠ΅Ρ€Π΅ΠΏΠΈΡˆΠ΅ΠΌ Π±Π΅Π· измСнСния. ΠŸΠΎΠ»ΡƒΡ‡ΠΈΠΌ ΡΠ»Π΅Π΄ΡƒΡŽΡ‰Π΅Π΅ Π²Ρ‹Ρ€Π°ΠΆΠ΅Π½ΠΈΠ΅:

МоТно сначала Π·Π°ΠΌΠ΅Π½ΠΈΡ‚ΡŒ Π²Ρ‹Ρ‡ΠΈΡ‚Π°Π½ΠΈΠ΅ слоТСниСм Ρ‚Π°ΠΌ, Π³Π΄Π΅ это ΠΌΠΎΠΆΠ½ΠΎ ΠΈ ΡΠ»ΠΎΠΆΠΈΡ‚ΡŒ Ρ€Π°Ρ†ΠΈΠΎΠ½Π°Π»ΡŒΠ½Ρ‹Π΅ числа ΠΎΠ΄Π½ΠΎ Π·Π° Π΄Ρ€ΡƒΠ³ΠΈΠΌ.

Π•ΡΡ‚ΡŒ ΠΈ Π²Ρ‚ΠΎΡ€ΠΎΠΉ Π²Π°Ρ€ΠΈΠ°Π½Ρ‚: сначала ΡΠ»ΠΎΠΆΠΈΡ‚ΡŒ Ρ€Π°Ρ†ΠΈΠΎΠ½Π°Π»ΡŒΠ½Ρ‹Π΅ числа ΠΈ , Π° Π·Π°Ρ‚Π΅ΠΌ ΠΈΠ· ΠΏΠΎΠ»ΡƒΡ‡Π΅Π½Π½ΠΎΠ³ΠΎ Ρ€Π΅Π·ΡƒΠ»ΡŒΡ‚Π°Ρ‚Π° Π²Ρ‹Ρ‡Π΅ΡΡ‚ΡŒΒ . Π­Ρ‚ΠΈΠΌ Π²Π°Ρ€ΠΈΠ°Π½Ρ‚ΠΎΠΌ ΠΈ Π²ΠΎΡΠΏΠΎΠ»ΡŒΠ·ΡƒΠ΅ΠΌΡΡ.

ΠŸΠ΅Ρ€Π²ΠΎΠ΅ дСйствиС:

Π’Ρ‚ΠΎΡ€ΠΎΠ΅ дСйствиС:

ΠžΡ‚Π²Π΅Ρ‚: Π·Π½Π°Ρ‡Π΅Π½ΠΈΠ΅ выраТСния  Ρ€Π°Π²Π½ΠΎΒ βˆ’2.


ΠŸΡ€ΠΈΠΌΠ΅Ρ€ 30. Найти Π·Π½Π°Ρ‡Π΅Π½ΠΈΠ΅ выраТСния

ΠŸΠ΅Ρ€Π΅Π²Π΅Π΄Ρ‘ΠΌ дСсятичныС Π΄Ρ€ΠΎΠ±ΠΈ Π² ΠΎΠ±Ρ‹ΠΊΠ½ΠΎΠ²Π΅Π½Π½Ρ‹Π΅. ΠžΡΡ‚Π°Π»ΡŒΠ½ΠΎΠ΅ ΠΏΠ΅Ρ€Π΅ΠΏΠΈΡˆΠ΅ΠΌ Π±Π΅Π· измСнСния:

ΠŸΠΎΠ»ΡƒΡ‡ΠΈΠ»ΠΈ сумму ΠΈΠ· Π½Π΅ΡΠΊΠΎΠ»ΡŒΠΊΠΈΡ… слагаСмых. Если сумма состоит ΠΈΠ· Π½Π΅ΡΠΊΠΎΠ»ΡŒΠΊΠΈΡ… слагаСмых, Ρ‚ΠΎ Π²Ρ‹Ρ€Π°ΠΆΠ΅Π½ΠΈΠ΅ ΠΌΠΎΠΆΠ½ΠΎ Π²Ρ‹Ρ‡ΠΈΡΠ»ΡΡ‚ΡŒ Π² любом порядкС. Π­Ρ‚ΠΎ слСдуСт ΠΈΠ· ΡΠΎΡ‡Π΅Ρ‚Π°Ρ‚Π΅Π»ΡŒΠ½ΠΎΠ³ΠΎ Π·Π°ΠΊΠΎΠ½Π° слоТСния.

ΠŸΠΎΡΡ‚ΠΎΠΌΡƒ ΠΌΡ‹ ΠΌΠΎΠΆΠ΅ΠΌ ΠΎΡ€Π³Π°Π½ΠΈΠ·ΠΎΠ²Π°Ρ‚ΡŒ Π½Π°ΠΈΠ±ΠΎΠ»Π΅Π΅ ΡƒΠ΄ΠΎΠ±Π½Ρ‹ΠΉ для нас Π²Π°Ρ€ΠΈΠ°Π½Ρ‚. Π’ ΠΏΠ΅Ρ€Π²ΡƒΡŽ ΠΎΡ‡Π΅Ρ€Π΅Π΄ΡŒ ΠΌΠΎΠΆΠ½ΠΎ ΡΠ»ΠΎΠΆΠΈΡ‚ΡŒ ΠΏΠ΅Ρ€Π²ΠΎΠ΅ ΠΈ послСднСС слагаСмоС, Π° ΠΈΠΌΠ΅Π½Π½ΠΎ Ρ€Π°Ρ†ΠΈΠΎΠ½Π°Π»ΡŒΠ½Ρ‹Π΅ числа Β  ΠΈΒ  . Π£ этих чисСл ΠΎΠ΄ΠΈΠ½Π°ΠΊΠΎΠ²Ρ‹Π΅ Π·Π½Π°ΠΌΠ΅Π½Π°Ρ‚Π΅Π»ΠΈ, Π° Π·Π½Π°Ρ‡ΠΈΡ‚ это освободит нас ΠΎΡ‚ нСобходимости ΠΏΡ€ΠΈΠ²ΠΎΠ΄ΠΈΡ‚ΡŒ ΠΈΡ… ΠΊ Π½Π΅ΠΌΡƒ.

ΠŸΠ΅Ρ€Π²ΠΎΠ΅ дСйствиС:

ΠŸΠΎΠ»ΡƒΡ‡Π΅Π½Π½ΠΎΠ΅ число ΠΌΠΎΠΆΠ½ΠΎ ΡΠ»ΠΎΠΆΠΈΡ‚ΡŒ со Π²Ρ‚ΠΎΡ€Ρ‹ΠΌ слагаСмым, Π° ΠΈΠΌΠ΅Π½Π½ΠΎ с Ρ€Π°Ρ†ΠΈΠΎΠ½Π°Π»ΡŒΠ½Ρ‹ΠΌ числом . Π£ Ρ€Π°Ρ†ΠΈΠΎΠ½Π°Π»ΡŒΠ½Ρ‹Ρ… чисСл ΠΈΒ Β  ΠΎΠ΄ΠΈΠ½Π°ΠΊΠΎΠ²Ρ‹Π΅ Π·Π½Π°ΠΌΠ΅Π½Π°Ρ‚Π΅Π»ΠΈ Π² Π΄Ρ€ΠΎΠ±Π½Ρ‹Ρ… частях, Ρ‡Ρ‚ΠΎ ΠΎΠΏΡΡ‚ΡŒ ΠΆΠ΅ являСтся прСимущСством для нас

Π’Ρ‚ΠΎΡ€ΠΎΠ΅ дСйствиС:

Ну ΠΈ слóТим ΠΏΠΎΠ»ΡƒΡ‡Π΅Π½Π½ΠΎΠ΅ число βˆ’7 с послСдним слагаСмым, Π° ΠΈΠΌΠ΅Π½Π½ΠΎ с Ρ€Π°Ρ†ΠΈΠΎΠ½Π°Π»ΡŒΠ½Ρ‹ΠΌ числом . Π£Π΄ΠΎΠ±Π½ΠΎ Ρ‚ΠΎ, Ρ‡Ρ‚ΠΎ ΠΏΡ€ΠΈ вычислСнии Π΄Π°Π½Π½ΠΎΠ³ΠΎ выраТСния, сСмёрки исчСзнут, ΠΏΠΎΡΠΊΠΎΠ»ΡŒΠΊΡƒ ΠΈΡ… сумма Π±ΡƒΠ΄Π΅Ρ‚ Ρ€Π°Π²Π½Π° Π½ΡƒΠ»ΡŽ:

Π’Ρ€Π΅Ρ‚ΡŒΠ΅ дСйствиС:

ΠžΡ‚Π²Π΅Ρ‚: Π·Π½Π°Ρ‡Π΅Π½ΠΈΠ΅ выраТСния  Ρ€Π°Π²Π½ΠΎ

Задания для ΡΠ°ΠΌΠΎΡΡ‚ΠΎΡΡ‚Π΅Π»ΡŒΠ½ΠΎΠ³ΠΎ Ρ€Π΅ΡˆΠ΅Π½ΠΈΡ

Π—Π°Π΄Π°Π½ΠΈΠ΅ 1. НайдитС Π·Π½Π°Ρ‡Π΅Π½ΠΈΠ΅ выраТСния:

РСшСниС:

Π—Π°Π΄Π°Π½ΠΈΠ΅ 2. НайдитС Π·Π½Π°Ρ‡Π΅Π½ΠΈΠ΅ выраТСния:

РСшСниС:

Π—Π°Π΄Π°Π½ΠΈΠ΅ 3. НайдитС Π·Π½Π°Ρ‡Π΅Π½ΠΈΠ΅ выраТСния:

РСшСниС:

Π—Π°Π΄Π°Π½ΠΈΠ΅ 4. НайдитС Π·Π½Π°Ρ‡Π΅Π½ΠΈΠ΅ выраТСния:

РСшСниС:

Π—Π°Π΄Π°Π½ΠΈΠ΅ 5. НайдитС Π·Π½Π°Ρ‡Π΅Π½ΠΈΠ΅ выраТСния:

РСшСниС:

Π—Π°Π΄Π°Π½ΠΈΠ΅ 6. НайдитС Π·Π½Π°Ρ‡Π΅Π½ΠΈΠ΅ выраТСния:

РСшСниС:

Π—Π°Π΄Π°Π½ΠΈΠ΅ 7. НайдитС Π·Π½Π°Ρ‡Π΅Π½ΠΈΠ΅ выраТСния:

РСшСниС:

Π—Π°Π΄Π°Π½ΠΈΠ΅ 8. НайдитС Π·Π½Π°Ρ‡Π΅Π½ΠΈΠ΅ выраТСния:

РСшСниС:

Π—Π°Π΄Π°Π½ΠΈΠ΅ 9. НайдитС Π·Π½Π°Ρ‡Π΅Π½ΠΈΠ΅ выраТСния:

РСшСниС:

Π—Π°Π΄Π°Π½ΠΈΠ΅ 10. НайдитС Π·Π½Π°Ρ‡Π΅Π½ΠΈΠ΅ выраТСния:

РСшСниС:


ΠŸΠΎΠ½Ρ€Π°Π²ΠΈΠ»ΡΡ ΡƒΡ€ΠΎΠΊ?
Вступай Π² Π½Π°ΡˆΡƒ Π½ΠΎΠ²ΡƒΡŽ Π³Ρ€ΡƒΠΏΠΏΡƒ Π’ΠΊΠΎΠ½Ρ‚Π°ΠΊΡ‚Π΅ ΠΈ Π½Π°Ρ‡Π½ΠΈ ΠΏΠΎΠ»ΡƒΡ‡Π°Ρ‚ΡŒ увСдомлСния ΠΎ Π½ΠΎΠ²Ρ‹Ρ… ΡƒΡ€ΠΎΠΊΠ°Ρ…

Π’ΠΎΠ·Π½ΠΈΠΊΠ»ΠΎ ΠΆΠ΅Π»Π°Π½ΠΈΠ΅ ΠΏΠΎΠ΄Π΄Π΅Ρ€ΠΆΠ°Ρ‚ΡŒ ΠΏΡ€ΠΎΠ΅ΠΊΡ‚?
Π˜ΡΠΏΠΎΠ»ΡŒΠ·ΡƒΠΉ ΠΊΠ½ΠΎΠΏΠΊΡƒ Π½ΠΈΠΆΠ΅

Навигация ΠΏΠΎ записям

Как ΠΈΠ· ΠΎΡ‚Ρ€ΠΈΡ†Π°Ρ‚Π΅Π»ΡŒΠ½ΠΎΠΉ стСпСни ΡΠ΄Π΅Π»Π°Ρ‚ΡŒ ΠΏΠΎΠ»ΠΎΠΆΠΈΡ‚Π΅Π»ΡŒΠ½ΡƒΡŽ

ΠŸΡ€Π΅ΠΆΠ΄Π΅ Ρ‡Π΅ΠΌ ΠΏΠ΅Ρ€Π΅ΠΉΡ‚ΠΈ ΠΊ ΠΈΠ·ΡƒΡ‡Π΅Π½ΠΈΡŽ опрСдСлСния Β«ΠΎΡ‚Ρ€ΠΈΡ†Π°Ρ‚Π΅Π»ΡŒΠ½Π°Ρ ΡΡ‚Π΅ΠΏΠ΅Π½ΡŒΒ» Ρ€Π΅ΠΊΠΎΠΌΠ΅Π½Π΄ΡƒΠ΅ΠΌ ΠΏΠΎΠ²Ρ‚ΠΎΡ€Π½ΠΎ ΠΏΡ€ΠΎΡ‡ΠΈΡ‚Π°Ρ‚ΡŒ ΡƒΡ€ΠΎΠΊ Β«Π‘Ρ‚Π΅ΠΏΠ΅Π½ΡŒΒ» ΠΈ «Бвойства стСпСнСй».

НСобходимо ΡƒΠ²Π΅Ρ€Π΅Π½Π½ΠΎ ΠΏΠΎΠ½ΠΈΠΌΠ°Ρ‚ΡŒ, Ρ‡Ρ‚ΠΎ Ρ‚Π°ΠΊΠΎΠ΅ ΠΏΠΎΠ»ΠΎΠΆΠΈΡ‚Π΅Π»ΡŒΠ½Π°Ρ ΡΡ‚Π΅ΠΏΠ΅Π½ΡŒ числа ΠΈ ΡƒΠ²Π΅Ρ€Π΅Π½Π½ΠΎ ΠΈΡΠΏΠΎΠ»ΡŒΠ·ΠΎΠ²Π°Ρ‚ΡŒ Π΅Ρ‘ свойства Π² Ρ€Π΅ΡˆΠ΅Π½ΠΈΠΈ ΠΏΡ€ΠΈΠΌΠ΅Ρ€ΠΎΠ².

Как возвСсти число Π² ΠΎΡ‚Ρ€ΠΈΡ†Π°Ρ‚Π΅Π»ΡŒΠ½ΡƒΡŽ ΡΡ‚Π΅ΠΏΠ΅Π½ΡŒ

Π§Ρ‚ΠΎΠ±Ρ‹ возвСсти число Π² ΠΎΡ‚Ρ€ΠΈΡ†Π°Ρ‚Π΅Π»ΡŒΠ½ΡƒΡŽ ΡΡ‚Π΅ΠΏΠ΅Π½ΡŒ Π½ΡƒΠΆΠ½ΠΎ:

  • Β«ΠΏΠ΅Ρ€Π΅Π²Π΅Ρ€Π½ΡƒΡ‚ΡŒΒ» число. Π—Π°ΠΏΠΈΡΠ°Ρ‚ΡŒ Π΅Π³ΠΎ Π² Π²ΠΈΠ΄Π΅ Π΄Ρ€ΠΎΠ±ΠΈ с Π΅Π΄ΠΈΠ½ΠΈΡ†ΠΎΠΉ Π½Π°Π²Π΅Ρ€Ρ…Ρƒ (Π² числитСлС) ΠΈ с исходным числом Π² стСпСни Π²Π½ΠΈΠ·Ρƒ;
  • Π·Π°ΠΌΠ΅Π½ΠΈΡ‚ΡŒ ΠΎΡ‚Ρ€ΠΈΡ†Π°Ρ‚Π΅Π»ΡŒΠ½ΡƒΡŽ ΡΡ‚Π΅ΠΏΠ΅Π½ΡŒ Π½Π° ΠΏΠΎΠ»ΠΎΠΆΠΈΡ‚Π΅Π»ΡŒΠ½ΡƒΡŽ ;
  • возвСсти число Π² ΠΏΠΎΠ»ΠΎΠΆΠΈΡ‚Π΅Π»ΡŒΠ½ΡƒΡŽ ΡΡ‚Π΅ΠΏΠ΅Π½ΡŒ.

ΠžΠ±Ρ‰Π°Ρ Ρ„ΠΎΡ€ΠΌΡƒΠ»Π° возвСдСния Π² ΠΎΡ‚Ρ€ΠΈΡ†Π°Ρ‚Π΅Π»ΡŒΠ½ΡƒΡŽ ΡΡ‚Π΅ΠΏΠ΅Π½ΡŒ выглядит ΡΠ»Π΅Π΄ΡƒΡŽΡ‰ΠΈΠΌ ΠΎΠ±Ρ€Π°Π·ΠΎΠΌ.

a βˆ’n =

,Π³Π΄Π΅ a β‰  0, n ∈ z ( n ΠΏΡ€ΠΈΠ½Π°Π΄Π»Π΅ΠΆΠΈΡ‚ Ρ†Π΅Π»Ρ‹ΠΌ числам).

ΠŸΡ€ΠΈΠΌΠ΅Ρ€Ρ‹ возвСдСния Π² ΠΎΡ‚Ρ€ΠΈΡ†Π°Ρ‚Π΅Π»ΡŒΠ½ΡƒΡŽ ΡΡ‚Π΅ΠΏΠ΅Π½ΡŒ.

  • 6 βˆ’2 =

    =

  • (βˆ’3) βˆ’3 =

    =

    = βˆ’

  • 0,2 βˆ’2 =

    =

Π›ΡŽΠ±ΠΎΠ΅ число Π² Π½ΡƒΠ»Π΅Π²ΠΎΠΉ стСпСни β€” Π΅Π΄ΠΈΠ½ΠΈΡ†Π°.

ΠŸΡ€ΠΈΠΌΠ΅Ρ€Ρ‹ возвСдСния Π² Π½ΡƒΠ»Π΅Π²ΡƒΡŽ ΡΡ‚Π΅ΠΏΠ΅Π½ΡŒ.

Как Π½Π°ΠΉΡ‚ΠΈ

10 Π² минус 1 стСпСни

Π’ ΡƒΡ€ΠΎΠΊΠ΅ 8 класса Β«Π‘Ρ‚Π°Π½Π΄Π°Ρ€Ρ‚Π½Ρ‹ΠΉ Π²ΠΈΠ΄ числа» ΠΌΡ‹ ΡƒΠΆΠ΅ ΡΡ‚Π°Π»ΠΊΠΈΠ²Π°Π»ΠΈΡΡŒ с записью:

Π’Π΅ΠΏΠ΅Ρ€ΡŒ, зная ΠΎΠΏΡ€Π΅Π΄Π΅Π»Π΅Π½ΠΈΠ΅ ΠΎΡ‚Ρ€ΠΈΡ†Π°Ρ‚Π΅Π»ΡŒΠ½ΠΎΠΉ стСпСни, Π΄Π°Π²Π°ΠΉΡ‚Π΅ разбСрСмся, ΠΏΠΎΡ‡Π΅ΠΌΡƒ Β« 10 Β» Π² минус ΠΏΠ΅Ρ€Π²ΠΎΠΉ стСпСни Ρ€Π°Π²Π½ΠΎ Β« 0,1 Β».

Π’ΠΎΠ·Π²Π΅Π΄Π΅ΠΌ Β« 10 βˆ’1 Β» ΠΏΠΎ ΠΏΡ€Π°Π²ΠΈΠ»Π°ΠΌ ΠΎΡ‚Ρ€ΠΈΡ†Π°Ρ‚Π΅Π»ΡŒΠ½ΠΎΠΉ стСпСни. ΠŸΠ΅Ρ€Π΅Π²Π΅Ρ€Π½Π΅ΠΌ Β« 10 Β» ΠΈ запишСм Π΅Ρ‘ Π² Π²ΠΈΠ΄Π΅ Π΄Ρ€ΠΎΠ±ΠΈ Β«

Β» ΠΈ Π·Π°ΠΌΠ΅Π½ΠΈΠΌ ΠΎΡ‚Ρ€ΠΈΡ†Π°Ρ‚Π΅Π»ΡŒΠ½ΡƒΡŽ ΡΡ‚Π΅ΠΏΠ΅Π½ΡŒ Β« βˆ’1 Β» Π½Π°
ΠΏΠΎΠ»ΠΎΠΆΠΈΡ‚Π΅Π»ΡŒΠ½ΡƒΡŽ ΡΡ‚Π΅ΠΏΠ΅Π½ΡŒ Β« 1 Β».

10 βˆ’1 =

Π’ΠΎΠ·Π²Π΅Π΄Π΅ΠΌ Β« 10 Β» Π² Β« 1 Β» ΡΡ‚Π΅ΠΏΠ΅Π½ΡŒ. Помним, Ρ‡Ρ‚ΠΎ любоС число Π² ΠΏΠ΅Ρ€Π²ΠΎΠΉ стСпСни Ρ€Π°Π²Π½ΠΎ самому числу.

10 βˆ’1 =

=

Π’Π΅ΠΏΠ΅Ρ€ΡŒ ΠΏΠΎ ΠΎΠΏΡ€Π΅Π΄Π΅Π»Π΅Π½ΠΈΡŽ дСсятичной Π΄Ρ€ΠΎΠ±ΠΈ запишСм ΠΎΠ±Ρ‹ΠΊΠ½ΠΎΠ²Π΅Π½Π½ΡƒΡŽ Π΄Ρ€ΠΎΠ±ΡŒ Π² Π²ΠΈΠ΄Π΅ дСсятичной.

10 βˆ’1 =

=

= 0,1

По Ρ‚Π°ΠΊΠΎΠΌΡƒ ΠΆΠ΅ ΠΏΡ€ΠΈΠ½Ρ†ΠΈΠΏΡƒ ΠΌΠΎΠΆΠ½ΠΎ Π½Π°ΠΉΡ‚ΠΈ Β« 10 Β» Π² минус Π²Ρ‚ΠΎΡ€ΠΎΠΉ, Ρ‚Ρ€Π΅Ρ‚ΡŒΠ΅ΠΉ ΠΈ Ρ‚.Π΄.

Для упрощСния ΠΏΠ΅Ρ€Π΅Π²ΠΎΠ΄Π° Β« 10 Β» Π² минус ΠΏΠ΅Ρ€Π²ΡƒΡŽ, Π²Ρ‚ΠΎΡ€ΡƒΡŽ ΠΈ Ρ‚.Π΄ стСпСни, Π½ΡƒΠΆΠ½ΠΎ Π·Π°ΠΏΠΎΠΌΠ½ΠΈΡ‚ΡŒ ΠΏΡ€Π°Π²ΠΈΠ»ΠΎ:
Β«ΠšΠΎΠ»ΠΈΡ‡Π΅ΡΡ‚Π²ΠΎ Π½ΡƒΠ»Π΅ΠΉ послС запятой Ρ€Π°Π²Π½ΠΎ ΠΏΠΎΠ»ΠΎΠΆΠΈΡ‚Π΅Π»ΡŒΠ½ΠΎΠΌΡƒ Π·Π½Π°Ρ‡Π΅Π½ΠΈΡŽ стСпСни минус ΠΎΠ΄ΠΈΠ½ Β».

ΠŸΡ€ΠΎΠ²Π΅Ρ€ΠΈΠΌ ΠΏΡ€Π°Π²ΠΈΠ»ΠΎ Π²Ρ‹ΡˆΠ΅ для Β« 10 βˆ’2 Β».

Π’.ΠΊ. Ρƒ нас ΡΡ‚Π΅ΠΏΠ΅Π½ΡŒ Β« βˆ’2 Β», Π·Π½Π°Ρ‡ΠΈΡ‚, Π±ΡƒΠ΄Π΅Ρ‚ всСго ΠΎΠ΄ΠΈΠ½ ноль (ΠΏΠΎΠ»ΠΎΠΆΠΈΡ‚Π΅Π»ΡŒΠ½ΠΎΠ΅ Π·Π½Π°Ρ‡Π΅Π½ΠΈΠ΅ стСпСни Β« 2 βˆ’ 1 = 1 Β». Π‘Ρ€Π°Π·Ρƒ послС запятой ставим ΠΎΠ΄ΠΈΠ½ ноль ΠΈ Π·Π° Π½ΠΈΠΌ Β« 1 Β».

Рассмотрим Β« 10 βˆ’1 Β».

Π’.ΠΊ. Ρƒ нас ΡΡ‚Π΅ΠΏΠ΅Π½ΡŒ Β« βˆ’1 Β», Π·Π½Π°Ρ‡ΠΈΡ‚, Π½ΡƒΠ»Π΅ΠΉ послС запятой Π½Π΅ Π±ΡƒΠ΄Π΅Ρ‚ (ΠΏΠΎΠ»ΠΎΠΆΠΈΡ‚Π΅Π»ΡŒΠ½ΠΎΠ΅ Π·Π½Π°Ρ‡Π΅Π½ΠΈΠ΅ стСпСни Β« 1 βˆ’ 1 = 0 Β». Π‘Ρ€Π°Π·Ρƒ послС запятой ставим Β« 1 Β».

Π’ΠΎ ΠΆΠ΅ самоС ΠΏΡ€Π°Π²ΠΈΠ»ΠΎ Ρ€Π°Π±ΠΎΡ‚Π°Π΅Ρ‚ ΠΈ для Β« 10 βˆ’12 Β». ΠŸΡ€ΠΈ ΠΏΠ΅Ρ€Π΅Π²ΠΎΠ΄Π΅ Π² Π΄Π΅ΡΡΡ‚ΠΈΡ‡Π½ΡƒΡŽ Π΄Ρ€ΠΎΠ±ΡŒ Π±ΡƒΠ΄Π΅Ρ‚ Β« 12 βˆ’ 1 = 11 Β» Π½ΡƒΠ»Π΅ΠΉ ΠΈ Β« 1 Β» Π² ΠΊΠΎΠ½Ρ†Π΅.

Как возвСсти Π² ΠΎΡ‚Ρ€ΠΈΡ†Π°Ρ‚Π΅Π»ΡŒΠ½ΡƒΡŽ ΡΡ‚Π΅ΠΏΠ΅Π½ΡŒ Π΄Ρ€ΠΎΠ±ΡŒ

Π§Ρ‚ΠΎΠ±Ρ‹ возвСсти Π΄Ρ€ΠΎΠ±ΡŒ Π² ΠΎΡ‚Ρ€ΠΈΡ†Π°Ρ‚Π΅Π»ΡŒΠ½ΡƒΡŽ ΡΡ‚Π΅ΠΏΠ΅Π½ΡŒ Π½ΡƒΠΆΠ½ΠΎ:

  • Β«ΠΏΠ΅Ρ€Π΅Π²Π΅Ρ€Π½ΡƒΡ‚ΡŒΒ» Π΄Ρ€ΠΎΠ±ΡŒ;
  • Π·Π°ΠΌΠ΅Π½ΠΈΡ‚ΡŒ ΠΎΡ‚Ρ€ΠΈΡ†Π°Ρ‚Π΅Π»ΡŒΠ½ΡƒΡŽ ΡΡ‚Π΅ΠΏΠ΅Π½ΡŒ Π½Π° ΠΏΠΎΠ»ΠΎΠΆΠΈΡ‚Π΅Π»ΡŒΠ½ΡƒΡŽ ;
  • возвСсти Π΄Ρ€ΠΎΠ±ΡŒ Π² ΠΏΠΎΠ»ΠΎΠΆΠΈΡ‚Π΅Π»ΡŒΠ½ΡƒΡŽ ΡΡ‚Π΅ΠΏΠ΅Π½ΡŒ.

ΠŸΡ€ΠΈΠΌΠ΅Ρ€. ВрСбуСтся возвСсти Π² ΠΎΡ‚Ρ€ΠΈΡ†Π°Ρ‚Π΅Π»ΡŒΠ½ΡƒΡŽ ΡΡ‚Π΅ΠΏΠ΅Π½ΡŒ Π΄Ρ€ΠΎΠ±ΡŒ.

(

) βˆ’3 =
ΠŸΠ΅Ρ€Π΅Π²Π΅Ρ€Π½Π΅ΠΌ Π΄Ρ€ΠΎΠ±ΡŒ Β«

Β» ΠΈ Π·Π°ΠΌΠ΅Π½ΠΈΠΌ ΠΎΡ‚Ρ€ΠΈΡ†Π°Ρ‚Π΅Π»ΡŒΠ½ΡƒΡŽ ΡΡ‚Π΅ΠΏΠ΅Π½ΡŒ Β« βˆ’3 Β» Π½Π° ΠΏΠΎΠ»ΠΎΠΆΠΈΡ‚Π΅Π»ΡŒΠ½ΡƒΡŽ Β« 3 Β».
(

) βˆ’3 = (

) 3

Π’ΠΎΠ·Π²Π΅Π΄Π΅ΠΌ Π΄Ρ€ΠΎΠ±ΡŒ Π² ΠΏΠΎΠ»ΠΎΠΆΠΈΡ‚Π΅Π»ΡŒΠ½ΡƒΡŽ ΡΡ‚Π΅ΠΏΠ΅Π½ΡŒ ΠΏΠΎ ΠΏΡ€Π°Π²ΠΈΠ»Ρƒ возвСдСния Π΄Ρ€ΠΎΠ±ΠΈ Π² ΠΏΠΎΠ»ΠΎΠΆΠΈΡ‚Π΅Π»ΡŒΠ½ΡƒΡŽ ΡΡ‚Π΅ΠΏΠ΅Π½ΡŒ. Π’.Π΅. Π²ΠΎΠ·Π²Π΅Π΄Π΅ΠΌ ΠΈ Ρ‡ΠΈΡΠ»ΠΈΡ‚Π΅Π»ΡŒ Β« 3 Β», ΠΈ Π·Π½Π°ΠΌΠ΅Π½Π°Ρ‚Π΅Π»ΡŒ Β« 10 Β» Π² Ρ‚Ρ€Π΅Ρ‚ΡŒΡŽ ΡΡ‚Π΅ΠΏΠ΅Π½ΡŒ.

(

) βˆ’3 = (

) 3 =

=

Для Π±ΠΎΠ»Π΅Π΅ Π³Ρ€Π°ΠΌΠΎΡ‚Π½ΠΎΠ³ΠΎ ΠΎΡ‚Π²Π΅Ρ‚Π° запишСм ΠΏΠΎΠ»ΡƒΡ‡Π΅Π½Π½Ρ‹ΠΉ Ρ€Π΅Π·ΡƒΠ»ΡŒΡ‚Π°Ρ‚ Π² Π²ΠΈΠ΄Π΅ дСсятичной Π΄Ρ€ΠΎΠ±ΠΈ.

(

) βˆ’3 = (

) 3 =

=

= 0,027

Как возвСсти ΠΎΡ‚Ρ€ΠΈΡ†Π°Ρ‚Π΅Π»ΡŒΠ½ΠΎΠ΅ число Π² ΠΎΡ‚Ρ€ΠΈΡ†Π°Ρ‚Π΅Π»ΡŒΠ½ΡƒΡŽ ΡΡ‚Π΅ΠΏΠ΅Π½ΡŒ

Как ΠΈ ΠΏΡ€ΠΈ Π²ΠΎΠ·Π²Π΅Π΄Π΅Π½ΠΈΠΈ ΠΎΡ‚Ρ€ΠΈΡ†Π°Ρ‚Π΅Π»ΡŒΠ½ΠΎΠ³ΠΎ числа Π² ΠΏΠΎΠ»ΠΎΠΆΠΈΡ‚Π΅Π»ΡŒΠ½ΡƒΡŽ ΡΡ‚Π΅ΠΏΠ΅Π½ΡŒ, Π² ΠΏΠ΅Ρ€Π²ΡƒΡŽ ΠΎΡ‡Π΅Ρ€Π΅Π΄ΡŒ Π½Π΅ΠΎΠ±Ρ…ΠΎΠ΄ΠΈΠΌΠΎ ΠΎΠΏΡ€Π΅Π΄Π΅Π»ΠΈΡ‚ΡŒ ΠΊΠΎΠ½Π΅Ρ‡Π½Ρ‹ΠΉ Π·Π½Π°ΠΊ Ρ€Π΅Π·ΡƒΠ»ΡŒΡ‚Π°Ρ‚Π° возвСдСния Π² ΡΡ‚Π΅ΠΏΠ΅Π½ΡŒ. Вспомним основныС ΠΏΡ€Π°Π²ΠΈΠ»Π° Π΅Ρ‰Π΅ Ρ€Π°Π·.

ΠžΡ‚Ρ€ΠΈΡ†Π°Ρ‚Π΅Π»ΡŒΠ½ΠΎΠ΅ число, Π²ΠΎΠ·Π²Π΅Π΄Ρ‘Π½Π½ΠΎΠ΅ Π² Ρ‡Ρ‘Ρ‚Π½ΡƒΡŽ ΡΡ‚Π΅ΠΏΠ΅Π½ΡŒ, β€” число ΠΏΠΎΠ»ΠΎΠΆΠΈΡ‚Π΅Π»ΡŒΠ½ΠΎΠ΅ .

ΠžΡ‚Ρ€ΠΈΡ†Π°Ρ‚Π΅Π»ΡŒΠ½ΠΎΠ΅ число, Π²ΠΎΠ·Π²Π΅Π΄Ρ‘Π½Π½ΠΎΠ΅ Π² Π½Π΅Ρ‡Ρ‘Ρ‚Π½ΡƒΡŽ ΡΡ‚Π΅ΠΏΠ΅Π½ΡŒ, β€” число ΠΎΡ‚Ρ€ΠΈΡ†Π°Ρ‚Π΅Π»ΡŒΠ½ΠΎΠ΅ .

ΠŸΠ΅Ρ€Π΅Π²Π΅Ρ€Π½Π΅ΠΌ число Β« βˆ’5 Β» ΠΈ Π·Π°ΠΌΠ΅Π½ΠΈΠΌ ΠΎΡ‚Ρ€ΠΈΡ†Π°Ρ‚Π΅Π»ΡŒΠ½ΡƒΡŽ ΡΡ‚Π΅ΠΏΠ΅Π½ΡŒ Β« βˆ’2 Β»
Π½Π° ΠΏΠΎΠ»ΠΎΠΆΠΈΡ‚Π΅Π»ΡŒΠ½ΡƒΡŽ Β« 2 Β».

(βˆ’5) βˆ’2 = (βˆ’

) 2 =

Π’Π°ΠΊ ΠΊΠ°ΠΊ ΡΡ‚Π΅ΠΏΠ΅Π½ΡŒ Β« 2 Β» β€” чСтная , Π·Π½Π°Ρ‡ΠΈΡ‚, Ρ€Π΅Π·ΡƒΠ»ΡŒΡ‚Π°Ρ‚ возвСдСния Π² ΡΡ‚Π΅ΠΏΠ΅Π½ΡŒ Π±ΡƒΠ΄Π΅Ρ‚ ΠΏΠΎΠ»ΠΎΠΆΠΈΡ‚Π΅Π»ΡŒΠ½Ρ‹ΠΉ . ΠŸΠΎΡΡ‚ΠΎΠΌΡƒ ΡƒΠ±ΠΈΡ€Π°Π΅ΠΌ Π·Π½Π°ΠΊ минуса ΠΏΡ€ΠΈ раскрытии скобок.

Π”Π°Π»Π΅Π΅ ΠΎΡ‚ΠΊΡ€ΠΎΠ΅ΠΌ скобки ΠΈ Π²ΠΎΠ·Π²Π΅Π΄Π΅ΠΌ Π²ΠΎ Π²Ρ‚ΠΎΡ€ΡƒΡŽ ΡΡ‚Π΅ΠΏΠ΅Π½ΡŒ ΠΈ Ρ‡ΠΈΡΠ»ΠΈΡ‚Π΅Π»ΡŒ Β« 1 Β»,
ΠΈ Π·Π½Π°ΠΌΠ΅Π½Π°Ρ‚Π΅Π»ΡŒ Β« 5 Β».

(βˆ’5) βˆ’2 = (βˆ’

) 2 =

=

Как возвСсти ΠΎΡ‚Ρ€ΠΈΡ†Π°Ρ‚Π΅Π»ΡŒΠ½ΡƒΡŽ Π΄Ρ€ΠΎΠ±ΡŒ Π² ΠΎΡ‚Ρ€ΠΈΡ†Π°Ρ‚Π΅Π»ΡŒΠ½ΡƒΡŽ ΡΡ‚Π΅ΠΏΠ΅Π½ΡŒ

ΠšΠΎΠ½Π΅Ρ‡Π½Ρ‹ΠΉ Π·Π½Π°ΠΊ Ρ€Π΅Π·ΡƒΠ»ΡŒΡ‚Π°Ρ‚Π° возвСдСния Π² ΡΡ‚Π΅ΠΏΠ΅Π½ΡŒ ΠΎΡ‚Ρ€ΠΈΡ†Π°Ρ‚Π΅Π»ΡŒΠ½ΠΎΠΉ Π΄Ρ€ΠΎΠ±ΠΈ опрСдСляСтся ΠΏΠΎ Ρ‚Π΅ΠΌ ΠΆΠ΅ ΠΏΡ€Π°Π²ΠΈΠ»Π°ΠΌ, Ρ‡Ρ‚ΠΎ ΠΈ для Ρ†Π΅Π»ΠΎΠ³ΠΎ ΠΎΡ‚Ρ€ΠΈΡ†Π°Ρ‚Π΅Π»ΡŒΠ½ΠΎΠ³ΠΎ числа.

ΠžΡ‚Ρ€ΠΈΡ†Π°Ρ‚Π΅Π»ΡŒΠ½Π°Ρ Π΄Ρ€ΠΎΠ±ΡŒ, возвСдённая Π² Ρ‡Ρ‘Ρ‚Π½ΡƒΡŽ ΡΡ‚Π΅ΠΏΠ΅Π½ΡŒ, β€” Π΄Ρ€ΠΎΠ±ΡŒ ΠΏΠΎΠ»ΠΎΠΆΠΈΡ‚Π΅Π»ΡŒΠ½Π°Ρ .

ΠžΡ‚Ρ€ΠΈΡ†Π°Ρ‚Π΅Π»ΡŒΠ½Π°Ρ Π΄Ρ€ΠΎΠ±ΡŒ, возвСдённая Π² Π½Π΅Ρ‡Ρ‘Ρ‚Π½ΡƒΡŽ ΡΡ‚Π΅ΠΏΠ΅Π½ΡŒ, β€” Π΄Ρ€ΠΎΠ±ΡŒ ΠΎΡ‚Ρ€ΠΈΡ†Π°Ρ‚Π΅Π»ΡŒΠ½Π°Ρ .

РазбСрСмся Π½Π° ΠΏΡ€ΠΈΠΌΠ΅Ρ€Π΅. Π—Π°Π΄Π°Π½ΠΈΠ΅: возвСсти ΠΎΡ‚Ρ€ΠΈΡ†Π°Ρ‚Π΅Π»ΡŒΠ½ΡƒΡŽ Π΄Ρ€ΠΎΠ±ΡŒ Β« (βˆ’

) Β» Π² Β« βˆ’3 Β» ΡΡ‚Π΅ΠΏΠ΅Π½ΡŒ.

По ΠΏΡ€Π°Π²ΠΈΠ»Ρƒ возвСдСния Π΄Ρ€ΠΎΠ±ΠΈ Π² ΠΎΡ‚Ρ€ΠΈΡ†Π°Ρ‚Π΅Π»ΡŒΠ½ΡƒΡŽ ΡΡ‚Π΅ΠΏΠ΅Π½ΡŒ ΠΏΠ΅Ρ€Π΅Π²Π΅Ρ€Π½Π΅ΠΌ Π΄Ρ€ΠΎΠ±ΡŒ ΠΈ Π·Π°ΠΌΠ΅Π½ΠΈΠΌ ΠΎΡ‚Ρ€ΠΈΡ†Π°Ρ‚Π΅Π»ΡŒΠ½ΡƒΡŽ ΡΡ‚Π΅ΠΏΠ΅Π½ΡŒ Β« βˆ’3 Β» Π½Π° ΠΏΠΎΠ»ΠΎΠΆΠΈΡ‚Π΅Π»ΡŒΠ½ΡƒΡŽ Β« 3 Β».

(βˆ’

) βˆ’3 = (βˆ’

) 3 =

Π’Π΅ΠΏΠ΅Ρ€ΡŒ ΠΎΠΏΡ€Π΅Π΄Π΅Π»ΠΈΠΌ ΠΊΠΎΠ½Π΅Ρ‡Π½Ρ‹ΠΉ Π·Π½Π°ΠΊ Ρ€Π΅Π·ΡƒΠ»ΡŒΡ‚Π°Ρ‚Π° возвСдСния Π² Β« 3 Β» ΡΡ‚Π΅ΠΏΠ΅Π½ΡŒ.

Π‘Ρ‚Π΅ΠΏΠ΅Π½ΡŒ Β« 3 Β» β€” нСчСтная , Π·Π½Π°Ρ‡ΠΈΡ‚, ΠΏΠΎ ΠΏΡ€Π°Π²ΠΈΠ»Ρƒ возвСдСния ΠΎΡ‚Ρ€ΠΈΡ†Π°Ρ‚Π΅Π»ΡŒΠ½ΠΎΠ³ΠΎ числа Π² ΡΡ‚Π΅ΠΏΠ΅Π½ΡŒ Π΄Ρ€ΠΎΠ±ΡŒ останСтся ΠΎΡ‚Ρ€ΠΈΡ†Π°Ρ‚Π΅Π»ΡŒΠ½ΠΎΠΉ .

Нам остаСтся Ρ‚ΠΎΠ»ΡŒΠΊΠΎ Ρ€Π°ΡΠΊΡ€Ρ‹Ρ‚ΡŒ скобки ΠΈ возвСсти Π² ΡΡ‚Π΅ΠΏΠ΅Π½ΡŒ ΠΈ Ρ‡ΠΈΡΠ»ΠΈΡ‚Π΅Π»ΡŒ Β« 3 Β», ΠΈ Π·Π½Π°ΠΌΠ΅Π½Π°Ρ‚Π΅Π»ΡŒ Β« 2 Β» Π² Ρ‚Ρ€Π΅Ρ‚ΡŒΡŽ ΡΡ‚Π΅ΠΏΠ΅Π½ΡŒ.

(βˆ’

) βˆ’3 = (βˆ’

) 3 = βˆ’

= βˆ’

Для ΠΎΠΊΠΎΠ½Ρ‡Π°Ρ‚Π΅Π»ΡŒΠ½ΠΎΠ³ΠΎ ΠΎΡ‚Π²Π΅Ρ‚Π° Π²Ρ‹Π΄Π΅Π»ΠΈΠΌ Ρ†Π΅Π»ΡƒΡŽ Ρ‡Π°ΡΡ‚ΡŒ ΠΈΠ· Π΄Ρ€ΠΎΠ±ΠΈ.

(βˆ’

) βˆ’3 = (βˆ’

) 3 = βˆ’

= βˆ’

= βˆ’ 3

Рассмотрим Π΄Ρ€ΡƒΠ³ΠΎΠΉ ΠΏΡ€ΠΈΠΌΠ΅Ρ€ возвСдСния ΠΎΡ‚Ρ€ΠΈΡ†Π°Ρ‚Π΅Π»ΡŒΠ½ΠΎΠΉ Π΄Ρ€ΠΎΠ±ΠΈ Π² ΠΎΡ‚Ρ€ΠΈΡ†Π°Ρ‚Π΅Π»ΡŒΠ½ΡƒΡŽ ΡΡ‚Π΅ΠΏΠ΅Π½ΡŒ.

ΠŸΡ€Π°Π²ΠΈΠ»ΠΎ возвСдСния ΠΎΡ‚Ρ€ΠΈΡ†Π°Ρ‚Π΅Π»ΡŒΠ½ΠΎΠ³ΠΎ числа Π² ΡΡ‚Π΅ΠΏΠ΅Π½ΡŒ гласит: Ссли ΡΡ‚Π΅ΠΏΠ΅Π½ΡŒ чСтная , Π·Π½Π°Ρ‡ΠΈΡ‚, Ρ€Π΅Π·ΡƒΠ»ΡŒΡ‚Π°Ρ‚ возвСдСния Π±ΡƒΠ΄Π΅Ρ‚ ΠΏΠΎΠ»ΠΎΠΆΠΈΡ‚Π΅Π»ΡŒΠ½Ρ‹ΠΌ .

(βˆ’

) βˆ’2 = (βˆ’

) 2 =

=

= 1

Бвойства ΠΎΡ‚Ρ€ΠΈΡ†Π°Ρ‚Π΅Π»ΡŒΠ½ΠΎΠΉ стСпСни

ВсС свойства стСпСни, ΠΊΠΎΡ‚ΠΎΡ€Ρ‹Π΅ ΠΈΡΠΏΠΎΠ»ΡŒΠ·ΡƒΡŽΡ‚ΡΡ для ΠΏΠΎΠ»ΠΎΠΆΠΈΡ‚Π΅Π»ΡŒΠ½ΠΎΠΉ стСпСни, Ρ‚ΠΎΡ‡Π½ΠΎ Ρ‚Π°ΠΊΠΆΠ΅ ΠΏΡ€ΠΈΠΌΠ΅Π½ΡΡŽΡ‚ΡΡ ΠΈ для ΠΎΡ‚Ρ€ΠΈΡ†Π°Ρ‚Π΅Π»ΡŒΠ½ΠΎΠΉ стСпСни.

Π’ этом ΡƒΡ€ΠΎΠΊΠ΅ ΠΌΡ‹ Π½Π΅ Π±ΡƒΠ΄Π΅ΠΌ ΠΏΠΎΠ²Ρ‚ΠΎΡ€Π½ΠΎ ΠΏΠΎΠ΄Ρ€ΠΎΠ±Π½ΠΎ Ρ€Π°Π·Π±ΠΈΡ€Π°Ρ‚ΡŒ ΠΊΠ°ΠΆΠ΄ΠΎΠ΅ свойство стСпСни, Π½ΠΎ Π΅Ρ‰Π΅ Ρ€Π°Π· ΠΏΡ€ΠΈΠ²Π΅Π΄Π΅ΠΌ основныС Ρ„ΠΎΡ€ΠΌΡƒΠ»Ρ‹ свойств стСпСни ΠΈ ΠΏΠΎΠΊΠ°ΠΆΠ΅ΠΌ ΠΏΡ€ΠΈΠΌΠ΅Ρ€Ρ‹ ΠΈΡ… использования.

Π—Π°ΠΏΠΎΠΌΠ½ΠΈΡ‚Π΅!

  • a m Β· a n = a m + n
  • = a m βˆ’ n

  • (a n ) m = a n Β· m
  • (a Β· b) n = a n Β· b n
ΠŸΡ€ΠΈΠΌΠ΅Ρ€Ρ‹ Ρ€Π΅ΡˆΠ΅Π½ΠΈΠΉ Π·Π°Π΄Π°Π½ΠΈΠΉ с ΠΎΡ‚Ρ€ΠΈΡ†Π°Ρ‚Π΅Π»ΡŒΠ½ΠΎΠΉ

ΡΡ‚Π΅ΠΏΠ΅Π½ΡŒΡŽ

Колягин 9 класс. Π—Π°Π΄Π°Π½ΠΈΠ΅ β„– 1

ΠŸΡ€Π΅Π΄ΡΡ‚Π°Π²ΠΈΡ‚ΡŒ Π² Π²ΠΈΠ΄Π΅ стСпСни.

2) a 6 Β· b 6 = (ab) 6

Колягин 9 класс. Π—Π°Π΄Π°Π½ΠΈΠ΅ β„– 5

Π—Π°ΠΏΠΈΡΠ°Ρ‚ΡŒ Π² Π²ΠΈΠ΄Π΅ стСпСни с ΠΎΡ‚Ρ€ΠΈΡ†Π°Ρ‚Π΅Π»ΡŒΠ½Ρ‹ΠΌ числом.

Π§Ρ‚ΠΎ Ρ‚Π°ΠΊΠΎΠ΅ ΡΡ‚Π΅ΠΏΠ΅Π½ΡŒ с ΠΎΡ‚Ρ€ΠΈΡ†Π°Ρ‚Π΅Π»ΡŒΠ½Ρ‹ΠΌ ΠΏΠΎΠΊΠ°Π·Π°Ρ‚Π΅Π»Π΅ΠΌ (ΠΎΡ‚Ρ€ΠΈΡ†Π°Ρ‚Π΅Π»ΡŒΠ½Π°Ρ ΡΡ‚Π΅ΠΏΠ΅Π½ΡŒ)? Как Π²Ρ‹ΠΏΠΎΠ»Π½ΠΈΡ‚ΡŒ Π²ΠΎΠ·Π²Π΅Π΄Π΅Π½ΠΈΠ΅ числа Π² ΠΎΡ‚Ρ€ΠΈΡ†Π°Ρ‚Π΅Π»ΡŒΠ½ΡƒΡŽ ΡΡ‚Π΅ΠΏΠ΅Π½ΡŒ? Как возвСсти Π² ΠΎΡ‚Ρ€ΠΈΡ†Π°Ρ‚Π΅Π»ΡŒΠ½ΡƒΡŽ ΡΡ‚Π΅ΠΏΠ΅Π½ΡŒ Π΄Ρ€ΠΎΠ±ΡŒ?

Π’ частности, число Π² стСпСни минус ΠΎΠ΄ΠΈΠ½ β€” это число, ΠΎΠ±Ρ€Π°Ρ‚Π½ΠΎΠ΅ Π΄Π°Π½Π½ΠΎΠΌΡƒ:

Если n β€” Ρ†Π΅Π»ΠΎΠ΅ число, Ρ‚ΠΎ Ρ€Π΅Ρ‡ΡŒ ΠΈΠ΄Π΅Ρ‚ ΠΎ стСпСни с Ρ†Π΅Π»Ρ‹ΠΌ ΠΎΡ‚Ρ€ΠΈΡ†Π°Ρ‚Π΅Π»ΡŒΠ½Ρ‹ΠΌ ΠΏΠΎΠΊΠ°Π·Π°Ρ‚Π΅Π»Π΅ΠΌ ΠΈ равСнство Π²Π΅Ρ€Π½ΠΎ для любого a, ΠΎΡ‚Π»ΠΈΡ‡Π½ΠΎΠ³ΠΎ ΠΎΡ‚ нуля (Ρ‚.Π΅. ΠΏΡ€ΠΈ aβ‰ 0).

Если n β€” Π΄Ρ€ΠΎΠ±Π½ΠΎΠ΅ число, Ρ‚ΠΎ Ρ€Π΅Ρ‡ΡŒ ΠΈΠ΄Π΅Ρ‚ ΠΎ стСпСни с Ρ€Π°Ρ†ΠΈΠΎΠ½Π°Π»ΡŒΠ½Ρ‹ΠΌ ΠΏΠΎΠΊΠ°Π·Π°Ρ‚Π΅Π»Π΅ΠΌ:

(m β€” Ρ†Π΅Π»ΠΎΠ΅ число, n β€” Π½Π°Ρ‚ΡƒΡ€Π°Π»ΡŒΠ½ΠΎΠ΅ число). Π‘Ρ‚Π΅ΠΏΠ΅Π½ΡŒ с Π΄Ρ€ΠΎΠ±Π½Ρ‹ΠΌ ΠΏΠΎΠΊΠ°Π·Π°Ρ‚Π΅Π»Π΅ΠΌ ΠΎΠΏΡ€Π΅Π΄Π΅Π»Π΅Π½Π° Ρ‚ΠΎΠ»ΡŒΠΊΠΎ для ΠΏΠΎΠ»ΠΎΠΆΠΈΡ‚Π΅Π»ΡŒΠ½Ρ‹Ρ… a (a>0).

Π”Ρ€ΠΎΠ±ΡŒ Π² стСпСни с ΠΎΡ‚Ρ€ΠΈΡ†Π°Ρ‚Π΅Π»ΡŒΠ½Ρ‹ΠΌ ΠΏΠΎΠΊΠ°Π·Π°Ρ‚Π΅Π»Π΅ΠΌ Ρ€Π°Π²Π½Π° ΠΎΠ±Ρ€Π°Ρ‚Π½ΠΎΠΌΡƒ этой Π΄Ρ€ΠΎΠ±ΠΈ числу Π² стСпСни с ΠΏΠΎΠΊΠ°Π·Π°Ρ‚Π΅Π»Π΅ΠΌ, ΠΏΡ€ΠΎΡ‚ΠΈΠ²ΠΎΠΏΠΎΠ»ΠΎΠΆΠ½Ρ‹ΠΌ Π΄Π°Π½Π½ΠΎΠΌΡƒ:

Π”Ρ€ΡƒΠ³ΠΈΠΌΠΈ словами, Ρ‡Ρ‚ΠΎΠ±Ρ‹ возвСсти Π΄Ρ€ΠΎΠ±ΡŒ Π² ΠΎΡ‚Ρ€ΠΈΡ†Π°Ρ‚Π΅Π»ΡŒΠ½ΡƒΡŽ ΡΡ‚Π΅ΠΏΠ΅Π½ΡŒ, Π½Π°Π΄ΠΎ эту Π΄Ρ€ΠΎΠ±ΡŒ Β«ΠΏΠ΅Ρ€Π΅Π²Π΅Ρ€Π½ΡƒΡ‚ΡŒΒ»(Ρ‡ΠΈΡΠ»ΠΈΡ‚Π΅Π»ΡŒ ΠΈ Π·Π½Π°ΠΌΠ΅Π½Π°Ρ‚Π΅Π»ΡŒ ΠΏΠΎΠΌΠ΅Π½ΡΡ‚ΡŒ мСстами) ΠΈ ΠΈΠ·ΠΌΠ΅Π½ΠΈΡ‚ΡŒ Π·Π½Π°ΠΊ Π² ΠΏΠΎΠΊΠ°Π·Π°Ρ‚Π΅Π»Π΅ стСпСни.

Π”Ρ€ΠΎΠ±ΡŒ Π² минус ΠΏΠ΅Ρ€Π²ΠΎΠΉ стСпСни β€” это «пСрСвСрнутая» Π΄Ρ€ΠΎΠ±ΡŒ.

Рассмотрим ΠΏΡ€ΠΈΠΌΠ΅Ρ€Ρ‹ возвСдСния чисСл Π² ΡΡ‚Π΅ΠΏΠ΅Π½ΡŒ с ΠΎΡ‚Ρ€ΠΈΡ†Π°Ρ‚Π΅Π»ΡŒΠ½Ρ‹ΠΌ ΠΏΠΎΠΊΠ°Π·Π°Ρ‚Π΅Π»Π΅ΠΌ.

Для ускорСния вычислСний ΠΈΡΠΏΠΎΠ»ΡŒΠ·ΡƒΠ΅ΠΌ Ρ‚Π°Π±Π»ΠΈΡ†Ρƒ стСпСнСй.

Π§Ρ‚ΠΎΠ±Ρ‹ возвСсти Π² ΠΎΡ‚Ρ€ΠΈΡ†Π°Ρ‚Π΅Π»ΡŒΠ½ΡƒΡŽ ΡΡ‚Π΅ΠΏΠ΅Π½ΡŒ смСшанноС число, Π½Π°Π΄ΠΎ сначала пСрСвСсти Π΅Π³ΠΎ Π² Π½Π΅ΠΏΡ€Π°Π²ΠΈΠ»ΡŒΠ½ΡƒΡŽ Π΄Ρ€ΠΎΠ±ΡŒ:

Π’ΠΎΠ·Π²Π΅Π΄Π΅ΠΌ числа Π² ΡΡ‚Π΅ΠΏΠ΅Π½ΡŒ с Π΄Ρ€ΠΎΠ±Π½Ρ‹ΠΌ ΠΎΡ‚Ρ€ΠΈΡ†Π°Ρ‚Π΅Π»ΡŒΠ½Ρ‹ΠΌ ΠΏΠΎΠΊΠ°Π·Π°Ρ‚Π΅Π»Π΅ΠΌ:

ΠŸΡ€ΠΈ Π²ΠΎΠ·Π²Π΅Π΄Π΅Π½ΠΈΠΈ Π² ΠΎΡ‚Ρ€ΠΈΡ†Π°Ρ‚Π΅Π»ΡŒΠ½ΡƒΡŽ ΡΡ‚Π΅ΠΏΠ΅Π½ΡŒ дСсятичной Π΄Ρ€ΠΎΠ±ΠΈ ΠΌΠΎΠΆΠ½ΠΎ сначала пСрСвСсти Π΅Π΅ Π² ΠΎΠ±Ρ‹ΠΊΠ½ΠΎΠ²Π΅Π½Π½ΡƒΡŽ ΠΈ, Ссли Π²ΠΎΠ·ΠΌΠΎΠΆΠ½ΠΎ, ΡΠΎΠΊΡ€Π°Ρ‚ΠΈΡ‚ΡŒ:

Если Π² ΠΏΠΎΠΊΠ°Π·Π°Ρ‚Π΅Π»Π΅ стСпСни стоит дСсятичная Π΄Ρ€ΠΎΠ±ΡŒ, Π½ΡƒΠΆΠ½ΠΎ пСрСвСсти Π΅Π΅ Π² ΠΎΠ±Ρ‹ΠΊΠ½ΠΎΠ²Π΅Π½Π½ΡƒΡŽ:

Π’ΠΎΠ·Π²Π΅Π΄Π΅Π½ΠΈΠ΅ Π² ΡΡ‚Π΅ΠΏΠ΅Π½ΡŒ с ΠΎΡ‚Ρ€ΠΈΡ†Π°Ρ‚Π΅Π»ΡŒΠ½Ρ‹ΠΌ ΠΏΠΎΠΊΠ°Π·Π°Ρ‚Π΅Π»Π΅ΠΌ Π² Π°Π»Π³Π΅Π±Ρ€Π΅ встрСчаСтся достаточно часто, поэтому Π²Π°ΠΆΠ½ΠΎ воврСмя ΡƒΡΠ²ΠΎΠΈΡ‚ΡŒ эту Ρ‚Π΅ΠΌΡƒ.

13 ΠΊΠΎΠΌΠΌΠ΅Π½Ρ‚Π°Ρ€ΠΈΠ΅Π²

Бпасибо! врубился) Таль, Ρ‡Ρ‚ΠΎ Π² школС Π½Π΅ учился(

Π§Ρ‚ΠΎ ΠΆ, ΡƒΡ‡ΠΈΡ‚ΡŒΡΡ Π½ΠΈΠΊΠΎΠ³Π΄Π° Π½Π΅ ΠΏΠΎΠ·Π΄Π½ΠΎ). Но всё ΠΆΠ΅ Π»ΡƒΡ‡ΡˆΠ΅ воврСмя.

Π—Π°Π±Π°Π²Π½ΠΎ, Ρ‡Ρ‚ΠΎ Π·Π° врСмя Ρ€Π°Π±ΠΎΡ‚Ρ‹ встрСчал мноТСство ΠΊΠΎΠ»Π»Π΅Π³, ΠΊΠΎΠΌΡƒ ΠΏΡ€ΠΈΡ…ΠΎΠ΄ΠΈΠ»ΠΎΡΡŒ Π½Π° Π²Π½ΡƒΡ‚Ρ€Π΅Π½Π½ΠΈΡ… курсах Ρ€Π°Π·ΠΆΡ‘Π²Ρ‹Π²Π°Ρ‚ΡŒ ΠΊΠ°ΠΊΠΈΠ΅ Π²Π΅Ρ‰ΠΈ Π½Π°Ρ‡Π°Π»ΡŒΠ½ΠΎΠ³ΠΎ уровня ΠΈ всС ΡΠΎΠΊΡ€ΡƒΡˆΠ°Π»ΠΈΡΡŒ: Β«Π§Ρ‚ΠΎ ΠΆΠ΅ я Π² школС-Ρ‚ΠΎ (институтС) Π½Π΅ ΡƒΡ‡ΠΈΠ» это? Π­Ρ‚ΠΎ ΠΆΠ΅ Ρ‚Π°ΠΊ просто, понятно, ΠΏΠΎΠ»Π΅Π·Π½ΠΎ ΠΈ Π˜ΠΠ’Π•Π Π•Π‘ΠΠž. Β»

А вся ΠΏΡ€ΠΎΠ±Π»Π΅ΠΌΠ° Π² Ρ‚ΠΎΠΌ, Ρ‡Ρ‚ΠΎ Π½ΠΈ Π² школС, Π½ΠΈ Π² институтС ΠΏΠ΅Ρ€Π΅Π΄ Ρ‚Π΅ΠΌ, ΠΊΠ°ΠΊ Ρ‡Ρ‚ΠΎ-Ρ‚ΠΎ Π½Π°Ρ‡Π°Ρ‚ΡŒ Ρ€Π°ΡΡΠΊΠ°Π·Ρ‹Π²Π°Ρ‚ΡŒ Π½Π΅ проводят красочныС, Π·Π°Π²Π»Π΅ΠΊΠ°Ρ‚Π΅Π»ΡŒΠ½Ρ‹Π΅, ΠΏΠΎΠ·Π½Π°Π²Π°Ρ‚Π΅Π»ΡŒΠ½Ρ‹Π΅, вСсёлыС ΠΈ ΠΈΠ³Ρ€ΠΎΠ²Ρ‹Π΅ ΠΏΡ€Π΅Π·Π΅Π½Ρ‚Π°Ρ†ΠΈΠΈ Π±ΡƒΠ΄ΡƒΡ‰Π΅Π³ΠΎ курса, Ρ‡Ρ‚ΠΎΠ±Ρ‹ Π±Ρ‹Π»ΠΎ понятно, Π° Π³Π΄Π΅ ΠΆΠ΅ Ρ‚ΠΎ, Ρ‡Ρ‚ΠΎ Π±ΡƒΠ΄Π΅ΠΌ скоро ΠΈΠ·ΡƒΡ‡Π°Ρ‚ΡŒ, примСняСтся Π² ΠΆΠΈΠ·Π½ΠΈ? Каким профСссиям ΠΈ Π² ΠΊΠ°ΠΊΠΈΡ… ТитСйских ситуациях это ΠΌΠΎΠΆΠ΅Ρ‚ Π±Ρ‹Ρ‚ΡŒ ΠΏΠΎΠ»Π΅Π·Π½ΠΎ?

Π£Ρ‡Π°Ρ‚ ΠΊΠ°ΠΊΠΈΠΌ-Ρ‚ΠΎ абстрактным Ρ„ΠΎΡ€ΠΌΡƒΠ»Π°ΠΌ вмСсто Ρ‚ΠΎΠ³ΠΎ, Ρ‡Ρ‚ΠΎΠ±Ρ‹ Ρ€Π°ΡΡΠΊΠ°Π·Π°Ρ‚ΡŒ, Ρ‡Ρ‚ΠΎ это пригодится Π½Π° ΠΊΡƒΡ…Π½Π΅, ΠΏΡ€ΠΈ Ρ€Π°Π·Π΄Π΅Π»Π΅ Π·Π΅ΠΌΠ»ΠΈ, ΠΏΡ€ΠΈ ΡΡ‚Ρ€ΠΎΠΈΡ‚Π΅Π»ΡŒΡΡ‚Π²Π΅ сарая Π½Π° Π΄Π°Ρ‡Π΅, ΠΏΡ€ΠΈ ΡΡ‚Ρ€Π΅Π»ΡŒΠ±Π΅ ΠΈΠ· ΠΏΡƒΡˆΠΊΠΈ, ΠΏΡ€ΠΈ запускС спутника ΠΈ Ρ‚. Π΄.

ΠŸΡ€ΠΈ Ρ€Π°Π·Π±Π°Π²Π»Π΅Π½ΠΈΠΈ спирта Π²ΠΎΠ΄ΠΎΠΉ, Π² ΠΊΠΎΠ½Ρ†Π΅ ΠΊΠΎΠ½Ρ†ΠΎΠ²! :))

Π’Π΅Π΄ΡŒ часто ΠΆΠ΅Π½Ρ‰ΠΈΠ½Ρ‹ Π²ΡΡ‚Π°ΡŽΡ‚ Π² ступор ΠΎΡ‚ элСмСнтарной Π·Π°Π΄Π°Ρ‡ΠΈ:

Π’ Ρ€Π΅Ρ†Π΅ΠΏΡ‚Π΅ ΡƒΠΊΠ°Π·Π°Π½ΠΎ Β«1 ст. Π»ΠΎΠΆΠΊΠ° 3 %-Π³ΠΎ уксуса», Π° Ρƒ Π½Π΅Ρ‘ Π½Π° ΠΊΡƒΡ…Π½Π΅ Ρ‚ΠΎΠ»ΡŒΠΊΠΎ 9 % ΠΈΠ»ΠΈ («О, Π‘ΠžΠ–Π•! ΠšΡ€Π°Ρ…! ΠŸΡ€ΠΎΠ²Π°Π»!Β») Π²ΠΎΠΎΠ±Ρ‰Π΅ уксусная эссСнция! А ΠΏΠΎ сути Ρ‚Π° ΠΆΠ΅ кислота, Π½ΠΎ Π² ΠΊΠΎΠ½Ρ†Π΅Π½Ρ‚Ρ€Π°Ρ†ΠΈΠΈ 70 %…

Π’ΠΎΠ·Π²Π΅Π΄Π΅Π½ΠΈΠ΅ Π² ΠΎΡ‚Ρ€ΠΈΡ†Π°Ρ‚Π΅Π»ΡŒΠ½ΡƒΡŽ ΡΡ‚Π΅ΠΏΠ΅Π½ΡŒ – ΠΎΠ΄ΠΈΠ½ ΠΈΠ· основных элСмСнтов ΠΌΠ°Ρ‚Π΅ΠΌΠ°Ρ‚ΠΈΠΊΠΈ, ΠΊΠΎΡ‚ΠΎΡ€Ρ‹ΠΉ часто встрСчаСтся ΠΏΡ€ΠΈ Ρ€Π΅ΡˆΠ΅Π½ΠΈΠΈ алгСбраичСских Π·Π°Π΄Π°Ρ‡. НиТС ΠΏΡ€ΠΈΠ²Π΅Π΄Π΅Π½Π° подробная инструкция.

Как Π²ΠΎΠ·Π²ΠΎΠ΄ΠΈΡ‚ΡŒ Π² ΠΎΡ‚Ρ€ΠΈΡ†Π°Ρ‚Π΅Π»ΡŒΠ½ΡƒΡŽ ΡΡ‚Π΅ΠΏΠ΅Π½ΡŒ – тСория

Когда ΠΌΡ‹ число Π² ΠΎΠ±Ρ‹Ρ‡Π½ΡƒΡŽ ΡΡ‚Π΅ΠΏΠ΅Π½ΡŒ, ΠΌΡ‹ ΡƒΠΌΠ½ΠΎΠΆΠ°Π΅ΠΌ Π΅Π³ΠΎ Π·Π½Π°Ρ‡Π΅Π½ΠΈΠ΅ нСсколько Ρ€Π°Π·. НапримСр, 3 3 = 3Γ—3Γ—3 = 27. Π‘ ΠΎΡ‚Ρ€ΠΈΡ†Π°Ρ‚Π΅Π»ΡŒΠ½ΠΎΠΉ Π΄Ρ€ΠΎΠ±ΡŒΡŽ всС Π½Π°ΠΎΠ±ΠΎΡ€ΠΎΡ‚. ΠžΠ±Ρ‰ΠΈΠΉ Π²ΠΈΠ΄ ΠΏΠΎ Ρ„ΠΎΡ€ΠΌΡƒΠ»Π΅ Π±ΡƒΠ΄Π΅Ρ‚ ΠΈΠΌΠ΅Ρ‚ΡŒ ΡΠ»Π΅Π΄ΡƒΡŽΡ‰ΠΈΠΉ Π²ΠΈΠ΄: a -n = 1/a n . Π’Π°ΠΊΠΈΠΌ ΠΎΠ±Ρ€Π°Π·ΠΎΠΌ, Ρ‡Ρ‚ΠΎΠ±Ρ‹ возвСсти число Π² ΠΎΡ‚Ρ€ΠΈΡ†Π°Ρ‚Π΅Π»ΡŒΠ½ΡƒΡŽ ΡΡ‚Π΅ΠΏΠ΅Π½ΡŒ, Π½ΡƒΠΆΠ½ΠΎ Π΅Π΄ΠΈΠ½ΠΈΡ†Ρƒ ΠΏΠΎΠ΄Π΅Π»ΠΈΡ‚ΡŒ Π½Π° Π΄Π°Π½Π½ΠΎΠ΅ число, Π½ΠΎ ΡƒΠΆΠ΅ Π² ΠΏΠΎΠ»ΠΎΠΆΠΈΡ‚Π΅Π»ΡŒΠ½ΠΎΠΉ стСпСни.

Как Π²ΠΎΠ·Π²ΠΎΠ΄ΠΈΡ‚ΡŒ Π² ΠΎΡ‚Ρ€ΠΈΡ†Π°Ρ‚Π΅Π»ΡŒΠ½ΡƒΡŽ ΡΡ‚Π΅ΠΏΠ΅Π½ΡŒ – ΠΏΡ€ΠΈΠΌΠ΅Ρ€Ρ‹ Π½Π° ΠΎΠ±Ρ‹Ρ‡Π½Ρ‹Ρ… числах

Π”Π΅Ρ€ΠΆΠ° Π²Ρ‹ΡˆΠ΅ΠΏΡ€ΠΈΠ²Π΅Π΄Π΅Π½Π½ΠΎΠ΅ ΠΏΡ€Π°Π²ΠΈΠ»ΠΎ Π½Π° ΡƒΠΌΠ΅, Ρ€Π΅ΡˆΠΈΠΌ нСсколько ΠΏΡ€ΠΈΠΌΠ΅Ρ€ΠΎΠ².

4 -2 = 1/4 2 = 1/16
ΠžΡ‚Π²Π΅Ρ‚: 4 -2 = 1/16

4 -2 = 1/-4 2 = 1/16.
ΠžΡ‚Π²Π΅Ρ‚ -4 -2 = 1/16.

Но ΠΏΠΎΡ‡Π΅ΠΌΡƒ ΠΎΡ‚Π²Π΅Ρ‚ Π² ΠΏΠ΅Ρ€Π²ΠΎΠΌ ΠΈ Π²Ρ‚ΠΎΡ€ΠΎΠΌ ΠΏΡ€ΠΈΠΌΠ΅Ρ€Π°Ρ… ΠΎΠ΄ΠΈΠ½Π°ΠΊΠΎΠ²Ρ‹ΠΉ? Π”Π΅Π»ΠΎ Π² Ρ‚ΠΎΠΌ, Ρ‡Ρ‚ΠΎ ΠΏΡ€ΠΈ Π²ΠΎΠ·Π²Π΅Π΄Π΅Π½ΠΈΠΈ ΠΎΡ‚Ρ€ΠΈΡ†Π°Ρ‚Π΅Π»ΡŒΠ½ΠΎΠ³ΠΎ числа Π² Ρ‡Π΅Ρ‚Π½ΡƒΡŽ ΡΡ‚Π΅ΠΏΠ΅Π½ΡŒ (2, 4, 6 ΠΈ Ρ‚. Π΄.), Π·Π½Π°ΠΊ становится ΠΏΠΎΠ»ΠΎΠΆΠΈΡ‚Π΅Π»ΡŒΠ½Ρ‹ΠΌ. Если Π±Ρ‹ ΡΡ‚Π΅ΠΏΠ΅Π½ΡŒ Π±Ρ‹Π»Π° Ρ‡Π΅Ρ‚Π½ΠΎΠΉ, Ρ‚ΠΎ минус сохранился:

Как Π²ΠΎΠ·Π²ΠΎΠ΄ΠΈΡ‚ΡŒ Π² ΠΎΡ‚Ρ€ΠΈΡ†Π°Ρ‚Π΅Π»ΡŒΠ½ΡƒΡŽ ΡΡ‚Π΅ΠΏΠ΅Π½ΡŒ – числа ΠΎΡ‚ 0 Π΄ΠΎ 1

Вспомним, Ρ‡Ρ‚ΠΎ ΠΏΡ€ΠΈ Π²ΠΎΠ·Π²Π΅Π΄Π΅Π½ΠΈΠΈ числа Π² ΠΏΡ€ΠΎΠΌΠ΅ΠΆΡƒΡ‚ΠΊΠ΅ ΠΎΡ‚ 0 Π΄ΠΎ 1 Π² ΠΏΠΎΠ»ΠΎΠΆΠΈΡ‚Π΅Π»ΡŒΠ½ΡƒΡŽ ΡΡ‚Π΅ΠΏΠ΅Π½ΡŒ, Π·Π½Π°Ρ‡Π΅Π½ΠΈΠ΅ ΡƒΠΌΠ΅Π½ΡŒΡˆΠ°Π΅Ρ‚ΡΡ с возрастаниСм стСпСни. Π’Π°ΠΊ Π½Π°ΠΏΡ€ΠΈΠΌΠ΅Ρ€, 0,5 2 = 0,25. 0,25 1/1/4 = 4

ΠŸΡ€ΠΈΠΌΠ΅Ρ€ 4: Π’Ρ‹Ρ‡ΠΈΡΠ»ΠΈΡ‚ΡŒ 0,5 -3
РСшСниС: 0,5 -3 = (1/2) -3 = 1/(1/2) 3 = 1/(1/8) = 8

ΠŸΡ€ΠΈΠΌΠ΅Ρ€ 5: Π’Ρ‹Ρ‡ΠΈΡΠ»ΠΈΡ‚ΡŒ -0,5 -3
РСшСниС: -0,5 -3 = (-1/2) -3 = 1/(-1/2) 3 = 1/(-1/8) = -8
ΠžΡ‚Π²Π΅Ρ‚: -0,5 -3 = -8

Π˜ΡΡ…ΠΎΠ΄Ρ ΠΈΠ· 4-Π³ΠΎ ΠΈ 5-ΠΎΠ³ΠΎ ΠΏΡ€ΠΈΠΌΠ΅Ρ€ΠΎΠ², сдСлаСм нСсколько Π²Ρ‹Π²ΠΎΠ΄ΠΎΠ²:

  • Для ΠΏΠΎΠ»ΠΎΠΆΠΈΡ‚Π΅Π»ΡŒΠ½ΠΎΠ³ΠΎ числа Π² ΠΏΡ€ΠΎΠΌΠ΅ΠΆΡƒΡ‚ΠΊΠ΅ ΠΎΡ‚ 0 Π΄ΠΎ 1 (ΠΏΡ€ΠΈΠΌΠ΅Ρ€ 4), Π²ΠΎΠ·Π²ΠΎΠ΄ΠΈΠΌΠΎΠ³ΠΎ Π² ΠΎΡ‚Ρ€ΠΈΡ†Π°Ρ‚Π΅Π»ΡŒΠ½ΡƒΡŽ ΡΡ‚Π΅ΠΏΠ΅Π½ΡŒ, Ρ‡Π΅Ρ‚Π½ΠΎΡΡ‚ΡŒ ΠΈΠ»ΠΈ Π½Π΅Ρ‡Π΅Ρ‚Π½ΠΎΡΡ‚ΡŒ стСпСни Π½Π΅ Π²Π°ΠΆΠ½Π°, Π·Π½Π°Ρ‡Π΅Π½ΠΈΠ΅ выраТСния Π±ΡƒΠ΄Π΅Ρ‚ ΠΏΠΎΠ»ΠΎΠΆΠΈΡ‚Π΅Π»ΡŒΠ½Ρ‹ΠΌ. ΠŸΡ€ΠΈ этом, Ρ‡Π΅ΠΌ большС ΡΡ‚Π΅ΠΏΠ΅Π½ΡŒ, Ρ‚Π΅ΠΌ большС Π·Π½Π°Ρ‡Π΅Π½ΠΈΠ΅.
  • Для ΠΎΡ‚Ρ€ΠΈΡ†Π°Ρ‚Π΅Π»ΡŒΠ½ΠΎΠ³ΠΎ числа Π² ΠΏΡ€ΠΎΠΌΠ΅ΠΆΡƒΡ‚ΠΊΠ΅ ΠΎΡ‚ 0 Π΄ΠΎ 1 (ΠΏΡ€ΠΈΠΌΠ΅Ρ€ 5), Π²ΠΎΠ·Π²ΠΎΠ΄ΠΈΠΌΠΎΠ³ΠΎ Π² ΠΎΡ‚Ρ€ΠΈΡ†Π°Ρ‚Π΅Π»ΡŒΠ½ΡƒΡŽ ΡΡ‚Π΅ΠΏΠ΅Π½ΡŒ, Ρ‡Π΅Ρ‚Π½ΠΎΡΡ‚ΡŒ ΠΈΠ»ΠΈ Π½Π΅Ρ‡Π΅Ρ‚Π½ΠΎΡΡ‚ΡŒ стСпСни Π½Π΅Π²Π°ΠΆΠ½Π°, Π·Π½Π°Ρ‡Π΅Π½ΠΈΠ΅ выраТСния Π±ΡƒΠ΄Π΅Ρ‚ ΠΎΡ‚Ρ€ΠΈΡ†Π°Ρ‚Π΅Π»ΡŒΠ½Ρ‹ΠΌ. ΠŸΡ€ΠΈ этом, Ρ‡Π΅ΠΌ большС ΡΡ‚Π΅ΠΏΠ΅Π½ΡŒ, Ρ‚Π΅ΠΌ мСньшС Π·Π½Π°Ρ‡Π΅Π½ΠΈΠ΅.

Как Π²ΠΎΠ·Π²ΠΎΠ΄ΠΈΡ‚ΡŒ Π² ΠΎΡ‚Ρ€ΠΈΡ†Π°Ρ‚Π΅Π»ΡŒΠ½ΡƒΡŽ ΡΡ‚Π΅ΠΏΠ΅Π½ΡŒ – ΡΡ‚Π΅ΠΏΠ΅Π½ΡŒ Π² Π²ΠΈΠ΄Π΅ Π΄Ρ€ΠΎΠ±Π½ΠΎΠ³ΠΎ числа

ВыраТСния Π΄Π°Π½Π½ΠΎΠ³ΠΎ Ρ‚ΠΈΠΏΠ° ΠΈΠΌΠ΅ΡŽΡ‚ ΡΠ»Π΅Π΄ΡƒΡŽΡ‰ΠΈΠΉ Π²ΠΈΠ΄: a -m/n , Π³Π΄Π΅ a – ΠΎΠ±Ρ‹Ρ‡Π½ΠΎΠ΅ число, m – Ρ‡ΠΈΡΠ»ΠΈΡ‚Π΅Π»ΡŒ стСпСни, n – Π·Π½Π°ΠΌΠ΅Π½Π°Ρ‚Π΅Π»ΡŒ стСпСни.

Рассмотрим ΠΏΡ€ΠΈΠΌΠ΅Ρ€:
Π’Ρ‹Ρ‡ΠΈΡΠ»ΠΈΡ‚ΡŒ: 8 -1/3

РСшСниС (ΠΏΠΎΡΠ»Π΅Π΄ΠΎΠ²Π°Ρ‚Π΅Π»ΡŒΠ½ΠΎΡΡ‚ΡŒ дСйствий):

  • ВспоминаСм ΠΏΡ€Π°Π²ΠΈΠ»ΠΎ возвСдСния числа Π² ΠΎΡ‚Ρ€ΠΈΡ†Π°Ρ‚Π΅Π»ΡŒΠ½ΡƒΡŽ ΡΡ‚Π΅ΠΏΠ΅Π½ΡŒ. ΠŸΠΎΠ»ΡƒΡ‡ΠΈΠΌ: 8 -1/3 = 1/(8) 1/3 .
  • Π—Π°ΠΌΠ΅Ρ‚ΡŒΡ‚Π΅, Π² Π·Π½Π°ΠΌΠ΅Π½Π°Ρ‚Π΅Π»Π΅ число 8 Π² Π΄Ρ€ΠΎΠ±Π½ΠΎΠΉ стСпСни. ΠžΠ±Ρ‰ΠΈΠΉ Π²ΠΈΠ΄ вычислСния Π΄Ρ€ΠΎΠ±Π½ΠΎΠΉ стСпСни Ρ‚Π°ΠΊΠΎΠ²: a m/n = n √8 m .
  • Π’Π°ΠΊΠΈΠΌ ΠΎΠ±Ρ€Π°Π·ΠΎΠΌ, 1/(8) 1/3 = 1/(3 √8 1). ΠŸΠΎΠ»ΡƒΡ‡Π°Π΅ΠΌ кубичСский ΠΊΠΎΡ€Π΅Π½ΡŒ ΠΈΠ· восьми, ΠΊΠΎΡ‚ΠΎΡ€Ρ‹ΠΉ Ρ€Π°Π²Π΅Π½ 2. Π˜ΡΡ…ΠΎΠ΄Ρ ΠΎΡ‚ΡΡŽΠ΄Π°, 1/(8) 1/3 = 1/(1/2) = 2.
  • ΠžΡ‚Π²Π΅Ρ‚: 8 -1/3 = 2

ΠœΡ‹ Ρ€Π°Π·ΠΎΠ±Ρ€Π°Π»ΠΈΡΡŒ, Ρ‡Ρ‚ΠΎ Π²ΠΎΠΎΠ±Ρ‰Π΅ ΠΈΠ· сСбя прСдставляСт ΡΡ‚Π΅ΠΏΠ΅Π½ΡŒ числа. Π’Π΅ΠΏΠ΅Ρ€ΡŒ Π½Π°ΠΌ Π½Π°Π΄ΠΎ ΠΏΠΎΠ½ΡΡ‚ΡŒ, ΠΊΠ°ΠΊ ΠΏΡ€Π°Π²ΠΈΠ»ΡŒΠ½ΠΎ Π²Ρ‹ΠΏΠΎΠ»Π½ΡΡ‚ΡŒ Π΅Π΅ вычислСниС, Ρ‚.Π΅. Π²ΠΎΠ·Π²ΠΎΠ΄ΠΈΡ‚ΡŒ числа Π² ΡΡ‚Π΅ΠΏΠ΅Π½ΡŒ. Π’ этом ΠΌΠ°Ρ‚Π΅Ρ€ΠΈΠ°Π»Π΅ ΠΌΡ‹ Ρ€Π°Π·Π±Π΅Ρ€Π΅ΠΌ основныС ΠΏΡ€Π°Π²ΠΈΠ»Π° вычислСния стСпСни Π² случаС Ρ†Π΅Π»ΠΎΠ³ΠΎ, Π½Π°Ρ‚ΡƒΡ€Π°Π»ΡŒΠ½ΠΎΠ³ΠΎ, Π΄Ρ€ΠΎΠ±Π½ΠΎΠ³ΠΎ, Ρ€Π°Ρ†ΠΈΠΎΠ½Π°Π»ΡŒΠ½ΠΎΠ³ΠΎ ΠΈ ΠΈΡ€Ρ€Π°Ρ†ΠΈΠΎΠ½Π°Π»ΡŒΠ½ΠΎΠ³ΠΎ показатСля. ВсС опрСдСлСния Π±ΡƒΠ΄ΡƒΡ‚ ΠΏΡ€ΠΎΠΈΠ»Π»ΡŽΡΡ‚Ρ€ΠΈΡ€ΠΎΠ²Π°Π½Ρ‹ ΠΏΡ€ΠΈΠΌΠ΅Ρ€Π°ΠΌΠΈ.

ΠŸΠΎΠ½ΡΡ‚ΠΈΠ΅ возвСдСния Π² ΡΡ‚Π΅ΠΏΠ΅Π½ΡŒ

НачнСм с формулирования Π±Π°Π·ΠΎΠ²Ρ‹Ρ… ΠΎΠΏΡ€Π΅Π΄Π΅Π»Π΅Π½ΠΈΠΉ.

Π’ΠΎΠ·Π²Π΅Π΄Π΅Π½ΠΈΠ΅ Π² ΡΡ‚Π΅ΠΏΠ΅Π½ΡŒ – это вычислСниС значСния стСпСни Π½Π΅ΠΊΠΎΡ‚ΠΎΡ€ΠΎΠ³ΠΎ числа.

Π’ΠΎ Π΅ΡΡ‚ΡŒ слова «вычислСниС Π·Π½Π°Ρ‡Π΅Π½ΠΈΠ΅ стСпСни» ΠΈ Β«Π²ΠΎΠ·Π²Π΅Π΄Π΅Π½ΠΈΠ΅ Π² ΡΡ‚Π΅ΠΏΠ΅Π½ΡŒΒ» ΠΎΠ·Π½Π°Ρ‡Π°ΡŽΡ‚ ΠΎΠ΄Π½ΠΎ ΠΈ Ρ‚ΠΎ ΠΆΠ΅. Π’Π°ΠΊ, Ссли Π² Π·Π°Π΄Π°Ρ‡Π΅ стоит Β«Π’ΠΎΠ·Π²Π΅Π΄ΠΈΡ‚Π΅ число 0 , 5 Π² ΠΏΡΡ‚ΡƒΡŽ ΡΡ‚Π΅ΠΏΠ΅Π½ΡŒΒ», это слСдуСт ΠΏΠΎΠ½ΠΈΠΌΠ°Ρ‚ΡŒ ΠΊΠ°ΠΊ «вычислитС Π·Π½Π°Ρ‡Π΅Π½ΠΈΠ΅ стСпСни (0 , 5) 5 .

Π’Π΅ΠΏΠ΅Ρ€ΡŒ ΠΏΡ€ΠΈΠ²Π΅Π΄Π΅ΠΌ основныС ΠΏΡ€Π°Π²ΠΈΠ»Π°, ΠΊΠΎΡ‚ΠΎΡ€Ρ‹ΠΌ Π½ΡƒΠΆΠ½ΠΎ ΠΏΡ€ΠΈΠ΄Π΅Ρ€ΠΆΠΈΠ²Π°Ρ‚ΡŒΡΡ ΠΏΡ€ΠΈ Ρ‚Π°ΠΊΠΈΡ… вычислСниях.

Вспомним, Ρ‡Ρ‚ΠΎ Ρ‚Π°ΠΊΠΎΠ΅ ΡΡ‚Π΅ΠΏΠ΅Π½ΡŒ числа с Π½Π°Ρ‚ΡƒΡ€Π°Π»ΡŒΠ½Ρ‹ΠΌ ΠΏΠΎΠΊΠ°Π·Π°Ρ‚Π΅Π»Π΅ΠΌ. Для стСпСни с основаниСм a ΠΈ ΠΏΠΎΠΊΠ°Π·Π°Ρ‚Π΅Π»Π΅ΠΌ n это Π±ΡƒΠ΄Π΅Ρ‚ ΠΏΡ€ΠΎΠΈΠ·Π²Π΅Π΄Π΅Π½ΠΈΠ΅ n -Π½ΠΎΠ³ΠΎ числа ΠΌΠ½ΠΎΠΆΠΈΡ‚Π΅Π»Π΅ΠΉ, ΠΊΠ°ΠΆΠ΄Ρ‹ΠΉ ΠΈΠ· ΠΊΠΎΡ‚ΠΎΡ€Ρ‹Ρ… Ρ€Π°Π²Π΅Π½ a . Π­Ρ‚ΠΎ ΠΌΠΎΠΆΠ½ΠΎ Π·Π°ΠΏΠΈΡΠ°Ρ‚ΡŒ Ρ‚Π°ΠΊ:

Π§Ρ‚ΠΎΠ±Ρ‹ Π²Ρ‹Ρ‡ΠΈΡΠ»ΠΈΡ‚ΡŒ Π·Π½Π°Ρ‡Π΅Π½ΠΈΠ΅ стСпСни, Π½ΡƒΠΆΠ½ΠΎ Π²Ρ‹ΠΏΠΎΠ»Π½ΠΈΡ‚ΡŒ дСйствиС умноТСния, Ρ‚ΠΎ Π΅ΡΡ‚ΡŒ ΠΏΠ΅Ρ€Π΅ΠΌΠ½ΠΎΠΆΠΈΡ‚ΡŒ основания стСпСни ΡƒΠΊΠ°Π·Π°Π½Π½ΠΎΠ΅ число Ρ€Π°Π·. На ΡƒΠΌΠ΅Π½ΠΈΠΈ быстро ΡƒΠΌΠ½ΠΎΠΆΠ°Ρ‚ΡŒ ΠΈ основано само понятиС стСпСни с Π½Π°Ρ‚ΡƒΡ€Π°Π»ΡŒΠ½Ρ‹ΠΌ ΠΏΠΎΠΊΠ°Π·Π°Ρ‚Π΅Π»Π΅ΠΌ. ΠŸΡ€ΠΈΠ²Π΅Π΄Π΅ΠΌ ΠΏΡ€ΠΈΠΌΠ΅Ρ€Ρ‹.

УсловиС: Π²ΠΎΠ·Π²Π΅Π΄ΠΈΡ‚Π΅ – 2 Π² ΡΡ‚Π΅ΠΏΠ΅Π½ΡŒ 4 .

Π˜ΡΠΏΠΎΠ»ΡŒΠ·ΡƒΡ ΠΎΠΏΡ€Π΅Π΄Π΅Π»Π΅Π½ΠΈΠ΅ Π²Ρ‹ΡˆΠ΅, запишСм: (βˆ’ 2) 4 = (βˆ’ 2) Β· (βˆ’ 2) Β· (βˆ’ 2) Β· (βˆ’ 2) . Π”Π°Π»Π΅Π΅ Π½Π°ΠΌ Π½ΡƒΠΆΠ½ΠΎ просто Π²Ρ‹ΠΏΠΎΠ»Π½ΠΈΡ‚ΡŒ ΡƒΠΊΠ°Π·Π°Π½Π½Ρ‹Π΅ дСйствия ΠΈ ΠΏΠΎΠ»ΡƒΡ‡ΠΈΡ‚ΡŒ 16 .

Π’ΠΎΠ·ΡŒΠΌΠ΅ΠΌ ΠΏΡ€ΠΈΠΌΠ΅Ρ€ послоТнСС.

ВычислитС Π·Π½Π°Ρ‡Π΅Π½ΠΈΠ΅ 3 2 7 2

Π”Π°Π½Π½ΡƒΡŽ запись ΠΌΠΎΠΆΠ½ΠΎ ΠΏΠ΅Ρ€Π΅ΠΏΠΈΡΠ°Ρ‚ΡŒ Π² Π²ΠΈΠ΄Π΅ 3 2 7 Β· 3 2 7 . Π Π°Π½Π΅Π΅ ΠΌΡ‹ рассматривали, ΠΊΠ°ΠΊ ΠΏΡ€Π°Π²ΠΈΠ»ΡŒΠ½ΠΎ ΡƒΠΌΠ½ΠΎΠΆΠ°Ρ‚ΡŒ ΡΠΌΠ΅ΡˆΠ°Π½Π½Ρ‹Π΅ числа, упомянутыС Π² условии.

Π’Ρ‹ΠΏΠΎΠ»Π½ΠΈΠΌ эти дСйствия ΠΈ ΠΏΠΎΠ»ΡƒΡ‡ΠΈΠΌ ΠΎΡ‚Π²Π΅Ρ‚: 3 2 7 Β· 3 2 7 = 23 7 Β· 23 7 = 529 49 = 10 39 49

Если Π² Π·Π°Π΄Π°Ρ‡Π΅ ΡƒΠΊΠ°Π·Π°Π½Π° Π½Π΅ΠΎΠ±Ρ…ΠΎΠ΄ΠΈΠΌΠΎΡΡ‚ΡŒ Π²ΠΎΠ·Π²ΠΎΠ΄ΠΈΡ‚ΡŒ ΠΈΡ€Ρ€Π°Ρ†ΠΈΠΎΠ½Π°Π»ΡŒΠ½Ρ‹Π΅ числа Π² Π½Π°Ρ‚ΡƒΡ€Π°Π»ΡŒΠ½ΡƒΡŽ ΡΡ‚Π΅ΠΏΠ΅Π½ΡŒ, Π½Π°ΠΌ потрСбуСтся ΠΏΡ€Π΅Π΄Π²Π°Ρ€ΠΈΡ‚Π΅Π»ΡŒΠ½ΠΎ ΠΎΠΊΡ€ΡƒΠ³Π»ΠΈΡ‚ΡŒ ΠΈΡ… основания Π΄ΠΎ разряда, ΠΊΠΎΡ‚ΠΎΡ€Ρ‹ΠΉ ΠΏΠΎΠ·Π²ΠΎΠ»ΠΈΡ‚ Π½Π°ΠΌ ΠΏΠΎΠ»ΡƒΡ‡ΠΈΡ‚ΡŒ ΠΎΡ‚Π²Π΅Ρ‚ Π½ΡƒΠΆΠ½ΠΎΠΉ точности. Π Π°Π·Π±Π΅Ρ€Π΅ΠΌ ΠΏΡ€ΠΈΠΌΠ΅Ρ€.

Π’Ρ‹ΠΏΠΎΠ»Π½ΠΈΡ‚Π΅ Π²ΠΎΠ·Π²Π΅Π΄Π΅Π½ΠΈΠ΅ Π² ΠΊΠ²Π°Π΄Ρ€Π°Ρ‚ числа Ο€ .

Для Π½Π°Ρ‡Π°Π»Π° ΠΎΠΊΡ€ΡƒΠ³Π»ΠΈΠΌ Π΅Π³ΠΎ Π΄ΠΎ сотых. Π’ΠΎΠ³Π΄Π° Ο€ 2 β‰ˆ (3 , 14) 2 = 9 , 8596 . Если ΠΆΠ΅ Ο€ β‰ˆ 3 . 14159 , Ρ‚ΠΎ ΠΌΡ‹ ΠΏΠΎΠ»ΡƒΡ‡ΠΈΠΌ Π±ΠΎΠ»Π΅Π΅ Ρ‚ΠΎΡ‡Π½Ρ‹ΠΉ Ρ€Π΅Π·ΡƒΠ»ΡŒΡ‚Π°Ρ‚: Ο€ 2 β‰ˆ (3 , 14159) 2 = 9 , 8695877281 .

ΠžΡ‚ΠΌΠ΅Ρ‚ΠΈΠΌ, Ρ‡Ρ‚ΠΎ Π½Π΅ΠΎΠ±Ρ…ΠΎΠ΄ΠΈΠΌΠΎΡΡ‚ΡŒ Π²Ρ‹ΡΡ‡ΠΈΡ‚Ρ‹Π²Π°Ρ‚ΡŒ стСпСни ΠΈΡ€Ρ€Π°Ρ†ΠΈΠΎΠ½Π°Π»ΡŒΠ½Ρ‹Ρ… чисСл Π½Π° ΠΏΡ€Π°ΠΊΡ‚ΠΈΠΊΠ΅ Π²ΠΎΠ·Π½ΠΈΠΊΠ°Π΅Ρ‚ ΡΡ€Π°Π²Π½ΠΈΡ‚Π΅Π»ΡŒΠ½ΠΎ Ρ€Π΅Π΄ΠΊΠΎ. ΠœΡ‹ ΠΌΠΎΠΆΠ΅ΠΌ Ρ‚ΠΎΠ³Π΄Π° Π·Π°ΠΏΠΈΡΠ°Ρ‚ΡŒ ΠΎΡ‚Π²Π΅Ρ‚ Π² Π²ΠΈΠ΄Π΅ самой стСпСни (ln 6) 3 ΠΈΠ»ΠΈ ΠΏΡ€Π΅ΠΎΠ±Ρ€Π°Π·ΠΎΠ²Π°Ρ‚ΡŒ, Ссли это Π²ΠΎΠ·ΠΌΠΎΠΆΠ½ΠΎ: 5 7 = 125 5 .

ΠžΡ‚Π΄Π΅Π»ΡŒΠ½ΠΎ слСдуСт ΡƒΠΊΠ°Π·Π°Ρ‚ΡŒ, Ρ‡Ρ‚ΠΎ Ρ‚Π°ΠΊΠΎΠ΅ пСрвая ΡΡ‚Π΅ΠΏΠ΅Π½ΡŒ числа. Π’ΡƒΡ‚ ΠΌΠΎΠΆΠ½ΠΎ просто Π·Π°ΠΏΠΎΠΌΠ½ΠΈΡ‚ΡŒ, Ρ‡Ρ‚ΠΎ любоС число, Π²ΠΎΠ·Π²Π΅Π΄Π΅Π½Π½ΠΎΠ΅ Π² ΠΏΠ΅Ρ€Π²ΡƒΡŽ ΡΡ‚Π΅ΠΏΠ΅Π½ΡŒ, останСтся самим собой:

Π­Ρ‚ΠΎ понятно ΠΈΠ· записи .

ΠžΡ‚ основания стСпСни это Π½Π΅ зависит.

Π’Π°ΠΊ, (βˆ’ 9) 1 = βˆ’ 9 , Π° 7 3 , Π²ΠΎΠ·Π²Π΅Π΄Π΅Π½Π½ΠΎΠ΅ Π² ΠΏΠ΅Ρ€Π²ΡƒΡŽ ΡΡ‚Π΅ΠΏΠ΅Π½ΡŒ, останСтся Ρ€Π°Π²Π½ΠΎ 7 3 .

Для удобства Ρ€Π°Π·Π±Π΅Ρ€Π΅ΠΌ ΠΎΡ‚Π΄Π΅Π»ΡŒΠ½ΠΎ Ρ‚Ρ€ΠΈ случая: Ссли ΠΏΠΎΠΊΠ°Π·Π°Ρ‚Π΅Π»ΡŒ стСпСни – Ρ†Π΅Π»ΠΎΠ΅ ΠΏΠΎΠ»ΠΎΠΆΠΈΡ‚Π΅Π»ΡŒΠ½ΠΎΠ΅ число, Ссли это ноль ΠΈ Ссли это Ρ†Π΅Π»ΠΎΠ΅ ΠΎΡ‚Ρ€ΠΈΡ†Π°Ρ‚Π΅Π»ΡŒΠ½ΠΎΠ΅ число.

Π’ ΠΏΠ΅Ρ€Π²ΠΎΠ΅ случаС это Ρ‚ΠΎ ΠΆΠ΅ самоС, Ρ‡Ρ‚ΠΎ ΠΈ Π²ΠΎΠ·Π²Π΅Π΄Π΅Π½ΠΈΠ΅ Π² Π½Π°Ρ‚ΡƒΡ€Π°Π»ΡŒΠ½ΡƒΡŽ ΡΡ‚Π΅ΠΏΠ΅Π½ΡŒ: вСдь Ρ†Π΅Π»Ρ‹Π΅ ΠΏΠΎΠ»ΠΎΠΆΠΈΡ‚Π΅Π»ΡŒΠ½Ρ‹Π΅ числа ΠΏΡ€ΠΈΠ½Π°Π΄Π»Π΅ΠΆΠ°Ρ‚ ΠΊΠΎ мноТСству Π½Π°Ρ‚ΡƒΡ€Π°Π»ΡŒΠ½Ρ‹Ρ…. О Ρ‚ΠΎΠΌ, ΠΊΠ°ΠΊ Ρ€Π°Π±ΠΎΡ‚Π°Ρ‚ΡŒ с Ρ‚Π°ΠΊΠΈΠΌΠΈ стСпСнями, ΠΌΡ‹ ΡƒΠΆΠ΅ рассказали Π²Ρ‹ΡˆΠ΅.

Π’Π΅ΠΏΠ΅Ρ€ΡŒ посмотрим, ΠΊΠ°ΠΊ ΠΏΡ€Π°Π²ΠΈΠ»ΡŒΠ½ΠΎ Π²ΠΎΠ·Π²ΠΎΠ΄ΠΈΡ‚ΡŒ Π² Π½ΡƒΠ»Π΅Π²ΡƒΡŽ ΡΡ‚Π΅ΠΏΠ΅Π½ΡŒ. ΠŸΡ€ΠΈ основании, ΠΊΠΎΡ‚ΠΎΡ€ΠΎΠ΅ отличаСтся ΠΎΡ‚ нуля, это вычислСниС всСгда Π΄Π°Π΅Ρ‚ Π½Π° Π²Ρ‹Ρ…ΠΎΠ΄Π΅ 1 . Π Π°Π½Π΅Π΅ ΠΌΡ‹ ΡƒΠΆΠ΅ поясняли, Ρ‡Ρ‚ΠΎ 0 -я ΡΡ‚Π΅ΠΏΠ΅Π½ΡŒ a ΠΌΠΎΠΆΠ΅Ρ‚ Π±Ρ‹Ρ‚ΡŒ ΠΎΠΏΡ€Π΅Π΄Π΅Π»Π΅Π½Π° для любого Π΄Π΅ΠΉΡΡ‚Π²ΠΈΡ‚Π΅Π»ΡŒΠ½ΠΎΠ³ΠΎ числа, Π½Π΅ Ρ€Π°Π²Π½ΠΎΠ³ΠΎ 0 , ΠΈ a 0 = 1 .

5 0 = 1 , (- 2 , 56) 0 = 1 2 3 0 = 1

0 0 – Π½Π΅ ΠΎΠΏΡ€Π΅Π΄Π΅Π»Π΅Π½.

Π£ нас остался Ρ‚ΠΎΠ»ΡŒΠΊΠΎ случай стСпСни с Ρ†Π΅Π»Ρ‹ΠΌ ΠΎΡ‚Ρ€ΠΈΡ†Π°Ρ‚Π΅Π»ΡŒΠ½Ρ‹ΠΌ ΠΏΠΎΠΊΠ°Π·Π°Ρ‚Π΅Π»Π΅ΠΌ. ΠœΡ‹ ΡƒΠΆΠ΅ Ρ€Π°Π·Π±ΠΈΡ€Π°Π»ΠΈ, Ρ‡Ρ‚ΠΎ Ρ‚Π°ΠΊΠΈΠ΅ стСпСни ΠΌΠΎΠΆΠ½ΠΎ Π·Π°ΠΏΠΈΡΠ°Ρ‚ΡŒ Π² Π²ΠΈΠ΄Π΅ Π΄Ρ€ΠΎΠ±ΠΈ 1 a z , Π³Π΄Π΅ Π° – любоС число, Π° z – Ρ†Π΅Π»Ρ‹ΠΉ ΠΎΡ‚Ρ€ΠΈΡ†Π°Ρ‚Π΅Π»ΡŒΠ½Ρ‹ΠΉ ΠΏΠΎΠΊΠ°Π·Π°Ρ‚Π΅Π»ΡŒ. ΠœΡ‹ Π²ΠΈΠ΄ΠΈΠΌ, Ρ‡Ρ‚ΠΎ Π·Π½Π°ΠΌΠ΅Π½Π°Ρ‚Π΅Π»ΡŒ этой Π΄Ρ€ΠΎΠ±ΠΈ Π΅ΡΡ‚ΡŒ Π½Π΅ Ρ‡Ρ‚ΠΎ ΠΈΠ½ΠΎΠ΅, ΠΊΠ°ΠΊ обыкновСнная ΡΡ‚Π΅ΠΏΠ΅Π½ΡŒ с Ρ†Π΅Π»Ρ‹ΠΌ ΠΏΠΎΠ»ΠΎΠΆΠΈΡ‚Π΅Π»ΡŒΠ½Ρ‹ΠΌ ΠΏΠΎΠΊΠ°Π·Π°Ρ‚Π΅Π»Π΅ΠΌ, Π° Π΅Π΅ Π²Ρ‹Ρ‡ΠΈΡΠ»ΡΡ‚ΡŒ ΠΌΡ‹ ΡƒΠΆΠ΅ Π½Π°ΡƒΡ‡ΠΈΠ»ΠΈΡΡŒ. ΠŸΡ€ΠΈΠ²Π΅Π΄Π΅ΠΌ ΠΏΡ€ΠΈΠΌΠ΅Ρ€Ρ‹ Π·Π°Π΄Π°Ρ‡.

Π’ΠΎΠ·Π²Π΅Π΄ΠΈΡ‚Π΅ 3 Π² ΡΡ‚Π΅ΠΏΠ΅Π½ΡŒ – 2 .

Π˜ΡΠΏΠΎΠ»ΡŒΠ·ΡƒΡ ΠΎΠΏΡ€Π΅Π΄Π΅Π»Π΅Π½ΠΈΠ΅ Π²Ρ‹ΡˆΠ΅, запишСм: 2 – 3 = 1 2 3

ΠŸΠΎΠ΄ΡΡ‡ΠΈΡ‚Π°Π΅ΠΌ Π·Π½Π°ΠΌΠ΅Π½Π°Ρ‚Π΅Π»ΡŒ этой Π΄Ρ€ΠΎΠ±ΠΈ ΠΈ ΠΏΠΎΠ»ΡƒΡ‡ΠΈΠΌ 8: 2 3 = 2 Β· 2 Β· 2 = 8 .

Π’ΠΎΠ³Π΄Π° ΠΎΡ‚Π²Π΅Ρ‚ Ρ‚Π°ΠΊΠΎΠ²: 2 – 3 = 1 2 3 = 1 8

Π’ΠΎΠ·Π²Π΅Π΄ΠΈΡ‚Π΅ 1 , 43 Π² ΡΡ‚Π΅ΠΏΠ΅Π½ΡŒ – 2 .

ΠŸΠ΅Ρ€Π΅Ρ„ΠΎΡ€ΠΌΡƒΠ»ΠΈΡ€ΡƒΠ΅ΠΌ: 1 , 43 – 2 = 1 (1 , 43) 2

ВычисляСм ΠΊΠ²Π°Π΄Ρ€Π°Ρ‚ Π² Π·Π½Π°ΠΌΠ΅Π½Π°Ρ‚Π΅Π»Π΅: 1,43Β·1,43. ДСсятичныС Π΄Ρ€ΠΎΠ±ΠΈ ΠΌΠΎΠΆΠ½ΠΎ ΡƒΠΌΠ½ΠΎΠΆΠΈΡ‚ΡŒ Ρ‚Π°ΠΊΠΈΠΌ способом:

Π’ ΠΈΡ‚ΠΎΠ³Π΅ Ρƒ нас Π²Ρ‹ΡˆΠ»ΠΎ (1 , 43) – 2 = 1 (1 , 43) 2 = 1 2 , 0449 . Π­Ρ‚ΠΎΡ‚ Ρ€Π΅Π·ΡƒΠ»ΡŒΡ‚Π°Ρ‚ Π½Π°ΠΌ ΠΎΡΡ‚Π°Π»ΠΎΡΡŒ Π·Π°ΠΏΠΈΡΠ°Ρ‚ΡŒ Π² Π²ΠΈΠ΄Π΅ ΠΎΠ±Ρ‹ΠΊΠ½ΠΎΠ²Π΅Π½Π½ΠΎΠΉ Π΄Ρ€ΠΎΠ±ΠΈ, для Ρ‡Π΅Π³ΠΎ Π½Π΅ΠΎΠ±Ρ…ΠΎΠ΄ΠΈΠΌΠΎ ΡƒΠΌΠ½ΠΎΠΆΠΈΡ‚ΡŒ Π΅Π΅ Π½Π° 10 тысяч (см. ΠΌΠ°Ρ‚Π΅Ρ€ΠΈΠ°Π» ΠΎ ΠΏΡ€Π΅ΠΎΠ±Ρ€Π°Π·ΠΎΠ²Π°Π½ΠΈΠΈ Π΄Ρ€ΠΎΠ±Π΅ΠΉ).

ΠžΡ‚Π²Π΅Ρ‚: (1 , 43) – 2 = 10000 20449

ΠžΡ‚Π΄Π΅Π»ΡŒΠ½Ρ‹ΠΉ случай – Π²ΠΎΠ·Π²Π΅Π΄Π΅Π½ΠΈΠ΅ числа Π² минус ΠΏΠ΅Ρ€Π²ΡƒΡŽ ΡΡ‚Π΅ΠΏΠ΅Π½ΡŒ. Π—Π½Π°Ρ‡Π΅Π½ΠΈΠ΅ Ρ‚Π°ΠΊΠΎΠΉ стСпСни Ρ€Π°Π²Π½ΠΎ числу, ΠΎΠ±Ρ€Π°Ρ‚Π½ΠΎΠΌΡƒ исходному Π·Π½Π°Ρ‡Π΅Π½ΠΈΡŽ основания: a – 1 = 1 a 1 = 1 a .

ΠŸΡ€ΠΈΠΌΠ΅Ρ€: 3 βˆ’ 1 = 1 / 3

9 13 – 1 = 13 9 6 4 – 1 = 1 6 4 .

Как возвСсти число Π² Π΄Ρ€ΠΎΠ±Π½ΡƒΡŽ ΡΡ‚Π΅ΠΏΠ΅Π½ΡŒ

Для выполнСния Ρ‚Π°ΠΊΠΎΠΉ ΠΎΠΏΠ΅Ρ€Π°Ρ†ΠΈΠΈ Π½Π°ΠΌ потрСбуСтся Π²ΡΠΏΠΎΠΌΠ½ΠΈΡ‚ΡŒ Π±Π°Π·ΠΎΠ²ΠΎΠ΅ ΠΎΠΏΡ€Π΅Π΄Π΅Π»Π΅Π½ΠΈΠ΅ стСпСни с Π΄Ρ€ΠΎΠ±Π½Ρ‹ΠΌ ΠΏΠΎΠΊΠ°Π·Π°Ρ‚Π΅Π»Π΅ΠΌ: a m n = a m n ΠΏΡ€ΠΈ любом ΠΏΠΎΠ»ΠΎΠΆΠΈΡ‚Π΅Π»ΡŒΠ½ΠΎΠΌ a , Ρ†Π΅Π»ΠΎΠΌ m ΠΈ Π½Π°Ρ‚ΡƒΡ€Π°Π»ΡŒΠ½ΠΎΠΌ n .

Π’Π°ΠΊΠΈΠΌ ΠΎΠ±Ρ€Π°Π·ΠΎΠΌ, вычислСниС Π΄Ρ€ΠΎΠ±Π½ΠΎΠΉ стСпСни Π½ΡƒΠΆΠ½ΠΎ Π²Ρ‹ΠΏΠΎΠ»Π½ΡΡ‚ΡŒ Π² Π΄Π²Π° дСйствия: Π²ΠΎΠ·Π²Π΅Π΄Π΅Π½ΠΈΠ΅ Π² Ρ†Π΅Π»ΡƒΡŽ ΡΡ‚Π΅ΠΏΠ΅Π½ΡŒ ΠΈ Π½Π°Ρ…ΠΎΠΆΠ΄Π΅Π½ΠΈΠ΅ корня n -Π½ΠΎΠΉ стСпСни.

Π£ нас Π΅ΡΡ‚ΡŒ равСнство a m n = a m n , ΠΊΠΎΡ‚ΠΎΡ€ΠΎΠ΅, учитывая свойства ΠΊΠΎΡ€Π½Π΅ΠΉ, ΠΎΠ±Ρ‹Ρ‡Π½ΠΎ примСняСтся для Ρ€Π΅ΡˆΠ΅Π½ΠΈΡ Π·Π°Π΄Π°Ρ‡ Π² Π²ΠΈΠ΄Π΅ a m n = a n m . Π­Ρ‚ΠΎ Π·Π½Π°Ρ‡ΠΈΡ‚, Ρ‡Ρ‚ΠΎ Ссли ΠΌΡ‹ Π²ΠΎΠ·Π²ΠΎΠ΄ΠΈΠΌ число a Π² Π΄Ρ€ΠΎΠ±Π½ΡƒΡŽ ΡΡ‚Π΅ΠΏΠ΅Π½ΡŒ m / n , Ρ‚ΠΎ сначала ΠΌΡ‹ ΠΈΠ·Π²Π»Π΅ΠΊΠ°Π΅ΠΌ ΠΊΠΎΡ€Π΅Π½ΡŒ n -Π½ΠΎΠΉ стСпСни ΠΈΠ· Π°, ΠΏΠΎΡ‚ΠΎΠΌ Π²ΠΎΠ·Π²ΠΎΠ΄ΠΈΠΌ Ρ€Π΅Π·ΡƒΠ»ΡŒΡ‚Π°Ρ‚ Π² ΡΡ‚Π΅ΠΏΠ΅Π½ΡŒ с Ρ†Π΅Π»Ρ‹ΠΌ ΠΏΠΎΠΊΠ°Π·Π°Ρ‚Π΅Π»Π΅ΠΌ m .

ΠŸΡ€ΠΎΠΈΠ»Π»ΡŽΡΡ‚Ρ€ΠΈΡ€ΡƒΠ΅ΠΌ Π½Π° ΠΏΡ€ΠΈΠΌΠ΅Ρ€Π΅.

ВычислитС 8 – 2 3 .

Бпособ 1. Богласно основному ΠΎΠΏΡ€Π΅Π΄Π΅Π»Π΅Π½ΠΈΡŽ, ΠΌΡ‹ ΠΌΠΎΠΆΠ΅ΠΌ ΠΏΡ€Π΅Π΄ΡΡ‚Π°Π²ΠΈΡ‚ΡŒ это Π² Π²ΠΈΠ΄Π΅: 8 – 2 3 = 8 – 2 3

Π’Π΅ΠΏΠ΅Ρ€ΡŒ подсчитаСм ΡΡ‚Π΅ΠΏΠ΅Π½ΡŒ ΠΏΠΎΠ΄ ΠΊΠΎΡ€Π½Π΅ΠΌ ΠΈ ΠΈΠ·Π²Π»Π΅Ρ‡Π΅ΠΌ ΠΊΠΎΡ€Π΅Π½ΡŒ Ρ‚Ρ€Π΅Ρ‚ΡŒΠ΅ΠΉ стСпСни ΠΈΠ· Ρ€Π΅Π·ΡƒΠ»ΡŒΡ‚Π°Ρ‚Π°: 8 – 2 3 = 1 64 3 = 1 3 3 64 3 = 1 3 3 4 3 3 = 1 4

Бпособ 2. ΠŸΡ€Π΅ΠΎΠ±Ρ€Π°Π·ΡƒΠ΅ΠΌ основноС равСнство: 8 – 2 3 = 8 – 2 3 = 8 3 – 2

ПослС этого ΠΈΠ·Π²Π»Π΅Ρ‡Π΅ΠΌ ΠΊΠΎΡ€Π΅Π½ΡŒ 8 3 – 2 = 2 3 3 – 2 = 2 – 2 ΠΈ Ρ€Π΅Π·ΡƒΠ»ΡŒΡ‚Π°Ρ‚ Π²ΠΎΠ·Π²Π΅Π΄Π΅ΠΌ Π² ΠΊΠ²Π°Π΄Ρ€Π°Ρ‚: 2 – 2 = 1 2 2 = 1 4

Π’ΠΈΠ΄ΠΈΠΌ, Ρ‡Ρ‚ΠΎ Ρ€Π΅ΡˆΠ΅Π½ΠΈΡ ΠΈΠ΄Π΅Π½Ρ‚ΠΈΡ‡Π½Ρ‹. МоТно ΠΏΠΎΠ»ΡŒΠ·ΠΎΠ²Π°Ρ‚ΡŒΡΡ Π»ΡŽΠ±Ρ‹ΠΌ ΠΏΠΎΠ½Ρ€Π°Π²ΠΈΠ²ΡˆΠΈΠΌΡΡ способом.

Π‘Ρ‹Π²Π°ΡŽΡ‚ случаи, ΠΊΠΎΠ³Π΄Π° ΡΡ‚Π΅ΠΏΠ΅Π½ΡŒ ΠΈΠΌΠ΅Π΅Ρ‚ ΠΏΠΎΠΊΠ°Π·Π°Ρ‚Π΅Π»ΡŒ, Π²Ρ‹Ρ€Π°ΠΆΠ΅Π½Π½Ρ‹ΠΉ ΡΠΌΠ΅ΡˆΠ°Π½Π½Ρ‹ΠΌ числом ΠΈΠ»ΠΈ дСсятичной Π΄Ρ€ΠΎΠ±ΡŒΡŽ. Для простоты вычислСний Π΅Π³ΠΎ Π»ΡƒΡ‡ΡˆΠ΅ Π·Π°ΠΌΠ΅Π½ΠΈΡ‚ΡŒ ΠΎΠ±Ρ‹Ρ‡Π½ΠΎΠΉ Π΄Ρ€ΠΎΠ±ΡŒΡŽ ΠΈ ΡΡ‡ΠΈΡ‚Π°Ρ‚ΡŒ, ΠΊΠ°ΠΊ ΡƒΠΊΠ°Π·Π°Π½ΠΎ Π²Ρ‹ΡˆΠ΅.

Π’ΠΎΠ·Π²Π΅Π΄ΠΈΡ‚Π΅ 44 , 89 Π² ΡΡ‚Π΅ΠΏΠ΅Π½ΡŒ 2 , 5 .

ΠŸΡ€Π΅ΠΎΠ±Ρ€Π°Π·ΡƒΠ΅ΠΌ Π·Π½Π°Ρ‡Π΅Π½ΠΈΠ΅ показатСля Π² ΠΎΠ±Ρ‹ΠΊΠ½ΠΎΠ²Π΅Π½Π½ΡƒΡŽ Π΄Ρ€ΠΎΠ±ΡŒ – 44 , 89 2 , 5 = 49 , 89 5 2 .

А Ρ‚Π΅ΠΏΠ΅Ρ€ΡŒ выполняСм ΠΏΠΎ порядку всС дСйствия, ΡƒΠΊΠ°Π·Π°Π½Π½Ρ‹Π΅ Π²Ρ‹ΡˆΠ΅: 44 , 89 5 2 = 44 , 89 5 = 44 , 89 5 = 4489 100 5 = 4489 100 5 = 67 2 10 2 5 = 67 10 5 = = 1350125107 100000 = 13 501 , 25107

ΠžΡ‚Π²Π΅Ρ‚: 13 501 , 25107 . s=frac$.

Π‘ΠΏΡ€Π°Π²ΠΎΡ‡Π½Ρ‹ΠΉ ΠΌΠ°Ρ‚Π΅Ρ€ΠΈΠ°Π» ΠΏΠΎ ΠΌΠ°Ρ‚Π΅ΠΌΠ°Ρ‚ΠΈΠΊΠ΅ (6 класс) Π”Ρ€ΠΎΠ±ΠΈ

Π‘ΠΏΡ€Π°Π²ΠΎΡ‡Π½Ρ‹ΠΉ ΠΌΠ°Ρ‚Π΅Ρ€ΠΈΠ°Π» ΠΏΠΎ ΠΌΠ°Ρ‚Π΅ΠΌΠ°Ρ‚ΠΈΠΊΠ΅ (6 класс) Π”Ρ€ΠΎΠ±ΠΈ β€” страница β„–1/1

Π‘ΠΏΡ€Π°Π²ΠΎΡ‡Π½Ρ‹ΠΉ ΠΌΠ°Ρ‚Π΅Ρ€ΠΈΠ°Π» ΠΏΠΎ ΠΌΠ°Ρ‚Π΅ΠΌΠ°Ρ‚ΠΈΠΊΠ΅ (6 класс)


  1. Π”Ρ€ΠΎΠ±ΠΈ

ОсновноС свойство Π΄Ρ€ΠΎΠ±ΠΈ

: Если Ρ‡ΠΈΡΠ»ΠΈΡ‚Π΅Π»ΡŒ ΠΈ Π·Π½Π°ΠΌΠ΅Π½Π°Ρ‚Π΅Π»ΡŒ Π΄Ρ€ΠΎΠ±ΠΈ ΡƒΠΌΠ½ΠΎΠΆΠΈΡ‚ΡŒ ΠΈΠ»ΠΈ Ρ€Π°Π·Π΄Π΅Π»ΠΈΡ‚ΡŒ Π½Π° ΠΎΠ΄Π½ΠΎ ΠΈ Ρ‚ΠΎ ΠΆΠ΅ Π½Π°Ρ‚ΡƒΡ€Π°Π»ΡŒΠ½ΠΎΠ΅ число, Ρ‚ΠΎ получится равная Π΅ΠΉ Π΄Ρ€ΠΎΠ±ΡŒ (ΠΈΡΠΏΠΎΠ»ΡŒΠ·ΡƒΠ΅Ρ‚ΡΡ ΠΏΡ€ΠΈ сокращСнии Π΄Ρ€ΠΎΠ±Π΅ΠΉ).

ΠŸΠ΅Ρ€Π΅Π²ΠΎΠ΄ Π΄Ρ€ΠΎΠ±Π΅ΠΉ

Π§Ρ‚ΠΎΠ±Ρ‹ Π·Π°ΠΏΠΈΡΠ°Ρ‚ΡŒ ΠΎΠ±Ρ‹ΠΊΠ½ΠΎΠ²Π΅Π½Π½ΡƒΡŽ Π΄Ρ€ΠΎΠ±ΡŒ Π² Π²ΠΈΠ΄Π΅ дСсятичной, Π½ΡƒΠΆΠ½ΠΎ Ρ€Π°Π·Π΄Π΅Π»ΠΈΡ‚ΡŒ Ρ‡ΠΈΡΠ»ΠΈΡ‚Π΅Π»ΡŒ Π½Π° Π·Π½Π°ΠΌΠ΅Π½Π°Ρ‚Π΅Π»ΡŒ

ДСйствия с дробями


  1. Π§Ρ‚ΠΎΠ±Ρ‹ ΡΠ»ΠΎΠΆΠΈΡ‚ΡŒ Π΄Ρ€ΠΎΠ±ΠΈ с Ρ€Π°Π·Π½Ρ‹ΠΌΠΈ знамСнатСлями, Π½Π°Π΄ΠΎ: привСсти Π΄Π°Π½Π½Ρ‹Π΅ Π΄Ρ€ΠΎΠ±ΠΈ ΠΊ Π½Π°ΠΈΠΌΠ΅Π½ΡŒΡˆΠ΅ΠΌΡƒ ΠΎΠ±Ρ‰Π΅ΠΌΡƒ Π·Π½Π°ΠΌΠ΅Π½Π°Ρ‚Π΅Π»ΡŽ; ΡΠ»ΠΎΠΆΠΈΡ‚ΡŒ ΠΏΠΎΠ»ΡƒΡ‡Π΅Π½Π½Ρ‹Π΅ Π΄Ρ€ΠΎΠ±ΠΈ (ΡΠ»ΠΎΠΆΠΈΡ‚ΡŒ числитСли Π΄Ρ€ΠΎΠ±Π΅ΠΉ, Π·Π½Π°ΠΌΠ΅Π½Π°Ρ‚Π΅Π»ΡŒ ΠΏΠ΅Ρ€Π΅ΠΏΠΈΡΠ°Ρ‚ΡŒ Π±Π΅Π· ΠΈΠ·ΠΌΠ΅Π½Π΅Π½ΠΈΠΉ).

  2. Π§Ρ‚ΠΎΠ±Ρ‹ ΡƒΠΌΠ½ΠΎΠΆΠΈΡ‚ΡŒ Π΄Ρ€ΠΎΠ±ΡŒ Π½Π° Π΄Ρ€ΠΎΠ±ΡŒ, Π½Π°Π΄ΠΎ: Π½Π°ΠΉΡ‚ΠΈ ΠΏΡ€ΠΎΠΈΠ·Π²Π΅Π΄Π΅Π½ΠΈΠ΅ числитСлСй ΠΈ ΠΏΡ€ΠΎΠΈΠ·Π²Π΅Π΄Π΅Π½ΠΈΠ΅ Π·Π½Π°ΠΌΠ΅Π½Π°Ρ‚Π΅Π»Π΅ΠΉ этих Π΄Ρ€ΠΎΠ±Π΅ΠΉ; ΠΏΠ΅Ρ€Π²ΠΎΠ΅ ΠΏΡ€ΠΎΠΈΠ·Π²Π΅Π΄Π΅Π½ΠΈΠ΅ Π·Π°ΠΏΠΈΡΠ°Ρ‚ΡŒ числитСлСм, Π° Π²Ρ‚ΠΎΡ€ΠΎΠ΅ – Π·Π½Π°ΠΌΠ΅Π½Π°Ρ‚Π΅Π»Π΅ΠΌ.

  3. Π§Ρ‚ΠΎΠ±Ρ‹ Ρ€Π°Π·Π΄Π΅Π»ΠΈΡ‚ΡŒ ΠΎΠ΄Π½Ρƒ Π΄Ρ€ΠΎΠ±ΡŒ Π½Π° Π΄Ρ€ΡƒΠ³ΡƒΡŽ, Π½Π°Π΄ΠΎ Π΄Π΅Π»ΠΈΠΌΠΎΠ΅ (ΠΏΠ΅Ρ€Π²ΡƒΡŽ Π΄Ρ€ΠΎΠ±ΡŒ) ΡƒΠΌΠ½ΠΎΠΆΠΈΡ‚ΡŒ Π½Π° число, ΠΎΠ±Ρ€Π°Ρ‚Π½ΠΎΠ΅ Π΄Π΅Π»ΠΈΡ‚Π΅Π»ΡŽ («пСрСвСрнутая» вторая Π΄Ρ€ΠΎΠ±ΡŒ).

  1. ΠŸΡ€ΠΎΠΏΠΎΡ€Ρ†ΠΈΠΈ

ЧастноС Π΄Π²ΡƒΡ… чисСл Π½Π°Π·Ρ‹Π²Π°ΡŽΡ‚

ΠΎΡ‚Π½ΠΎΡˆΠ΅Π½ΠΈΠ΅ΠΌ

этих чисСл. (4:5)

РавСнство Π΄Π²ΡƒΡ… ΠΎΡ‚Π½ΠΎΡˆΠ΅Π½ΠΈΠΉ называСтся ΠΏΡ€ΠΎΠΏΠΎΡ€Ρ†ΠΈΠ΅ΠΉ. (a:b=c:d)

Бвойство ΠΏΡ€ΠΎΠΏΠΎΡ€Ρ†ΠΈΠΈ: Π² Π²Π΅Ρ€Π½ΠΎΠΉ ΠΏΡ€ΠΎΠΏΠΎΡ€Ρ†ΠΈΠΈ ΠΏΡ€ΠΎΠΈΠ·Π²Π΅Π΄Π΅Π½ΠΈΠ΅ ΠΊΡ€Π°ΠΉΠ½ΠΈΡ… Ρ‡Π»Π΅Π½ΠΎΠ² Ρ€Π°Π²Π½ΠΎ ΠΏΡ€ΠΎΠΈΠ·Π²Π΅Π΄Π΅Π½ΠΈΡŽ срСдних. (ad=bc)


  1. ΠŸΠΎΠ»ΠΎΠΆΠΈΡ‚Π΅Π»ΡŒΠ½Ρ‹Π΅ ΠΈ ΠΎΡ‚Ρ€ΠΈΡ†Π°Ρ‚Π΅Π»ΡŒΠ½Ρ‹Π΅ числа

Числа, Π»Π΅ΠΆΠ°Ρ‰ΠΈΠ΅ Π½Π° ΠΊΠΎΠΎΡ€Π΄ΠΈΠ½Π°Ρ‚Π½ΠΎΠΉ прямой

Π»Π΅Π²Π΅Π΅

0, Π½Π°Π·Ρ‹Π²Π°ΡŽΡ‚ΡΡ

ΠΎΡ‚Ρ€ΠΈΡ†Π°Ρ‚Π΅Π»ΡŒΠ½Ρ‹ΠΌΠΈ

, Π°

ΠΏΡ€Π°Π²Π΅Π΅ 0

–

ΠΏΠΎΠ»ΠΎΠΆΠΈΡ‚Π΅Π»ΡŒΠ½Ρ‹ΠΌΠΈ

.

Π‘Ρ€Π°Π²Π½Π΅Π½ΠΈΠ΅ чисСл

На ΠΊΠΎΠΎΡ€Π΄ΠΈΠ½Π°Ρ‚Π½ΠΎΠΉ прямой Ρ‚ΠΎΡ‡ΠΊΠ° с большСй ΠΊΠΎΠΎΡ€Π΄ΠΈΠ½Π°Ρ‚ΠΎΠΉ Π»Π΅ΠΆΠΈΡ‚ ΠΏΡ€Π°Π²Π΅Π΅ Ρ‚ΠΎΡ‡ΠΊΠΈ с мСньшСй ΠΊΠΎΠΎΡ€Π΄ΠΈΠ½Π°Ρ‚ΠΎΠΉ.

ΠœΠΎΠ΄ΡƒΠ»ΡŒ числа

ΠœΠΎΠ΄ΡƒΠ»Π΅ΠΌ числа Π° Π½Π°Π·Ρ‹Π²Π°ΡŽΡ‚ расстояниС (Π² Π΅Π΄ΠΈΠ½ΠΈΡ‡Π½Ρ‹Ρ… ΠΎΡ‚Ρ€Π΅Π·ΠΊΠ°Ρ…) ΠΎΡ‚ Π½Π°Ρ‡Π°Π»Π° ΠΊΠΎΠΎΡ€Π΄ΠΈΠ½Π°Ρ‚ Π΄ΠΎ Ρ‚ΠΎΡ‡ΠΊΠΈ А(Π°).

ДСйствия с ΠΎΡ‚Ρ€ΠΈΡ†Π°Ρ‚Π΅Π»ΡŒΠ½Ρ‹ΠΌΠΈ ΠΈ ΠΏΠΎΠ»ΠΎΠΆΠΈΡ‚Π΅Π»ΡŒΠ½Ρ‹ΠΌΠΈ числами

Π§Ρ‚ΠΎΠ±Ρ‹ ΡΠ»ΠΎΠΆΠΈΡ‚ΡŒ Π΄Π²Π° ΠΎΡ‚Ρ€ΠΈΡ†Π°Ρ‚Π΅Π»ΡŒΠ½Ρ‹Ρ… числа, Π½Π°Π΄ΠΎ: ΡΠ»ΠΎΠΆΠΈΡ‚ΡŒ ΠΈΡ… ΠΌΠΎΠ΄ΡƒΠ»ΠΈ, ΠΏΠΎΡΡ‚Π°Π²ΠΈΡ‚ΡŒ ΠΏΠ΅Ρ€Π΅Π΄ ΠΏΠΎΠ»ΡƒΡ‡Π΅Π½Π½Ρ‹ΠΌ числом Π·Π½Π°ΠΊ Β«-Β». Π Π΅Π·ΡƒΠ»ΡŒΡ‚Π°Ρ‚ слоТСния Π΄Π²ΡƒΡ… ΠΎΡ‚Ρ€ΠΈΡ†Π°Ρ‚Π΅Π»ΡŒΠ½Ρ‹Ρ… чисСл – ΠΎΡ‚Ρ€ΠΈΡ†Π°Ρ‚Π΅Π»ΡŒΠ½ΠΎΠ΅ число! (-4-5=-4+(-5)=-9)

Π§Ρ‚ΠΎΠ±Ρ‹ ΡΠ»ΠΎΠΆΠΈΡ‚ΡŒ Π΄Π²Π° числа с Ρ€Π°Π·Π½Ρ‹ΠΌΠΈ Π·Π½Π°ΠΊΠ°ΠΌΠΈ, Π½Π°Π΄ΠΎ ΠΈΠ· большСго модуля слагаСмых Π²Ρ‹Ρ‡Π΅ΡΡ‚ΡŒ мСньший, ΠΏΠΎΡΡ‚Π°Π²ΠΈΡ‚ΡŒ ΠΏΠ΅Ρ€Π΅Π΄ ΠΏΠΎΠ»ΡƒΡ‡Π΅Π½Π½Ρ‹ΠΌ числом Π·Π½Π°ΠΊ Ρ‚ΠΎΠ³ΠΎ слагаСмого, ΠΌΠΎΠ΄ΡƒΠ»ΡŒ ΠΊΠΎΡ‚ΠΎΡ€ΠΎΠ³ΠΎ большС.

-2+3=3-2=1, 5-8=-(8-5)=-3

Π§Ρ‚ΠΎΠ±Ρ‹ ΠΏΠ΅Ρ€Π΅ΠΌΠ½ΠΎΠΆΠΈΡ‚ΡŒ Π΄Π²Π° ΠΎΡ‚Ρ€ΠΈΡ†Π°Ρ‚Π΅Π»ΡŒΠ½Ρ‹Ρ… числа, Π½Π°Π΄ΠΎ ΠΏΠ΅Ρ€Π΅ΠΌΠ½ΠΎΠΆΠΈΡ‚ΡŒ ΠΈΡ… ΠΌΠΎΠ΄ΡƒΠ»ΠΈ. Π Π΅Π·ΡƒΠ»ΡŒΡ‚Π°Ρ‚ умноТСния Π΄Π²ΡƒΡ… ΠΎΡ‚Ρ€ΠΈΡ†Π°Ρ‚Π΅Π»ΡŒΠ½Ρ‹Ρ… чисСл – число ΠΏΠΎΠ»ΠΎΠΆΠΈΡ‚Π΅Π»ΡŒΠ½ΠΎΠ΅! (-4. (-5)=20) (Π°Π½Π°Π»ΠΎΠ³ΠΈΡ‡Π½ΠΎ выполняСтся Π΄Π΅Π»Π΅Π½ΠΈΠ΅ ΠΎΡ‚Ρ€ΠΈΡ†Π°Ρ‚Π΅Π»ΡŒΠ½Ρ‹Ρ… чисСл).

Π§Ρ‚ΠΎΠ±Ρ‹ ΠΏΠ΅Ρ€Π΅ΠΌΠ½ΠΎΠΆΠΈΡ‚ΡŒ Π΄Π²Π° числа с Ρ€Π°Π·Π½Ρ‹ΠΌΠΈ Π·Π½Π°ΠΊΠ°ΠΌΠΈ Π½ΡƒΠΆΠ½ΠΎ ΠΏΠ΅Ρ€Π΅ΠΌΠ½ΠΎΠΆΠΈΡ‚ΡŒ ΠΈΡ… ΠΌΠΎΠ΄ΡƒΠ»ΠΈ ΠΈ ΠΏΠ΅Ρ€Π΅Π΄ ΠΏΠΎΠ»ΡƒΡ‡Π΅Π½Π½Ρ‹ΠΌ числом ΠΏΠΎΡΡ‚Π°Π²ΠΈΡ‚ΡŒ Π·Π½Π°ΠΊ Β«-Β». Π Π΅Π·ΡƒΠ»ΡŒΡ‚Π°Ρ‚ умноТСния ΠΏΠΎΠ»ΠΎΠΆΠΈΡ‚Π΅Π»ΡŒΠ½ΠΎΠ³ΠΎ ΠΈ ΠΎΡ‚Ρ€ΠΈΡ†Π°Ρ‚Π΅Π»ΡŒΠ½ΠΎΠ³ΠΎ чисСл – ΠΎΡ‚Ρ€ΠΈΡ†Π°Ρ‚Π΅Π»ΡŒΠ½ΠΎΠ΅ число! (-4. 5=-20)



  1. РСшСниС ΡƒΡ€Π°Π²Π½Π΅Π½ΠΈΠΉ

РаскрытиС скобок

Если ΠΏΠ΅Ρ€Π΅Π΄ скобками стоит Π·Π½Π°ΠΊ Β«+Β», Ρ‚ΠΎ ΠΌΠΎΠΆΠ½ΠΎ ΠΎΠΏΡƒΡΡ‚ΠΈΡ‚ΡŒ скобки ΠΈ этот Π·Π½Π°ΠΊ, сохранив Π·Π½Π°ΠΊΠΈ слагаСмых, стоящих Π² скобках.

Если ΠΏΠ΅Ρ€Π΅Π΄ скобками стоит Π·Π½Π°ΠΊ Β«-Β», Π½Π°Π΄ΠΎ Π·Π°ΠΌΠ΅Π½ΠΈΡ‚ΡŒ этот Π·Π½Π°ΠΊ Π½Π° Β«+Β», помСняв Π·Π½Π°ΠΊΠΈ всСх слагаСмых Π² скобках Π½Π° ΠΏΡ€ΠΎΡ‚ΠΈΠ²ΠΎΠΏΠΎΠ»ΠΎΠΆΠ½Ρ‹Π΅, Π° ΠΏΠΎΡ‚ΠΎΠΌ Ρ€Π°ΡΠΊΡ€Ρ‹Ρ‚ΡŒ скобки.

Π‘Π»Π°Π³Π°Π΅ΠΌΡ‹Π΅, ΠΈΠΌΠ΅ΡŽΡ‰ΠΈΠ΅ ΠΎΠ΄ΠΈΠ½Π°ΠΊΠΎΠ²ΡƒΡŽ Π±ΡƒΠΊΠ²Π΅Π½Π½ΡƒΡŽ Ρ‡Π°ΡΡ‚ΡŒ, Π½Π°Π·Ρ‹Π²Π°ΡŽΡ‚ ΠΏΠΎΠ΄ΠΎΠ±Π½Ρ‹ΠΌΠΈ.

Π§Ρ‚ΠΎΠ±Ρ‹ привСсти ΠΏΠΎΠ΄ΠΎΠ±Π½Ρ‹Π΅ слагаСмыС, Π½Π°Π΄ΠΎ ΡΠ»ΠΎΠΆΠΈΡ‚ΡŒ ΠΈΡ… коэффициСнты ΠΈ Ρ€Π΅Π·ΡƒΠ»ΡŒΡ‚Π°Ρ‚ ΡƒΠΌΠ½ΠΎΠΆΠΈΡ‚ΡŒ Π½Π° ΠΈΡ… ΠΎΠ±Ρ‰ΡƒΡŽ Π±ΡƒΠΊΠ²Π΅Π½Π½ΡƒΡŽ Ρ‡Π°ΡΡ‚ΡŒ.

Алгоритм Ρ€Π΅ΡˆΠ΅Π½ΠΈΡ Π»ΠΈΠ½Π΅ΠΉΠ½ΠΎΠ³ΠΎ уравнСния


  1. Π Π°ΡΠΊΡ€Ρ‹Ρ‚ΡŒ скобки (Ссли Ρ‚Π°ΠΊΠΎΠ²Ρ‹Π΅ ΠΈΠΌΠ΅ΡŽΡ‚ΡΡ), ΠΈΡΠΏΠΎΠ»ΡŒΠ·ΡƒΡ ΠΏΡ€Π°Π²ΠΈΠ»Π° (см.Π²Ρ‹ΡˆΠ΅) ΠΈΠ»ΠΈ свойства умноТСния.

  2. ΠŸΠ΅Ρ€Π΅Π½Π΅ΡΡ‚ΠΈ всС слагаСмыС с ΠΏΠ΅Ρ€Π΅ΠΌΠ΅Π½Π½ΠΎΠΉ Π² ΠΎΠ΄Π½Ρƒ Ρ‡Π°ΡΡ‚ΡŒ уравнСния, Π° извСстныС слагаСмыС – Π² Π΄Ρ€ΡƒΠ³ΡƒΡŽ. ΠŸΡ€ΠΈ пСрСносС слагаСмого ΠΈΠ· ΠΎΠ΄Π½ΠΎΠΉ части уравнСния Π² Π΄Ρ€ΡƒΠ³ΡƒΡŽ Π΅Π³ΠΎ Π·Π½Π°ΠΊ мСняСтся!

  3. ΠŸΡ€ΠΈΠ²Π΅ΡΡ‚ΠΈ ΠΏΠΎΠ΄ΠΎΠ±Π½Ρ‹Π΅ слагаСмыС, ΠΈΡΠΏΠΎΠ»ΡŒΠ·ΡƒΡ дСйствия с ΠΏΠΎΠ»ΠΎΠΆΠΈΡ‚Π΅Π»ΡŒΠ½Ρ‹ΠΌΠΈ ΠΈ ΠΎΡ‚Ρ€ΠΈΡ†Π°Ρ‚Π΅Π»ΡŒΠ½Ρ‹ΠΌΠΈ числами.

  4. Π Π°Π·Π΄Π΅Π»ΠΈΡ‚ΡŒ число, стоящСС справа, Π½Π° коэффициСнт, стоящий ΠΏΠ΅Ρ€Π΅Π΄ ΠΏΠ΅Ρ€Π΅ΠΌΠ΅Π½Π½ΠΎΠΉ (слСва).

Π£ΠΌΠ½ΠΎΠΆΠ΅Π½ΠΈΠ΅ ΠΎΡ‚Ρ€ΠΈΡ†Π°Ρ‚Π΅Π»ΡŒΠ½Ρ‹Ρ… Π΄Ρ€ΠΎΠ±Π΅ΠΉ β€” Π’ΠΈΠ΄Π΅ΠΎ ΠΈ стСнограмма ΡƒΡ€ΠΎΠΊΠ°

Π£ΠΌΠ½ΠΎΠΆΠ΅Π½ΠΈΠ΅ ΠΎΡ‚Ρ€ΠΈΡ†Π°Ρ‚Π΅Π»ΡŒΠ½Ρ‹Ρ… Π΄Ρ€ΠΎΠ±Π΅ΠΉ

Π’ΠΎΡ‚ Ρ‡Π΅Ρ‚Ρ‹Ρ€Π΅ шага для умноТСния ΠΎΡ‚Ρ€ΠΈΡ†Π°Ρ‚Π΅Π»ΡŒΠ½Ρ‹Ρ… Π΄Ρ€ΠΎΠ±Π΅ΠΉ. Π”Π°Π²Π°ΠΉΡ‚Π΅ Π²ΠΎΡΠΏΠΎΠ»ΡŒΠ·ΡƒΠ΅ΠΌΡΡ ΡΠ»Π΅Π΄ΡƒΡŽΡ‰Π΅ΠΉ Π·Π°Π΄Π°Ρ‡Π΅ΠΉ, Ρ‡Ρ‚ΠΎΠ±Ρ‹ ΠΏΡ€ΠΎΠΈΠ»Π»ΡŽΡΡ‚Ρ€ΠΈΡ€ΠΎΠ²Π°Ρ‚ΡŒ шаги.

ΠŸΠΎΠΌΠ½ΠΈΡ‚Π΅, Ρ‡Ρ‚ΠΎ числитСли β€” это числа Π½Π°Π΄ Ρ‡Π΅Ρ€Ρ‚ΠΎΠΉ Π² Π΄Ρ€ΠΎΠ±Π½ΠΎΠΉ части, Π° Π·Π½Π°ΠΌΠ΅Π½Π°Ρ‚Π΅Π»ΠΈ β€” это числа ΠΏΠΎΠ΄ Ρ‡Π΅Ρ€Ρ‚ΠΎΠΉ Π² дробях.

ΠŸΠΎΠΌΠ½ΠΈΡ‚Π΅, Ρ‡Ρ‚ΠΎ Ρ†Π΅Π»ΠΎΠ΅ число ΠΌΠΎΠΆΠ½ΠΎ ΠΏΡ€Π΅Π²Ρ€Π°Ρ‚ΠΈΡ‚ΡŒ Π² Π΄Ρ€ΠΎΠ±ΡŒ, просто помСстив Ρ†Π΅Π»ΠΎΠ΅ число Π½Π°Π΄ Π΅Π΄ΠΈΠ½ΠΈΡ†Π΅ΠΉ.НапримСр, Ρ†Π΅Π»ΠΎΠ΅ число 7 становится 7/1.

ΠŸΠΎΠΏΡ€Π°ΠΊΡ‚ΠΈΠΊΡƒΠ΅ΠΌΡΡ

Π”Π°Π²Π°ΠΉΡ‚Π΅ вмСстС ΠΏΠΎΠΏΡ€ΠΎΠ±ΡƒΠ΅ΠΌ. Π”Π°Π²Π°ΠΉΡ‚Π΅ Π½Π°ΠΉΠ΄Π΅ΠΌ ΠΏΡ€ΠΎΠΈΠ·Π²Π΅Π΄Π΅Π½ΠΈΠ΅ -2/7 * 3/4 ​​

Π¨Π°Π³ 1) Когда ΠΌΡ‹ ΡƒΠΌΠ½ΠΎΠΆΠΈΠΌ числитСли (2 * 3), ΠΎΡ‚Π²Π΅Ρ‚ Π±ΡƒΠ΄Π΅Ρ‚ 6.

Π¨Π°Π³ 2) Когда ΠΌΡ‹ ΡƒΠΌΠ½ΠΎΠΆΠΈΠΌ Π·Π½Π°ΠΌΠ΅Π½Π°Ρ‚Π΅Π»ΠΈ (7 * 4), ΠΎΡ‚Π²Π΅Ρ‚ Ρ€Π°Π²Π½ΠΎ 28.

Π¨Π°Π³ 3) Новая Π΄Ρ€ΠΎΠ±ΡŒ β€” 6/28. Оба числа дСлятся Π½Π° 2 ΠΈ ΠΌΠΎΠ³ΡƒΡ‚ Π±Ρ‹Ρ‚ΡŒ ΡƒΠΏΡ€ΠΎΡ‰Π΅Π½Ρ‹ Π΄ΠΎ 3/14.

Π¨Π°Π³ 4) Π£ Π΄Ρ€ΠΎΠ±Π΅ΠΉ Ρ€Π°Π·Π½Ρ‹Π΅ Π·Π½Π°ΠΊΠΈ, Ρ‚Π°ΠΊ ΠΊΠ°ΠΊ ΠΎΠ΄Π½Π° ΠΏΠΎΠ»ΠΎΠΆΠΈΡ‚Π΅Π»ΡŒΠ½Π°Ρ, Π° другая ΠΎΡ‚Ρ€ΠΈΡ†Π°Ρ‚Π΅Π»ΡŒΠ½Π°Ρ, поэтому ΠΎΡ‚Π²Π΅Ρ‚ ΠΎΡ‚Ρ€ΠΈΡ†Π°Ρ‚Π΅Π»ΡŒΠ½Ρ‹ΠΉ.

ΠŸΡ€ΠΎΠΈΠ·Π²Π΅Π΄Π΅Π½ΠΈΠ΅ -2/7 * 3/4 ​​= -3/14.

ΠŸΠΎΠΏΡ€ΠΎΠ±ΡƒΠ΅ΠΌ Π΅Ρ‰Π΅ ΠΏΠ°Ρ€ΠΎΡ‡ΠΊΡƒ. НС ΡΡ‚Π΅ΡΠ½ΡΠΉΡ‚Π΅ΡΡŒ Π±Ρ€Π°Ρ‚ΡŒ лист Π±ΡƒΠΌΠ°Π³ΠΈ ΠΈ ΠΊΠ°Ρ€Π°Π½Π΄Π°Ρˆ ΠΈ ΡΠ°ΠΌΠΎΡΡ‚ΠΎΡΡ‚Π΅Π»ΡŒΠ½ΠΎ Ρ€Π΅ΡˆΠ°Ρ‚ΡŒ ΡΠ»Π΅Π΄ΡƒΡŽΡ‰ΠΈΠ΅ Π·Π°Π΄Π°Ρ‡ΠΈ, ΠΏΡ€Π΅ΠΆΠ΄Π΅ Ρ‡Π΅ΠΌ ΠΈΡΠΊΠ°Ρ‚ΡŒ Ρ€Π΅ΡˆΠ΅Π½ΠΈΠ΅.

Π Π΅ΡˆΠΈΡ‚ΡŒ: -3/4 * 2/3

Π¨Π°Π³ 1) Π£ΠΌΠ½ΠΎΠΆΠΈΡ‚ΡŒ числитСли: 3 * 2 = 6

Π¨Π°Π³ 2) Π£ΠΌΠ½ΠΎΠΆΠΈΡ‚ΡŒ Π·Π½Π°ΠΌΠ΅Π½Π°Ρ‚Π΅Π»ΠΈ: 4 * 3 = 12

Π¨Π°Π³ 3) Π£ΠΏΡ€ΠΎΡΡ‚ΠΈΡ‚ΡŒ 6/12 ( ΠΎΠ±Π° дСлятся Π½Π° 6) Π΄ΠΎ 1/2

Π¨Π°Π³ 4) Π—Π½Π°ΠΊΠΈ Ρ€Π°Π·Π½Ρ‹Π΅, поэтому ΠΎΡ‚Π²Π΅Ρ‚ ΠΎΡ‚Ρ€ΠΈΡ†Π°Ρ‚Π΅Π»ΡŒΠ½Ρ‹ΠΉ.

ΠžΡ‚Π²Π΅Ρ‚ -1/2.

Π Π΅ΡˆΠΈΡ‚ΡŒ: -1/2 * 3/4 ​​

Π¨Π°Π³ 1) Π£ΠΌΠ½ΠΎΠΆΠΈΡ‚ΡŒ числитСли: 1 * 3 = 3

Π¨Π°Π³ 2) Π£ΠΌΠ½ΠΎΠΆΠΈΡ‚ΡŒ Π·Π½Π°ΠΌΠ΅Π½Π°Ρ‚Π΅Π»ΠΈ: 2 * 4 = 8

Π¨Π°Π³ 3) 3/8 ΡƒΠΆΠ΅ Π² ΠΏΡ€ΠΎΡΡ‚Π΅ΠΉΡˆΠ΅ΠΉ Ρ„ΠΎΡ€ΠΌΠ΅

Π¨Π°Π³ 4) Π—Π½Π°ΠΊΠΈ Ρ€Π°Π·Π½Ρ‹Π΅, поэтому ΠΎΡ‚Π²Π΅Ρ‚ ΠΎΡ‚Ρ€ΠΈΡ†Π°Ρ‚Π΅Π»ΡŒΠ½Ρ‹ΠΉ.

Π˜Ρ‚Π°ΠΊ, -1/2 * 3/4 ​​= -3/8.

ΠšΡ€Π°Ρ‚ΠΊΠΎΠ΅ содСрТаниС ΡƒΡ€ΠΎΠΊΠ°

ΠŸΡ€ΠΈ ΡƒΠΌΠ½ΠΎΠΆΠ΅Π½ΠΈΠΈ Π΄Ρ€ΠΎΠ±Π΅ΠΉ ΠΏΠΎΠΌΠ½ΠΈΡ‚Π΅, Ρ‡Ρ‚ΠΎ Ссли Π΄Ρ€ΠΎΠ±ΠΈ ΠΈΠΌΠ΅ΡŽΡ‚ ΠΎΠ΄ΠΈΠ½Π°ΠΊΠΎΠ²Ρ‹ΠΉ Π·Π½Π°ΠΊ, ΠΎΡ‚Π²Π΅Ρ‚ Π±ΡƒΠ΄Π΅Ρ‚ ΠΏΠΎΠ»ΠΎΠΆΠΈΡ‚Π΅Π»ΡŒΠ½Ρ‹ΠΌ, Π° Ссли Π΄Ρ€ΠΎΠ±ΠΈ ΠΈΠΌΠ΅ΡŽΡ‚ Ρ€Π°Π·Π½Ρ‹Π΅ Π·Π½Π°ΠΊΠΈ, ΠΎΡ‚Π²Π΅Ρ‚ Π±ΡƒΠ΄Π΅Ρ‚ ΠΎΡ‚Ρ€ΠΈΡ†Π°Ρ‚Π΅Π»ΡŒΠ½Ρ‹ΠΌ.Π’Π°ΠΊΠΆΠ΅ ΠΏΠΎΠΌΠ½ΠΈΡ‚Π΅, Ρ‡Ρ‚ΠΎ ΡƒΠΌΠ½ΠΎΠΆΠ΅Π½ΠΈΠ΅ ΠΎΡ‚Ρ€ΠΈΡ†Π°Ρ‚Π΅Π»ΡŒΠ½Ρ‹Ρ… Π΄Ρ€ΠΎΠ±Π΅ΠΉ состоит ΠΈΠ· Ρ‡Π΅Ρ‚Ρ‹Ρ€Π΅Ρ… простых шагов, ΠΈ это, бСзусловно, ΠΏΠΎΠ»ΠΎΠΆΠΈΡ‚Π΅Π»ΡŒΠ½Ρ‹ΠΉ ΠΌΠΎΠΌΠ΅Π½Ρ‚!

Π£ΠΏΡ€ΠΎΡ‰Π΅Π½ΠΈΠ΅ выраТСния с ΠΏΠΎΠΌΠΎΡ‰ΡŒΡŽ Π΄Ρ€ΠΎΠ±ΠΈ

Π Π΅Π·ΡƒΠ»ΡŒΡ‚Π°Ρ‚Ρ‹ обучСния
  • ΠžΠΏΡ€Π΅Π΄Π΅Π»ΠΈΡ‚Π΅ ΠΎΡ‚Ρ€ΠΈΡ†Π°Ρ‚Π΅Π»ΡŒΠ½Ρ‹Π΅ Π΄Ρ€ΠΎΠ±ΠΈ, ΠΊΠΎΡ‚ΠΎΡ€Ρ‹Π΅ эквивалСнтны, учитывая, Ρ‡Ρ‚ΠΎ ΠΈΡ… ΠΎΡ‚Ρ€ΠΈΡ†Π°Ρ‚Π΅Π»ΡŒΠ½Ρ‹ΠΉ Π·Π½Π°ΠΊ находится Π² Π΄Ρ€ΡƒΠ³ΠΎΠΌ мСстС
  • УпроститС выраТСния, содСрТащиС Π΄Ρ€ΠΎΠ±Π½Ρ‹Π΅ Ρ‡Π΅Ρ€Ρ‚Ρ‹, ΠΈΡΠΏΠΎΠ»ΡŒΠ·ΡƒΡ порядок ΠΎΠΏΠ΅Ρ€Π°Ρ†ΠΈΠΉ

Π“Π΄Π΅ ΠΈΠ΄Π΅Ρ‚ Π·Π½Π°ΠΊ минус Π² Π΄Ρ€ΠΎΠ±ΠΈ? ΠžΠ±Ρ‹Ρ‡Π½ΠΎ ΠΏΠ΅Ρ€Π΅Π΄ Π΄Ρ€ΠΎΠ±ΡŒΡŽ ставится Π·Π½Π°ΠΊ минус, Π½ΠΎ ΠΈΠ½ΠΎΠ³Π΄Π° ΠΌΠΎΠΆΠ½ΠΎ ΡƒΠ²ΠΈΠ΄Π΅Ρ‚ΡŒ Π΄Ρ€ΠΎΠ±ΡŒ с ΠΎΡ‚Ρ€ΠΈΡ†Π°Ρ‚Π΅Π»ΡŒΠ½Ρ‹ΠΌ числитСлСм ΠΈΠ»ΠΈ Π·Π½Π°ΠΌΠ΅Π½Π°Ρ‚Π΅Π»Π΅ΠΌ. ΠŸΠΎΠΌΠ½ΠΈΡ‚Π΅, Ρ‡Ρ‚ΠΎ Π΄Ρ€ΠΎΠ±ΠΈ ΠΏΡ€Π΅Π΄ΡΡ‚Π°Π²Π»ΡΡŽΡ‚ собой Π΄Π΅Π»Π΅Π½ΠΈΠ΅. Π”Ρ€ΠΎΠ±ΡŒ [латСкс] β€” \ frac {1} {3} [/ latex] ΠΌΠΎΠΆΠ΅Ρ‚ Π±Ρ‹Ρ‚ΡŒ Ρ€Π΅Π·ΡƒΠ»ΡŒΡ‚Π°Ρ‚ΠΎΠΌ дСлСния [latex] \ frac {-1} {3} [/ latex], ΠΎΡ‚Ρ€ΠΈΡ†Π°Ρ‚Π΅Π»ΡŒΠ½ΠΎΠ³ΠΎ Π½Π° ΠΏΠΎΠ»ΠΎΠΆΠΈΡ‚Π΅Π»ΡŒΠ½Ρ‹ΠΉ ΠΈΠ»ΠΈ дСлСния [latex] \ frac {1} {- 3} [/ latex], ΠΏΠΎΠ»ΠΎΠΆΠΈΡ‚Π΅Π»ΡŒΠ½ΠΎΠ΅ Π·Π° ΠΎΡ‚Ρ€ΠΈΡ†Π°Ρ‚Π΅Π»ΡŒΠ½Ρ‹ΠΌ. Когда Ρ‡ΠΈΡΠ»ΠΈΡ‚Π΅Π»ΡŒ ΠΈ Π·Π½Π°ΠΌΠ΅Π½Π°Ρ‚Π΅Π»ΡŒ ΠΈΠΌΠ΅ΡŽΡ‚ Ρ€Π°Π·Π½Ρ‹Π΅ Π·Π½Π°ΠΊΠΈ, частноС ΠΎΡ‚Ρ€ΠΈΡ†Π°Ρ‚Π΅Π»ΡŒΠ½ΠΎΠ΅.


Если ΠΈ Ρ‡ΠΈΡΠ»ΠΈΡ‚Π΅Π»ΡŒ, ΠΈ Π·Π½Π°ΠΌΠ΅Π½Π°Ρ‚Π΅Π»ΡŒ ΠΎΡ‚Ρ€ΠΈΡ†Π°Ρ‚Π΅Π»ΡŒΠ½Ρ‹, Ρ‚ΠΎΠ³Π΄Π° сама Π΄Ρ€ΠΎΠ±ΡŒ ΠΏΠΎΠ»ΠΎΠΆΠΈΡ‚Π΅Π»ΡŒΠ½Π°, ΠΏΠΎΡ‚ΠΎΠΌΡƒ Ρ‡Ρ‚ΠΎ ΠΌΡ‹ Π΄Π΅Π»ΠΈΠΌ ΠΎΡ‚Ρ€ΠΈΡ†Π°Ρ‚Π΅Π»ΡŒΠ½ΠΎΠ΅ Π½Π° ΠΎΡ‚Ρ€ΠΈΡ†Π°Ρ‚Π΅Π»ΡŒΠ½ΠΎΠ΅.

[латСкс] \ frac {-1} {- 3} = \ frac {1} {3} \ frac {\ text {negative}} {\ text {negative}} = \ text {positive} [/ latex]

Π Π°Π·ΠΌΠ΅Ρ‰Π΅Π½ΠΈΠ΅ ΠΎΡ‚Ρ€ΠΈΡ†Π°Ρ‚Π΅Π»ΡŒΠ½ΠΎΠ³ΠΎ Π·Π½Π°ΠΊΠ° Π² Π΄Ρ€ΠΎΠ±ΠΈ

Для Π»ΡŽΠ±Ρ‹Ρ… ΠΏΠΎΠ»ΠΎΠΆΠΈΡ‚Π΅Π»ΡŒΠ½Ρ‹Ρ… чисСл [латСкс] a \ text {ΠΈ} b [/ latex],

[латСкс] \ frac {-a} {b} = \ frac {a} {- b} = β€” \ frac {a} {b} [/ latex]

ΠŸΡ€ΠΈΠΌΠ΅Ρ€

Какая ΠΈΠ· ΡΠ»Π΅Π΄ΡƒΡŽΡ‰ΠΈΡ… Ρ„Ρ€Π°ΠΊΡ†ΠΈΠΉ эквивалСнтна [latex] \ frac {7} {- 8}? [/ Latex]

[латСкс] \ frac {-7} {- 8}, \ frac {-7} {8}, \ frac {7} {8}, β€” \ frac {7} {8} [/ latex]

РСшСниС:
ЧастноС ΠΏΠΎΠ»ΠΎΠΆΠΈΡ‚Π΅Π»ΡŒΠ½ΠΎΠ³ΠΎ ΠΈ ΠΎΡ‚Ρ€ΠΈΡ†Π°Ρ‚Π΅Π»ΡŒΠ½ΠΎΠ³ΠΎ ΠΎΡ‚Ρ€ΠΈΡ†Π°Ρ‚Π΅Π»ΡŒΠ½ΠΎ, поэтому [latex] \ frac {7} {- 8} [/ latex] ΠΎΡ‚Ρ€ΠΈΡ†Π°Ρ‚Π΅Π»ΡŒΠ½ΠΎ. Из пСрСчислСнных Ρ„Ρ€Π°ΠΊΡ†ΠΈΠΉ [latex] \ frac {-7} {8} \ text {ΠΈ} β€” \ frac {7} {8} [/ latex] Ρ‚Π°ΠΊΠΆΠ΅ ΠΎΡ‚Ρ€ΠΈΡ†Π°Ρ‚Π΅Π»ΡŒΠ½Ρ‹.

Π£ΠΏΡ€ΠΎΡ‰Π΅Π½ΠΈΠ΅ выраТСния с ΠΏΠΎΠΌΠΎΡ‰ΡŒΡŽ Π΄Ρ€ΠΎΠ±ΠΈ

Полоски Π΄Ρ€ΠΎΠ±ΠΈ Π΄Π΅ΠΉΡΡ‚Π²ΡƒΡŽΡ‚ ΠΊΠ°ΠΊ символы Π³Ρ€ΡƒΠΏΠΏΠΈΡ€ΠΎΠ²ΠΊΠΈ. ВыраТСния Π½Π°Π΄ ΠΈ ΠΏΠΎΠ΄ Π΄Ρ€ΠΎΠ±Π½ΠΎΠΉ Ρ‡Π΅Ρ€Ρ‚ΠΎΠΉ слСдуСт Ρ€Π°ΡΡΠΌΠ°Ρ‚Ρ€ΠΈΠ²Π°Ρ‚ΡŒ Ρ‚Π°ΠΊ, ΠΊΠ°ΠΊ Ссли Π±Ρ‹ ΠΎΠ½ΠΈ Π±Ρ‹Π»ΠΈ Π·Π°ΠΊΠ»ΡŽΡ‡Π΅Π½Ρ‹ Π² ΠΊΡ€ΡƒΠ³Π»Ρ‹Π΅ скобки. НапримСр, [латСкс] \ frac {4 + 8} {5 β€” 3} [/ latex] ΠΎΠ·Π½Π°Ρ‡Π°Π΅Ρ‚ [латСкс] \ left (4 + 8 \ right) \ div \ left (5 β€” 3 \ right) [/ latex] . ΠŸΠΎΡ€ΡΠ΄ΠΎΠΊ ΠΎΠΏΠ΅Ρ€Π°Ρ†ΠΈΠΉ Π³ΠΎΠ²ΠΎΡ€ΠΈΡ‚ Π½Π°ΠΌ сначала ΡƒΠΏΡ€ΠΎΡΡ‚ΠΈΡ‚ΡŒ Ρ‡ΠΈΡΠ»ΠΈΡ‚Π΅Π»ΡŒ ΠΈ Π·Π½Π°ΠΌΠ΅Π½Π°Ρ‚Π΅Π»ΡŒ β€” ΠΊΠ°ΠΊ Ссли Π±Ρ‹ Π±Ρ‹Π»ΠΈ ΠΊΡ€ΡƒΠ³Π»Ρ‹Π΅ скобки β€” ΠΏΠ΅Ρ€Π΅Π΄ Ρ‚Π΅ΠΌ, ΠΊΠ°ΠΊ Π΄Π΅Π»ΠΈΡ‚ΡŒ.
ΠœΡ‹ Π΄ΠΎΠ±Π°Π²ΠΈΠΌ Π΄Ρ€ΠΎΠ±Π½Ρ‹Π΅ Ρ‡Π΅Ρ€Ρ‚Ρ‹ ΠΊ Π½Π°ΡˆΠ΅ΠΌΡƒ Π½Π°Π±ΠΎΡ€Ρƒ символов Π³Ρ€ΡƒΠΏΠΏΠΈΡ€ΠΎΠ²ΠΊΠΈ ΠΈΠ· Ρ€Π°Π·Π΄Π΅Π»Π° «ИспользованиС языка Π°Π»Π³Π΅Π±Ρ€Ρ‹Β», Ρ‡Ρ‚ΠΎΠ±Ρ‹ ΠΏΠΎΠ»ΡƒΡ‡ΠΈΡ‚ΡŒ здСсь Π±ΠΎΠ»Π΅Π΅ ΠΏΠΎΠ»Π½Ρ‹ΠΉ Π½Π°Π±ΠΎΡ€.

Π“Ρ€ΡƒΠΏΠΏΠΈΡ€ΠΎΠ²ΠΊΠ° символов
УпроститС Π²Ρ‹Ρ€Π°ΠΆΠ΅Π½ΠΈΠ΅ с ΠΏΠΎΠΌΠΎΡ‰ΡŒΡŽ Π΄Ρ€ΠΎΠ±Π½ΠΎΠΉ Π»ΠΈΠ½Π΅ΠΉΠΊΠΈ
  1. Упростим Ρ‡ΠΈΡΠ»ΠΈΡ‚Π΅Π»ΡŒ.
  2. Упростим Π·Π½Π°ΠΌΠ΅Π½Π°Ρ‚Π΅Π»ΡŒ.
  3. УпроститС Π΄Ρ€ΠΎΠ±ΡŒ.
ΠŸΡ€ΠΈΠΌΠ΅Ρ€

Π£ΠΏΡ€ΠΎΡΡ‚ΠΈΡ‚ΡŒ: [латСкс] \ frac {4 + 8} {5 β€” 3} [/ латСкс]

ΠŸΠΎΠΊΠ°Π·Π°Ρ‚ΡŒ Ρ€Π΅ΡˆΠ΅Π½ΠΈΠ΅

РСшСниС:

» data-label=»»>
[латСкс] \ frac {4 + 8} {5 β€” 3} [/ латСкс]
УпроститС Π²Ρ‹Ρ€Π°ΠΆΠ΅Π½ΠΈΠ΅ Π² числитСлС. [латСкс] \ frac {12} {5 β€” 3} [/ латСкс]
Упростим Π²Ρ‹Ρ€Π°ΠΆΠ΅Π½ΠΈΠ΅ Π² Π·Π½Π°ΠΌΠ΅Π½Π°Ρ‚Π΅Π»Π΅. {2} +2} [/ latex]
Π˜ΡΠΏΠΎΠ»ΡŒΠ·ΡƒΠΉΡ‚Π΅ порядок ΠΎΠΏΠ΅Ρ€Π°Ρ†ΠΈΠΉ.{2}} {64 β€” 16} [/ латСкс]
Упростим Ρ‡ΠΈΡΠ»ΠΈΡ‚Π΅Π»ΡŒ ΠΈ Π·Π½Π°ΠΌΠ΅Π½Π°Ρ‚Π΅Π»ΡŒ. [латСкс] \ frac {16} {48} [/ латСкс]
Упростим Π΄Ρ€ΠΎΠ±ΡŒ. [латСкс] \ frac {1} {3} [/ латСкс]
ΠŸΡ€ΠΈΠΌΠ΅Ρ€

Π£ΠΏΡ€ΠΎΡΡ‚ΠΈΡ‚ΡŒ: [латСкс] \ frac {4 \ left (-3 \ right) +6 \ left (-2 \ right)} {- 3 \ left (2 \ right) -2} [/ latex]

ΠŸΠΎΠΊΠ°Π·Π°Ρ‚ΡŒ Ρ€Π΅ΡˆΠ΅Π½ΠΈΠ΅

РСшСниС:

[латСкс] \ frac {4 \ left (-3 \ right) +6 \ left (-2 \ right)} {- 3 \ left (2 \ right) -2} [/ латСкс]
Π£ΠΌΠ½ΠΎΠΆΠΈΡ‚ΡŒ. [латСкс] \ frac {-12+ \ left (-12 \ right)} {- 6 β€” 2} [/ латСкс]
Π£ΠΏΡ€ΠΎΡΡ‚ΠΈΡ‚ΡŒ. [латСкс] \ frac {-24} {- 8} [/ латСкс]
Π Π°Π·Π΄Π΅Π»ΠΈΡ‚ΡŒ. [латСкс] 3 [/ латСкс]

ΠŸΠΎΡΠΌΠΎΡ‚Ρ€ΠΈΡ‚Π΅ это Π²ΠΈΠ΄Π΅ΠΎ, Ρ‡Ρ‚ΠΎΠ±Ρ‹ ΡƒΠ²ΠΈΠ΄Π΅Ρ‚ΡŒ Π΅Ρ‰Π΅ ΠΎΠ΄ΠΈΠ½ ΠΏΡ€ΠΈΠΌΠ΅Ρ€ Ρ‚ΠΎΠ³ΠΎ, ΠΊΠ°ΠΊ ΡƒΠΏΡ€ΠΎΡΡ‚ΠΈΡ‚ΡŒ Π²Ρ‹Ρ€Π°ΠΆΠ΅Π½ΠΈΠ΅ с ΠΏΠΎΠΌΠΎΡ‰ΡŒΡŽ Π΄Ρ€ΠΎΠ±Π½ΠΎΠΉ Π»ΠΈΠ½Π΅ΠΉΠΊΠΈ, содСрТащСй нСсколько Ρ€Π°Π·Π»ΠΈΡ‡Π½Ρ‹Ρ… ΠΎΠΏΠ΅Ρ€Π°Ρ†ΠΈΠΉ.

Π£ΠΌΠ½ΠΎΠΆΠ΅Π½ΠΈΠ΅ ΠΏΠΎΠ»ΠΎΠΆΠΈΡ‚Π΅Π»ΡŒΠ½Ρ‹Ρ… ΠΈ ΠΎΡ‚Ρ€ΠΈΡ†Π°Ρ‚Π΅Π»ΡŒΠ½Ρ‹Ρ… чисСл: 3 простых ΠΏΡ€Π°Π²ΠΈΠ»Π°

ΠŸΡ€ΠΈ ΡƒΠΌΠ½ΠΎΠΆΠ΅Π½ΠΈΠΈ ΠΏΠΎΠ»ΠΎΠΆΠΈΡ‚Π΅Π»ΡŒΠ½Ρ‹Ρ… ΠΈ ΠΎΡ‚Ρ€ΠΈΡ†Π°Ρ‚Π΅Π»ΡŒΠ½Ρ‹Ρ… чисСл сущСствуСт мСньшС ΠΏΡ€Π°Π²ΠΈΠ», Ρ‡Π΅ΠΌ ΠΏΡ€ΠΈ слоТСнии ΠΈ Π²Ρ‹Ρ‡ΠΈΡ‚Π°Π½ΠΈΠΈ ΠΏΠΎΠ»ΠΎΠΆΠΈΡ‚Π΅Π»ΡŒΠ½Ρ‹Ρ… ΠΈ ΠΎΡ‚Ρ€ΠΈΡ†Π°Ρ‚Π΅Π»ΡŒΠ½Ρ‹Ρ… чисСл.Π‘Π»Π΅Π΄ΡƒΠ΅Ρ‚ Π·Π°ΠΏΠΎΠΌΠ½ΠΈΡ‚ΡŒ всСго Ρ‚Ρ€ΠΈ ΠΏΡ€Π°Π²ΠΈΠ»Π°:

ΠŸΡ€Π°Π²ΠΈΠ»ΠΎ 1. ΠŸΠΎΠ»ΠΎΠΆΠΈΡ‚Π΅Π»ΡŒΠ½ΠΎΠ΅ число, ΡƒΠΌΠ½ΠΎΠΆΠ΅Π½Π½ΠΎΠ΅ Π½Π° ΠΏΠΎΠ»ΠΎΠΆΠΈΡ‚Π΅Π»ΡŒΠ½ΠΎΠ΅, равняСтся ΠΏΠΎΠ»ΠΎΠΆΠΈΡ‚Π΅Π»ΡŒΠ½ΠΎΠΌΡƒ числу.

Π­Ρ‚ΠΎ ΡƒΠΌΠ½ΠΎΠΆΠ΅Π½ΠΈΠ΅, ΠΊΠΎΡ‚ΠΎΡ€ΠΎΠ΅ Π²Ρ‹ Π΄Π΅Π»Π°Π»ΠΈ всС врСмя: ΠΏΠΎΠ»ΠΎΠΆΠΈΡ‚Π΅Π»ΡŒΠ½Ρ‹Π΅ числа, ΡƒΠΌΠ½ΠΎΠΆΠ΅Π½Π½Ρ‹Π΅ Π½Π° ΠΏΠΎΠ»ΠΎΠΆΠΈΡ‚Π΅Π»ΡŒΠ½Ρ‹Π΅, Ρ€Π°Π²Π½Ρ‹ ΠΏΠΎΠ»ΠΎΠΆΠΈΡ‚Π΅Π»ΡŒΠ½Ρ‹ΠΌ числам.

НапримСр, 5 x 3 = 15. 5 β€” ΠΏΠΎΠ»ΠΎΠΆΠΈΡ‚Π΅Π»ΡŒΠ½ΠΎΠ΅ число, 3 β€” ΠΏΠΎΠ»ΠΎΠΆΠΈΡ‚Π΅Π»ΡŒΠ½ΠΎΠ΅ число, Π° ΡƒΠΌΠ½ΠΎΠΆΠ΅Π½ΠΈΠ΅ Π΄Π°Π΅Ρ‚ ΠΏΠΎΠ»ΠΎΠΆΠΈΡ‚Π΅Π»ΡŒΠ½ΠΎΠ΅ число: 15.

ΠžΡ‚Π²Π΅Ρ‚: 5 x 3 = 15.

ΠŸΡ€Π°Π²ΠΈΠ»ΠΎ 2: ΠžΡ‚Ρ€ΠΈΡ†Π°Ρ‚Π΅Π»ΡŒΠ½ΠΎΠ΅ число, ΡƒΠΌΠ½ΠΎΠΆΠ΅Π½Π½ΠΎΠ΅ Π½Π° ΠΏΠΎΠ»ΠΎΠΆΠΈΡ‚Π΅Π»ΡŒΠ½ΠΎΠ΅, равняСтся ΠΎΡ‚Ρ€ΠΈΡ†Π°Ρ‚Π΅Π»ΡŒΠ½ΠΎΠΌΡƒ числу.

Когда Π²Ρ‹ ΡƒΠΌΠ½ΠΎΠΆΠ°Π΅Ρ‚Π΅ ΠΎΡ‚Ρ€ΠΈΡ†Π°Ρ‚Π΅Π»ΡŒΠ½ΠΎΠ΅ число Π½Π° ΠΏΠΎΠ»ΠΎΠΆΠΈΡ‚Π΅Π»ΡŒΠ½ΠΎΠ΅, ваш ΠΎΡ‚Π²Π΅Ρ‚ β€” ΠΎΡ‚Ρ€ΠΈΡ†Π°Ρ‚Π΅Π»ΡŒΠ½ΠΎΠ΅ число. НСваТно, Π² ΠΊΠ°ΠΊΠΎΠΌ порядкС Π²Ρ‹ ΡƒΠΌΠ½ΠΎΠΆΠ°Π΅Ρ‚Π΅ ΠΏΠΎΠ»ΠΎΠΆΠΈΡ‚Π΅Π»ΡŒΠ½Ρ‹Π΅ ΠΈ ΠΎΡ‚Ρ€ΠΈΡ†Π°Ρ‚Π΅Π»ΡŒΠ½Ρ‹Π΅ числа, ΠΎΡ‚Π²Π΅Ρ‚ всСгда Π±ΡƒΠ΄Π΅Ρ‚ ΠΎΡ‚Ρ€ΠΈΡ†Π°Ρ‚Π΅Π»ΡŒΠ½Ρ‹ΠΌ.

НапримСр: -2 x 4, Ρ‡Ρ‚ΠΎ ΠΏΠΎ сути Ρ‚ΠΎ ΠΆΠ΅ самоС, Ρ‡Ρ‚ΠΎ -2 + (-2) + (-2) + (-2)

ΠžΡ‚Π²Π΅Ρ‚: -2 Ρ… 4 = -8.

И, ΠΊΠ°ΠΊ ΠΌΡ‹ ΡƒΠΆΠ΅ сказали, Ссли ΠΎΠΊΠΎΠ»ΠΎ 4 x -2 Π½Π°ΠΎΠ±ΠΎΡ€ΠΎΡ‚, ΠΎΡ‚Π²Π΅Ρ‚ всС Ρ‚ΠΎΡ‚ ΠΆΠ΅: -8.

ΠžΡ‚Π²Π΅Ρ‚: 4 Ρ… -2 = -8.

ΠŸΡ€Π°Π²ΠΈΠ»ΠΎ 3. ΠžΡ‚Ρ€ΠΈΡ†Π°Ρ‚Π΅Π»ΡŒΠ½ΠΎΠ΅ число, ΡƒΠΌΠ½ΠΎΠΆΠ΅Π½Π½ΠΎΠ΅ Π½Π° ΠΎΡ‚Ρ€ΠΈΡ†Π°Ρ‚Π΅Π»ΡŒΠ½ΠΎΠ΅ число, равняСтся ΠΏΠΎΠ»ΠΎΠΆΠΈΡ‚Π΅Π»ΡŒΠ½ΠΎΠΌΡƒ числу.

Π”Π²Π° ΠΎΡ‚Ρ€ΠΈΡ†Π°Ρ‚Π΅Π»ΡŒΠ½Ρ‹Ρ… числа Π΄Π°ΡŽΡ‚ ΠΏΠΎΠ»ΠΎΠΆΠΈΡ‚Π΅Π»ΡŒΠ½ΠΎΠ΅ число, поэтому ΡƒΠΌΠ½ΠΎΠΆΠ΅Π½Π½ΠΎΠ΅ Π½Π° ΠΎΡ‚Ρ€ΠΈΡ†Π°Ρ‚Π΅Π»ΡŒΠ½ΠΎΠ΅ число Π΄Π°Π΅Ρ‚ ΠΏΠΎΠ»ΠΎΠΆΠΈΡ‚Π΅Π»ΡŒΠ½ΠΎΠ΅ число. Если Π²Ρ‹ посмотритС Π½Π° Π½Π΅Π³ΠΎ Π½Π° числовой прямой, идя Π½Π°Π·Π°Π΄ ΠΈ ΠΏΠΎΠ²Π΅Ρ€Π½ΡƒΠ²ΡˆΠΈΡΡŒ Π»ΠΈΡ†ΠΎΠΌ Π² ΠΎΡ‚Ρ€ΠΈΡ†Π°Ρ‚Π΅Π»ΡŒΠ½ΠΎΠΌ Π½Π°ΠΏΡ€Π°Π²Π»Π΅Π½ΠΈΠΈ, Π²Ρ‹ Π΄Π²ΠΈΠ½Π΅Ρ‚Π΅ΡΡŒ Π² ΠΏΠΎΠ»ΠΎΠΆΠΈΡ‚Π΅Π»ΡŒΠ½ΠΎΠΌ Π½Π°ΠΏΡ€Π°Π²Π»Π΅Π½ΠΈΠΈ.

НапримСр. -2 x -4 ΠΎΡ‚Ρ€ΠΈΡ†Π°Ρ‚Π΅Π»ΡŒΠ½Ρ‹, поэтому ΠΌΡ‹ Π·Π½Π°Π΅ΠΌ, Ρ‡Ρ‚ΠΎ ΠΎΡ‚Π²Π΅Ρ‚ Π±ΡƒΠ΄Π΅Ρ‚ ΠΏΠΎΠ»ΠΎΠΆΠΈΡ‚Π΅Π»ΡŒΠ½Ρ‹ΠΌ.

ΠžΡ‚Π²Π΅Ρ‚: -2 x -4 = 8.

Π’ΠΎΡ‚ ΠΎΠ±Ρ‰Π΅Π΅ ΠΏΡ€Π°Π²ΠΈΠ»ΠΎ, ΠΊΠΎΡ‚ΠΎΡ€ΠΎΠ΅ слСдуСт ΠΏΠΎΠΌΠ½ΠΈΡ‚ΡŒ ΠΏΡ€ΠΈ ΡƒΠΌΠ½ΠΎΠΆΠ΅Π½ΠΈΠΈ ΠΏΠΎΠ»ΠΎΠΆΠΈΡ‚Π΅Π»ΡŒΠ½Ρ‹Ρ… ΠΈ ΠΎΡ‚Ρ€ΠΈΡ†Π°Ρ‚Π΅Π»ΡŒΠ½Ρ‹Ρ… чисСл:

Π”Π²Π° ΠΎΠ΄ΠΈΠ½Π°ΠΊΠΎΠ²Ρ‹Ρ… Π·Π½Π°ΠΊΠ° Π΄Π°ΡŽΡ‚ ΠΏΠΎΠ»ΠΎΠΆΠΈΡ‚Π΅Π»ΡŒΠ½Ρ‹ΠΉ Π·Π½Π°ΠΊ:

Π”Π²Π° Π½Π΅ΠΏΠΎΡ…ΠΎΠΆΠΈΡ… Π·Π½Π°ΠΊΠ° ΠΎΠ±Ρ€Π°Π·ΡƒΡŽΡ‚ ΠΎΡ‚Ρ€ΠΈΡ†Π°Ρ‚Π΅Π»ΡŒΠ½Ρ‹ΠΉ Π·Π½Π°ΠΊ:

Если Π²Ρ‹ всС Π΅Ρ‰Π΅ Π½Π΅ ΠΏΠΎΠ½ΠΈΠΌΠ°Π΅Ρ‚Π΅, ΠΏΠΎΡ‡Π΅ΠΌΡƒ ΠΎΡ‚Ρ€ΠΈΡ†Π°Ρ‚Π΅Π»ΡŒΠ½ΠΎΠ΅ число, ΡƒΠΌΠ½ΠΎΠΆΠ΅Π½Π½ΠΎΠ΅ Π½Π° ΠΎΡ‚Ρ€ΠΈΡ†Π°Ρ‚Π΅Π»ΡŒΠ½ΠΎΠ΅, Π΄Π°Π΅Ρ‚ ΠΏΠΎΠ»ΠΎΠΆΠΈΡ‚Π΅Π»ΡŒΠ½ΠΎΠ΅ число, Π”ΠΈΠ°Π½Π° Π‘Ρ€Π°ΡƒΠ½ с Ρ„Π°ΠΊΡƒΠ»ΡŒΡ‚Π΅Ρ‚Π° ΠΌΠ°Ρ‚Π΅ΠΌΠ°Ρ‚ΠΈΠΊΠΈ УнивСрситСта Π”ΠΆΠΎΡ€Π΄ΠΆΠΈΠΈ ΠΎΠ±ΡŠΡΡΠ½ΡΠ΅Ρ‚ это Ρ€Π°Π·Π½Ρ‹ΠΌΠΈ способами Π² этой ΡΡ‚Π°Ρ‚ΡŒΠ΅.

Π‘ΠΊΠΎΡ‚Ρ‚ ΠΈΠ· About.com Ρ‚Π°ΠΊΠΆΠ΅ собрал ΡƒΠ΄ΠΎΠ±Π½ΠΎΠ΅ Π²ΠΈΠ΄Π΅ΠΎ ΠΎ Ρ‚ΠΎΠΌ, ΠΊΠ°ΠΊ ΡΠΎΠ·Π΄Π°Ρ‚ΡŒ ΡˆΠΏΠ°Ρ€Π³Π°Π»ΠΊΡƒ для умноТСния ΠΎΡ‚Ρ€ΠΈΡ†Π°Ρ‚Π΅Π»ΡŒΠ½Ρ‹Ρ… ΠΈ ΠΏΠΎΠ»ΠΎΠΆΠΈΡ‚Π΅Π»ΡŒΠ½Ρ‹Ρ… чисСл (ΠΏΡ€ΠΎΠΊΡ€ΡƒΡ‚ΠΈΡ‚Π΅ страницу Π²Π½ΠΈΠ·, ΠΈ Π²Ρ‹ Π½Π°ΠΉΠ΄Π΅Ρ‚Π΅ Π²ΠΈΠ΄Π΅ΠΎ).

Π£ΠΌΠ½ΠΎΠΆΠ΅Π½ΠΈΠ΅ ΠΈ Π΄Π΅Π»Π΅Π½ΠΈΠ΅ ΠΎΡ‚Ρ€ΠΈΡ†Π°Ρ‚Π΅Π»ΡŒΠ½Ρ‹Ρ… Π΄Ρ€ΠΎΠ±Π΅ΠΉ ΠΈ ΡΠΌΠ΅ΡˆΠ°Π½Π½Ρ‹Ρ… чисСл

Π”Ρ€ΠΎΠ±ΠΈ Π‘ΠΌΠ΅ΡˆΠ°Π½Π½Ρ‹Π΅ числа ΠΈ Π½Π΅ΠΏΡ€Π°Π²ΠΈΠ»ΡŒΠ½Ρ‹Π΅ Π΄Ρ€ΠΎΠ±ΠΈ. БмСшанноС число содСрТит Ρ†Π΅Π»ΡƒΡŽ Ρ‡Π°ΡΡ‚ΡŒ ΠΈ Π΄Ρ€ΠΎΠ±Π½ΡƒΡŽ Ρ‡Π°ΡΡ‚ΡŒ. смСшанноС число. Он содСрТит ΠΊΠ°ΠΊ Ρ†Π΅Π»ΡƒΡŽ Ρ‡Π°ΡΡ‚ΡŒ, 3, Ρ‚Π°ΠΊ ΠΈ Π΄Ρ€ΠΎΠ±Π½ΡƒΡŽ Ρ‡Π°ΡΡ‚ΡŒ, 2/5. ΠœΡ‹ Ρ‡ΠΈΡ‚Π°Π΅ΠΌ Π΄Ρ€ΠΎΠ±ΡŒ ΠΊΠ°ΠΊ Β«Ρ‚Ρ€ΠΈ ΠΈ Π΄Π²Π΅ пятых», ΠΈ это ΠΈΠΌΠ΅Π½Π½ΠΎ Ρ‚ΠΎ, Ρ‡Ρ‚ΠΎ ΠΌΡ‹ ΠΈΠΌΠ΅Π΅ΠΌ Π² Π²ΠΈΠ΄Ρƒ.= Π”ΠΎΠ±Π°Π²Π»Π΅Π½ΠΈΠ΅ Ρ†Π΅Π»ΠΎΠ³ΠΎ числа ΠΊ Π΄Ρ€ΠΎΠ±ΠΈ β€” это особый случай слоТСния Π΄Π²ΡƒΡ… … БСсплатныС Ρ€Π°Π±ΠΎΡ‡ΠΈΠ΅ листы ΠΏΠΎ ΠΌΠ°Ρ‚Π΅ΠΌΠ°Ρ‚ΠΈΠΊΠ΅ для слоТСния, вычитания, умноТСния, срСднСго, дСлСния, Π°Π»Π³Π΅Π±Ρ€Ρ‹ ΠΈ Ρ‚Π΅ΠΌ мСньшС, Ρ‡Π΅ΠΌ большС, Π² соотвСтствии с ΠΎΠ±Ρ‰ΠΈΠΌΠΈ основными стандартами для 5-Π³ΠΎ ΠΈ 4-Π³ΠΎ классов класс, 3 класс, 2 класс, 1 класс, срСдняя школа ΠΈ дошкольноС ΡƒΡ‡Ρ€Π΅ΠΆΠ΄Π΅Π½ΠΈΠ΅

Π‘ΠΏΠΎΡΠΎΠ±Π½ΠΎΡΡ‚ΡŒ Π·Π°ΠΏΠΈΡΡ‹Π²Π°Ρ‚ΡŒ ΡΠΌΠ΅ΡˆΠ°Π½Π½Ρ‹Π΅ Π΄Ρ€ΠΎΠ±ΠΈ ΠΊΠ°ΠΊ Π½Π΅ΠΏΡ€Π°Π²ΠΈΠ»ΡŒΠ½Ρ‹Π΅ Π΄Ρ€ΠΎΠ±ΠΈ ΠΈ Π½Π°ΠΎΠ±ΠΎΡ€ΠΎΡ‚ являСтся Π²Π°ΠΆΠ½ΠΎΠΉ прСдпосылкой для слоТСния ΠΈ вычитания ΡΠΌΠ΅ΡˆΠ°Π½Π½Ρ‹Ρ… Π΄Ρ€ΠΎΠ±Π΅ΠΉ (CCSS. Math.4.NF.3c): ΡΠΊΠ»Π°Π΄Ρ‹Π²Π°Ρ‚ΡŒ ΠΈ Π²Ρ‹Ρ‡ΠΈΡ‚Π°Ρ‚ΡŒ ΡΠΌΠ΅ΡˆΠ°Π½Π½Ρ‹Π΅ числа с ΠΎΠ΄ΠΈΠ½Π°ΠΊΠΎΠ²Ρ‹ΠΌΠΈ знамСнатСлями, Π½Π°ΠΏΡ€ΠΈΠΌΠ΅Ρ€, замСняя ΠΊΠ°ΠΆΠ΄ΠΎΠ΅ смСшанноС число эквивалСнтной Π΄Ρ€ΠΎΠ±ΡŒΡŽ ΠΈ / ΠΈΠ»ΠΈ ΠΈΡΠΏΠΎΠ»ΡŒΠ·ΡƒΡ свойства ΠΎΠΏΠ΅Ρ€Π°Ρ†ΠΈΠΉ ΠΈ … На этой страницС Π²Ρ‹ Π½Π°ΠΉΠ΄Π΅Ρ‚Π΅ наши Ρ€Π°Π±ΠΎΡ‡ΠΈΠ΅ листы с ΠΎΡ‚Ρ€ΠΈΡ†Π°Ρ‚Π΅Π»ΡŒΠ½Ρ‹ΠΌΠΈ Ρ†Π΅Π»Ρ‹ΠΌΠΈ числами, ΠΎΡ‚Ρ€ΠΈΡ†Π°Ρ‚Π΅Π»ΡŒΠ½Ρ‹ΠΌΠΈ дСсятичными Π·Π½Π°ΠΊΠ°ΠΌΠΈ. ΠΈ ΠΎΡ‚Ρ€ΠΈΡ†Π°Ρ‚Π΅Π»ΡŒΠ½Ρ‹Π΅ Π΄Ρ€ΠΎΠ±ΠΈ. Наши Ρ€Π°Π±ΠΎΡ‡ΠΈΠ΅ листы с ΠΎΡ‚Ρ€ΠΈΡ†Π°Ρ‚Π΅Π»ΡŒΠ½Ρ‹ΠΌΠΈ числами подходят для 6 ΠΈ 7 классов ΠΌΠ°Ρ‚Π΅ΠΌΠ°Ρ‚ΠΈΠΊΠΈ ΠΈ ΡΠ²Π»ΡΡŽΡ‚ΡΡ ΠΎΡ‚Π»ΠΈΡ‡Π½Ρ‹ΠΌ матСматичСским рСсурсом для ΠΊΠΎΡ€Ρ€Π΅ΠΊΡ‚ΠΈΡ€ΡƒΡŽΡ‰ΠΈΡ… Ρ†Π΅Π»Π΅ΠΉ ΠΏΠΎ ΠΌΠ°Ρ‚Π΅ΠΌΠ°Ρ‚ΠΈΠΊΠ΅ ΠΈΠ»ΠΈ матСматичСскому ΠΎΠ±ΡƒΡ‡Π΅Π½ΠΈΡŽ. Π£ нас Π΅ΡΡ‚ΡŒ Ρ†Π΅Π»Ρ‹Π΅ листы, Π² ΠΊΠΎΡ‚ΠΎΡ€Ρ‹Ρ… описываСтся слоТСниС ΠΈ Π²Ρ‹Ρ‡ΠΈΡ‚Π°Π½ΠΈΠ΅ Ρ†Π΅Π»Ρ‹Ρ… ΠΈ ΠΎΡ‚Ρ€ΠΈΡ†Π°Ρ‚Π΅Π»ΡŒΠ½Ρ‹Ρ… чисСл ΠΈΠ· числа…

Π£ΠΌΠ½ΠΎΠΆΠ°ΠΉΡ‚Π΅ ΠΈΠ»ΠΈ Π΄Π΅Π»ΠΈΡ‚Π΅ ΡΠΌΠ΅ΡˆΠ°Π½Π½Ρ‹Π΅ числа. ΠŸΡ€Π΅ΠΎΠ±Ρ€Π°Π·ΡƒΠΉΡ‚Π΅ ΡΠΌΠ΅ΡˆΠ°Π½Π½Ρ‹Π΅ числа Π² Π½Π΅ΠΏΡ€Π°Π²ΠΈΠ»ΡŒΠ½Ρ‹Π΅ Π΄Ρ€ΠΎΠ±ΠΈ. Π‘Π»Π΅Π΄ΡƒΠΉΡ‚Π΅ ΠΏΡ€Π°Π²ΠΈΠ»Π°ΠΌ умноТСния ΠΈΠ»ΠΈ дСлСния Π΄Ρ€ΠΎΠ±Π΅ΠΉ. Если Π²ΠΎΠ·ΠΌΠΎΠΆΠ½ΠΎ, упроститС. УпроститС ΡΠ»ΠΎΠΆΠ½ΡƒΡŽ Π΄Ρ€ΠΎΠ±ΡŒ. ΠŸΠ΅Ρ€Π΅ΠΏΠΈΡˆΠΈΡ‚Π΅ ΡΠ»ΠΎΠΆΠ½ΡƒΡŽ Π΄Ρ€ΠΎΠ±ΡŒ ΠΊΠ°ΠΊ Π·Π°Π΄Π°Ρ‡Ρƒ дСлСния. Π‘ΠΎΠ±Π»ΡŽΠ΄Π°ΠΉΡ‚Π΅ ΠΏΡ€Π°Π²ΠΈΠ»Π° дСлСния Π΄Ρ€ΠΎΠ±Π΅ΠΉ. Если Π²ΠΎΠ·ΠΌΠΎΠΆΠ½ΠΎ, упроститС. Π Π°Π·ΠΌΠ΅Ρ‰Π΅Π½ΠΈΠ΅ Π·Π½Π°ΠΊΠ° минус Π² Π΄Ρ€ΠΎΠ±ΠΈ CCBC Math 081 Π£ΠΌΠ½ΠΎΠΆΠ΅Π½ΠΈΠ΅ Π΄Ρ€ΠΎΠ±Π΅ΠΉ ΠΈ ΡΠΌΠ΅ΡˆΠ°Π½Π½Ρ‹Ρ… чисСл Π Π°Π·Π΄Π΅Π» 3. 2 Π’Ρ€Π΅Ρ‚ΡŒΠ΅ ΠΈΠ·Π΄Π°Π½ΠΈΠ΅ 17 страниц 162 ΠŸΡ€ΠΈΠΌΠ΅Ρ€ 4a: Π£ΠΌΠ½ΠΎΠΆΠ΅Π½ΠΈΠ΅: 29 3 14 u 2 9 2 9 18 3 14 3 14 42 uuu Π£ΠΌΠ½ΠΎΠΆΡŒΡ‚Π΅ числитСли вмСстС ΠΈ Π·Π½Π°ΠΌΠ΅Π½Π°Ρ‚Π΅Π»ΠΈ вмСстС.ΠŸΡ€ΠΎΠ²Π΅Ρ€ΡŒΡ‚Π΅, ΠΌΠΎΠΆΠ½ΠΎ Π»ΠΈ ΡƒΠΏΡ€ΠΎΡΡ‚ΠΈΡ‚ΡŒ ΠΎΡ‚Π²Π΅Ρ‚. Π•ΡΡ‚ΡŒ ΠΎΠ±Ρ‰ΠΈΠ΅ ΠΌΠ½ΠΎΠΆΠΈΡ‚Π΅Π»ΠΈ для 18 ΠΈ 42, Ρ‚Π°ΠΊ Ρ‡Ρ‚ΠΎ это

9 апрСля 2015 Π³. Β· ИдСя состоит Π² Ρ‚ΠΎΠΌ, Ρ‡Ρ‚ΠΎ ΠΏΠ°Ρ€Ρ‹ чисСл сами ΠΏΠΎ сСбС связаны с ассоциациями. Π£ этого утвСрТдСния Π΅ΡΡ‚ΡŒ ТСсткая вСрсия, ΠΊΠΎΡ‚ΠΎΡ€ΡƒΡŽ я Π½Π΅ ΡΠΎΠ±ΠΈΡ€Π°ΡŽΡΡŒ Π΄Π΅Π»Π°Ρ‚ΡŒ. Π― Π½Π΅ Ρ…ΠΎΡ‡Ρƒ ΡΠΊΠ°Π·Π°Ρ‚ΡŒ, Ρ‡Ρ‚ΠΎ нСзависимо ΠΎΡ‚ контСкста, Π²Ρ‹ ΠΎΠΆΠΈΠ΄Π°Π΅Ρ‚Π΅, Ρ‡Ρ‚ΠΎ ΡƒΡ‡Π΅Π½ΠΈΠΊ слоТит вмСстС 5 1/2 ΠΈ 2 1/4. Π― Π΄ΡƒΠΌΠ°ΡŽ, Ρ‡Ρ‚ΠΎ ΠΏΡ€ΠΎΠ±Π»Π΅ΠΌΠ° дСлСния ΡΠΌΠ΅ΡˆΠ°Π½Π½Ρ‹Ρ… чисСл вряд Π»ΠΈ Π²Ρ‹Π·ΠΎΠ²Π΅Ρ‚ ассоциации с … Как ΠΏΠΎΠ»ΡŒΠ·ΠΎΠ²Π°Ρ‚ΡŒΡΡ ΠΊΠ°Π»ΡŒΠΊΡƒΠ»ΡΡ‚ΠΎΡ€ΠΎΠΌ. Π’Π²Π΅Π΄ΠΈΡ‚Π΅ свою Π·Π°Π΄Π°Ρ‡Ρƒ ΠΏΠΎ Π°Π»Π³Π΅Π±Ρ€Π΅ Π² тСкстовоС ΠΏΠΎΠ»Π΅.НапримСр, Π²Π²Π΅Π΄ΠΈΡ‚Π΅ 3x + 2 = 14 Π² тСкстовоС ΠΏΠΎΠ»Π΅, Ρ‡Ρ‚ΠΎΠ±Ρ‹ ΠΏΠΎΠ»ΡƒΡ‡ΠΈΡ‚ΡŒ пошаговоС объяснСниС Ρ‚ΠΎΠ³ΠΎ, ΠΊΠ°ΠΊ Ρ€Π΅ΡˆΠΈΡ‚ΡŒ 3x + 2 = 14.

Π’Ρ‹Ρ‡ΠΈΡ‚Π°Π½ΠΈΠ΅ Ρ†Π΅Π»Ρ‹Ρ… чисСл ΠΎΡ‚ (-15) Π΄ΠΎ (+15) (ΠΎΡ‚Ρ€ΠΈΡ†Π°Ρ‚Π΅Π»ΡŒΠ½Ρ‹Π΅ числа Π² ΠΊΡ€ΡƒΠ³Π»Ρ‹Ρ… скобках) (106 просмотров Π½Π° этой Π½Π΅Π΄Π΅Π»Π΅) ВсС ΠΎΠΏΠ΅Ρ€Π°Ρ†ΠΈΠΈ с Ρ†Π΅Π»Ρ‹ΠΌΠΈ числами (Π΄ΠΈΠ°ΠΏΠ°Π·ΠΎΠ½ ΠΎΡ‚ -9 Π΄ΠΎ 9) со всСми Ρ†Π΅Π»Ρ‹ΠΌΠΈ числами Π² ΠΊΡ€ΡƒΠ³Π»Ρ‹Ρ… скобках (81 просмотр Π½Π° этой Π½Π΅Π΄Π΅Π»Π΅) ЦСлочислСнноС слоТСниС ΠΈ Π²Ρ‹Ρ‡ΠΈΡ‚Π°Π½ΠΈΠ΅ с ΠΊΡ€ΡƒΠ³Π»Ρ‹ΠΌΠΈ скобками Π²ΠΎΠΊΡ€ΡƒΠ³ всСх Ρ†Π΅Π»Ρ‹Ρ… чисСл (Π΄ΠΈΠ°ΠΏΠ°Π·ΠΎΠ½ ΠΎΡ‚ -25 Π΄ΠΎ 25) (60 просмотров Π½Π° этой Π½Π΅Π΄Π΅Π»Π΅) Π”ΠΎΠ±Π°Π²Π»Π΅Π½ΠΈΠ΅ Ρ†Π΅Π»Ρ‹Ρ… чисСл ΠΎΡ‚ (-15) Π΄ΠΎ (+15) (всС числа Π² ΠΊΡ€ΡƒΠ³Π»Ρ‹Ρ… скобках) (49 просмотров Π½Π° этой нСдСлС… ΠŸΡ€Π°ΠΊΡ‚ΠΈΠΊΠ° слоТСния, вычитания, умноТСния ΠΈ дСлСния Π½Π° ΡΠΌΠ΅ΡˆΠ°Π½Π½Ρ‹Π΅ Π΄Ρ€ΠΎΠ±ΠΈ, рабочая Ρ‚Π΅Ρ‚Ρ€Π°Π΄ΡŒ ΡƒΠ»ΡƒΡ‡ΡˆΠΈΡ‚ Π²Π°ΡˆΡƒ Π±Π΅Π³Π»ΠΎΡΡ‚ΡŒ ΠΌΠ°Ρ‚Π΅ΠΌΠ°Ρ‚ΠΈΠΊΠΈ Ρ‚ΠΎΠΌ 14 25 октября 2020 Π³. Автор: Π”ΠΆΠ΅ΠΊΠΈ Коллинз Паблишинг …

Π£ΠΌΠ½ΠΎΠΆΠ΅Π½ΠΈΠ΅ β€” это матСматичСская опСрация, которая ΠΌΠ°ΡΡˆΡ‚Π°Π±ΠΈΡ€ΡƒΠ΅Ρ‚ ΠΎΠ΄Π½ΠΎ число Π½Π° Π΄Ρ€ΡƒΠ³ΠΎΠ΅ ( ΡƒΠΌΠ½ΠΎΠΆΠ΅Π½ΠΈΠ΅ β€” Π·Π½Π°ΠΊ: x). Он замСняСт слоТСниС большого количСства чисСл подряд. Π£ΠΌΠ½ΠΎΠΆΠ΅Π½ΠΈΠ΅ β€” матСматичСская опСрация, противополоТная дСлСнию. НапримСр: Π²Ρ‹ ΠΌΠΎΠΆΠ΅Ρ‚Π΅ Π·Π°ΠΌΠ΅Π½ΠΈΡ‚ΡŒ матСматичСскоС Π²Ρ‹Ρ€Π°ΠΆΠ΅Π½ΠΈΠ΅ 5 + 5 + 5 + 5 Π½Π° 5 Γ— 4 (число 5, ΡƒΠΌΠ½ΠΎΠΆΠ΅Π½Π½ΠΎΠ΅ Π½Π° количСство Ρ€Π°Π·, ΠΊΠΎΡ‚ΠΎΡ€ΠΎΠ΅ ΠΎΠ½ΠΎ появилось Π² строкС).

Π£ΠΌΠ½ΠΎΠΆΠ΅Π½ΠΈΠ΅ ΠΈΠΌΠ΅Π΅Ρ‚ нСсколько свойств. Когда число умноТаСтся Π½Π° число 1, ΠΏΡ€ΠΎΠΈΠ·Π²Π΅Π΄Π΅Π½ΠΈΠ΅ (Ρ€Π΅Π·ΡƒΠ»ΡŒΡ‚Π°Ρ‚ умноТСния) совпадаСт с этим числом. Когда число умноТаСтся Π½Π° ноль, ΠΏΡ€ΠΎΠΈΠ·Π²Π΅Π΄Π΅Π½ΠΈΠ΅ всСгда Ρ€Π°Π²Π½ΠΎ Π½ΡƒΠ»ΡŽ. Когда число умноТаСтся Π½Π° ΠΎΡ‚Ρ€ΠΈΡ†Π°Ρ‚Π΅Π»ΡŒΠ½ΠΎΠ΅, Ρ€Π΅Π·ΡƒΠ»ΡŒΡ‚Π°Ρ‚ΠΎΠΌ являСтся ΠΎΡ‚Ρ€ΠΈΡ†Π°Ρ‚Π΅Π»ΡŒΠ½Π°Ρ вСрсия этого числа. Он Ρ‚Π°ΠΊΠΆΠ΅ ΠΎΠ±Π»Π°Π΄Π°Π΅Ρ‚ свойствами коммутативности, ассоциативности ΠΈ дистрибутивности.

ΠšΠΎΠΌΠΌΡƒΡ‚Π°Ρ‚ΠΈΠ²Π½Ρ‹Π΅ ΠΈ ассоциативныС свойства Π°Π½Π°Π»ΠΎΠ³ΠΈΡ‡Π½Ρ‹ этим свойствам.Π Π°ΡΠΏΡ€Π΅Π΄Π΅Π»ΠΈΡ‚Π΅Π»ΡŒΠ½ΠΎΡΡ‚ΡŒ примСняСтся ΠΏΡ€ΠΈ объСдинСнии слоТСния ΠΈ умноТСния, ΠΊΠ°ΠΊ Π² этом ΠΏΡ€ΠΈΠΌΠ΅Ρ€Π΅:

Π° * (Π± + Π²) = Π° * Π± + Π° * Π²

ΠžΡ‡Π΅Π½ΡŒ Π²Π°ΠΆΠ½ΠΎ ΠΏΠΎΠΌΠ½ΠΈΡ‚ΡŒ, Ρ‡Ρ‚ΠΎ порядок ваТности умноТСния всСгда большС, Ρ‡Π΅ΠΌ порядок ваТности слоТСния ΠΈΠ»ΠΈ вычитания. Π‘Π½Π°Ρ‡Π°Π»Π° Π²Ρ‹ ΡƒΠΌΠ½ΠΎΠΆΠ°Π΅Ρ‚Π΅ числа (Π³Π΄Π΅ это Π½Π΅ΠΎΠ±Ρ…ΠΎΠ΄ΠΈΠΌΠΎ), Π° Π·Π°Ρ‚Π΅ΠΌ складываСтС ΠΈΠ»ΠΈ Π²Ρ‹Ρ‡ΠΈΡ‚Π°Π΅Ρ‚Π΅ ΠΈΡ…. Π’ΠΎΡ‚ ΠΏΡ€ΠΈΠΌΠ΅Ρ€:

5 * 4 + 3 = 20 + 3 = 23

ЕдинствСнным ΠΈΡΠΊΠ»ΡŽΡ‡Π΅Π½ΠΈΠ΅ΠΌ ΠΈΠ· ΠΏΡ€Π°Π²ΠΈΠ»Π° являСтся Ρ‚ΠΎ, Ρ‡Ρ‚ΠΎ слоТСниС ΠΈΠ»ΠΈ Π²Ρ‹Ρ‡ΠΈΡ‚Π°Π½ΠΈΠ΅ ΠΈΠΌΠ΅Π΅Ρ‚ ΠΏΡ€ΠΈΠΎΡ€ΠΈΡ‚Π΅Ρ‚ Π½Π°Π΄ ΡƒΠΌΠ½ΠΎΠΆΠ΅Π½ΠΈΠ΅ΠΌ, ΠΊΠΎΠ³Π΄Π° числа, ΠΊΠΎΡ‚ΠΎΡ€Ρ‹Π΅ Π½ΡƒΠΆΠ½ΠΎ ΡΠ»ΠΎΠΆΠΈΡ‚ΡŒ, ΠΈ ΠΎΠΏΠ΅Ρ€Π°Ρ‚ΠΎΡ€ Π·Π°ΠΊΠ»ΡŽΡ‡Π΅Π½Ρ‹ Π² ΠΊΠ²Π°Π΄Ρ€Π°Ρ‚Π½Ρ‹Π΅ скобки, Π° ΠΎΠΏΠ΅Ρ€Π°Ρ‚ΠΎΡ€ умноТСния β€” Π½Π΅Ρ‚.

Π£ΠΌΠ½ΠΎΠΆΠ΅Π½ΠΈΠ΅ экзамСнов для ΡƒΡ‡ΠΈΡ‚Π΅Π»Π΅ΠΉ

НазваниС экзамСна Π Π°Π·ΠΌΠ΅Ρ€ Ρ„Π°ΠΉΠ»Π° Π—Π°Π³Ρ€ΡƒΠ·ΠΊΠΈ Π”Π°Ρ‚Π° Π·Π°Π³Ρ€ΡƒΠ·ΠΊΠΈ

Π¦Π΅Π»Ρ‹Π΅ ΠΏΠΎΠ»ΠΎΠΆΠΈΡ‚Π΅Π»ΡŒΠ½Ρ‹Π΅ числа

Π£ΠΌΠ½ΠΎΠΆΠ΅Π½ΠΈΠ΅ ΠΏΠΎΠ»ΠΎΠΆΠΈΡ‚Π΅Π»ΡŒΠ½Ρ‹Ρ… Ρ†Π΅Π»Ρ‹Ρ… чисСл β€” ΠΎΡ‡Π΅Π½ΡŒ просто 78 ΠšΠ±Π°ΠΉΡ‚ 2899 3 сСнтября 2019 Π³.
Π£ΠΌΠ½ΠΎΠΆΠ΅Π½ΠΈΠ΅ ΠΏΠΎΠ»ΠΎΠΆΠΈΡ‚Π΅Π»ΡŒΠ½Ρ‹Ρ… Ρ†Π΅Π»Ρ‹Ρ… чисСл β€” Π»Π΅Π³ΠΊΠΎ 143 ΠšΠ±Π°ΠΉΡ‚ 2902 3 сСнтября 2019 Π³.
Π£ΠΌΠ½ΠΎΠΆΠ΅Π½ΠΈΠ΅ ΠΏΠΎΠ»ΠΎΠΆΠΈΡ‚Π΅Π»ΡŒΠ½Ρ‹Ρ… Ρ†Π΅Π»Ρ‹Ρ… чисСл β€” срСднСС 166.3 ΠΊΠ‘ 3194 3 сСнтября 2019 Π³.
Π£ΠΌΠ½ΠΎΠΆΠ΅Π½ΠΈΠ΅ ΠΏΠΎΠ»ΠΎΠΆΠΈΡ‚Π΅Π»ΡŒΠ½Ρ‹Ρ… Ρ†Π΅Π»Ρ‹Ρ… чисСл β€” срСднСй слоТности 173.1 ΠšΠ±Π°ΠΉΡ‚ 2675 3 сСнтября 2019 Π³.
Π£ΠΌΠ½ΠΎΠΆΠ΅Π½ΠΈΠ΅ ΠΏΠΎΠ»ΠΎΠΆΠΈΡ‚Π΅Π»ΡŒΠ½Ρ‹Ρ… Ρ†Π΅Π»Ρ‹Ρ… чисСл β€” ТСсткоС 561.5 ΠΊΠ‘ 2082 3 сСнтября 2019 Π³.
Π£ΠΌΠ½ΠΎΠΆΠ΅Π½ΠΈΠ΅ ΠΏΠΎΠ»ΠΎΠΆΠΈΡ‚Π΅Π»ΡŒΠ½Ρ‹Ρ… Ρ†Π΅Π»Ρ‹Ρ… чисСл β€” ΠΎΡ‡Π΅Π½ΡŒ слоТно 571.8 ΠšΠ±Π°ΠΉΡ‚ 2294 3 сСнтября 2019 Π³.

ΠŸΠΎΠ»ΠΎΠΆΠΈΡ‚Π΅Π»ΡŒΠ½Ρ‹Π΅ дСсятичныС Π·Π½Π°ΠΊΠΈ

Π£ΠΌΠ½ΠΎΠΆΠ΅Π½ΠΈΠ΅ ΠΏΠΎΠ»ΠΎΠΆΠΈΡ‚Π΅Π»ΡŒΠ½Ρ‹Ρ… дСсятичных Π·Π½Π°ΠΊΠΎΠ² β€” ΠΎΡ‡Π΅Π½ΡŒ просто 554.5 ΠΊΠ‘ 1936 3 сСнтября 2019 Π³.
Π£ΠΌΠ½ΠΎΠΆΠ΅Π½ΠΈΠ΅ ΠΏΠΎΠ»ΠΎΠΆΠΈΡ‚Π΅Π»ΡŒΠ½Ρ‹Ρ… дСсятичных Π·Π½Π°ΠΊΠΎΠ² β€” Π»Π΅Π³ΠΊΠΎ 555.5 ΠšΠ±Π°ΠΉΡ‚ 1927 3 сСнтября 2019 Π³.
Π£ΠΌΠ½ΠΎΠΆΠ΅Π½ΠΈΠ΅ ΠΏΠΎΠ»ΠΎΠΆΠΈΡ‚Π΅Π»ΡŒΠ½Ρ‹Ρ… дСсятичных Π·Π½Π°ΠΊΠΎΠ² β€” срСднСС 579.2 ΠšΠ±Π°ΠΉΡ‚ 1752 3 сСнтября 2019 Π³.
Π£ΠΌΠ½ΠΎΠΆΠ΅Π½ΠΈΠ΅ ΠΏΠΎΠ»ΠΎΠΆΠΈΡ‚Π΅Π»ΡŒΠ½Ρ‹Ρ… дСсятичных Π·Π½Π°ΠΊΠΎΠ² β€” срСднС-ТСсткий 580. 2 ΠšΠ±Π°ΠΉΡ‚ 1573 3 сСнтября 2019 Π³.
Π£ΠΌΠ½ΠΎΠΆΠ΅Π½ΠΈΠ΅ ΠΏΠΎΠ»ΠΎΠΆΠΈΡ‚Π΅Π»ΡŒΠ½Ρ‹Ρ… дСсятичных Π·Π½Π°ΠΊΠΎΠ² β€” ТСсткоС 569.5 ΠΊΠ‘ 1736 3 сСнтября 2019 Π³.
Π£ΠΌΠ½ΠΎΠΆΠ΅Π½ΠΈΠ΅ ΠΏΠΎΠ»ΠΎΠΆΠΈΡ‚Π΅Π»ΡŒΠ½Ρ‹Ρ… дСсятичных Π·Π½Π°ΠΊΠΎΠ² β€” ΠΎΡ‡Π΅Π½ΡŒ слоТно 567.2 ΠšΠ±Π°ΠΉΡ‚ 1576 3 сСнтября 2019 Π³.

ΠŸΠΎΠ»ΠΎΠΆΠΈΡ‚Π΅Π»ΡŒΠ½Ρ‹Π΅ Π΄Ρ€ΠΎΠ±ΠΈ

Π£ΠΌΠ½ΠΎΠΆΠ΅Π½ΠΈΠ΅ ΠΏΠΎΠ»ΠΎΠΆΠΈΡ‚Π΅Π»ΡŒΠ½Ρ‹Ρ… Π΄Ρ€ΠΎΠ±Π΅ΠΉ β€” ΠΎΡ‡Π΅Π½ΡŒ слоТно 567.2 ΠšΠ±Π°ΠΉΡ‚ 1576 3 сСнтября 2019 Π³.
Π£ΠΌΠ½ΠΎΠΆΠ΅Π½ΠΈΠ΅ ΠΏΠΎΠ»ΠΎΠΆΠΈΡ‚Π΅Π»ΡŒΠ½Ρ‹Ρ… Π΄Ρ€ΠΎΠ±Π΅ΠΉ β€” ΠΎΡ‡Π΅Π½ΡŒ просто 172.7 ΠšΠ±Π°ΠΉΡ‚ 2353 3 сСнтября 2019 Π³.
Π£ΠΌΠ½ΠΎΠΆΠ΅Π½ΠΈΠ΅ ΠΏΠΎΠ»ΠΎΠΆΠΈΡ‚Π΅Π»ΡŒΠ½Ρ‹Ρ… Π΄Ρ€ΠΎΠ±Π΅ΠΉ β€” Π»Π΅Π³ΠΊΠΎ 188.9 ΠΊΠ‘ 2049 3 сСнтября 2019 Π³.
Π£ΠΌΠ½ΠΎΠΆΠ΅Π½ΠΈΠ΅ ΠΏΠΎΠ»ΠΎΠΆΠΈΡ‚Π΅Π»ΡŒΠ½Ρ‹Ρ… Π΄Ρ€ΠΎΠ±Π΅ΠΉ β€” срСднСС 577.2 ΠšΠ±Π°ΠΉΡ‚ 1765 3 сСнтября 2019 Π³.
Π£ΠΌΠ½ΠΎΠΆΠ΅Π½ΠΈΠ΅ ΠΏΠΎΠ»ΠΎΠΆΠΈΡ‚Π΅Π»ΡŒΠ½Ρ‹Ρ… Π΄Ρ€ΠΎΠ±Π΅ΠΉ β€” срСднСй твСрдости 591. 9 ΠšΠ±Π°ΠΉΡ‚ 1557 3 сСнтября 2019 Π³.
Π£ΠΌΠ½ΠΎΠΆΠ΅Π½ΠΈΠ΅ ΠΏΠΎΠ»ΠΎΠΆΠΈΡ‚Π΅Π»ΡŒΠ½Ρ‹Ρ… Π΄Ρ€ΠΎΠ±Π΅ΠΉ β€” ТСсткоС 575.7 ΠšΠ±Π°ΠΉΡ‚ 1569 3 сСнтября 2019 Π³.
Π£ΠΌΠ½ΠΎΠΆΠ΅Π½ΠΈΠ΅ ΠΏΠΎΠ»ΠΎΠΆΠΈΡ‚Π΅Π»ΡŒΠ½Ρ‹Ρ… Π΄Ρ€ΠΎΠ±Π΅ΠΉ β€” ΠΎΡ‡Π΅Π½ΡŒ слоТно 581 ΠšΠ±Π°ΠΉΡ‚ 1553 3 сСнтября 2019 Π³.

ΠŸΠΎΠ»ΠΎΠΆΠΈΡ‚Π΅Π»ΡŒΠ½Ρ‹Π΅ ΡΠΌΠ΅ΡˆΠ°Π½Π½Ρ‹Π΅ числа

Π£ΠΌΠ½ΠΎΠΆΠ΅Π½ΠΈΠ΅ ΠΏΠΎΠ»ΠΎΠΆΠΈΡ‚Π΅Π»ΡŒΠ½Ρ‹Ρ… ΡΠΌΠ΅ΡˆΠ°Π½Π½Ρ‹Ρ… чисСл β€” ΠΎΡ‡Π΅Π½ΡŒ просто 176.8 ΠΊΠ‘ 1749 3 сСнтября 2019 Π³.
Π£ΠΌΠ½ΠΎΠΆΠ΅Π½ΠΈΠ΅ ΠΏΠΎΠ»ΠΎΠΆΠΈΡ‚Π΅Π»ΡŒΠ½Ρ‹Ρ… ΡΠΌΠ΅ΡˆΠ°Π½Π½Ρ‹Ρ… чисСл β€” Π»Π΅Π³ΠΊΠΎ 209.8 ΠšΠ±Π°ΠΉΡ‚ 1761 3 сСнтября 2019 Π³.
Π£ΠΌΠ½ΠΎΠΆΠ΅Π½ΠΈΠ΅ ΡΠΌΠ΅ΡˆΠ°Π½Π½Ρ‹Ρ… ΠΏΠΎΠ»ΠΎΠΆΠΈΡ‚Π΅Π»ΡŒΠ½Ρ‹Ρ… чисСл β€” срСднСС 585.6 ΠšΠ±Π°ΠΉΡ‚ 1592 3 сСнтября 2019 Π³.
Π£ΠΌΠ½ΠΎΠΆΠ΅Π½ΠΈΠ΅ ΡΠΌΠ΅ΡˆΠ°Π½Π½Ρ‹Ρ… ΠΏΠΎΠ»ΠΎΠΆΠΈΡ‚Π΅Π»ΡŒΠ½Ρ‹Ρ… чисСл β€” срСднСй слоТности 592.9 ΠšΠ±Π°ΠΉΡ‚ 1513 3 сСнтября 2019 Π³.
Π£ΠΌΠ½ΠΎΠΆΠ΅Π½ΠΈΠ΅ ΠΏΠΎΠ»ΠΎΠΆΠΈΡ‚Π΅Π»ΡŒΠ½Ρ‹Ρ… ΡΠΌΠ΅ΡˆΠ°Π½Π½Ρ‹Ρ… чисСл β€” ТСсткоС 589. 3 ΠΊΠ‘ 1517 3 сСнтября 2019 Π³.
Π£ΠΌΠ½ΠΎΠΆΠ΅Π½ΠΈΠ΅ ΠΏΠΎΠ»ΠΎΠΆΠΈΡ‚Π΅Π»ΡŒΠ½Ρ‹Ρ… ΡΠΌΠ΅ΡˆΠ°Π½Π½Ρ‹Ρ… чисСл β€” ΠΎΡ‡Π΅Π½ΡŒ слоТно 596.6 ΠšΠ±Π°ΠΉΡ‚ 1483 3 сСнтября 2019 Π³.

ΠŸΠΎΠ»ΠΎΠΆΠΈΡ‚Π΅Π»ΡŒΠ½Ρ‹Π΅ Π½Π΅ΠΏΡ€Π°Π²ΠΈΠ»ΡŒΠ½Ρ‹Π΅ Π΄Ρ€ΠΎΠ±ΠΈ

Π£ΠΌΠ½ΠΎΠΆΠ΅Π½ΠΈΠ΅ ΠΏΠΎΠ»ΠΎΠΆΠΈΡ‚Π΅Π»ΡŒΠ½Ρ‹Ρ… Π½Π΅ΠΏΡ€Π°Π²ΠΈΠ»ΡŒΠ½Ρ‹Ρ… Π΄Ρ€ΠΎΠ±Π΅ΠΉ β€” ΠΎΡ‡Π΅Π½ΡŒ просто 166 ΠšΠ±Π°ΠΉΡ‚ 1539 3 сСнтября 2019 Π³.
Π£ΠΌΠ½ΠΎΠΆΠ΅Π½ΠΈΠ΅ ΠΏΠΎΠ»ΠΎΠΆΠΈΡ‚Π΅Π»ΡŒΠ½Ρ‹Ρ… Π½Π΅ΠΏΡ€Π°Π²ΠΈΠ»ΡŒΠ½Ρ‹Ρ… Π΄Ρ€ΠΎΠ±Π΅ΠΉ β€” Π»Π΅Π³ΠΊΠΎ 169.9 ΠΊΠ‘ 1640 3 сСнтября 2019 Π³.
Π£ΠΌΠ½ΠΎΠΆΠ΅Π½ΠΈΠ΅ ΠΏΠΎΠ»ΠΎΠΆΠΈΡ‚Π΅Π»ΡŒΠ½Ρ‹Ρ… Π½Π΅ΠΏΡ€Π°Π²ΠΈΠ»ΡŒΠ½Ρ‹Ρ… Π΄Ρ€ΠΎΠ±Π΅ΠΉ β€” срСднСС 571.6 ΠšΠ±Π°ΠΉΡ‚ 1520 3 сСнтября 2019 Π³.
Π£ΠΌΠ½ΠΎΠΆΠ΅Π½ΠΈΠ΅ ΠΏΠΎΠ»ΠΎΠΆΠΈΡ‚Π΅Π»ΡŒΠ½Ρ‹Ρ… Π½Π΅ΠΏΡ€Π°Π²ΠΈΠ»ΡŒΠ½Ρ‹Ρ… Π΄Ρ€ΠΎΠ±Π΅ΠΉ β€” срСднСй твСрдости 569.5 ΠšΠ±Π°ΠΉΡ‚ 1483 3 сСнтября 2019 Π³.
Π£ΠΌΠ½ΠΎΠΆΠ΅Π½ΠΈΠ΅ ΠΏΠΎΠ»ΠΎΠΆΠΈΡ‚Π΅Π»ΡŒΠ½Ρ‹Ρ… Π½Π΅ΠΏΡ€Π°Π²ΠΈΠ»ΡŒΠ½Ρ‹Ρ… Π΄Ρ€ΠΎΠ±Π΅ΠΉ β€” ТСсткоС 560.5 ΠšΠ±Π°ΠΉΡ‚ 1426 3 сСнтября 2019 Π³.
Π£ΠΌΠ½ΠΎΠΆΠ΅Π½ΠΈΠ΅ ΠΏΠΎΠ»ΠΎΠΆΠΈΡ‚Π΅Π»ΡŒΠ½Ρ‹Ρ… Π½Π΅ΠΏΡ€Π°Π²ΠΈΠ»ΡŒΠ½Ρ‹Ρ… Π΄Ρ€ΠΎΠ±Π΅ΠΉ β€” ΠΎΡ‡Π΅Π½ΡŒ слоТно 571.8 ΠΊΠ‘ 1494 3 сСнтября 2019 Π³.

Π¦Π΅Π»Ρ‹Π΅ Π½Π΅ΠΏΠΎΠ»ΠΎΠΆΠΈΡ‚Π΅Π»ΡŒΠ½Ρ‹Π΅ числа

Π£ΠΌΠ½ΠΎΠΆΠ΅Π½ΠΈΠ΅ Ρ†Π΅Π»Ρ‹Ρ… чисСл β€” ΠΎΡ‡Π΅Π½ΡŒ просто 137.9 ΠšΠ±Π°ΠΉΡ‚ 2107 3 сСнтября 2019 Π³.
Π£ΠΌΠ½ΠΎΠΆΠ΅Π½ΠΈΠ΅ Ρ†Π΅Π»Ρ‹Ρ… чисСл β€” Π»Π΅Π³ΠΊΠΎ 143.6 ΠšΠ±Π°ΠΉΡ‚ 2143 3 сСнтября 2019 Π³.
Π£ΠΌΠ½ΠΎΠΆΠ΅Π½ΠΈΠ΅ Ρ†Π΅Π»Ρ‹Ρ… чисСл β€” срСднСС 173.3 ΠšΠ±Π°ΠΉΡ‚ 2097 3 сСнтября 2019 Π³.
Π£ΠΌΠ½ΠΎΠΆΠ΅Π½ΠΈΠ΅ Ρ†Π΅Π»Ρ‹Ρ… чисСл β€” срСднСй слоТности 173.5 ΠΊΠ‘ 1957 3 сСнтября 2019 Π³.
Π£ΠΌΠ½ΠΎΠΆΠ΅Π½ΠΈΠ΅ Ρ†Π΅Π»Ρ‹Ρ… чисСл β€” ТСсткоС 565.9 ΠšΠ±Π°ΠΉΡ‚ 1746 3 сСнтября 2019 Π³.
Π£ΠΌΠ½ΠΎΠΆΠ΅Π½ΠΈΠ΅ Ρ†Π΅Π»Ρ‹Ρ… чисСл β€” ΠΎΡ‡Π΅Π½ΡŒ слоТно 568 ΠšΠ±Π°ΠΉΡ‚ 1641 3 сСнтября 2019 Π³.

ΠΠ΅ΠΏΠΎΠ»ΠΎΠΆΠΈΡ‚Π΅Π»ΡŒΠ½Ρ‹Π΅ дСсятичныС числа

Π£ΠΌΠ½ΠΎΠΆΠ΅Π½ΠΈΠ΅ дСсятичных Π·Π½Π°ΠΊΠΎΠ² β€” ΠΎΡ‡Π΅Π½ΡŒ просто 186. 6 ΠšΠ±Π°ΠΉΡ‚ 1983 3 сСнтября 2019 Π³.
Π£ΠΌΠ½ΠΎΠΆΠ΅Π½ΠΈΠ΅ дСсятичных Π·Π½Π°ΠΊΠΎΠ² β€” просто 187 ΠšΠ±Π°ΠΉΡ‚ 2103 3 сСнтября 2019 Π³.
Π£ΠΌΠ½ΠΎΠΆΠ΅Π½ΠΈΠ΅ дСсятичных Π·Π½Π°ΠΊΠΎΠ² β€” срСднСС 578.8 ΠΊΠ‘ 1601 3 сСнтября 2019 Π³.
Π£ΠΌΠ½ΠΎΠΆΠ΅Π½ΠΈΠ΅ дСсятичных Π·Π½Π°ΠΊΠΎΠ² β€” срСднСй ТСсткости 577 ΠšΠ±Π°ΠΉΡ‚ 1520 3 сСнтября 2019 Π³.
Π£ΠΌΠ½ΠΎΠΆΠ΅Π½ΠΈΠ΅ дСсятичных Π·Π½Π°ΠΊΠΎΠ² β€” ТСсткоС 564.9 ΠšΠ±Π°ΠΉΡ‚ 1591 3 сСнтября 2019 Π³.
Π£ΠΌΠ½ΠΎΠΆΠ΅Π½ΠΈΠ΅ дСсятичных Π·Π½Π°ΠΊΠΎΠ² β€” ΠΎΡ‡Π΅Π½ΡŒ слоТно 567.9 ΠšΠ±Π°ΠΉΡ‚ 1501 3 сСнтября 2019 Π³.

ΠΠ΅ΠΏΠΎΠ»ΠΎΠΆΠΈΡ‚Π΅Π»ΡŒΠ½Ρ‹Π΅ Π΄Ρ€ΠΎΠ±ΠΈ

Π£ΠΌΠ½ΠΎΠΆΠ΅Π½ΠΈΠ΅ Π΄Ρ€ΠΎΠ±Π΅ΠΉ β€” ΠΎΡ‡Π΅Π½ΡŒ просто 179.6 ΠΊΠ‘ 1594 3 сСнтября 2019 Π³.
Π£ΠΌΠ½ΠΎΠΆΠ΅Π½ΠΈΠ΅ Π΄Ρ€ΠΎΠ±Π΅ΠΉ β€” Π»Π΅Π³ΠΊΠΎ 186.7 ΠšΠ±Π°ΠΉΡ‚ 1866 3 сСнтября 2019 Π³.
Π£ΠΌΠ½ΠΎΠΆΠ΅Π½ΠΈΠ΅ Π΄Ρ€ΠΎΠ±Π΅ΠΉ β€” срСднСС 583.8 ΠšΠ±Π°ΠΉΡ‚ 1596 3 сСнтября 2019 Π³.
Π£ΠΌΠ½ΠΎΠΆΠ΅Π½ΠΈΠ΅ Π΄Ρ€ΠΎΠ±Π΅ΠΉ β€” срСднСй твСрдости 589 ΠšΠ±Π°ΠΉΡ‚ 1511 3 сСнтября 2019 Π³.
Π£ΠΌΠ½ΠΎΠΆΠ΅Π½ΠΈΠ΅ Π΄Ρ€ΠΎΠ±Π΅ΠΉ β€” ТСсткоС 588 ΠšΠ±Π°ΠΉΡ‚ 1422 3 сСнтября 2019 Π³.
Π£ΠΌΠ½ΠΎΠΆΠ΅Π½ΠΈΠ΅ Π΄Ρ€ΠΎΠ±Π΅ΠΉ β€” ΠΎΡ‡Π΅Π½ΡŒ слоТно 594.9 ΠΊΠ‘ 1436 3 сСнтября 2019 Π³.

ΠΠ΅ΠΏΠΎΠ»ΠΎΠΆΠΈΡ‚Π΅Π»ΡŒΠ½Ρ‹Π΅ ΡΠΌΠ΅ΡˆΠ°Π½Π½Ρ‹Π΅ числа

Π£ΠΌΠ½ΠΎΠΆΠ΅Π½ΠΈΠ΅ ΡΠΌΠ΅ΡˆΠ°Π½Π½Ρ‹Ρ… чисСл β€” ΠΎΡ‡Π΅Π½ΡŒ просто 192.9 ΠΊΠ‘ 1468 3 сСнтября 2019 Π³.
Π£ΠΌΠ½ΠΎΠΆΠ΅Π½ΠΈΠ΅ ΡΠΌΠ΅ΡˆΠ°Π½Π½Ρ‹Ρ… чисСл β€” просто 209 ΠšΠ±Π°ΠΉΡ‚ 1635 3 сСнтября 2019 Π³.
Π£ΠΌΠ½ΠΎΠΆΠ΅Π½ΠΈΠ΅ ΡΠΌΠ΅ΡˆΠ°Π½Π½Ρ‹Ρ… чисСл β€” срСднСС 594.7 ΠšΠ±Π°ΠΉΡ‚ 1510 3 сСнтября 2019 Π³.
Π£ΠΌΠ½ΠΎΠΆΠ΅Π½ΠΈΠ΅ ΡΠΌΠ΅ΡˆΠ°Π½Π½Ρ‹Ρ… чисСл β€” срСднСй слоТности 602. 2 ΠΊΠ‘ 1354 3 сСнтября 2019 Π³.
Π£ΠΌΠ½ΠΎΠΆΠ΅Π½ΠΈΠ΅ ΡΠΌΠ΅ΡˆΠ°Π½Π½Ρ‹Ρ… чисСл β€” слоТноС 586.3 ΠšΠ±Π°ΠΉΡ‚ 1380 3 сСнтября 2019 Π³.
Π£ΠΌΠ½ΠΎΠΆΠ΅Π½ΠΈΠ΅ ΡΠΌΠ΅ΡˆΠ°Π½Π½Ρ‹Ρ… чисСл β€” ΠΎΡ‡Π΅Π½ΡŒ слоТно 591.3 ΠšΠ±Π°ΠΉΡ‚ 1360 3 сСнтября 2019 Π³.

ΠΠ΅ΠΏΠΎΠ»ΠΎΠΆΠΈΡ‚Π΅Π»ΡŒΠ½Ρ‹Π΅ Π½Π΅ΠΏΡ€Π°Π²ΠΈΠ»ΡŒΠ½Ρ‹Π΅ Π΄Ρ€ΠΎΠ±ΠΈ

Π£ΠΌΠ½ΠΎΠΆΠ΅Π½ΠΈΠ΅ Π½Π΅ΠΏΡ€Π°Π²ΠΈΠ»ΡŒΠ½Ρ‹Ρ… Π΄Ρ€ΠΎΠ±Π΅ΠΉ β€” ΠΎΡ‡Π΅Π½ΡŒ просто 171.2 ΠšΠ±Π°ΠΉΡ‚ 1411 3 сСнтября 2019 Π³.
Π£ΠΌΠ½ΠΎΠΆΠ΅Π½ΠΈΠ΅ Π½Π΅ΠΏΡ€Π°Π²ΠΈΠ»ΡŒΠ½Ρ‹Ρ… Π΄Ρ€ΠΎΠ±Π΅ΠΉ β€” Π»Π΅Π³ΠΊΠΎ 174 ΠšΠ±Π°ΠΉΡ‚ 1454 3 сСнтября 2019 Π³.
Π£ΠΌΠ½ΠΎΠΆΠ΅Π½ΠΈΠ΅ Π½Π΅ΠΏΡ€Π°Π²ΠΈΠ»ΡŒΠ½Ρ‹Ρ… Π΄Ρ€ΠΎΠ±Π΅ΠΉ β€” срСднСС 574.7 ΠΊΠ‘ 1419 3 сСнтября 2019 Π³.
Π£ΠΌΠ½ΠΎΠΆΠ΅Π½ΠΈΠ΅ Π½Π΅ΠΏΡ€Π°Π²ΠΈΠ»ΡŒΠ½Ρ‹Ρ… Π΄Ρ€ΠΎΠ±Π΅ΠΉ β€” срСднСй твСрдости 573.9 ΠšΠ±Π°ΠΉΡ‚ 1333 3 сСнтября 2019 Π³.
Π£ΠΌΠ½ΠΎΠΆΠ΅Π½ΠΈΠ΅ Π½Π΅ΠΏΡ€Π°Π²ΠΈΠ»ΡŒΠ½Ρ‹Ρ… Π΄Ρ€ΠΎΠ±Π΅ΠΉ β€” ТСсткоС 570. 7 ΠšΠ±Π°ΠΉΡ‚ 1351 3 сСнтября 2019 Π³.
Π£ΠΌΠ½ΠΎΠΆΠ΅Π½ΠΈΠ΅ Π½Π΅ΠΏΡ€Π°Π²ΠΈΠ»ΡŒΠ½Ρ‹Ρ… Π΄Ρ€ΠΎΠ±Π΅ΠΉ β€” ΠΎΡ‡Π΅Π½ΡŒ слоТно 577.4 ΠšΠ±Π°ΠΉΡ‚ 1348 3 сСнтября 2019 Π³.

Π£ΠΌΠ½ΠΎΠΆΠ΅Π½ΠΈΠ΅ Ρ€Π°Π±ΠΎΡ‡ΠΈΡ… листов для студСнтов

НазваниС листа Π Π°Π·ΠΌΠ΅Ρ€ Ρ„Π°ΠΉΠ»Π° Π—Π°Π³Ρ€ΡƒΠ·ΠΊΠΈ Π”Π°Ρ‚Π° Π·Π°Π³Ρ€ΡƒΠ·ΠΊΠΈ

ΠŸΠΎΠ»ΠΎΠΆΠΈΡ‚Π΅Π»ΡŒΠ½ΠΎ

Π£ΠΌΠ½ΠΎΠΆΠ΅Π½ΠΈΠ΅ Π½Π°Ρ‚ΡƒΡ€Π°Π»ΡŒΠ½Ρ‹Ρ… чисСл 496.1 ΠΊΠ‘ 2432 3 сСнтября 2019 Π³.
Π£ΠΌΠ½ΠΎΠΆΠ΅Π½ΠΈΠ΅ ΠΏΠΎΠ»ΠΎΠΆΠΈΡ‚Π΅Π»ΡŒΠ½Ρ‹Ρ… дСсятичных Π·Π½Π°ΠΊΠΎΠ² 51.8 ΠšΠ±Π°ΠΉΡ‚ 1768 3 сСнтября 2019 Π³.
Π£ΠΌΠ½ΠΎΠΆΠ΅Π½ΠΈΠ΅ ΠΏΠΎΠ»ΠΎΠΆΠΈΡ‚Π΅Π»ΡŒΠ½Ρ‹Ρ… Π΄Ρ€ΠΎΠ±Π΅ΠΉ 632.1 ΠšΠ±Π°ΠΉΡ‚ 1571 3 сСнтября 2019 Π³.
Π£ΠΌΠ½ΠΎΠΆΠ΅Π½ΠΈΠ΅ ΡΠΌΠ΅ΡˆΠ°Π½Π½Ρ‹Ρ… ΠΏΠΎΠ»ΠΎΠΆΠΈΡ‚Π΅Π»ΡŒΠ½Ρ‹Ρ… чисСл 637.8 ΠšΠ±Π°ΠΉΡ‚ 1784 3 сСнтября 2019 Π³.
Π£ΠΌΠ½ΠΎΠΆΠ΅Π½ΠΈΠ΅ ΠΏΠΎΠ»ΠΎΠΆΠΈΡ‚Π΅Π»ΡŒΠ½Ρ‹Ρ… Π½Π΅ΠΏΡ€Π°Π²ΠΈΠ»ΡŒΠ½Ρ‹Ρ… Π΄Ρ€ΠΎΠ±Π΅ΠΉ 594. 6 ΠΊΠ‘ 1390 3 сСнтября 2019 Π³.

ΠΠ΅ΠΏΠΎΠ»ΠΎΠΆΠΈΡ‚Π΅Π»ΡŒΠ½Ρ‹ΠΉ

Π£ΠΌΠ½ΠΎΠΆΠ΅Π½ΠΈΠ΅ Ρ†Π΅Π»Ρ‹Ρ… чисСл 166.8 ΠšΠ±Π°ΠΉΡ‚ 1810 3 сСнтября 2019 Π³.
Π£ΠΌΠ½ΠΎΠΆΠ΅Π½ΠΈΠ΅ дСсятичных Π·Π½Π°ΠΊΠΎΠ² 246.3 ΠšΠ±Π°ΠΉΡ‚ 1644 3 сСнтября 2019 Π³.
Π£ΠΌΠ½ΠΎΠΆΠ΅Π½ΠΈΠ΅ Π΄Ρ€ΠΎΠ±Π΅ΠΉ 393.1 ΠšΠ±Π°ΠΉΡ‚ 1639 3 сСнтября 2019 Π³.
Π£ΠΌΠ½ΠΎΠΆΠ΅Π½ΠΈΠ΅ ΡΠΌΠ΅ΡˆΠ°Π½Π½Ρ‹Ρ… чисСл 412.4 ΠΊΠ‘ 1534 3 сСнтября 2019 Π³.
Π£ΠΌΠ½ΠΎΠΆΠ΅Π½ΠΈΠ΅ Π½Π΅ΠΏΡ€Π°Π²ΠΈΠ»ΡŒΠ½Ρ‹Ρ… Π΄Ρ€ΠΎΠ±Π΅ΠΉ 334.1 ΠšΠ±Π°ΠΉΡ‚ 1440 3 сСнтября 2019 Π³.

ΠžΠ±Ρ€Π°Ρ‚Π½Π°Ρ Π΄Ρ€ΠΎΠ±ΡŒ β€” ChiliMath

ΠŸΠΎΠΌΠ½ΠΈΡ‚Π΅, Ρ‡Ρ‚ΠΎ Π΄Ρ€ΠΎΠ±ΡŒ состоит ΠΈΠ· Ρ‚Ρ€Π΅Ρ… частСй.

ЧАБВИ ЀРАКЦИИ

Как Π½Π°ΠΉΡ‚ΠΈ Π²Π΅Π»ΠΈΡ‡ΠΈΠ½Ρƒ, ΠΎΠ±Ρ€Π°Ρ‚Π½ΡƒΡŽ Π΄Ρ€ΠΎΠ±ΠΈ

Π§Ρ‚ΠΎΠ±Ρ‹ ΠΏΠΎΠ»ΡƒΡ‡ΠΈΡ‚ΡŒ Π²Π΅Π»ΠΈΡ‡ΠΈΠ½Ρƒ, ΠΎΠ±Ρ€Π°Ρ‚Π½ΡƒΡŽ Π΄Ρ€ΠΎΠ±ΠΈ

просто мСняСт мСстами ΠΈΠ»ΠΈ мСстами Ρ‡ΠΈΡΠ»ΠΈΡ‚Π΅Π»ΡŒ ΠΈ Π·Π½Π°ΠΌΠ΅Π½Π°Ρ‚Π΅Π»ΡŒ. Π’Ρ‹ ΠΌΠΎΠΆΠ΅Ρ‚Π΅ ΡΠΊΠ°Π·Π°Ρ‚ΡŒ, Ρ‡Ρ‚ΠΎ ΠΌΡ‹ просто ΠΏΠ΅Ρ€Π΅Π²ΠΎΡ€Π°Ρ‡ΠΈΠ²Π°Π΅ΠΌ ΠΈΡΡ…ΠΎΠ΄Π½ΡƒΡŽ Π΄Ρ€ΠΎΠ±ΡŒ Π²Π²Π΅Ρ€Ρ… Π΄Π½ΠΎΠΌ.


ΠŸΡ€ΠΈΠΌΠ΅Ρ€Ρ‹ нахоТдСния ΠΎΠ±Ρ€Π°Ρ‚Π½ΠΎΠΉ Π΄Ρ€ΠΎΠ±ΠΈ

ΠŸΡ€ΠΈΠΌΠ΅Ρ€ 1: НайдитС Π²Π΅Π»ΠΈΡ‡ΠΈΠ½Ρƒ, ΠΎΠ±Ρ€Π°Ρ‚Π½ΡƒΡŽ Π΄Ρ€ΠΎΠ±ΠΈ, ΡƒΠΊΠ°Π·Π°Π½Π½ΠΎΠΉ Π½ΠΈΠΆΠ΅.

Π”Π°Π²Π°ΠΉΡ‚Π΅ ΠΏΠ΅Ρ€Π΅Π²Π΅Ρ€Π½Π΅ΠΌ Π΄Ρ€ΠΎΠ±ΡŒ {3 \ over 7} Π²Π²Π΅Ρ€Ρ… Π΄Π½ΠΎΠΌ, Ρ‡Ρ‚ΠΎΠ±Ρ‹ ΠΏΠΎΠ»ΡƒΡ‡ΠΈΡ‚ΡŒ ΠΎΠ±Ρ€Π°Ρ‚Π½ΡƒΡŽ Π²Π΅Π»ΠΈΡ‡ΠΈΠ½Ρƒ. Π­Ρ‚ΠΎ Π΄Π°Π΅Ρ‚ Π½Π°ΠΌ


ΠŸΡ€ΠΈΠΌΠ΅Ρ€ 2: НайдитС Π²Π΅Π»ΠΈΡ‡ΠΈΠ½Ρƒ, ΠΎΠ±Ρ€Π°Ρ‚Π½ΡƒΡŽ Π΄Ρ€ΠΎΠ±ΠΈ, ΡƒΠΊΠ°Π·Π°Π½Π½ΠΎΠΉ Π½ΠΈΠΆΠ΅.

Для Π½Π΅ΠΊΠΎΡ‚ΠΎΡ€Ρ‹Ρ… это ΠΌΠΎΠΆΠ΅Ρ‚ Π±Ρ‹Ρ‚ΡŒ нСпросто. ΠžΠ±Ρ€Π°Ρ‚ΠΈΡ‚Π΅ Π²Π½ΠΈΠΌΠ°Π½ΠΈΠ΅, Ρ‡Ρ‚ΠΎ Ρ‡ΠΈΡΠ»ΠΈΡ‚Π΅Π»ΡŒ ΠΎΡ‚Ρ€ΠΈΡ†Π°Ρ‚Π΅Π»ΡŒΠ½Ρ‹ΠΉ. Если ΠΏΠ΅Ρ€Π΅Π²Π΅Ρ€Π½ΡƒΡ‚ΡŒ эту Π΄Ρ€ΠΎΠ±ΡŒ Π²Π²Π΅Ρ€Ρ… Π΄Π½ΠΎΠΌ, Ρ‚ΠΎ СстСствСнно Π΄ΠΎΠ»ΠΆΠ½ΠΎ ΠΏΠΎΠ»ΡƒΡ‡ΠΈΡ‚ΡŒΡΡ

.

ΠŸΠΎΡΠΊΠΎΠ»ΡŒΠΊΡƒ это ΠΎΡ‚Ρ€ΠΈΡ†Π°Ρ‚Π΅Π»ΡŒΠ½Π°Ρ Π΄Ρ€ΠΎΠ±ΡŒ, ΠΎΡ‚Ρ€ΠΈΡ†Π°Ρ‚Π΅Π»ΡŒΠ½Ρ‹ΠΉ символ Π½Π΅ всСгда ΠΌΠΎΠΆΠ΅Ρ‚ Β«ΡΠ»Π΅Π΄ΠΎΠ²Π°Ρ‚ΡŒΒ» Π·Π° числом, ΠΊ ΠΊΠΎΡ‚ΠΎΡ€ΠΎΠΌΡƒ ΠΎΠ½ ΠΈΠ·Π½Π°Ρ‡Π°Π»ΡŒΠ½ΠΎ ΠΏΡ€ΠΈΠΊΡ€Π΅ΠΏΠ»Π΅Π½.

Π”Ρ€ΡƒΠ³ΠΈΠΌΠΈ словами, ΠΎΡ‚Ρ€ΠΈΡ†Π°Ρ‚Π΅Π»ΡŒΠ½Ρ‹ΠΉ символ ΠΌΠΎΠΆΠ΅Ρ‚ ΠΎΡΡ‚Π°Π²Π°Ρ‚ΡŒΡΡ Π² числитСлС. Π˜Ρ‚Π°ΠΊ, послС инвСртирования Π΄Π²ΡƒΡ… чисСл ΠΎΡ‚Ρ€ΠΈΡ†Π°Ρ‚Π΅Π»ΡŒΠ½Ρ‹ΠΉ символ присоСдиняСтся ΠΊ 2. Π­Ρ‚ΠΎ Ρ‚Π°ΠΊΠΆΠ΅ ΠΏΡ€Π°Π²ΠΈΠ»ΡŒΠ½Ρ‹ΠΉ ΠΎΡ‚Π²Π΅Ρ‚, ΠΏΠΎΡΠΊΠΎΠ»ΡŒΠΊΡƒ ΠΎΠ½ являСтся ΠΎΠ±Ρ€Π°Ρ‚Π½ΠΎΠΉ Π²Π΅Π»ΠΈΡ‡ΠΈΠ½ΠΎΠΉ исходной Π΄Ρ€ΠΎΠ±ΠΈ.

Π•ΡΡ‚ΡŒ Π΅Ρ‰Π΅ ΠΎΠ΄ΠΈΠ½ способ Π·Π°ΠΏΠΈΡΠ°Ρ‚ΡŒ ΠΎΠ±Ρ€Π°Ρ‚Π½ΡƒΡŽ Π²Π΅Π»ΠΈΡ‡ΠΈΠ½Ρƒ этой ΠΎΡ‚Ρ€ΠΈΡ†Π°Ρ‚Π΅Π»ΡŒΠ½ΠΎΠΉ Π΄Ρ€ΠΎΠ±ΠΈ. ΠžΡ‚Ρ€ΠΈΡ†Π°Ρ‚Π΅Π»ΡŒΠ½Ρ‹ΠΉ символ Π½Π΅ ΠΌΠΎΠΆΠ΅Ρ‚ Π±Ρ‹Ρ‚ΡŒ присоСдинСн ΠΊ Ρ‡ΠΈΡΠ»ΠΈΡ‚Π΅Π»ΡŽ ΠΈΠ»ΠΈ Π·Π½Π°ΠΌΠ΅Π½Π°Ρ‚Π΅Π»ΡŽ , .

Π’Π°ΠΊΠΆΠ΅ ΠΏΡ€Π°Π²ΠΈΠ»ΡŒΠ½ΠΎ Ρ€Π°Π·ΠΌΠ΅Ρ‰Π°Ρ‚ΡŒ ΠΎΡ‚Ρ€ΠΈΡ†Π°Ρ‚Π΅Π»ΡŒΠ½Ρ‹ΠΉ символ нСпосрСдствСнно слСва ΠΎΡ‚ Π΄Ρ€ΠΎΠ±ΠΈ .Π’ΠΎΡ‚ ΠΊΠ°ΠΊ это выглядит!

Π˜Ρ‚Π°ΠΊ, Ссли Π΄Ρ€ΠΎΠ±ΡŒ ΠΎΡ‚Ρ€ΠΈΡ†Π°Ρ‚Π΅Π»ΡŒΠ½Π°Ρ, Π΅Π΅ обратная Π²Π΅Π»ΠΈΡ‡ΠΈΠ½Π° ΠΌΠΎΠΆΠ΅Ρ‚ Π±Ρ‹Ρ‚ΡŒ записана трСмя способами.

  • ΠžΡ‚Ρ€ΠΈΡ†Π°Ρ‚Π΅Π»ΡŒΠ½Ρ‹ΠΉ символ остаСтся с числитСлСм
  • ΠžΡ‚Ρ€ΠΈΡ†Π°Ρ‚Π΅Π»ΡŒΠ½Ρ‹ΠΉ символ остаСтся со Π·Π½Π°ΠΌΠ΅Π½Π°Ρ‚Π΅Π»Π΅ΠΌ
  • ΠžΡ‚Ρ€ΠΈΡ†Π°Ρ‚Π΅Π»ΡŒΠ½Ρ‹ΠΉ символ остаСтся с Π΄Ρ€ΠΎΠ±Π½ΠΎΠΉ Ρ‡Π΅Ρ€Ρ‚ΠΎΠΉ

ΠŸΡ€ΠΈΠΌΠ΅Ρ€ 3: Π—Π°ΠΏΠΈΡˆΠΈΡ‚Π΅ ΠΎΠ±Ρ€Π°Ρ‚Π½ΡƒΡŽ Π²Π΅Π»ΠΈΡ‡ΠΈΠ½Ρƒ ΠΎΡ‚Ρ€ΠΈΡ†Π°Ρ‚Π΅Π»ΡŒΠ½ΠΎΠΉ Π΄Ρ€ΠΎΠ±ΠΈ Π½ΠΈΠΆΠ΅ трСмя Ρ€Π°Π·Π»ΠΈΡ‡Π½Ρ‹ΠΌΠΈ способами.

РСшСниС:


ΠŸΡ€ΠΈΠΌΠ΅Ρ€ 4: НайдитС Π·Π½Π°Ρ‡Π΅Π½ΠΈΠ΅, ΠΎΠ±Ρ€Π°Ρ‚Π½ΠΎΠ΅ Ρ†Π΅Π»ΠΎΠΌΡƒ числу 15.

Π›ΡŽΠ±ΠΎΠ΅ Π½Π΅Π½ΡƒΠ»Π΅Π²ΠΎΠ΅ Ρ†Π΅Π»ΠΎΠ΅ число ΠΌΠΎΠΆΠ΅Ρ‚ Π±Ρ‹Ρ‚ΡŒ Π²Ρ‹Ρ€Π°ΠΆΠ΅Π½ΠΎ Π·Π½Π°ΠΌΠ΅Π½Π°Ρ‚Π΅Π»Π΅ΠΌ , Ρ€Π°Π²Π½Ρ‹ΠΌ 1.

ИмСя Ρ‡Π΅Ρ‚ΠΊΠΈΠΉ Π·Π½Π°ΠΌΠ΅Π½Π°Ρ‚Π΅Π»ΡŒ, ΠΌΡ‹ ΠΌΠΎΠΆΠ΅ΠΌ Π»Π΅Π³ΠΊΠΎ ΠΏΠ΅Ρ€Π΅Π²Π΅Ρ€Π½ΡƒΡ‚ΡŒ эту Π΄Ρ€ΠΎΠ±ΡŒ Π²Π²Π΅Ρ€Ρ… Π΄Π½ΠΎΠΌ, Ρ‡Ρ‚ΠΎΠ±Ρ‹ ΠΏΠΎΠ»ΡƒΡ‡ΠΈΡ‚ΡŒ ΠΎΠ±Ρ€Π°Ρ‚Π½ΡƒΡŽ Π²Π΅Π»ΠΈΡ‡ΠΈΠ½Ρƒ.


ΠŸΡ€ΠΈΠΌΠ΅Ρ€ 5: НайдитС ΠΎΠ±Ρ€Π°Ρ‚Π½ΠΎΠ΅ Ρ†Π΅Π»ΠΎΠΌΡƒ числу β€” 11.

Π‘Π½Π°Ρ‡Π°Π»Π° ΠΏΠ΅Ρ€Π΅ΠΏΠΈΡˆΠΈΡ‚Π΅ это Ρ†Π΅Π»ΠΎΠ΅ число со Π·Π½Π°ΠΌΠ΅Π½Π°Ρ‚Π΅Π»Π΅ΠΌ , Ρ€Π°Π²Π½Ρ‹ΠΌ 1.

Π’Π΅ΠΏΠ΅Ρ€ΡŒ, имСя Ρ‡Π΅Ρ‚ΠΊΠΈΠΉ Π·Π½Π°ΠΌΠ΅Π½Π°Ρ‚Π΅Π»ΡŒ, ΠΌΡ‹ ΠΌΠΎΠΆΠ΅ΠΌ Π½Π°ΠΉΡ‚ΠΈ Π΅Π³ΠΎ ΠΎΠ±Ρ€Π°Ρ‚Π½ΡƒΡŽ Π²Π΅Π»ΠΈΡ‡ΠΈΠ½Ρƒ. Π£ нас Π΅ΡΡ‚ΡŒ ΠΎΡ‚Ρ€ΠΈΡ†Π°Ρ‚Π΅Π»ΡŒΠ½Ρ‹ΠΉ символ Π² числитСлС, ΠΏΠΎΠΌΠ½ΠΈΡ‚Π΅, Ρ‡Ρ‚ΠΎ Π΅Π³ΠΎ ΠΌΠΎΠΆΠ½ΠΎ Ρ€Π°ΡΠΏΠΎΠ»ΠΎΠΆΠΈΡ‚ΡŒ Π² Ρ‚Ρ€Π΅Ρ… Ρ€Π°Π·Π½Ρ‹Ρ… мСстах: Ρ‡ΠΈΡΠ»ΠΈΡ‚Π΅Π»ΡŒ, Π·Π½Π°ΠΌΠ΅Π½Π°Ρ‚Π΅Π»ΡŒ ΠΈ Π΄Ρ€ΠΎΠ±ΡŒ.Π’ΠΎΡ‚ Π²ΠΎΠ·ΠΌΠΎΠΆΠ½Ρ‹Π΅ ΠΎΡ‚Π²Π΅Ρ‚Ρ‹!


Вас Ρ‚Π°ΠΊΠΆΠ΅ ΠΌΠΎΠΆΠ΅Ρ‚ Π·Π°ΠΈΠ½Ρ‚Π΅Ρ€Π΅ΡΠΎΠ²Π°Ρ‚ΡŒ:

Π‘Π»ΠΎΠΆΠ΅Π½ΠΈΠ΅ ΠΈ Π²Ρ‹Ρ‡ΠΈΡ‚Π°Π½ΠΈΠ΅ Π΄Ρ€ΠΎΠ±Π΅ΠΉ с ΠΎΠ΄Π½ΠΈΠΌ Π·Π½Π°ΠΌΠ΅Π½Π°Ρ‚Π΅Π»Π΅ΠΌ
Π‘Π»ΠΎΠΆΠ΅Π½ΠΈΠ΅ ΠΈ Π²Ρ‹Ρ‡ΠΈΡ‚Π°Π½ΠΈΠ΅ Π΄Ρ€ΠΎΠ±Π΅ΠΉ с Ρ€Π°Π·Π½Ρ‹ΠΌΠΈ знамСнатСлями
Π£ΠΌΠ½ΠΎΠΆΠ΅Π½ΠΈΠ΅ Π΄Ρ€ΠΎΠ±Π΅ΠΉ
Π”Π΅Π»Π΅Π½ΠΈΠ΅ Π΄Ρ€ΠΎΠ±Π΅ΠΉ
Π£ΠΏΡ€ΠΎΡ‰Π΅Π½ΠΈΠ΅ Π΄Ρ€ΠΎΠ±Π΅ΠΉ
Π­ΠΊΠ²ΠΈΠ²Π°Π»Π΅Π½Ρ‚Π½Ρ‹Π΅ Π΄Ρ€ΠΎΠ±ΠΈ

Π Π°Π±ΠΎΡ‡ΠΈΠΉ лист

Π΄Ρ€ΠΎΠ±Π΅ΠΉ

Π”ΠΎΠ±Ρ€ΠΎ ΠΏΠΎΠΆΠ°Π»ΠΎΠ²Π°Ρ‚ΡŒ Π½Π° страницу Ρ€Π°Π±ΠΎΡ‡ΠΈΡ… листов для Π΄Ρ€ΠΎΠ±Π΅ΠΉ Π½Π° Math-Drills.com, Π³Π΄Π΅ Ρ‡Π°ΡˆΠΊΠ° Π½Π°ΠΏΠΎΠ»ΠΎΠ²ΠΈΠ½Ρƒ Π·Π°ΠΏΠΎΠ»Π½Π΅Π½Π°! Π‘ΠΊΠΎΡ€Π΅Π΅ всСго, это ΠΎΠ΄Π½Π° ΠΈΠ· самых популярных Π½Π°ΡˆΠΈΡ… страниц, ΠΏΠΎΡ‚ΠΎΠΌΡƒ Ρ‡Ρ‚ΠΎ ΠΈΠ·ΡƒΡ‡Π΅Π½ΠΈΠ΅ Π΄Ρ€ΠΎΠ±Π΅ΠΉ нСвСроятно Π²Π°ΠΆΠ½ΠΎ Π² ΠΆΠΈΠ·Π½ΠΈ Ρ‡Π΅Π»ΠΎΠ²Π΅ΠΊΠ°, ΠΈ это матСматичСская Ρ‚Π΅ΠΌΠ°, ΠΊ ΠΊΠΎΡ‚ΠΎΡ€ΠΎΠΉ ΠΌΠ½ΠΎΠ³ΠΈΠ΅ подходят с Ρ‚Ρ€Π΅ΠΏΠ΅Ρ‚ΠΎΠΌ ΠΈΠ·-Π·Π° ΠΏΠ»ΠΎΡ…ΠΎΠΉ Ρ€Π΅ΠΏΡƒΡ‚Π°Ρ†ΠΈΠΈ Π½Π° протяТСнии ΠΌΠ½ΠΎΠ³ΠΈΡ… Π»Π΅Ρ‚. ΠžΡΠ²ΠΎΠΈΡ‚ΡŒ Π΄Ρ€ΠΎΠ±ΠΈ Π½Π° самом Π΄Π΅Π»Π΅ Π½Π΅ Ρ‚Π°ΠΊ слоТно, особСнно с ΠΏΠΎΠ΄Π΄Π΅Ρ€ΠΆΠΊΠΎΠΉ нашСго ΡˆΠΈΡ€ΠΎΠΊΠΎΠ³ΠΎ Π²Ρ‹Π±ΠΎΡ€Π° Ρ€Π°Π±ΠΎΡ‡ΠΈΡ… листов.

Π­Ρ‚Π° страница Π²ΠΊΠ»ΡŽΡ‡Π°Π΅Ρ‚ Π² сСбя Ρ€Π°Π±ΠΎΡ‡ΠΈΠ΅ листы Π΄Ρ€ΠΎΠ±Π΅ΠΉ для понимания Π΄Ρ€ΠΎΠ±Π΅ΠΉ, Π²ΠΊΠ»ΡŽΡ‡Π°Ρ ΠΌΠΎΠ΄Π΅Π»ΠΈΡ€ΠΎΠ²Π°Π½ΠΈΠ΅, сравнСниС, упорядочСниС, ΡƒΠΏΡ€ΠΎΡ‰Π΅Π½ΠΈΠ΅ ΠΈ ΠΏΡ€Π΅ΠΎΠ±Ρ€Π°Π·ΠΎΠ²Π°Π½ΠΈΠ΅ Π΄Ρ€ΠΎΠ±Π΅ΠΉ ΠΈ ΠΎΠΏΠ΅Ρ€Π°Ρ†ΠΈΠΈ с дробями. НачнСм с ΠΎΡ‡Π΅Π²ΠΈΠ΄Π½ΠΎΠ³ΠΎ: ΠΌΠΎΠ΄Π΅Π»ΠΈΡ€ΠΎΠ²Π°Π½ΠΈΠ΅ Π΄Ρ€ΠΎΠ±Π΅ΠΉ. Π­Ρ‚ΠΎ отличная идСя, Ссли студСнты Π΄Π΅ΠΉΡΡ‚Π²ΠΈΡ‚Π΅Π»ΡŒΠ½ΠΎ ΠΌΠΎΠ³ΡƒΡ‚ ΠΏΠΎΠ½ΡΡ‚ΡŒ, Ρ‡Ρ‚ΠΎ Ρ‚Π°ΠΊΠΎΠ΅ Π΄Ρ€ΠΎΠ±ΡŒ, поэтому, поТалуйста, ΡƒΠ΄Π΅Π»ΠΈΡ‚Π΅ Π½Π΅ΠΌΠ½ΠΎΠ³ΠΎ Π²Ρ€Π΅ΠΌΠ΅Π½ΠΈ аспСкту модСлирования. Бвязь модСлирования с Ρ€Π΅Π°Π»ΡŒΠ½ΠΎΠΉ Тизнью Ρ‚ΠΎΠΆΠ΅ ΠΎΡ‡Π΅Π½ΡŒ ΠΏΠΎΠΌΠΎΠ³Π°Π΅Ρ‚, Ρ‚Π°ΠΊ ΠΊΠ°ΠΊ Π³ΠΎΡ€Π°Π·Π΄ΠΎ Π»Π΅Π³Ρ‡Π΅ ΡΠ²ΡΠ·Π°Ρ‚ΡŒ ΠΏΠΎΠ»ΠΎΠ²ΠΈΠ½Ρƒ ΠΏΠ΅Ρ‡Π΅Π½ΡŒΡ, Ρ‡Π΅ΠΌ ΠΏΠΎΠ»ΠΎΠ²ΠΈΠ½Ρƒ ΠΊΠ²Π°Π΄Ρ€Π°Ρ‚Π°.БпроситС Ρƒ Π±ΠΎΠ»ΡŒΡˆΠΈΠ½ΡΡ‚Π²Π° студСнтов, Ρ‡Ρ‚ΠΎ Π²Ρ‹ ΠΏΠΎΠ»ΡƒΡ‡ΠΈΡ‚Π΅, Ссли Π΄ΠΎΠ±Π°Π²ΠΈΡ‚Π΅ ΠΏΠΎΠ»ΠΎΠ²ΠΈΠ½Ρƒ ΠΏΠ΅Ρ‡Π΅Π½ΡŒΡ ΠΈ ΠΏΠΎΠ»ΠΎΠ²ΠΈΠ½Ρƒ ΠΏΠ΅Ρ‡Π΅Π½ΡŒΡ, ΠΈ ΠΎΠ½ΠΈ, вСроятно, Π΄Π°Π΄ΡƒΡ‚ Π²Π°ΠΌ Π·Π½Π°Ρ‚ΡŒ, Ρ‡Ρ‚ΠΎ ΠΈΠ· этого получится ΠΎΠ΄Π½Π° вкусная закуска.

Π”Ρ€ΡƒΠ³ΠΈΠ΅ Ρ€Π°Π±ΠΎΡ‡ΠΈΠ΅ листы Π½Π° этой страницС посвящСны Ρ‚ΠΎΠΌΡƒ, Ρ‡Ρ‚ΠΎΠ±Ρ‹ ΠΏΠΎΠΌΠΎΡ‡ΡŒ студСнтам ΠΏΠΎΠ½ΡΡ‚ΡŒ ΠΊΠΎΠ½Ρ†Π΅ΠΏΡ†ΠΈΡŽ Π΄Ρ€ΠΎΠ±Π΅ΠΉ. ΠžΡ‚ сравнСния ΠΈ упорядочСния Π΄ΠΎ упрощСния ΠΈ прСобразования … ΠΊ Ρ‚ΠΎΠΌΡƒ Π²Ρ€Π΅ΠΌΠ΅Π½ΠΈ, ΠΊΠΎΠ³Π΄Π° студСнты усвоят ΠΌΠ°Ρ‚Π΅Ρ€ΠΈΠ°Π» Π½Π° этой страницС, ΠΎΠΏΠ΅Ρ€Π°Ρ†ΠΈΠΈ с дробями Π±ΡƒΠ΄ΡƒΡ‚ ΠΏΠΎΡ…ΠΎΠΆΠΈ Π½Π° ΠΏΡ€ΠΎΠ³ΡƒΠ»ΠΊΡƒ ΠΏΠΎ ΠΏΠ°Ρ€ΠΊΡƒ.

Π Π°Π±ΠΎΡ‡ΠΈΠ΅ листы с самыми популярными дробями Π½Π° этой Π½Π΅Π΄Π΅Π»Π΅

РаспСчатки с дробями ΠΎΠ±Ρ‰Π΅Π³ΠΎ назначСния

Π”Ρ€ΠΎΠ±Π½Ρ‹Π΅ ΠΊΡ€ΡƒΠ³ΠΈ

Π§Π΅Ρ€Π½Ρ‹Π΅ ΠΈ Π±Π΅Π»Ρ‹Π΅ ΠΊΡ€ΡƒΠ³ΠΈ Π΄Ρ€ΠΎΠ±Π΅ΠΉ ΠΌΠΎΠΆΠ½ΠΎ ΠΈΡΠΏΠΎΠ»ΡŒΠ·ΠΎΠ²Π°Ρ‚ΡŒ ΠΊΠ°ΠΊ манипуляторы для сравнСния Π΄Ρ€ΠΎΠ±Π΅ΠΉ.Π‘Π΄Π΅Π»Π°ΠΉΡ‚Π΅ Ρ„ΠΎΡ‚ΠΎΠΊΠΎΠΏΠΈΡŽ Ρ€Π°Π±ΠΎΡ‡Π΅Π³ΠΎ листа Π½Π° слайд с ΠΏΡ€ΠΎΠ΅ΠΊΡ†ΠΈΠ΅ΠΉ свСрху. ΠšΠ°Ρ€Π°Π½Π΄Π°ΡˆΠΎΠΌ слСгка Π·Π°ΠΊΡ€Π°ΡΡŒΡ‚Π΅ ΡΠΎΠΎΡ‚Π²Π΅Ρ‚ΡΡ‚Π²ΡƒΡŽΡ‰ΠΈΠΉ ΠΊΡ€ΡƒΠΆΠΎΠΊ, Ρ‡Ρ‚ΠΎΠ±Ρ‹ ΠΈΠ·ΠΎΠ±Ρ€Π°Π·ΠΈΡ‚ΡŒ ΠΏΠ΅Ρ€Π²ΡƒΡŽ Π΄Ρ€ΠΎΠ±ΡŒ Π½Π° Π±ΡƒΠΌΠ°ΠΆΠ½ΠΎΠΉ ΠΊΠΎΠΏΠΈΠΈ. Π˜ΡΠΏΠΎΠ»ΡŒΠ·ΡƒΠΉΡ‚Π΅ Π½Π΅ΠΏΠΎΡΡ‚ΠΎΡΠ½Π½ΡƒΡŽ Ρ€ΡƒΡ‡ΠΊΡƒ Π½Π°Π΄ Π³ΠΎΠ»ΠΎΠ²ΠΎΠΉ, Ρ‡Ρ‚ΠΎΠ±Ρ‹ Ρ€Π°ΡΠΊΡ€Π°ΡΠΈΡ‚ΡŒ ΡΠΎΠΎΡ‚Π²Π΅Ρ‚ΡΡ‚Π²ΡƒΡŽΡ‰ΠΈΠΉ ΠΊΡ€ΡƒΠ³, Ρ‡Ρ‚ΠΎΠ±Ρ‹ ΠΎΠ±ΠΎΠ·Π½Π°Ρ‡ΠΈΡ‚ΡŒ Π²Ρ‚ΠΎΡ€ΡƒΡŽ Π΄Ρ€ΠΎΠ±ΡŒ. ΠŸΠΎΠ»ΠΎΠΆΠΈΡ‚Π΅ слайд Π½Π° Π±ΡƒΠΌΠ°Π³Ρƒ ΠΈ сравнитС Π΄Π²Π° ΠΊΡ€ΡƒΠ³Π°. Π’Ρ‹ Π»Π΅Π³ΠΊΠΎ смоТСтС ΠΎΠΏΡ€Π΅Π΄Π΅Π»ΠΈΡ‚ΡŒ, какая Π΄Ρ€ΠΎΠ±ΡŒ большС ΠΈΠ»ΠΈ мСньшС ΠΈΠ»ΠΈ Ρ€Π°Π²Π½Ρ‹ Π΄Π²Π΅ Π΄Ρ€ΠΎΠ±ΠΈ. Π˜ΡΠΏΠΎΠ»ΡŒΠ·ΡƒΠΉΡ‚Π΅ ΠΎΠ±Π° листа ΠΏΠΎΠ²Ρ‚ΠΎΡ€Π½ΠΎ, стирая ΠΊΠ°Ρ€Π°Π½Π΄Π°Ρˆ ΠΈ смывая ΠΌΠ°Ρ€ΠΊΠ΅Ρ€.

Π€Ρ€Π°ΠΊΡ†ΠΈΠΎΠ½Π½Ρ‹Π΅ полоски

Полоски с дробями ΠΌΠΎΠΆΠ½ΠΎ Π»Π°ΠΌΠΈΠ½ΠΈΡ€ΠΎΠ²Π°Ρ‚ΡŒ для ΠΏΠΎΠ²Ρ‹ΡˆΠ΅Π½ΠΈΡ прочности ΠΈ Π²Ρ‹Ρ€Π΅Π·Π°Ρ‚ΡŒ для сравнСния, упорядочивания, слоТСния ΠΈ вычитания Π΄Ρ€ΠΎΠ±Π΅ΠΉ.Они ΠΎΡ‡Π΅Π½ΡŒ ΠΏΠΎΠ»Π΅Π·Π½Ρ‹ для сравнСния Π΄Ρ€ΠΎΠ±Π΅ΠΉ. Π’Ρ‹ Ρ‚Π°ΠΊΠΆΠ΅ ΠΌΠΎΠΆΠ΅Ρ‚Π΅ ΡΠΊΠΎΠΏΠΈΡ€ΠΎΠ²Π°Ρ‚ΡŒ полоски с дробями Π½Π° слайды ΠΏΡ€ΠΎΠ΅ΠΊΡ†ΠΈΠΈ ΠΈ Π²Ρ‹Ρ€Π΅Π·Π°Ρ‚ΡŒ ΠΈΡ…. Они Π±ΡƒΠ΄ΡƒΡ‚ Π½Π΅ Ρ‚ΠΎΠ»ΡŒΠΊΠΎ Π΄ΠΎΠ»Π³ΠΎΠ²Π΅Ρ‡Π½Ρ‹ΠΌΠΈ, Π½ΠΎ ΠΈ ΠΏΡ€ΠΎΠ·Ρ€Π°Ρ‡Π½Ρ‹ΠΌΠΈ, Ρ‡Ρ‚ΠΎ ΠΏΠΎΠ»Π΅Π·Π½ΠΎ ΠΏΡ€ΠΈ использовании Π² сочСтании с Π±ΡƒΠΌΠ°ΠΆΠ½Ρ‹ΠΌΠΈ вСрсиями (Π½Π°ΠΏΡ€ΠΈΠΌΠ΅Ρ€, для сравнСния Π΄Ρ€ΠΎΠ±Π΅ΠΉ).

Π Π°Π±ΠΎΡ‡ΠΈΠ΅ листы модСлирования Π΄Ρ€ΠΎΠ±Π΅ΠΉ

Помимо использования ΠΏΡ€ΠΈΠ²Π΅Π΄Π΅Π½Π½Ρ‹Ρ… Π½ΠΈΠΆΠ΅ Ρ‚Π°Π±Π»ΠΈΡ†, Π²Ρ‹ Ρ‚Π°ΠΊΠΆΠ΅ ΠΌΠΎΠΆΠ΅Ρ‚Π΅ ΠΏΠΎΠΏΡ€ΠΎΠ±ΠΎΠ²Π°Ρ‚ΡŒ Π΅Ρ‰Π΅ нСсколько интСрСсных способов модСлирования Π΄Ρ€ΠΎΠ±Π΅ΠΉ. Π—Π΄ΠΎΡ€ΠΎΠ²Ρ‹Π΅ закуски ΠΌΠΎΠ³ΡƒΡ‚ ΡΡ‚Π°Ρ‚ΡŒ ΠΎΡ‚Π»ΠΈΡ‡Π½Ρ‹ΠΌΠΈ модСлями для Π΄Ρ€ΠΎΠ±Π΅ΠΉ.МоТно Π»ΠΈ Ρ€Π°Π·Ρ€Π΅Π·Π°Ρ‚ΡŒ ΠΎΠ³ΡƒΡ€Π΅Ρ† ΠΏΠΎΠΏΠΎΠ»Π°ΠΌ? ΠŸΠΎΠΌΠΈΠ΄ΠΎΡ€ Π½Π° Ρ‡Π΅Ρ‚Π²Π΅Ρ€Ρ‚ΠΈΠ½ΠΊΠΈ? МоТно Π»ΠΈ ΡΠ΄Π΅Π»Π°Ρ‚ΡŒ Π΄Π²Π΅ Ρ‚Ρ€Π΅Ρ‚ΠΈ Π²ΠΈΠ½ΠΎΠ³Ρ€Π°Π΄Π° красными, Π° ΠΎΠ΄Π½Ρƒ Ρ‚Ρ€Π΅Ρ‚ΡŒ β€” Π·Π΅Π»Π΅Π½Ρ‹ΠΌΠΈ?

ΠœΠΎΠ΄Π΅Π»ΠΈΡ€ΠΎΠ²Π°Π½ΠΈΠ΅ Ρ„Ρ€Π°ΠΊΡ†ΠΈΠΉ с Π³Ρ€ΡƒΠΏΠΏΠ°ΠΌΠΈ Ρ„ΠΈΠ³ΡƒΡ€

Π”Ρ€ΠΎΠ±ΠΈ ΠΌΠΎΠ³ΡƒΡ‚ ΠΏΡ€Π΅Π΄ΡΡ‚Π°Π²Π»ΡΡ‚ΡŒ части Π³Ρ€ΡƒΠΏΠΏΡ‹ ΠΈΠ»ΠΈ части Ρ†Π΅Π»ΠΎΠ³ΠΎ. Π’ этих Ρ‚Π°Π±Π»ΠΈΡ†Π°Ρ… Π΄Ρ€ΠΎΠ±ΠΈ ΠΌΠΎΠ΄Π΅Π»ΠΈΡ€ΡƒΡŽΡ‚ΡΡ ΠΊΠ°ΠΊ части Π³Ρ€ΡƒΠΏΠΏΡ‹.

ΠœΠΎΠ΄Π΅Π»ΠΈΡ€ΠΎΠ²Π°Π½ΠΈΠ΅ Ρ„Ρ€Π°ΠΊΡ†ΠΈΠΉ ΠΏΡ€ΡΠΌΠΎΡƒΠ³ΠΎΠ»ΡŒΠ½ΠΈΠΊΠ°ΠΌΠΈ

ΠœΠΎΠ΄Π΅Π»ΠΈΡ€ΠΎΠ²Π°Π½ΠΈΠ΅ Ρ„Ρ€Π°ΠΊΡ†ΠΈΠΉ с ΠΊΡ€ΡƒΠ³Π°ΠΌΠΈ

Π’Π°Π±Π»ΠΈΡ†Ρ‹ ΡΠΎΠΎΡ‚Π½ΠΎΡˆΠ΅Π½ΠΈΠΉ ΠΈ ΠΏΡ€ΠΎΠΏΠΎΡ€Ρ†ΠΈΠΉ

Π‘ΠΎΠΎΡ‚Π½ΠΎΡˆΠ΅Π½ΠΈΠ΅ ΠΈΠ·ΠΎΠ±Ρ€Π°ΠΆΠ΅Π½ΠΈΠΉ

Π­ΠΊΠ²ΠΈΠ²Π°Π»Π΅Π½Ρ‚Π½Ρ‹Π΅ Π΄Ρ€ΠΎΠ±ΠΈ

Π Π°Π±ΠΎΡ‡ΠΈΠ΅ листы ΠΌΠΎΠ΄Π΅Π»Π΅ΠΉ эквивалСнтных Ρ„Ρ€Π°ΠΊΡ†ΠΈΠΉ Π²ΠΊΠ»ΡŽΡ‡Π°ΡŽΡ‚ Ρ‚ΠΎΠ»ΡŒΠΊΠΎ Β«Ρ„Ρ€Π°ΠΊΡ†ΠΈΠΈ Π²Ρ‹ΠΏΠ΅Ρ‡ΠΊΠΈΒ» Π² вСрсиях A. Π§Ρ‚ΠΎΠ±Ρ‹ ΡƒΠ²ΠΈΠ΄Π΅Ρ‚ΡŒ Π±ΠΎΠ»Π΅Π΅ слоТныС ΠΈ Ρ€Π°Π·Π½ΠΎΠΎΠ±Ρ€Π°Π·Π½Ρ‹Π΅ Π΄Ρ€ΠΎΠ±ΠΈ, Π²Ρ‹Π±Π΅Ρ€ΠΈΡ‚Π΅ вСрсии ΠΎΡ‚ B Π΄ΠΎ J послС Π·Π°Π³Ρ€ΡƒΠ·ΠΊΠΈ вСрсии A.

Π­ΠΊΠ²ΠΈΠ²Π°Π»Π΅Π½Ρ‚Π½Ρ‹Π΅ ΡΠΎΠΎΡ‚Π½ΠΎΡˆΠ΅Π½ΠΈΡ

Π’Π°Π±Π»ΠΈΡ†Ρ‹ сравнСния ΠΈ сортировки Π΄Ρ€ΠΎΠ±Π΅ΠΉ

Π‘Ρ€Π°Π²Π½Π΅Π½ΠΈΠ΅ простых Π΄Ρ€ΠΎΠ±Π΅ΠΉ

Π•ΡΡ‚ΡŒ ΠΌΠ½ΠΎΠ³ΠΎ Ρ€Π°Π·Π½Ρ‹Ρ… стратСгий, ΠΊΡ€ΠΎΠΌΠ΅ просмотра страницы, ΠΊΠΎΡ‚ΠΎΡ€Ρ‹Π΅ ΠΏΠΎΠΌΠΎΠ³ΡƒΡ‚ Π² сравнСнии Π΄Ρ€ΠΎΠ±Π΅ΠΉ. ΠŸΠΎΠΏΡ€ΠΎΠ±ΡƒΠΉΡ‚Π΅ Π½Π°Ρ‡Π°Ρ‚ΡŒ с Ρ‡Π΅Π³ΠΎ-Π½ΠΈΠ±ΡƒΠ΄ΡŒ наглядного, Ρ‡Ρ‚ΠΎ Π±ΡƒΠ΄Π΅Ρ‚ ΠΈΠ·ΠΎΠ±Ρ€Π°ΠΆΠ°Ρ‚ΡŒ рассматриваСмыС Π΄Ρ€ΠΎΠ±ΠΈ.ΠœΡ‹ Π½Π°ΡΡ‚ΠΎΡΡ‚Π΅Π»ΡŒΠ½ΠΎ Ρ€Π΅ΠΊΠΎΠΌΠ΅Π½Π΄ΡƒΠ΅ΠΌ наши Π΄Ρ€ΠΎΠ±Π½Ρ‹Π΅ полоски (ΠΏΡ€ΠΎΠΊΡ€ΡƒΡ‚ΠΈΡ‚Π΅ Π½Π΅ΠΌΠ½ΠΎΠ³ΠΎ Π²Π²Π΅Ρ€Ρ…). ИспользованиС Π»ΠΈΠ½Π΅ΠΉΠΊΠΈ, Ρ‚Π°ΠΊΠΎΠΉ ΠΊΠ°ΠΊ Π»ΠΈΠ½Π΅ΠΉΠΊΠ°, ΠΊΠ½ΠΈΠ³Π° ΠΈΠ»ΠΈ складка, ΠΏΠΎΠΌΠΎΠΆΠ΅Ρ‚ учащимся Π»Π΅Π³ΠΊΠΎ ΡƒΠ²ΠΈΠ΄Π΅Ρ‚ΡŒ, какая Π΄Ρ€ΠΎΠ±ΡŒ большС ΠΈΠ»ΠΈ Ρ€Π°Π²Π½Π° Π»ΠΈ ΠΎΠ½ΠΈ. Π‘Π»Π΅Π΄ΡƒΠ΅Ρ‚ Ρ‚Π°ΠΊΠΆΠ΅ ΠΎΡ‚ΠΌΠ΅Ρ‚ΠΈΡ‚ΡŒ, Ρ‡Ρ‚ΠΎ сравниваСмыС Π²Π΅Ρ‰ΠΈ Π΄ΠΎΠ»ΠΆΠ½Ρ‹ Π±Ρ‹Ρ‚ΡŒ ΠΎΠ΄ΠΈΠ½Π°ΠΊΠΎΠ²Ρ‹ΠΌΠΈ. НапримСр, каТдая фракционная полоска ΠΈΠΌΠ΅Π΅Ρ‚ ΠΎΠ΄ΠΈΠ½Π°ΠΊΠΎΠ²Ρ‹ΠΉ Ρ€Π°Π·ΠΌΠ΅Ρ€, Ρ‚ΠΎΠ³Π΄Π° ΠΊΠ°ΠΊ Ссли Π²Ρ‹ Π²ΠΎΠ·ΡŒΠΌΠ΅Ρ‚Π΅ Ρ‚Ρ€Π΅Ρ‚ΡŒ Π°Ρ€Π±ΡƒΠ·Π° ΠΈ ΠΏΠΎΠ»ΠΎΠ²ΠΈΠ½Ρƒ Π²ΠΈΠ½ΠΎΠ³Ρ€Π°Π΄Π°, Π°Ρ€Π±ΡƒΠ·, вСроятно, ΠΏΠΎΠ±Π΅Π΄ΠΈΡ‚.

Π‘Ρ€Π°Π²Π½Π΅Π½ΠΈΠ΅ простых ΠΈ Π½Π΅ΠΏΡ€Π°Π²ΠΈΠ»ΡŒΠ½Ρ‹Ρ… Π΄Ρ€ΠΎΠ±Π΅ΠΉ

Π•Ρ‰Π΅ ΠΎΠ΄Π½Π° стратСгия, ΠΊΠΎΡ‚ΠΎΡ€ΡƒΡŽ ΠΌΠΎΠΆΠ½ΠΎ ΠΈΡΠΏΠΎΠ»ΡŒΠ·ΠΎΠ²Π°Ρ‚ΡŒ ΠΏΡ€ΠΈ сравнСнии Π΄Ρ€ΠΎΠ±Π΅ΠΉ, β€” ΠΈΡΠΏΠΎΠ»ΡŒΠ·ΠΎΠ²Π°Ρ‚ΡŒ Ρ‡ΠΈΡΠ»ΠΎΠ²ΡƒΡŽ линию ΠΈ ΠΈΡΠΏΠΎΠ»ΡŒΠ·ΠΎΠ²Π°Ρ‚ΡŒ ΠΊΠΎΠ½Ρ‚Ρ€ΠΎΠ»ΡŒΠ½Ρ‹Π΅ Ρ†ΠΈΡ„Ρ€Ρ‹, Ρ‚Π°ΠΊΠΈΠ΅ ΠΊΠ°ΠΊ 0, 1, 1/2, Ρ‡Ρ‚ΠΎΠ±Ρ‹ Π²Ρ‹ΡΡΠ½ΠΈΡ‚ΡŒ, ΠΊΡƒΠ΄Π° ΠΈΠ΄Π΅Ρ‚ каТдая Π΄Ρ€ΠΎΠ±ΡŒ, Π° Π·Π°Ρ‚Π΅ΠΌ ΠΏΠΎΡΠΌΠΎΡ‚Ρ€Π΅Ρ‚ΡŒ, какая ΠΈΠ· Π½ΠΈΡ… большС. Π‘Ρ‚ΡƒΠ΄Π΅Π½Ρ‚Ρ‹ Π½Π° самом Π΄Π΅Π»Π΅ Π΄Π΅Π»Π°ΡŽΡ‚ это всС врСмя, Ρ‚Π°ΠΊ ΠΊΠ°ΠΊ ΠΎΠ½ΠΈ часто ΠΌΠΎΠ³ΡƒΡ‚ ΡΡ€Π°Π²Π½ΠΈΠ²Π°Ρ‚ΡŒ Π΄Ρ€ΠΎΠ±ΠΈ, осознавая, Ρ‡Ρ‚ΠΎ ΠΎΠ΄Π½Π° мСньшС ΠΏΠΎΠ»ΠΎΠ²ΠΈΠ½Ρ‹, Π° другая большС ΠΏΠΎΠ»ΠΎΠ²ΠΈΠ½Ρ‹. Они Ρ‚Π°ΠΊΠΆΠ΅ ΠΌΠΎΠ³ΡƒΡ‚ ΡƒΠ²ΠΈΠ΄Π΅Ρ‚ΡŒ, Ρ‡Ρ‚ΠΎ ΠΎΠ΄Π½Π° фракция Π½Π°ΠΌΠ½ΠΎΠ³ΠΎ Π±Π»ΠΈΠΆΠ΅ ΠΊ Ρ†Π΅Π»ΠΎΠΌΡƒ, Ρ‡Π΅ΠΌ другая фракция, Π΄Π°ΠΆΠ΅ Ссли ΠΎΠ½ΠΈ ΠΎΠ±Π΅ ΠΌΠΎΠ³ΡƒΡ‚ Π±Ρ‹Ρ‚ΡŒ большС ΠΏΠΎΠ»ΠΎΠ²ΠΈΠ½Ρ‹.

Π‘Ρ€Π°Π²Π½Π΅Π½ΠΈΠ΅ простых, Π½Π΅ΠΏΡ€Π°Π²ΠΈΠ»ΡŒΠ½Ρ‹Ρ… ΠΈ ΡΠΌΠ΅ΡˆΠ°Π½Π½Ρ‹Ρ… Π΄Ρ€ΠΎΠ±Π΅ΠΉ

Другая стратСгия сравнСния Π΄Ρ€ΠΎΠ±Π΅ΠΉ β€” ΠΏΡ€Π΅ΠΎΠ±Ρ€Π°Π·ΠΎΠ²Π°Ρ‚ΡŒ ΠΊΠ°ΠΆΠ΄ΡƒΡŽ Π΄Ρ€ΠΎΠ±ΡŒ Π² Π΄Π΅ΡΡΡ‚ΠΈΡ‡Π½ΡƒΡŽ Π΄Ρ€ΠΎΠ±ΡŒ ΠΈ ΡΡ€Π°Π²Π½ΠΈΡ‚ΡŒ дСсятичныС Π΄Ρ€ΠΎΠ±ΠΈ.ДСсятичныС прСобразования ΠΌΠΎΠΆΠ½ΠΎ Π·Π°ΠΏΠΎΠΌΠ½ΠΈΡ‚ΡŒ (особСнно для ΠΎΠ±Ρ‹Ρ‡Π½Ρ‹Ρ… Π΄Ρ€ΠΎΠ±Π΅ΠΉ), рассчитанныС ΠΏΡƒΡ‚Π΅ΠΌ дСлСния Π² столбик, с ΠΏΠΎΠΌΠΎΡ‰ΡŒΡŽ ΠΊΠ°Π»ΡŒΠΊΡƒΠ»ΡΡ‚ΠΎΡ€Π° ΠΈΠ»ΠΈ справочной Ρ‚Π°Π±Π»ΠΈΡ†Ρ‹. ΠœΡ‹ ΠΏΡ€Π΅Π΄Π»Π°Π³Π°Π΅ΠΌ послСднСС, ΠΏΠΎΡΠΊΠΎΠ»ΡŒΠΊΡƒ использованиС справочной Ρ‚Π°Π±Π»ΠΈΡ†Ρ‹ часто ΠΏΡ€ΠΈΠ²ΠΎΠ΄ΠΈΡ‚ ΠΊ мыслСнному вспоминанию.

Π—Π°ΠΊΠ°Π· Π΄Ρ€ΠΎΠ±Π΅ΠΉ Π² числовой строкС

МногиС ΠΈΠ· Ρ‚Π΅Ρ… ΠΆΠ΅ стратСгий, ΠΊΠΎΡ‚ΠΎΡ€Ρ‹Π΅ Ρ€Π°Π±ΠΎΡ‚Π°ΡŽΡ‚ для сравнСния Π΄Ρ€ΠΎΠ±Π΅ΠΉ, Ρ‚Π°ΠΊΠΆΠ΅ Ρ€Π°Π±ΠΎΡ‚Π°ΡŽΡ‚ для упорядочивания Π΄Ρ€ΠΎΠ±Π΅ΠΉ. ИспользованиС Ρ‚Π°ΠΊΠΈΡ… манипуляторов, ΠΊΠ°ΠΊ полоски Π΄Ρ€ΠΎΠ±Π΅ΠΉ, использованиС числовых Π»ΠΈΠ½ΠΈΠΉ ΠΈΠ»ΠΈ поиск дСсятичных эквивалСнтов, заставит Π²Π°ΡˆΠΈΡ… ΡƒΡ‡Π΅Π½ΠΈΠΊΠΎΠ² быстро Ρ€Π°ΡΡΡ‚Π°Π²ΠΈΡ‚ΡŒ Π΄Ρ€ΠΎΠ±ΠΈ Π² ΠΏΡ€Π°Π²ΠΈΠ»ΡŒΠ½ΠΎΠΌ порядкС.ΠœΡ‹, вСроятно, Π³ΠΎΠ²ΠΎΡ€ΠΈΠ»ΠΈ ΠΎΠ± этом Ρ€Π°Π½ΡŒΡˆΠ΅, Π½ΠΎ ΡƒΠ±Π΅Π΄ΠΈΡ‚Π΅ΡΡŒ, Ρ‡Ρ‚ΠΎ Π²Ρ‹ ΠΏΠΎΠ΄Ρ‡Π΅Ρ€ΠΊΠ½ΡƒΠ»ΠΈ, Ρ‡Ρ‚ΠΎ ΠΏΡ€ΠΈ сравнСнии ΠΈΠ»ΠΈ упорядочивании Π΄Ρ€ΠΎΠ±Π΅ΠΉ учащиСся ΠΏΠΎΠ½ΠΈΠΌΠ°ΡŽΡ‚, Ρ‡Ρ‚ΠΎ Ρ†Π΅Π»ΠΎΠ΅ Π΄ΠΎΠ»ΠΆΠ½ΠΎ Π±Ρ‹Ρ‚ΡŒ ΠΎΠ΄ΠΈΠ½Π°ΠΊΠΎΠ²Ρ‹ΠΌ. Π‘Ρ€Π°Π²Π½Π΅Π½ΠΈΠ΅ ΠΏΠΎΠ»ΠΎΠ²ΠΈΠ½Ρ‹ насСлСния ΠšΠ°Π½Π°Π΄Ρ‹ с Ρ‚Ρ€Π΅Ρ‚ΡŒΡŽ насСлСния Π‘ΠΎΠ΅Π΄ΠΈΠ½Π΅Π½Π½Ρ‹Ρ… Π¨Ρ‚Π°Ρ‚ΠΎΠ² Π½Π΅ ΠΏΠΎΠΌΠΎΠΆΠ΅Ρ‚. ΠŸΠΎΠΏΡ€ΠΎΠ±ΡƒΠΉΡ‚Π΅ ΠΈΡΠΏΠΎΠ»ΡŒΠ·ΠΎΠ²Π°Ρ‚ΡŒ Π²ΠΈΠ·ΡƒΠ°Π»ΡŒΠ½Ρ‹Π΅ эффСкты, Ρ‡Ρ‚ΠΎΠ±Ρ‹ ΡƒΡΠΈΠ»ΠΈΡ‚ΡŒ эту Π²Π°ΠΆΠ½ΡƒΡŽ ΠΊΠΎΠ½Ρ†Π΅ΠΏΡ†ΠΈΡŽ. НСсмотря Π½Π° Ρ‚ΠΎ, Ρ‡Ρ‚ΠΎ ΠΌΡ‹ Π²ΠΊΠ»ΡŽΡ‡ΠΈΠ»ΠΈ числовыС Π»ΠΈΠ½ΠΈΠΈ Π½ΠΈΠΆΠ΅, Π²Ρ‹ ΠΌΠΎΠΆΠ΅Ρ‚Π΅ свободно ΠΈΡΠΏΠΎΠ»ΡŒΠ·ΠΎΠ²Π°Ρ‚ΡŒ свои собствСнныС стратСгии.

Π—Π°ΠΊΠ°Π· Π΄Ρ€ΠΎΠ±Π΅ΠΉ

Π Π°Π±ΠΎΡ‡ΠΈΠ΅ листы с упорядочСниСм Π΄Ρ€ΠΎΠ±Π΅ΠΉ Π² этом Ρ€Π°Π·Π΄Π΅Π»Π΅ Π½Π΅ Π²ΠΊΠ»ΡŽΡ‡Π°ΡŽΡ‚ Ρ‡ΠΈΡΠ»ΠΎΠ²ΡƒΡŽ строку, Ρ‡Ρ‚ΠΎΠ±Ρ‹ студСнты ΠΌΠΎΠ³Π»ΠΈ ΠΈΡΠΏΠΎΠ»ΡŒΠ·ΠΎΠ²Π°Ρ‚ΡŒ Ρ€Π°Π·Π»ΠΈΡ‡Π½Ρ‹Π΅ стратСгии сортировки.

Π Π°Π±ΠΎΡ‡ΠΈΠ΅ листы для упрощСния ΠΈ прСобразования Π΄Ρ€ΠΎΠ±Π΅ΠΉ

ΠžΠΊΡ€ΡƒΠ³Π»Π΅Π½ΠΈΠ΅ Π΄Ρ€ΠΎΠ±Π΅ΠΉ

ΠžΠΊΡ€ΡƒΠ³Π»Π΅Π½ΠΈΠ΅ Π΄Ρ€ΠΎΠ±Π΅ΠΉ ΠΏΠΎΠΌΠΎΠ³Π°Π΅Ρ‚ учащимся Π½Π΅ΠΌΠ½ΠΎΠ³ΠΎ Π»ΡƒΡ‡ΡˆΠ΅ ΠΏΠΎΠ½ΠΈΠΌΠ°Ρ‚ΡŒ Π΄Ρ€ΠΎΠ±ΠΈ ΠΈ ΠΌΠΎΠΆΠ΅Ρ‚ ΠΏΡ€ΠΈΠΌΠ΅Π½ΡΡ‚ΡŒΡΡ для ΠΎΡ†Π΅Π½ΠΊΠΈ ΠΎΡ‚Π²Π΅Ρ‚ΠΎΠ² Π½Π° вопросы ΠΎ дробях. НапримСр, Ссли Π½ΡƒΠΆΠ½ΠΎ Π±Ρ‹Π»ΠΎ ΠΎΡ†Π΅Π½ΠΈΡ‚ΡŒ 1 4/7 Γ— 6, ΠΎΠ½ΠΈ, вСроятно, ΠΌΠΎΠ³Π»ΠΈ Π±Ρ‹ ΡΠΊΠ°Π·Π°Ρ‚ΡŒ, Ρ‡Ρ‚ΠΎ ΠΎΡ‚Π²Π΅Ρ‚ Π±ΡƒΠ΄Π΅Ρ‚ ΠΏΡ€ΠΈΠΌΠ΅Ρ€Π½ΠΎ 9, ΠΏΠΎΡΠΊΠΎΠ»ΡŒΠΊΡƒ 1 4/7 составляСт ΠΏΡ€ΠΈΠΌΠ΅Ρ€Π½ΠΎ 1 1/2, Π° 1 1/2 Γ— 6 β€” 9.

Π£ΠΏΡ€ΠΎΡ‰Π΅Π½ΠΈΠ΅ Π΄Ρ€ΠΎΠ±Π΅ΠΉ

Π˜Π·ΡƒΡ‡Π΅Π½ΠΈΠ΅ Ρ‚ΠΎΠ³ΠΎ, ΠΊΠ°ΠΊ ΡƒΠΏΡ€ΠΎΡ‰Π°Ρ‚ΡŒ Π΄Ρ€ΠΎΠ±ΠΈ, Π·Π½Π°Ρ‡ΠΈΡ‚Π΅Π»ΡŒΠ½ΠΎ ΠΎΠ±Π»Π΅Π³Ρ‡Π°Π΅Ρ‚ Тизнь студСнта Π² дальнСйшСм, ΠΊΠΎΠ³Π΄Π° ΠΎΠ½ Π±ΡƒΠ΄Π΅Ρ‚ ΠΈΠ·ΡƒΡ‡Π°Ρ‚ΡŒ ΠΎΠΏΠ΅Ρ€Π°Ρ†ΠΈΠΈ с дробями.Π­Ρ‚ΠΎ Ρ‚Π°ΠΊΠΆΠ΅ ΠΏΠΎΠΌΠΎΠ³Π°Π΅Ρ‚ ΠΈΠΌ ΡƒΠ·Π½Π°Ρ‚ΡŒ, Ρ‡Ρ‚ΠΎ Π΄Ρ€ΠΎΠ±ΠΈ, ΠΊΠΎΡ‚ΠΎΡ€Ρ‹Π΅ выглядят ΠΏΠΎ-Ρ€Π°Π·Π½ΠΎΠΌΡƒ, ΠΌΠΎΠ³ΡƒΡ‚ Π±Ρ‹Ρ‚ΡŒ эквивалСнтными. Один ΠΈΠ· способов ΠΏΡ€ΠΎΠ΄Π΅ΠΌΠΎΠ½ΡΡ‚Ρ€ΠΈΡ€ΠΎΠ²Π°Ρ‚ΡŒ это β€” Ρ€Π°Π·Π΄Π΅Π»ΠΈΡ‚ΡŒ Π΄Π²Π΅ эквивалСнтныС Π΄Ρ€ΠΎΠ±ΠΈ. НапримСр, 3/2 ΠΈ 6/4 ΠΏΡ€ΠΈ Π΄Π΅Π»Π΅Π½ΠΈΠΈ Π΄Π°ΡŽΡ‚ частноС 1,5. Π‘ΡƒΠ΄Π΅ΠΌ Π½Π°Π΄Π΅ΡΡ‚ΡŒΡΡ, Ρ‡Ρ‚ΠΎ практикуя ΡƒΠΏΡ€ΠΎΡ‰Π΅Π½ΠΈΠ΅ Π΄Ρ€ΠΎΠ±Π΅ΠΉ, студСнты Π±ΡƒΠ΄ΡƒΡ‚ Ρ€Π°ΡΠΏΠΎΠ·Π½Π°Π²Π°Ρ‚ΡŒ Π½Π΅ΡƒΠΏΡ€ΠΎΡ‰Π΅Π½Π½Ρ‹Π΅ Π΄Ρ€ΠΎΠ±ΠΈ, ΠΊΠΎΠ³Π΄Π° ΠΎΠ½ΠΈ Π½Π°Ρ‡Π½ΡƒΡ‚ ΡΠΊΠ»Π°Π΄Ρ‹Π²Π°Ρ‚ΡŒ, Π²Ρ‹Ρ‡ΠΈΡ‚Π°Ρ‚ΡŒ, ΡƒΠΌΠ½ΠΎΠΆΠ°Ρ‚ΡŒ ΠΈ Π΄Π΅Π»ΠΈΡ‚ΡŒ Π½Π° Π΄Ρ€ΠΎΠ±ΠΈ.

ΠŸΡ€Π΅ΠΎΠ±Ρ€Π°Π·ΠΎΠ²Π°Π½ΠΈΠ΅ ΠΈΠ· Π½Π΅ΠΏΡ€Π°Π²ΠΈΠ»ΡŒΠ½ΠΎΠΉ Π΄Ρ€ΠΎΠ±ΠΈ Π² ΡΠΌΠ΅ΡˆΠ°Π½Π½ΡƒΡŽ

ΠŸΡ€Π΅ΠΎΠ±Ρ€Π°Π·ΠΎΠ²Π°Π½ΠΈΠ΅ ΠΌΠ΅ΠΆΠ΄Ρƒ дробями, дСсятичными Π·Π½Π°ΠΊΠ°ΠΌΠΈ, ΠΏΡ€ΠΎΡ†Π΅Π½Ρ‚Π°ΠΌΠΈ ΠΈ ΠΎΡ‚Π½ΠΎΡˆΠ΅Π½ΠΈΡΠΌΠΈ ΠŸΡ€Π΅ΠΎΠ±Ρ€Π°Π·ΠΎΠ²Π°Π½ΠΈΠ΅ Π΄Ρ€ΠΎΠ±Π΅ΠΉ Π² Π·Π°Π²Π΅Ρ€ΡˆΠ°ΡŽΡ‰ΠΈΠ΅ дСсятичныС Π΄Ρ€ΠΎΠ±ΠΈ ΠŸΡ€Π΅ΠΎΠ±Ρ€Π°Π·ΠΎΠ²Π°Π½ΠΈΠ΅ Π΄Ρ€ΠΎΠ±Π΅ΠΉ Π² Π·Π°Π²Π΅Ρ€ΡˆΠ°ΡŽΡ‰ΠΈΠ΅ ΠΈ ΠΏΠΎΠ²Ρ‚ΠΎΡ€ΡΡŽΡ‰ΠΈΠ΅ΡΡ дСсятичныС Π΄Ρ€ΠΎΠ±ΠΈ ΠŸΡ€Π΅ΠΎΠ±Ρ€Π°Π·ΠΎΠ²Π°Π½ΠΈΠ΅ Π·Π°Π²Π΅Ρ€ΡˆΠ°ΡŽΡ‰ΠΈΡ… дСсятичных Π·Π½Π°ΠΊΠΎΠ² Π² Π΄Ρ€ΠΎΠ±ΠΈ ΠŸΡ€Π΅ΠΎΠ±Ρ€Π°Π·ΠΎΠ²Π°Π½ΠΈΠ΅ Π·Π°Π²Π΅Ρ€ΡˆΠ°ΡŽΡ‰ΠΈΡ… ΠΈ ΠΏΠΎΠ²Ρ‚ΠΎΡ€ΡΡŽΡ‰ΠΈΡ…ΡΡ дСсятичных Π·Π½Π°ΠΊΠΎΠ² Π² Π΄Ρ€ΠΎΠ±ΠΈ ΠŸΡ€Π΅ΠΎΠ±Ρ€Π°Π·ΠΎΠ²Π°Π½ΠΈΠ΅ Π΄Ρ€ΠΎΠ±Π΅ΠΉ Π² сотыС

ΠŸΡ€Π΅ΠΎΠ±Ρ€Π°Π·ΠΎΠ²Π°Π½ΠΈΠ΅ Π΄Ρ€ΠΎΠ±Π΅ΠΉ Π² дСсятичныС, ΠΏΡ€ΠΎΡ†Π΅Π½Ρ‚Ρ‹ ΠΈ частичныС Π² Части ΡΠΎΠΎΡ‚Π½ΠΎΡˆΠ΅Π½ΠΈΠΉ ( Ρ‚ΠΎΠ»ΡŒΠΊΠΎ Π·Π°Π²Π΅Ρ€ΡˆΠ°ΡŽΡ‰ΠΈΠ΅ дСсятичныС Π΄Ρ€ΠΎΠ±ΠΈ) ΠŸΡ€Π΅ΠΎΠ±Ρ€Π°Π·ΠΎΠ²Π°Π½ΠΈΠ΅ Π΄Ρ€ΠΎΠ±Π΅ΠΉ Π² дСсятичныС Π΄Ρ€ΠΎΠ±ΠΈ, ΠΏΡ€ΠΎΡ†Π΅Π½Ρ‚Ρ‹ ΠΈ части-ΠΊ- Ρ†Π΅Π»Ρ‹Π΅ ΠΎΡ‚Π½ΠΎΡˆΠ΅Π½ΠΈΡ ( Ρ‚ΠΎΠ»ΡŒΠΊΠΎ Π·Π°Π²Π΅Ρ€ΡˆΠ°ΡŽΡ‰ΠΈΠ΅ дСсятичныС Π΄Ρ€ΠΎΠ±ΠΈ) ΠŸΡ€Π΅ΠΎΠ±Ρ€Π°Π·ΠΎΠ²Π°Π½ΠΈΠ΅ дСсятичных Π΄Ρ€ΠΎΠ±Π΅ΠΉ Π² Π΄Ρ€ΠΎΠ±ΠΈ, ΠΏΡ€ΠΎΡ†Π΅Π½Ρ‚Ρ‹ ΠΈ частичныС-ΠΊ- Части ΠΎΡ‚Π½ΠΎΡˆΠ΅Π½ΠΈΡ ( Ρ‚ΠΎΠ»ΡŒΠΊΠΎ Π·Π°Π²Π΅Ρ€ΡˆΠ°ΡŽΡ‰ΠΈΠ΅ дСсятичныС Π΄Ρ€ΠΎΠ±ΠΈ) ΠŸΡ€Π΅ΠΎΠ±Ρ€Π°Π·ΠΎΠ²Π°Π½ΠΈΠ΅ дСсятичных Π΄Ρ€ΠΎΠ±Π΅ΠΉ Π² Π΄Ρ€ΠΎΠ±ΠΈ, ΠΏΡ€ΠΎΡ†Π΅Π½Ρ‚Ρ‹ ΠΈ части-ΠΊ- Ρ†Π΅Π»Ρ‹Π΅ ΠΎΡ‚Π½ΠΎΡˆΠ΅Π½ΠΈΡ ( Ρ‚ΠΎΠ»ΡŒΠΊΠΎ Π·Π°Π²Π΅Ρ€ΡˆΠ°ΡŽΡ‰ΠΈΠ΅ дСсятичныС числа) ΠŸΡ€Π΅ΠΎΠ±Ρ€Π°Π·ΠΎΠ²Π°Π½ΠΈΠ΅ ΠΏΡ€ΠΎΡ†Π΅Π½Ρ‚ΠΎΠ² Π² Π΄Ρ€ΠΎΠ±ΠΈ, дСсятичныС Π΄Ρ€ΠΎΠ±ΠΈ ΠΈ частичныС-ΠΊ- частичныС ΠΎΡ‚Π½ΠΎΡˆΠ΅Π½ΠΈΡ ( Ρ‚ΠΎΠ»ΡŒΠΊΠΎ Π·Π°Π²Π΅Ρ€ΡˆΠ°ΡŽΡ‰ΠΈΠ΅ дСсятичныС Π΄Ρ€ΠΎΠ±ΠΈ) ΠŸΡ€Π΅ΠΎΠ±Ρ€Π°Π·ΠΎΠ²Π°Π½ΠΈΠ΅ ΠΏΡ€ΠΎΡ†Π΅Π½Ρ‚ΠΎΠ² Π² Π΄Ρ€ΠΎΠ±ΠΈ, дСсятичныС Π΄Ρ€ΠΎΠ±ΠΈ ΠΈ частичныС-ΠΊ- Ρ†Π΅Π»Ρ‹Π΅ ΠΎΡ‚Π½ΠΎΡˆΠ΅Π½ΠΈΡ ( Ρ‚ΠΎΠ»ΡŒΠΊΠΎ Π·Π°Π²Π΅Ρ€ΡˆΠ°ΡŽΡ‰ΠΈΠ΅ дСсятичныС Π΄Ρ€ΠΎΠ±ΠΈ) ΠŸΡ€Π΅ΠΎΠ±Ρ€Π°Π·ΠΎΠ²Π°Π½ΠΈΠ΅ Π΄Ρ€ΠΎΠ±Π½Ρ‹Ρ… ΡΠΎΠΎΡ‚Π½ΠΎΡˆΠ΅Π½ΠΈΠΉ Π² Π΄Ρ€ΠΎΠ±ΠΈ, дСсятичныС Π΄Ρ€ΠΎΠ±ΠΈ ΠΈ ΠΏΡ€ΠΎΡ†Π΅Π½Ρ‚Ρ‹ ( Ρ‚ΠΎΠ»ΡŒΠΊΠΎ Π·Π°Π²Π΅Ρ€ΡˆΠ°Π΅Ρ‚ дСсятичныС Π΄Ρ€ΠΎΠ±ΠΈ) ΠŸΡ€Π΅ΠΎΠ±Ρ€Π°Π·ΠΎΠ²Π°Π½ΠΈΠ΅ ΠΎΡ‚Π½ΠΎΡˆΠ΅Π½ΠΈΡ части ΠΊ Ρ†Π΅Π»ΠΎΠΌΡƒ Π² Π΄Ρ€ΠΎΠ±ΠΈ, дСсятичныС Π΄Ρ€ΠΎΠ±ΠΈ ΠΈ ΠΏΡ€ΠΎΡ†Π΅Π½Ρ‚Ρ‹ ( Ρ‚ΠΎΠ»ΡŒΠΊΠΎ Π·Π°Π²Π΅Ρ€ΡˆΠ°Π΅Ρ‚ дСсятичныС Π΄Ρ€ΠΎΠ±ΠΈ) ΠŸΡ€Π΅ΠΎΠ±Ρ€Π°Π·ΠΎΠ²Π°Π½ΠΈΠ΅ Ρ€Π°Π·Π»ΠΈΡ‡Π½Ρ‹Ρ… Π΄Ρ€ΠΎΠ±Π΅ΠΉ, дСсятичных Π·Π½Π°ΠΊΠΎΠ², ΠΏΡ€ΠΎΡ†Π΅Π½Ρ‚ΠΎΠ² ΠΈ Π΄Ρ€ΠΎΠ±Π½Ρ‹Ρ… чисСл Π² Части ΡΠΎΠΎΡ‚Π½ΠΎΡˆΠ΅Π½ΠΈΠΉ (, Π·Π°Π²Π΅Ρ€ΡˆΠ°ΡŽΡ‰Π΅Π΅ дСсятичных Π·Π½Π°ΠΊΠΎΠ²) ΠŸΡ€Π΅ΠΎΠ±Ρ€Π°Π·ΠΎΠ²Π°Π½ΠΈΠ΅ Ρ€Π°Π·Π»ΠΈΡ‡Π½Ρ‹Ρ… Π΄Ρ€ΠΎΠ±Π΅ΠΉ, дСсятичных Π·Π½Π°ΠΊΠΎΠ², ΠΏΡ€ΠΎΡ†Π΅Π½Ρ‚ΠΎΠ² ΠΈ частСй Π² Ρ†Π΅Π»Ρ‹Ρ… ΡΠΎΠΎΡ‚Π½ΠΎΡˆΠ΅Π½ΠΈΠΉ (, Π·Π°Π²Π΅Ρ€ΡˆΠ°ΡŽΡ‰Π΅Π΅ дСсятичных Π·Π½Π°ΠΊΠΎΠ²) ΠŸΡ€Π΅ΠΎΠ±Ρ€Π°Π·ΠΎΠ²Π°Π½ΠΈΠ΅ Π΄Ρ€ΠΎΠ±Π΅ΠΉ Π² дСсятичныС Π΄Ρ€ΠΎΠ±ΠΈ, ΠΏΡ€ΠΎΡ†Π΅Π½Ρ‚Ρ‹ ΠΈ Π΄Ρ€ΠΎΠ±Π½Ρ‹Π΅ части Π§Π°ΡΡ‚ΡŒ ΡΠΎΠΎΡ‚Π½ΠΎΡˆΠ΅Π½ΠΈΠΉ ΠŸΡ€Π΅ΠΎΠ±Ρ€Π°Π·ΠΎΠ²Π°Π½ΠΈΠ΅ Π΄Ρ€ΠΎΠ±Π΅ΠΉ Π² дСсятичныС, ΠΏΡ€ΠΎΡ†Π΅Π½Ρ‚Ρ‹ ΠΈ частичныС Ρ†Π΅Π»Ρ‹Π΅ ΠΎΡ‚Π½ΠΎΡˆΠ΅Π½ΠΈΡ ΠŸΡ€Π΅ΠΎΠ±Ρ€Π°Π·ΠΎΠ²Π°Π½ΠΈΠ΅ дСсятичных Π΄Ρ€ΠΎΠ±Π΅ΠΉ Π² Π΄Ρ€ΠΎΠ±ΠΈ, ΠΏΡ€ΠΎΡ†Π΅Π½Ρ‚Ρ‹ ΠΈ частичныС ΠΊ Части ΠΎΡ‚Π½ΠΎΡˆΠ΅Π½ΠΈΡ ΠŸΡ€Π΅ΠΎΠ±Ρ€Π°Π·ΠΎΠ²Π°Π½ΠΈΠ΅ дСсятичных Π΄Ρ€ΠΎΠ±Π΅ΠΉ Π² Π΄Ρ€ΠΎΠ±ΠΈ, ΠΏΡ€ΠΎΡ†Π΅Π½Ρ‚Ρ‹ ΠΈ Π΄Ρ€ΠΎΠ±Π½Ρ‹Π΅ части Π¦Π΅Π»Ρ‹Π΅ ΠΎΡ‚Π½ΠΎΡˆΠ΅Π½ΠΈΡ ΠŸΡ€Π΅ΠΎΠ±Ρ€Π°Π·ΠΎΠ²Π°Π½ΠΈΠ΅ ΠΏΡ€ΠΎΡ†Π΅Π½Ρ‚ΠΎΠ² Π² Π΄Ρ€ΠΎΠ±ΠΈ, дСсятичныС Π΄Ρ€ΠΎΠ±ΠΈ ΠΈ части Π§Π°ΡΡ‚ΡŒ ΠΎΡ‚Π½ΠΎΡˆΠ΅Π½ΠΈΠΉ ΠŸΡ€Π΅ΠΎΠ±Ρ€Π°Π·ΠΎΠ²Π°Π½ΠΈΠ΅ ΠΏΡ€ΠΎΡ†Π΅Π½Ρ‚ΠΎΠ² Π² Π΄Ρ€ΠΎΠ±ΠΈ, дСсятичныС Π΄Ρ€ΠΎΠ±ΠΈ ΠΈ Π΄Ρ€ΠΎΠ±ΠΈ Π² Ρ†Π΅Π»Ρ‹Π΅ ΠΎΡ‚Π½ΠΎΡˆΠ΅Π½ΠΈΡ ΠŸΡ€Π΅ΠΎΠ±Ρ€Π°Π·ΠΎΠ²Π°Π½ΠΈΠ΅ частичных ΡΠΎΠΎΡ‚Π½ΠΎΡˆΠ΅Π½ΠΈΠΉ Π² Π΄Ρ€ΠΎΠ±ΠΈ, дСсятичныС Π΄Ρ€ΠΎΠ±ΠΈ ΠΈ ΠΏΡ€ΠΎΡ†Π΅Π½Ρ‚Ρ‹ ΠŸΡ€Π΅ΠΎΠ±Ρ€Π°Π·ΠΎΠ²Π°Π½ΠΈΠ΅ Ρ†Π΅Π»ΠΎΠ³ΠΎ числа Π² Π΄Ρ€ΠΎΠ±ΠΈ, дСсятичныС Π΄Ρ€ΠΎΠ±ΠΈ ΠΈ ΠΏΡ€ΠΎΡ†Π΅Π½Ρ‚Ρ‹ ΠŸΡ€Π΅ΠΎΠ±Ρ€Π°Π·ΠΎΠ²Π°Π½ΠΈΠ΅ Ρ€Π°Π·Π»ΠΈΡ‡Π½Ρ‹Ρ… Π΄Ρ€ΠΎΠ±Π΅ΠΉ, дСсятичных Π·Π½Π°ΠΊΠΎΠ², ΠΏΡ€ΠΎΡ†Π΅Π½Ρ‚ΠΎΠ² ΠΈ Π΄Ρ€ΠΎΠ±Π΅ΠΉ Π² Части ΡΠΎΠΎΡ‚Π½ΠΎΡˆΠ΅Π½ΠΈΠΉ ΠŸΡ€Π΅ΠΎΠ±Ρ€Π°Π·ΠΎΠ²Π°Π½ΠΈΠ΅ Ρ€Π°Π·Π»ΠΈΡ‡Π½Ρ‹Ρ… Π΄Ρ€ΠΎΠ±Π΅ΠΉ, дСсятичных Π·Π½Π°ΠΊΠΎΠ², ΠΏΡ€ΠΎΡ†Π΅Π½Ρ‚ΠΎΠ² ΠΈ Π΄Ρ€ΠΎΠ±Π΅ΠΉ Π² Ρ†Π΅Π»Ρ‹Ρ… ΡΠΎΠΎΡ‚Π½ΠΎΡˆΠ΅Π½ΠΈΠΉ ΠŸΡ€Π΅ΠΎΠ±Ρ€Π°Π·ΠΎΠ²Π°Π½ΠΈΠ΅ Ρ€Π°Π·Π»ΠΈΡ‡Π½Ρ‹Ρ… Π΄Ρ€ΠΎΠ±Π΅ΠΉ, дСсятичных Π·Π½Π°ΠΊΠΎΠ², ΠΏΡ€ΠΎΡ†Π΅Π½Ρ‚ΠΎΠ² ΠΈ частСй Π² Части ΠΎΡ‚Π½ΠΎΡˆΠ΅Π½ΠΈΠΉ с 7-ΠΌΠΈ ΠΈ 11-ΠΌΠΈ ΠŸΡ€Π΅ΠΎΠ±Ρ€Π°Π·ΠΎΠ²Π°Π½ΠΈΠ΅ Ρ€Π°Π·Π»ΠΈΡ‡Π½Ρ‹Ρ… Π΄Ρ€ΠΎΠ±Π΅ΠΉ, дСсятичных Π·Π½Π°ΠΊΠΎΠ², ΠΏΡ€ΠΎΡ†Π΅Π½Ρ‚ΠΎΠ² ΠΈ частСй Π² Ρ†Π΅Π»Ρ‹Ρ… ΡΠΎΠΎΡ‚Π½ΠΎΡˆΠ΅Π½ΠΈΠΉ с 7 ΠΈ 11 (БВАРЫЙ) ΠŸΡ€Π΅ΠΎΠ±Ρ€Π°Π·ΠΎΠ²Π°Π½ΠΈΠ΅ Π΄Ρ€ΠΎΠ±Π΅ΠΉ, дСсятичных Π·Π½Π°ΠΊΠΎΠ², ΠΏΡ€ΠΎΡ†Π΅Π½Ρ‚ΠΎΠ² ΠΈ ΡΠΎΠΎΡ‚Π½ΠΎΡˆΠ΅Π½ΠΈΠΉ

Π Π°Π±ΠΎΡ‡ΠΈΠ΅ листы ΠΎΠΏΠ΅Ρ€Π°Ρ†ΠΈΠΉ с дробями

Π£ΠΌΠ½ΠΎΠΆΠ΅Π½ΠΈΠ΅ Π΄Ρ€ΠΎΠ±Π΅ΠΉ

Π£ΠΌΠ½ΠΎΠΆΠ΅Π½ΠΈΠ΅ Π΄Ρ€ΠΎΠ±Π΅ΠΉ ΠΎΠ±Ρ‹Ρ‡Π½ΠΎ ΠΌΠ΅Π½Π΅Π΅ Π·Π°ΠΏΡƒΡ‚Π°Π½Π½ΠΎ с ΠΎΠΏΠ΅Ρ€Π°Ρ†ΠΈΠΎΠ½Π½ΠΎΠΉ Ρ‚ΠΎΡ‡ΠΊΠΈ зрСния, Ρ‡Π΅ΠΌ любая другая опСрация, ΠΈ ΠΌΠΎΠΆΠ΅Ρ‚ Π±Ρ‹Ρ‚ΡŒ ΠΌΠ΅Π½Π΅Π΅ Π·Π°ΠΏΡƒΡ‚Π°Π½Π½Ρ‹ΠΌ Π² ΠΊΠΎΠ½Ρ†Π΅ΠΏΡ‚ΡƒΠ°Π»ΡŒΠ½ΠΎΠΌ ΠΏΠ»Π°Π½Π΅ ΠΏΡ€ΠΈ ΠΏΡ€Π°Π²ΠΈΠ»ΡŒΠ½ΠΎΠΌ ΠΏΠΎΠ΄Ρ…ΠΎΠ΄Π΅. Алгоритм умноТСния β€” это просто ΡƒΠΌΠ½ΠΎΠΆΠ΅Π½ΠΈΠ΅ числитСлСй, Π° Π·Π°Ρ‚Π΅ΠΌ Π·Π½Π°ΠΌΠ΅Π½Π°Ρ‚Π΅Π»Π΅ΠΉ. Π’ΠΎΠ»ΡˆΠ΅Π±Π½ΠΎΠ΅ слово Π² ΠΏΠΎΠ½ΠΈΠΌΠ°Π½ΠΈΠΈ умноТСния Π΄Ρ€ΠΎΠ±Π΅ΠΉ β€” Β«ΠΈΠ·Β». НапримСр, Ρ‡Ρ‚ΠΎ составляСт Π΄Π²Π΅ Ρ‚Ρ€Π΅Ρ‚ΠΈ ОВ ΡˆΠ΅ΡΡ‚ΠΈ? Π§Ρ‚ΠΎ Ρ‚Π°ΠΊΠΎΠ΅ Ρ‚Ρ€Π΅Ρ‚ΡŒ ΠΎΡ‚ ΠΏΠΎΠ»ΠΎΠ²ΠΈΠ½Ρ‹? Когда Π²Ρ‹ ΠΈΡΠΏΠΎΠ»ΡŒΠ·ΡƒΠ΅Ρ‚Π΅ слово Β«ΠΈΠ·Β», становится Π½Π°ΠΌΠ½ΠΎΠ³ΠΎ Π»Π΅Π³Ρ‡Π΅ Π²ΠΈΠ·ΡƒΠ°Π»ΠΈΠ·ΠΈΡ€ΠΎΠ²Π°Ρ‚ΡŒ ΡƒΠΌΠ½ΠΎΠΆΠ΅Π½ΠΈΠ΅ Π΄Ρ€ΠΎΠ±Π΅ΠΉ. ΠŸΡ€ΠΈΠΌΠ΅Ρ€: Ρ€Π°Π·Ρ€Π΅Π·Π°Ρ‚ΡŒ Π±ΡƒΡ…Π°Π½ΠΊΡƒ Ρ…Π»Π΅Π±Π° ΠΏΠΎΠΏΠΎΠ»Π°ΠΌ, Π·Π°Ρ‚Π΅ΠΌ ΠΏΠΎΠ»ΠΎΠ²ΠΈΠ½Ρƒ Ρ€Π°Π·Ρ€Π΅Π·Π°Ρ‚ΡŒ Π½Π° Ρ‚Ρ€ΠΈ части. Одна Ρ‚Ρ€Π΅Ρ‚ΡŒ ΠΏΠΎΠ»Π±ΡƒΡ…Π°Π½ΠΊΠΈ Ρ…Π»Π΅Π±Π° такая ΠΆΠ΅, ΠΊΠ°ΠΊ 1/3 x 1/2, ΠΈ вкусная с маслом.

На Π΄Ρ€ΠΎΠ±ΠΈ

По ΠΈΠ΄Π΅Π΅, Π΄Π΅Π»Π΅Π½ΠΈΠ΅ Π½Π° Π΄Ρ€ΠΎΠ±ΠΈ, вСроятно, самая слоТная ΠΈΠ· всСх ΠΎΠΏΠ΅Ρ€Π°Ρ†ΠΈΠΉ, Π½ΠΎ ΠΌΡ‹ собираСмся ΠΏΠΎΠΌΠΎΡ‡ΡŒ Π²Π°ΠΌ.Алгоритм дСлСния Π΄Ρ€ΠΎΠ±Π΅ΠΉ Π°Π½Π°Π»ΠΎΠ³ΠΈΡ‡Π΅Π½ ΡƒΠΌΠ½ΠΎΠΆΠ΅Π½ΠΈΡŽ Π΄Ρ€ΠΎΠ±Π΅ΠΉ, Π½ΠΎ Π²Ρ‹ Π½Π°Ρ…ΠΎΠ΄ΠΈΡ‚Π΅ ΠΎΠ±Ρ€Π°Ρ‚Π½ΠΎΠ΅ Π·Π½Π°Ρ‡Π΅Π½ΠΈΠ΅ Π²Ρ‚ΠΎΡ€ΠΎΠΉ Π΄Ρ€ΠΎΠ±ΠΈ ΠΈΠ»ΠΈ ΠΏΡ€ΠΎΠΈΠ·Π²ΠΎΠ΄ΠΈΡ‚Π΅ пСрСкрСстноС ΡƒΠΌΠ½ΠΎΠΆΠ΅Π½ΠΈΠ΅. Π­Ρ‚ΠΎ даст Π²Π°ΠΌ ΠΏΡ€Π°Π²ΠΈΠ»ΡŒΠ½Ρ‹ΠΉ ΠΎΡ‚Π²Π΅Ρ‚, Ρ‡Ρ‚ΠΎ Ρ‡Ρ€Π΅Π·Π²Ρ‹Ρ‡Π°ΠΉΠ½ΠΎ Π²Π°ΠΆΠ½ΠΎ, особСнно Ссли Π²Ρ‹ строитС мост. ΠœΡ‹ рассказали Π²Π°ΠΌ, ΠΊΠ°ΠΊ ΠΊΠΎΠ½Ρ†Π΅ΠΏΡ‚ΡƒΠ°Π»ΠΈΠ·ΠΈΡ€ΠΎΠ²Π°Ρ‚ΡŒ ΡƒΠΌΠ½ΠΎΠΆΠ΅Π½ΠΈΠ΅ Π΄Ρ€ΠΎΠ±Π΅ΠΉ, Π½ΠΎ ΠΊΠ°ΠΊ ΠΎΠ½ΠΎ Ρ€Π°Π±ΠΎΡ‚Π°Π΅Ρ‚ с Π΄Π΅Π»Π΅Π½ΠΈΠ΅ΠΌ? Π›Π΅Π³ΠΊΠΈΠΉ! Π’Π°ΠΌ просто Π½ΡƒΠΆΠ½ΠΎ Π²Ρ‹ΡƒΡ‡ΠΈΡ‚ΡŒ Π²ΠΎΠ»ΡˆΠ΅Π±Π½ΡƒΡŽ Ρ„Ρ€Π°Π·Ρƒ: «Бколько ____ Π² ______? НапримСр, Π² вопросС 6 & div; 1/2 Π²Ρ‹ спроситС:Β« Бколько ΠΏΠΎΠ»ΠΎΠ²ΠΈΠ½ΠΎΠΊ Π² 6? »Бтановится Π½Π΅ΠΌΠ½ΠΎΠ³ΠΎ большС. Ρ‚Ρ€ΡƒΠ΄Π½ΠΎ, ΠΊΠΎΠ³Π΄Π° ΠΎΠ±Π° числа ΡΠ²Π»ΡΡŽΡ‚ΡΡ Π΄Ρ€ΠΎΠ±Π½Ρ‹ΠΌΠΈ, Π½ΠΎ это Π½Π΅ гигантский скачок, Ρ‡Ρ‚ΠΎΠ±Ρ‹ ΠΏΠΎΠ½ΡΡ‚ΡŒ это.1/2 & div; 1/4 β€” довольно простой ΠΏΡ€ΠΈΠΌΠ΅Ρ€, особСнно Ссли Π²Ρ‹ Π΄ΡƒΠΌΠ°Π΅Ρ‚Π΅ ΠΎ ΠΌΠΎΠ½Π΅Ρ‚Π°Ρ… БША ΠΈΠ»ΠΈ ΠšΠ°Π½Π°Π΄Ρ‹. Бколько Ρ‡Π΅Ρ‚Π²Π΅Ρ€Ρ‚Π΅ΠΉ Π² ΠΏΠΎΠ»Π΄ΠΎΠ»Π»Π°Ρ€Π΅?

Π‘Π»ΠΎΠΆΠ΅Π½ΠΈΠ΅ Π΄Ρ€ΠΎΠ±Π΅ΠΉ

Для слоТСния Π΄Ρ€ΠΎΠ±Π΅ΠΉ Π½ΡƒΠΆΠ΅Π½ Π½Π°Π΄ΠΎΠ΅Π΄Π»ΠΈΠ²Ρ‹ΠΉ ΠΎΠ±Ρ‰ΠΈΠΉ Π·Π½Π°ΠΌΠ΅Π½Π°Ρ‚Π΅Π»ΡŒ. УпроститС Тизнь своих ΡƒΡ‡Π΅Π½ΠΈΠΊΠΎΠ², сначала ΠΎΠ±ΡƒΡ‡ΠΈΠ² ΠΈΡ… понятиям эквивалСнтных Π΄Ρ€ΠΎΠ±Π΅ΠΉ ΠΈ Π½Π°ΠΈΠΌΠ΅Π½ΡŒΡˆΠΈΡ… ΠΎΠ±Ρ‰ΠΈΡ… ΠΊΡ€Π°Ρ‚Π½Ρ‹Ρ…. Как Ρ‚ΠΎΠ»ΡŒΠΊΠΎ учащиСся Π·Π½Π°ΠΊΠΎΠΌΡ‹ с этими двумя концСпциями, идСя поиска Π΄Ρ€ΠΎΠ±Π΅ΠΉ с ΠΎΠ±Ρ‰ΠΈΠΌΠΈ знамСнатСлями для слоТСния становится Π½Π°ΠΌΠ½ΠΎΠ³ΠΎ ΠΏΡ€ΠΎΡ‰Π΅.ВрСмя, ΠΏΠΎΡ‚Ρ€Π°Ρ‡Π΅Π½Π½ΠΎΠ΅ Π½Π° ΠΌΠΎΠ΄Π΅Π»ΠΈΡ€ΠΎΠ²Π°Π½ΠΈΠ΅ Π΄Ρ€ΠΎΠ±Π΅ΠΉ, Ρ‚Π°ΠΊΠΆΠ΅ ΠΏΠΎΠΌΠΎΠΆΠ΅Ρ‚ учащимся ΠΏΠΎΠ½ΡΡ‚ΡŒ слоТСниС Π΄Ρ€ΠΎΠ±Π΅ΠΉ. Бвязь Π΄Ρ€ΠΎΠ±Π΅ΠΉ с Π·Π½Π°ΠΊΠΎΠΌΡ‹ΠΌΠΈ ΠΏΡ€ΠΈΠΌΠ΅Ρ€Π°ΠΌΠΈ, бСзусловно, ΠΏΠΎΠΌΠΎΠΆΠ΅Ρ‚. НапримСр, Ссли Π²Ρ‹ Π΄ΠΎΠ±Π°Π²ΠΈΡ‚Π΅ 1/2 Π±Π°Π½Π°Π½Π° ΠΈ 1/2 Π±Π°Π½Π°Π½Π°, Π²Ρ‹ ΠΏΠΎΠ»ΡƒΡ‡ΠΈΡ‚Π΅ Ρ†Π΅Π»Ρ‹ΠΉ Π±Π°Π½Π°Π½. Π§Ρ‚ΠΎ ΠΏΡ€ΠΎΠΈΠ·ΠΎΠΉΠ΄Π΅Ρ‚, Ссли Π²Ρ‹ Π΄ΠΎΠ±Π°Π²ΠΈΡ‚Π΅ 1/2 Π±Π°Π½Π°Π½Π° ΠΈ 3/4 Π΄Ρ€ΡƒΠ³ΠΎΠ³ΠΎ Π±Π°Π½Π°Π½Π°?

Π”ΠΎΠ±Π°Π²Π»Π΅Π½ΠΈΠ΅ ΡΠΌΠ΅ΡˆΠ°Π½Π½Ρ‹Ρ… Ρ„Ρ€Π°ΠΊΡ†ΠΈΠΉ Π Π°Π±ΠΎΡ‡ΠΈΠ΅ листы

ΠžΠ±Ρ‹Ρ‡Π½Π°Ρ стратСгия, ΠΊΠΎΡ‚ΠΎΡ€ΡƒΡŽ ΠΈΡΠΏΠΎΠ»ΡŒΠ·ΡƒΡŽΡ‚ ΠΏΡ€ΠΈ Π΄ΠΎΠ±Π°Π²Π»Π΅Π½ΠΈΠΈ ΡΠΌΠ΅ΡˆΠ°Π½Π½Ρ‹Ρ… Ρ„Ρ€Π°ΠΊΡ†ΠΈΠΉ, состоит Π² Ρ‚ΠΎΠΌ, Ρ‡Ρ‚ΠΎΠ±Ρ‹ ΠΏΡ€Π΅ΠΎΠ±Ρ€Π°Π·ΠΎΠ²Π°Ρ‚ΡŒ ΡΠΌΠ΅ΡˆΠ°Π½Π½Ρ‹Π΅ Ρ„Ρ€Π°ΠΊΡ†ΠΈΠΈ Π² Π½Π΅ΠΏΡ€Π°Π²ΠΈΠ»ΡŒΠ½Ρ‹Π΅ Ρ„Ρ€Π°ΠΊΡ†ΠΈΠΈ, Π·Π°Π²Π΅Ρ€ΡˆΠΈΡ‚ΡŒ слоТСниС, Π° Π·Π°Ρ‚Π΅ΠΌ ΠΏΠ΅Ρ€Π΅ΠΊΠ»ΡŽΡ‡ΠΈΡ‚ΡŒΡΡ ΠΎΠ±Ρ€Π°Ρ‚Π½ΠΎ. Другая стратСгия, Ρ‚Ρ€Π΅Π±ΡƒΡŽΡ‰Π°Ρ Π½Π΅ΠΌΠ½ΠΎΠ³ΠΎ ΠΌΠ΅Π½ΡŒΡˆΠΈΡ… умствСнных способностСй, β€” Ρ€Π°ΡΡΠΌΠ°Ρ‚Ρ€ΠΈΠ²Π°Ρ‚ΡŒ Ρ†Π΅Π»Ρ‹Π΅ числа ΠΈ Π΄Ρ€ΠΎΠ±ΠΈ ΠΏΠΎ ΠΎΡ‚Π΄Π΅Π»ΡŒΠ½ΠΎΡΡ‚ΠΈ. Π‘Π½Π°Ρ‡Π°Π»Π° слоТитС Ρ†Π΅Π»Ρ‹Π΅ числа. Π”ΠΎΠ±Π°Π²ΡŒΡ‚Π΅ Π΄ΠΎΠ»ΠΈ сСкунды. Если получСнная Π΄Ρ€ΠΎΠ±ΡŒ Π½Π΅ΠΏΡ€Π°Π²ΠΈΠ»ΡŒΠ½Π°Ρ, Π΅Π΅ Π½ΡƒΠΆΠ½ΠΎ ΠΏΡ€Π΅ΠΎΠ±Ρ€Π°Π·ΠΎΠ²Π°Ρ‚ΡŒ Π² смСшанноС число. Π§Π°ΡΡ‚ΡŒ Ρ†Π΅Π»ΠΎΠ³ΠΎ числа ΠΌΠΎΠΆΠ΅Ρ‚ Π±Ρ‹Ρ‚ΡŒ Π΄ΠΎΠ±Π°Π²Π»Π΅Π½Π° ​​к исходной части Ρ†Π΅Π»ΠΎΠ³ΠΎ числа.

Π’Ρ‹Ρ‡ΠΈΡ‚Π°Π½ΠΈΠ΅ Π”Ρ€ΠΎΠ±Π΅ΠΉ

НСт большой Ρ€Π°Π·Π½ΠΈΡ†Ρ‹ ΠΌΠ΅ΠΆΠ΄Ρƒ слоТСниСм ΠΈ Π²Ρ‹Ρ‡ΠΈΡ‚Π°Π½ΠΈΠ΅ΠΌ Π΄Ρ€ΠΎΠ±Π΅ΠΉ.Оба Ρ‚Ρ€Π΅Π±ΡƒΡŽΡ‚ ΠΎΠ±Ρ‰Π΅Π³ΠΎ знамСнатСля, Ρ‡Ρ‚ΠΎ Ρ‚Ρ€Π΅Π±ΡƒΠ΅Ρ‚ Π½Π΅ΠΊΠΎΡ‚ΠΎΡ€Ρ‹Ρ… ΠΏΡ€Π΅Π΄Π²Π°Ρ€ΠΈΡ‚Π΅Π»ΡŒΠ½Ρ‹Ρ… Π·Π½Π°Π½ΠΈΠΉ. ЕдинствСнная Ρ€Π°Π·Π½ΠΈΡ†Π° Π² Ρ‚ΠΎΠΌ, Ρ‡Ρ‚ΠΎ Π²Ρ‚ΠΎΡ€ΠΎΠΉ ΠΈ ΠΏΠΎΡΠ»Π΅Π΄ΡƒΡŽΡ‰ΠΈΠ΅ числитСли Π²Ρ‹Ρ‡ΠΈΡ‚Π°ΡŽΡ‚ΡΡ ΠΈΠ· ΠΏΠ΅Ρ€Π²ΠΎΠ³ΠΎ. БущСствуСт ΠΎΠΏΠ°ΡΠ½ΠΎΡΡ‚ΡŒ Ρ‚ΠΎΠ³ΠΎ, Ρ‡Ρ‚ΠΎ Π²Ρ‹ ΠΌΠΎΠΆΠ΅Ρ‚Π΅ ΠΏΠΎΠ»ΡƒΡ‡ΠΈΡ‚ΡŒ ΠΎΡ‚Ρ€ΠΈΡ†Π°Ρ‚Π΅Π»ΡŒΠ½ΠΎΠ΅ число ΠΏΡ€ΠΈ Π²Ρ‹Ρ‡ΠΈΡ‚Π°Π½ΠΈΠΈ Π΄Ρ€ΠΎΠ±Π΅ΠΉ, поэтому учащимся, Π²ΠΎΠ·ΠΌΠΎΠΆΠ½ΠΎ, потрСбуСтся ΡƒΠ·Π½Π°Ρ‚ΡŒ, Ρ‡Ρ‚ΠΎ это ΠΎΠ·Π½Π°Ρ‡Π°Π΅Ρ‚ Π² этом случаС. Когда Π΄Π΅Π»ΠΎ Π΄ΠΎΡ…ΠΎΠ΄ΠΈΡ‚ Π΄ΠΎ любого понятия Π² дробях, всСгда ΠΏΠΎΠ»Π΅Π·Π½ΠΎ ΡΠ²ΡΠ·Π°Ρ‚ΡŒ Π΅Π³ΠΎ с Π·Π½Π°ΠΊΠΎΠΌΠΎΠΉ ΠΈΠ»ΠΈ простой для понимания ситуациСй. НапримСр, 7/8 β€” 3/4 = 1/8 ΠΌΠΎΠΆΠ΅Ρ‚ ΠΈΠΌΠ΅Ρ‚ΡŒ Π·Π½Π°Ρ‡Π΅Π½ΠΈΠ΅ Π² контСкстС Π³ΠΎΠ½ΠΊΠΈ.ΠŸΠ΅Ρ€Π²Ρ‹ΠΉ Π±Π΅Π³ΡƒΠ½ шСл Π½Π° 7/8 ΠΏΠΎ трассС, Π° Π²Ρ‚ΠΎΡ€ΠΎΠΉ Π±Π΅Π³ΡƒΠ½ Π½Π° 3/4 ΠΏΠΎ трассС. Как Π΄Π°Π»Π΅ΠΊΠΎ Π²ΠΏΠ΅Ρ€Π΅Π΄ΠΈ Π±Ρ‹Π» ΠΏΠ΅Ρ€Π²Ρ‹ΠΉ Π±Π΅Π³ΡƒΠ½? (1/8 Ρ‚Ρ€Π΅ΠΊΠ°).

Π’Ρ‹Ρ‡ΠΈΡ‚Π°Π½ΠΈΠ΅ ΡΠΌΠ΅ΡˆΠ°Π½Π½Ρ‹Ρ… Π΄Ρ€ΠΎΠ±Π΅ΠΉ Π Π°Π±ΠΎΡ‡ΠΈΠ΅ листы

Π Π°Π±ΠΎΡ‡ΠΈΠ΅ листы с дробями Ρ€Π°Π·Π»ΠΈΡ‡Π½Ρ‹Ρ… ΠΎΠΏΠ΅Ρ€Π°Ρ†ΠΈΠΉ

БмСшиваниС Π·Π½Π°ΠΊΠΎΠ² ΠΎΠΏΠ΅Ρ€Π°Ρ†ΠΈΠΉ с Ρ€Π°Π±ΠΎΡ‡ΠΈΠΌΠΈ листами с дробями заставляСт учащихся ΡƒΠ΄Π΅Π»ΡΡ‚ΡŒ большС внимания Ρ‚ΠΎΠΌΡƒ, Ρ‡Ρ‚ΠΎ ΠΎΠ½ΠΈ Π΄Π΅Π»Π°ΡŽΡ‚, ΠΈ позволяСт Ρ…ΠΎΡ€ΠΎΡˆΠΎ ΠΏΡ€ΠΎΠ²Π΅Ρ€ΠΈΡ‚ΡŒ свои Π½Π°Π²Ρ‹ΠΊΠΈ Π² Π±ΠΎΠ»Π΅Π΅ Ρ‡Π΅ΠΌ ΠΎΠ΄Π½ΠΎΠΉ ΠΎΠΏΠ΅Ρ€Π°Ρ†ΠΈΠΈ.

Π‘Π»ΠΎΠΆΠ΅Π½ΠΈΠ΅ ΠΈ Π²Ρ‹Ρ‡ΠΈΡ‚Π°Π½ΠΈΠ΅ Π΄Ρ€ΠΎΠ±Π΅ΠΉ

Π£ΠΌΠ½ΠΎΠΆΠ΅Π½ΠΈΠ΅ ΠΈ Π΄Π΅Π»Π΅Π½ΠΈΠ΅ Π΄Ρ€ΠΎΠ±Π΅ΠΉ

Π‘ΠΌΠ΅ΡˆΠ°Π½Π½Ρ‹Π΅ ΠΎΠΏΠ΅Ρ€Π°Ρ†ΠΈΠΈ с дробями

ΠžΠΏΠ΅Ρ€Π°Ρ†ΠΈΠΈ с

ΠžΡ‚Ρ€ΠΈΡ†Π°Ρ‚Π΅Π»ΡŒΠ½Ρ‹ΠΌΠΈ дробями

Π₯отя Π½Π΅ΠΊΠΎΡ‚ΠΎΡ€Ρ‹Π΅ ΠΈΠ· этих листов ΠΏΡ€Π΅Π΄ΡΡ‚Π°Π²Π»ΡΡŽΡ‚ собой ΠΎΡ‚Π΄Π΅Π»ΡŒΠ½Ρ‹Π΅ ΠΎΠΏΠ΅Ρ€Π°Ρ†ΠΈΠΈ, Π±Ρ‹Π»ΠΎ Π±Ρ‹ ΠΏΠΎΠ»Π΅Π·Π½ΠΎ ΠΈΠΌΠ΅Ρ‚ΡŒ всС ΠΎΠ½ΠΈ Π² ΠΎΠ΄Π½ΠΎΠΌ мСстС.ΠŸΡ€ΠΈ Π²Ρ‹ΠΏΠΎΠ»Π½Π΅Π½ΠΈΠΈ ΠΎΠΏΠ΅Ρ€Π°Ρ†ΠΈΠΉ с ΠΎΡ‚Ρ€ΠΈΡ†Π°Ρ‚Π΅Π»ΡŒΠ½Ρ‹ΠΌΠΈ дробями слСдуСт ΡƒΡ‡ΠΈΡ‚Ρ‹Π²Π°Ρ‚ΡŒ Π½Π΅ΠΊΠΎΡ‚ΠΎΡ€Ρ‹Π΅ особСнности. ΠžΠ±Ρ‹Ρ‡Π½ΠΎ ΠΎΡ‡Π΅Π½ΡŒ ΠΏΠΎΠ»Π΅Π·Π½ΠΎ ΠΏΠ΅Ρ€Π΅Π΄ Ρ‚Π΅ΠΌ, ΠΊΠ°ΠΊ ΠΏΡ€ΠΎΠ΄ΠΎΠ»ΠΆΠΈΡ‚ΡŒ, Π·Π°ΠΌΠ΅Π½ΠΈΡ‚ΡŒ Π»ΡŽΠ±Ρ‹Π΅ ΡΠΌΠ΅ΡˆΠ°Π½Π½Ρ‹Π΅ числа Π½Π° Π½Π΅ΠΏΡ€Π°Π²ΠΈΠ»ΡŒΠ½ΡƒΡŽ Π΄Ρ€ΠΎΠ±ΡŒ. Π’Π°ΠΆΠ½ΠΎ ΠΎΠ±Ρ€Π°Ρ‰Π°Ρ‚ΡŒ Π²Π½ΠΈΠΌΠ°Π½ΠΈΠ΅ Π½Π° Π·Π½Π°ΠΊΠΈ ΠΈ Π·Π½Π°Ρ‚ΡŒ ΠΏΡ€Π°Π²ΠΈΠ»Π° умноТСния ΠΏΠΎΠ»ΠΎΠΆΠΈΡ‚Π΅Π»ΡŒΠ½Ρ‹Ρ… ΠΈ ΠΎΡ‚Ρ€ΠΈΡ†Π°Ρ‚Π΅Π»ΡŒΠ½Ρ‹Ρ… (++ = +, + β€” = -, β€” + = β€” ΠΈ β€” = +).

ΠŸΠΎΡ€ΡΠ΄ΠΎΠΊ ΠΎΠΏΠ΅Ρ€Π°Ρ†ΠΈΠΉ с дробями Π Π°Π±ΠΎΡ‡ΠΈΠ΅ листы

ΠŸΠΎΡ€ΡΠ΄ΠΎΠΊ ΠΎΠΏΠ΅Ρ€Π°Ρ†ΠΈΠΉ с листами Π΄Ρ€ΠΎΠ±Π΅ΠΉ с опциями ΠΊΠ°ΠΊ ΠΏΠΎΠ»ΠΎΠΆΠΈΡ‚Π΅Π»ΡŒΠ½Ρ‹Ρ…, Ρ‚Π°ΠΊ ΠΈ ΠΎΡ‚Ρ€ΠΈΡ†Π°Ρ‚Π΅Π»ΡŒΠ½Ρ‹Ρ… Π΄Ρ€ΠΎΠ±Π΅ΠΉ Ρ€Π°Π·Π»ΠΈΡ‡Π½ΠΎΠΉ слоТности.

ΠŸΠΎΡ€ΡΠ΄ΠΎΠΊ ΠΎΠΏΠ΅Ρ€Π°Ρ†ΠΈΠΉ с

дробями

Как ΠΈ Π΄Ρ€ΡƒΠ³ΠΈΠ΅ Ρ€Π°Π±ΠΎΡ‡ΠΈΠ΅ листы с порядком ΠΎΠΏΠ΅Ρ€Π°Ρ†ΠΈΠΉ, Ρ€Π°Π±ΠΎΡ‡ΠΈΠ΅ листы с порядком Π΄Ρ€ΠΎΠ±ΠΈ ΠΎΠΏΠ΅Ρ€Π°Ρ†ΠΈΠΉ Ρ‚Ρ€Π΅Π±ΡƒΡŽΡ‚ Π½Π΅ΠΊΠΎΡ‚ΠΎΡ€Ρ‹Ρ… ΠΏΡ€Π΅Π΄Π²Π°Ρ€ΠΈΡ‚Π΅Π»ΡŒΠ½Ρ‹Ρ… Π·Π½Π°Π½ΠΈΠΉ. Если ваши ΡƒΡ‡Π΅Π½ΠΈΠΊΠΈ Π±ΠΎΡ€ΡŽΡ‚ΡΡ с этими вопросами, вСроятно, это большС связано с ΠΈΡ… ΡΠΏΠΎΡΠΎΠ±Π½ΠΎΡΡ‚ΡŒΡŽ Ρ€Π°Π±ΠΎΡ‚Π°Ρ‚ΡŒ с дробями, Ρ‡Π΅ΠΌ с самими вопросами. Π’Π½ΠΈΠΌΠ°Ρ‚Π΅Π»ΡŒΠ½ΠΎ Π½Π°Π±Π»ΡŽΠ΄Π°ΠΉΡ‚Π΅ ΠΈ ΠΏΠΎΡΡ‚Π°Ρ€Π°ΠΉΡ‚Π΅ΡΡŒ Ρ‚ΠΎΡ‡Π½ΠΎ ΡƒΠΊΠ°Π·Π°Ρ‚ΡŒ, ΠΊΠ°ΠΊΠΈΡ… Π½Π΅ΠΎΠ±Ρ…ΠΎΠ΄ΠΈΠΌΡ‹Ρ… Π·Π½Π°Π½ΠΈΠΉ Π½Π΅ Ρ…Π²Π°Ρ‚Π°Π΅Ρ‚, Π° Π·Π°Ρ‚Π΅ΠΌ ΠΏΠΎΡ‚Ρ€Π°Ρ‚ΡŒΡ‚Π΅ Π½Π΅ΠΊΠΎΡ‚ΠΎΡ€ΠΎΠ΅ врСмя Π½Π° ΠΈΠ·ΡƒΡ‡Π΅Π½ΠΈΠ΅ этих ΠΊΠΎΠ½Ρ†Π΅ΠΏΡ†ΠΈΠΉ / Π½Π°Π²Ρ‹ΠΊΠΎΠ², ΠΏΡ€Π΅ΠΆΠ΄Π΅ Ρ‡Π΅ΠΌ ΠΏΡ€ΠΎΠ΄ΠΎΠ»ΠΆΠΈΡ‚ΡŒ.Π’ ΠΏΡ€ΠΎΡ‚ΠΈΠ²Π½ΠΎΠΌ случаС ΠΏΡ€ΠΈΠ²Π΅Π΄Π΅Π½Π½Ρ‹Π΅ Π½ΠΈΠΆΠ΅ Ρ€Π°Π±ΠΎΡ‡ΠΈΠ΅ листы Π΄ΠΎΠ»ΠΆΠ½Ρ‹ ΡΠΎΠ΄Π΅Ρ€ΠΆΠ°Ρ‚ΡŒ довольно простыС ΠΎΡ‚Π²Π΅Ρ‚Ρ‹ ΠΈ Π½Π΅ Π΄ΠΎΠ»ΠΆΠ½Ρ‹ ΠΏΡ€ΠΈΠ²ΠΎΠ΄ΠΈΡ‚ΡŒ ΠΊ Ρ‡Ρ€Π΅Π·ΠΌΠ΅Ρ€Π½ΠΎΠΌΡƒ Π²Ρ‹ΠΏΠ°Π΄Π΅Π½ΠΈΡŽ волос.

ΠŸΠΎΡ€ΡΠ΄ΠΎΠΊ ΠΎΠΏΠ΅Ρ€Π°Ρ†ΠΈΠΉ с

дСсятичными ΠΈ ΡΠΌΠ΅ΡˆΠ°Π½Π½Ρ‹ΠΌΠΈ дробями .

Π£ΠΌΠ½ΠΎΠΆΠ΅Π½ΠΈΠ΅ Π΄Ρ€ΠΎΠ±Π΅ΠΉ с Ρ€Π°Π·Π½Ρ‹ΠΌΠΈ Π·Π½Π°ΠΊΠ°ΠΌΠΈ. Π£ΠΌΠ½ΠΎΠΆΠ΅Π½ΠΈΠ΅ ΠΈ Π΄Π΅Π»Π΅Π½ΠΈΠ΅ ΠΎΡ‚Ρ€ΠΈΡ†Π°Ρ‚Π΅Π»ΡŒΠ½Ρ‹Ρ… чисСл

Π’ Π΄Π°Π½Π½ΠΎΠΌ ΡƒΡ€ΠΎΠΊΠ΅ рассматриваСтся ΡƒΠΌΠ½ΠΎΠΆΠ΅Π½ΠΈΠ΅ ΠΈ Π΄Π΅Π»Π΅Π½ΠΈΠ΅ Ρ€Π°Ρ†ΠΈΠΎΠ½Π°Π»ΡŒΠ½Ρ‹Ρ… чисСл.

Π‘ΠΎΠ΄Π΅Ρ€ΠΆΠ°Π½ΠΈΠ΅ ΡƒΡ€ΠΎΠΊΠ°

Π£ΠΌΠ½ΠΎΠΆΠ΅Π½ΠΈΠ΅ Ρ€Π°Ρ†ΠΈΠΎΠ½Π°Π»ΡŒΠ½Ρ‹Ρ… чисСл

ΠŸΡ€Π°Π²ΠΈΠ»Π° умноТСния Ρ†Π΅Π»Ρ‹Ρ… чисСл справСдливы ΠΈ для Ρ€Π°Ρ†ΠΈΠΎΠ½Π°Π»ΡŒΠ½Ρ‹Ρ… чисСл. Π˜Π½Ρ‹ΠΌΠΈ словами, Ρ‡Ρ‚ΠΎΠ±Ρ‹ ΡƒΠΌΠ½ΠΎΠΆΠ°Ρ‚ΡŒ Ρ€Π°Ρ†ΠΈΠΎΠ½Π°Π»ΡŒΠ½Ρ‹Π΅ числа, Π½ΡƒΠΆΠ½ΠΎ ΡƒΠΌΠ΅Ρ‚ΡŒ

Π’Π°ΠΊΠΆΠ΅, Π½Π΅ΠΎΠ±Ρ…ΠΎΠ΄ΠΈΠΌΠΎ Π·Π½Π°Ρ‚ΡŒ основныС Π·Π°ΠΊΠΎΠ½Ρ‹ умноТСния, Ρ‚Π°ΠΊΠΈΠ΅ ΠΊΠ°ΠΊ: ΠΏΠ΅Ρ€Π΅ΠΌΠ΅ΡΡ‚ΠΈΡ‚Π΅Π»ΡŒΠ½Ρ‹ΠΉ Π·Π°ΠΊΠΎΠ½ умноТСния, ΡΠΎΡ‡Π΅Ρ‚Π°Ρ‚Π΅Π»ΡŒΠ½Ρ‹ΠΉ Π·Π°ΠΊΠΎΠ½ умноТСния, Ρ€Π°ΡΠΏΡ€Π΅Π΄Π΅Π»ΠΈΡ‚Π΅Π»ΡŒΠ½Ρ‹ΠΉ Π·Π°ΠΊΠΎΠ½ умноТСния ΠΈ ΡƒΠΌΠ½ΠΎΠΆΠ΅Π½ΠΈΠ΅ Π½Π° ноль.

ΠŸΡ€ΠΈΠΌΠ΅Ρ€ 1. Найти Π·Π½Π°Ρ‡Π΅Π½ΠΈΠ΅ выраТСния

Π­Ρ‚ΠΎ ΡƒΠΌΠ½ΠΎΠΆΠ΅Π½ΠΈΠ΅ Ρ€Π°Ρ†ΠΈΠΎΠ½Π°Π»ΡŒΠ½Ρ‹Ρ… чисСл с Ρ€Π°Π·Π½Ρ‹ΠΌΠΈ Π·Π½Π°ΠΊΠ°ΠΌΠΈ. Π§Ρ‚ΠΎΠ±Ρ‹ ΠΏΠ΅Ρ€Π΅ΠΌΠ½ΠΎΠΆΠΈΡ‚ΡŒ Ρ€Π°Ρ†ΠΈΠΎΠ½Π°Π»ΡŒΠ½Ρ‹Π΅ числа с Ρ€Π°Π·Π½Ρ‹ΠΌΠΈ Π·Π½Π°ΠΊΠ°ΠΌΠΈ, Π½ΡƒΠΆΠ½ΠΎ ΠΏΠ΅Ρ€Π΅ΠΌΠ½ΠΎΠΆΠΈΡ‚ΡŒ ΠΈΡ… ΠΌΠΎΠ΄ΡƒΠ»ΠΈ ΠΈ ΠΏΠ΅Ρ€Π΅Π΄ ΠΏΠΎΠ»ΡƒΡ‡Π΅Π½Π½Ρ‹ΠΌ ΠΎΡ‚Π²Π΅Ρ‚ΠΎΠΌ ΠΏΠΎΡΡ‚Π°Π²ΠΈΡ‚ΡŒ минус.

Π§Ρ‚ΠΎΠ±Ρ‹ Ρ…ΠΎΡ€ΠΎΡˆΠΎ ΡƒΠ²ΠΈΠ΄Π΅Ρ‚ΡŒ, Ρ‡Ρ‚ΠΎ ΠΌΡ‹ ΠΈΠΌΠ΅Π΅ΠΌ Π΄Π΅Π»ΠΎ с числами, Ρƒ ΠΊΠΎΡ‚ΠΎΡ€Ρ‹Ρ… Ρ€Π°Π·Π½Ρ‹Π΅ Π·Π½Π°ΠΊΠΈ, Π·Π°ΠΊΠ»ΡŽΡ‡ΠΈΠΌ ΠΊΠ°ΠΆΠ΄ΠΎΠ΅ Ρ€Π°Ρ†ΠΈΠΎΠ½Π°Π»ΡŒΠ½ΠΎΠ΅ число Π² скобки вмСстС со своими Π·Π½Π°ΠΊΠ°ΠΌΠΈ

ΠœΠΎΠ΄ΡƒΠ»ΡŒ числа Ρ€Π°Π²Π΅Π½ , Π° ΠΌΠΎΠ΄ΡƒΠ»ΡŒ числа Ρ€Π°Π²Π΅Π½ . ΠŸΠ΅Ρ€Π΅ΠΌΠ½ΠΎΠΆΠΈΠ² ΠΏΠΎΠ»ΡƒΡ‡Π΅Π½Π½Ρ‹Π΅ ΠΌΠΎΠ΄ΡƒΠ»ΠΈ, ΠΊΠ°ΠΊ ΠΏΠΎΠ»ΠΎΠΆΠΈΡ‚Π΅Π»ΡŒΠ½Ρ‹Π΅ Π΄Ρ€ΠΎΠ±ΠΈ, ΠΌΡ‹ ΠΏΠΎΠ»ΡƒΡ‡ΠΈΠ»ΠΈ ΠΎΡ‚Π²Π΅Ρ‚ , Π½ΠΎ ΠΏΠ΅Ρ€Π΅Π΄ ΠΎΡ‚Π²Π΅Ρ‚ΠΎΠΌ поставили минус, ΠΊΠ°ΠΊ ΠΎΡ‚ нас Ρ‚Ρ€Π΅Π±ΠΎΠ²Π°Π»ΠΎ ΠΏΡ€Π°Π²ΠΈΠ»ΠΎ. Π§Ρ‚ΠΎΠ±Ρ‹ ΠΎΠ±Π΅ΡΠΏΠ΅Ρ‡ΠΈΡ‚ΡŒ ΠΏΠ΅Ρ€Π΅Π΄ ΠΎΡ‚Π²Π΅Ρ‚ΠΎΠΌ этот минус, ΡƒΠΌΠ½ΠΎΠΆΠ΅Π½ΠΈΠ΅ ΠΌΠΎΠ΄ΡƒΠ»Π΅ΠΉ Π²Ρ‹ΠΏΠΎΠ»Π½ΡΠ»ΠΎΡΡŒ Π² скобках, ΠΏΠ΅Ρ€Π΅Π΄ ΠΊΠΎΡ‚ΠΎΡ€Ρ‹ΠΌΠΈ ΠΈ поставлСн минус.

ΠšΠΎΡ€ΠΎΡ‚ΠΊΠΎΠ΅ Ρ€Π΅ΡˆΠ΅Π½ΠΈΠ΅ выглядит ΡΠ»Π΅Π΄ΡƒΡŽΡ‰ΠΈΠΌ ΠΎΠ±Ρ€Π°Π·ΠΎΠΌ:

ΠŸΡ€ΠΈΠΌΠ΅Ρ€ 2. Найти Π·Π½Π°Ρ‡Π΅Π½ΠΈΠ΅ выраТСния

ΠŸΡ€ΠΈΠΌΠ΅Ρ€ 3. Найти Π·Π½Π°Ρ‡Π΅Π½ΠΈΠ΅ выраТСния

Π­Ρ‚ΠΎ ΡƒΠΌΠ½ΠΎΠΆΠ΅Π½ΠΈΠ΅ ΠΎΡ‚Ρ€ΠΈΡ†Π°Ρ‚Π΅Π»ΡŒΠ½Ρ‹Ρ… Ρ€Π°Ρ†ΠΈΠΎΠ½Π°Π»ΡŒΠ½Ρ‹Ρ… чисСл. Π§Ρ‚ΠΎΠ±Ρ‹ ΠΏΠ΅Ρ€Π΅ΠΌΠ½ΠΎΠΆΠΈΡ‚ΡŒ ΠΎΡ‚Ρ€ΠΈΡ†Π°Ρ‚Π΅Π»ΡŒΠ½Ρ‹Π΅ Ρ€Π°Ρ†ΠΈΠΎΠ½Π°Π»ΡŒΠ½Ρ‹Π΅ числа, Π½ΡƒΠΆΠ½ΠΎ ΠΏΠ΅Ρ€Π΅ΠΌΠ½ΠΎΠΆΠΈΡ‚ΡŒ ΠΈΡ… ΠΌΠΎΠ΄ΡƒΠ»ΠΈ ΠΈ ΠΏΠ΅Ρ€Π΅Π΄ ΠΏΠΎΠ»ΡƒΡ‡Π΅Π½Π½Ρ‹ΠΌ ΠΎΡ‚Π²Π΅Ρ‚ΠΎΠΌ ΠΏΠΎΡΡ‚Π°Π²ΠΈΡ‚ΡŒ плюс

РСшСниС для Π΄Π°Π½Π½ΠΎΠ³ΠΎ ΠΏΡ€ΠΈΠΌΠ΅Ρ€Π° ΠΌΠΎΠΆΠ½ΠΎ Π·Π°ΠΏΠΈΡΠ°Ρ‚ΡŒ ΠΏΠΎΠΊΠΎΡ€ΠΎΡ‡Π΅:

ΠŸΡ€ΠΈΠΌΠ΅Ρ€ 4. Найти Π·Π½Π°Ρ‡Π΅Π½ΠΈΠ΅ выраТСния

РСшСниС для Π΄Π°Π½Π½ΠΎΠ³ΠΎ ΠΏΡ€ΠΈΠΌΠ΅Ρ€Π° ΠΌΠΎΠΆΠ½ΠΎ Π·Π°ΠΏΠΈΡΠ°Ρ‚ΡŒ ΠΏΠΎΠΊΠΎΡ€ΠΎΡ‡Π΅:

ΠŸΡ€ΠΈΠΌΠ΅Ρ€ 5. Найти Π·Π½Π°Ρ‡Π΅Π½ΠΈΠ΅ выраТСния

Π­Ρ‚ΠΎ ΡƒΠΌΠ½ΠΎΠΆΠ΅Π½ΠΈΠ΅ Ρ€Π°Ρ†ΠΈΠΎΠ½Π°Π»ΡŒΠ½Ρ‹Ρ… чисСл с Ρ€Π°Π·Π½Ρ‹ΠΌΠΈ Π·Π½Π°ΠΊΠ°ΠΌΠΈ. ΠŸΠ΅Ρ€Π΅ΠΌΠ½ΠΎΠΆΠΈΠΌ ΠΌΠΎΠ΄ΡƒΠ»ΠΈ этих чисСл ΠΈ ΠΏΠ΅Ρ€Π΅Π΄ ΠΏΠΎΠ»ΡƒΡ‡Π΅Π½Π½Ρ‹ΠΌ ΠΎΡ‚Π²Π΅Ρ‚ΠΎΠΌ поставим минус

ΠšΠΎΡ€ΠΎΡ‚ΠΊΠΎΠ΅ Ρ€Π΅ΡˆΠ΅Π½ΠΈΠ΅ Π±ΡƒΠ΄Π΅Ρ‚ Π²Ρ‹Π³Π»ΡΠ΄Π΅Ρ‚ΡŒ Π·Π½Π°Ρ‡ΠΈΡ‚Π΅Π»ΡŒΠ½ΠΎ ΠΏΡ€ΠΎΡ‰Π΅:

ΠŸΡ€ΠΈΠΌΠ΅Ρ€ 6. Найти Π·Π½Π°Ρ‡Π΅Π½ΠΈΠ΅ выраТСния

ΠŸΠ΅Ρ€Π΅Π²Π΅Π΄Ρ‘ΠΌ смСшанноС число Π² Π½Π΅ΠΏΡ€Π°Π²ΠΈΠ»ΡŒΠ½ΡƒΡŽ Π΄Ρ€ΠΎΠ±ΡŒ. ΠžΡΡ‚Π°Π»ΡŒΠ½ΠΎΠ΅ ΠΏΠ΅Ρ€Π΅ΠΏΠΈΡˆΠ΅ΠΌ, ΠΊΠ°ΠΊ Π΅ΡΡ‚ΡŒ

ΠŸΠΎΠ»ΡƒΡ‡ΠΈΠ»ΠΈ ΡƒΠΌΠ½ΠΎΠΆΠ΅Π½ΠΈΠ΅ Ρ€Π°Ρ†ΠΈΠΎΠ½Π°Π»ΡŒΠ½Ρ‹Ρ… чисСл с Ρ€Π°Π·Π½Ρ‹ΠΌΠΈ Π·Π½Π°ΠΊΠ°ΠΌΠΈ. ΠŸΠ΅Ρ€Π΅ΠΌΠ½ΠΎΠΆΠΈΠΌ ΠΌΠΎΠ΄ΡƒΠ»ΠΈ этих чисСл ΠΈ ΠΏΠ΅Ρ€Π΅Π΄ ΠΏΠΎΠ»ΡƒΡ‡Π΅Π½Π½Ρ‹ΠΌ ΠΎΡ‚Π²Π΅Ρ‚ΠΎΠΌ поставим минус. Π—Π°ΠΏΠΈΡΡŒ с модулями ΠΌΠΎΠΆΠ½ΠΎ ΠΏΡ€ΠΎΠΏΡƒΡΡ‚ΠΈΡ‚ΡŒ, Ρ‡Ρ‚ΠΎΠ±Ρ‹ Π½Π΅ Π·Π°Π³Ρ€ΠΎΠΌΠΎΠΆΠ΄Π°Ρ‚ΡŒ Π²Ρ‹Ρ€Π°ΠΆΠ΅Π½ΠΈΠ΅

РСшСниС для Π΄Π°Π½Π½ΠΎΠ³ΠΎ ΠΏΡ€ΠΈΠΌΠ΅Ρ€Π° ΠΌΠΎΠΆΠ½ΠΎ Π·Π°ΠΏΠΈΡΠ°Ρ‚ΡŒ ΠΏΠΎΠΊΠΎΡ€ΠΎΡ‡Π΅

ΠŸΡ€ΠΈΠΌΠ΅Ρ€ 7. Найти Π·Π½Π°Ρ‡Π΅Π½ΠΈΠ΅ выраТСния

Π­Ρ‚ΠΎ ΡƒΠΌΠ½ΠΎΠΆΠ΅Π½ΠΈΠ΅ Ρ€Π°Ρ†ΠΈΠΎΠ½Π°Π»ΡŒΠ½Ρ‹Ρ… чисСл с Ρ€Π°Π·Π½Ρ‹ΠΌΠΈ Π·Π½Π°ΠΊΠ°ΠΌΠΈ. ΠŸΠ΅Ρ€Π΅ΠΌΠ½ΠΎΠΆΠΈΠΌ ΠΌΠΎΠ΄ΡƒΠ»ΠΈ этих чисСл ΠΈ ΠΏΠ΅Ρ€Π΅Π΄ ΠΏΠΎΠ»ΡƒΡ‡Π΅Π½Π½Ρ‹ΠΌ ΠΎΡ‚Π²Π΅Ρ‚ΠΎΠΌ поставим минус

Π‘Π½Π°Ρ‡Π°Π»Π° Π² ΠΎΡ‚Π²Π΅Ρ‚Π΅ ΠΏΠΎΠ»ΡƒΡ‡ΠΈΠ»Π°ΡΡŒ Π½Π΅ΠΏΡ€Π°Π²ΠΈΠ»ΡŒΠ½Π°Ρ Π΄Ρ€ΠΎΠ±ΡŒ , Π½ΠΎ ΠΌΡ‹ Π²Ρ‹Π΄Π΅Π»ΠΈΠ»ΠΈ Π² Π½Π΅ΠΉ Ρ†Π΅Π»ΡƒΡŽ Ρ‡Π°ΡΡ‚ΡŒ. ΠžΠ±Ρ€Π°Ρ‚ΠΈΡ‚Π΅ Π²Π½ΠΈΠΌΠ°Π½ΠΈΠ΅, Ρ‡Ρ‚ΠΎ цСлая Ρ‡Π°ΡΡ‚ΡŒ Π±Ρ‹Π»Π° Π²Ρ‹Π΄Π΅Π»Π΅Π½Π° ΠΎΡ‚ модуля Π΄Ρ€ΠΎΠ±ΠΈ . ΠŸΠΎΠ»ΡƒΡ‡ΠΈΠ²ΡˆΠ΅Π΅ΡΡ смСшанноС число Π±Ρ‹Π»ΠΎ Π·Π°ΠΊΠ»ΡŽΡ‡Π΅Π½ΠΎ Π² скобки, ΠΏΠ΅Ρ€Π΅Π΄ ΠΊΠΎΡ‚ΠΎΡ€Ρ‹ΠΌΠΈ поставлСн минус. Π­Ρ‚ΠΎ сдСлано для Ρ‚ΠΎΠ³ΠΎ, Ρ‡Ρ‚ΠΎΠ±Ρ‹ Π²Ρ‹ΠΏΠΎΠ»Π½ΡΠ»ΠΎΡΡŒ Ρ‚Ρ€Π΅Π±ΠΎΠ²Π°Π½ΠΈΠ΅ ΠΏΡ€Π°Π²ΠΈΠ»Π°. А ΠΏΡ€Π°Π²ΠΈΠ»ΠΎ Ρ‚Ρ€Π΅Π±ΠΎΠ²Π°Π»ΠΎ, Ρ‡Ρ‚ΠΎΠ±Ρ‹ ΠΏΠ΅Ρ€Π΅Π΄ ΠΏΠΎΠ»ΡƒΡ‡Π΅Π½Π½Ρ‹ΠΌ ΠΎΡ‚Π²Π΅Ρ‚ΠΎΠΌ стоял минус.

РСшСниС для Π΄Π°Π½Π½ΠΎΠ³ΠΎ ΠΏΡ€ΠΈΠΌΠ΅Ρ€Π° ΠΌΠΎΠΆΠ½ΠΎ Π·Π°ΠΏΠΈΡΠ°Ρ‚ΡŒ ΠΏΠΎΠΊΠΎΡ€ΠΎΡ‡Π΅:

ΠŸΡ€ΠΈΠΌΠ΅Ρ€ 8. Найти Π·Π½Π°Ρ‡Π΅Π½ΠΈΠ΅ выраТСния

Π‘Π½Π°Ρ‡Π°Π»Π° ΠΏΠ΅Ρ€Π΅ΠΌΠ½ΠΎΠΆΠΈΠΌ ΠΈ ΠΈ ΠΏΠΎΠ»ΡƒΡ‡Π΅Π½Π½ΠΎΠ΅ число ΠΏΠ΅Ρ€Π΅ΠΌΠ½ΠΎΠΆΠΈΠΌ с ΠΎΡΡ‚Π°Π²ΡˆΠΈΠΌΡΡ числом 5. Π—Π°ΠΏΠΈΡΡŒ с модулями пропустим, Ρ‡Ρ‚ΠΎΠ±Ρ‹ Π½Π΅ Π·Π°Π³Ρ€ΠΎΠΌΠΎΠΆΠ΄Π°Ρ‚ΡŒ Π²Ρ‹Ρ€Π°ΠΆΠ΅Π½ΠΈΠ΅.

ΠžΡ‚Π²Π΅Ρ‚: Π·Π½Π°Ρ‡Π΅Π½ΠΈΠ΅ выраТСния Ρ€Π°Π²Π½ΠΎ βˆ’2.

ΠŸΡ€ΠΈΠΌΠ΅Ρ€ 9. Найти Π·Π½Π°Ρ‡Π΅Π½ΠΈΠ΅ выраТСния:

ΠŸΠ΅Ρ€Π΅Π²Π΅Π΄Ρ‘ΠΌ ΡΠΌΠ΅ΡˆΠ°Π½Π½Ρ‹Π΅ числа Π² Π½Π΅ΠΏΡ€Π°Π²ΠΈΠ»ΡŒΠ½Ρ‹Π΅ Π΄Ρ€ΠΎΠ±ΠΈ:

ΠŸΠΎΠ»ΡƒΡ‡ΠΈΠ»ΠΈ ΡƒΠΌΠ½ΠΎΠΆΠ΅Π½ΠΈΠ΅ ΠΎΡ‚Ρ€ΠΈΡ†Π°Ρ‚Π΅Π»ΡŒΠ½Ρ‹Ρ… Ρ€Π°Ρ†ΠΈΠΎΠ½Π°Π»ΡŒΠ½Ρ‹Ρ… чисСл. ΠŸΠ΅Ρ€Π΅ΠΌΠ½ΠΎΠΆΠΈΠΌ ΠΌΠΎΠ΄ΡƒΠ»ΠΈ этих чисСл ΠΈ ΠΏΠ΅Ρ€Π΅Π΄ ΠΏΠΎΠ»ΡƒΡ‡Π΅Π½Π½Ρ‹ΠΌ ΠΎΡ‚Π²Π΅Ρ‚ΠΎΠΌ поставим плюс. Π—Π°ΠΏΠΈΡΡŒ с модулями ΠΌΠΎΠΆΠ½ΠΎ ΠΏΡ€ΠΎΠΏΡƒΡΡ‚ΠΈΡ‚ΡŒ, Ρ‡Ρ‚ΠΎΠ±Ρ‹ Π½Π΅ Π·Π°Π³Ρ€ΠΎΠΌΠΎΠΆΠ΄Π°Ρ‚ΡŒ Π²Ρ‹Ρ€Π°ΠΆΠ΅Π½ΠΈΠ΅

ΠŸΡ€ΠΈΠΌΠ΅Ρ€ 10. Найти Π·Π½Π°Ρ‡Π΅Π½ΠΈΠ΅ выраТСния

Π’Ρ‹Ρ€Π°ΠΆΠ΅Π½ΠΈΠ΅ состоит ΠΈΠ· Π½Π΅ΡΠΊΠΎΠ»ΡŒΠΊΠΈΡ… сомноТитСлСй. Богласно ΡΠΎΡ‡Π΅Ρ‚Π°Ρ‚Π΅Π»ΡŒΠ½ΠΎΠΌΡƒ Π·Π°ΠΊΠΎΠ½Ρƒ умноТСния, Ссли Π²Ρ‹Ρ€Π°ΠΆΠ΅Π½ΠΈΠ΅ состоит ΠΈΠ· Π½Π΅ΡΠΊΠΎΠ»ΡŒΠΊΠΈΡ… сомноТитСлСй, Ρ‚ΠΎ ΠΏΡ€ΠΎΠΈΠ·Π²Π΅Π΄Π΅Π½ΠΈΠ΅ Π½Π΅ Π±ΡƒΠ΄Π΅Ρ‚ Π·Π°Π²ΠΈΡΠ΅Ρ‚ΡŒ ΠΎΡ‚ порядка дСйствий. Π­Ρ‚ΠΎ позволяСт Π½Π°ΠΌ Π²Ρ‹Ρ‡ΠΈΡΠ»ΠΈΡ‚ΡŒ Π΄Π°Π½Π½ΠΎΠ΅ Π²Ρ‹Ρ€Π°ΠΆΠ΅Π½ΠΈΠ΅ Π² любом порядкС.

НС Π±ΡƒΠ΄Π΅ΠΌ ΠΈΠ·ΠΎΠ±Ρ€Π΅Ρ‚Π°Ρ‚ΡŒ вСлосипСд, Π° вычислим Π΄Π°Π½Π½ΠΎΠ΅ Π²Ρ‹Ρ€Π°ΠΆΠ΅Π½ΠΈΠ΅ слСва Π½Π°ΠΏΡ€Π°Π²ΠΎ Π² порядкС слСдования сомноТитСлСй. Π—Π°ΠΏΠΈΡΡŒ с модулями пропустим, Ρ‡Ρ‚ΠΎΠ±Ρ‹ Π½Π΅ Π·Π°Π³Ρ€ΠΎΠΌΠΎΠΆΠ΄Π°Ρ‚ΡŒ Π²Ρ‹Ρ€Π°ΠΆΠ΅Π½ΠΈΠ΅

Π’Ρ€Π΅Ρ‚ΡŒΠ΅ дСйствиС:

Π§Π΅Ρ‚Π²Ρ‘Ρ€Ρ‚ΠΎΠ΅ дСйствиС:

ΠžΡ‚Π²Π΅Ρ‚: Π·Π½Π°Ρ‡Π΅Π½ΠΈΠ΅ выраТСния Ρ€Π°Π²Π½ΠΎ

ΠŸΡ€ΠΈΠΌΠ΅Ρ€ 11. Найти Π·Π½Π°Ρ‡Π΅Π½ΠΈΠ΅ выраТСния

ВспоминаСм Π·Π°ΠΊΠΎΠ½ умноТСния Π½Π° ноль. Π­Ρ‚ΠΎΡ‚ Π·Π°ΠΊΠΎΠ½ гласит, Ρ‡Ρ‚ΠΎ ΠΏΡ€ΠΎΠΈΠ·Π²Π΅Π΄Π΅Π½ΠΈΠ΅ Ρ€Π°Π²Π½ΠΎ Π½ΡƒΠ»ΡŽ, Ссли хотя Π±Ρ‹ ΠΎΠ΄ΠΈΠ½ ΠΈΠ· сомноТитСлСй Ρ€Π°Π²Π΅Π½ Π½ΡƒΠ»ΡŽ.

Π’ нашСм ΠΏΡ€ΠΈΠΌΠ΅Ρ€Π΅ ΠΎΠ΄ΠΈΠ½ ΠΈΠ· сомноТитСлСй Ρ€Π°Π²Π΅Π½ Π½ΡƒΠ»ΡŽ, поэтому Π½Π΅ тСряя Π²Ρ€Π΅ΠΌΠ΅Π½ΠΈ ΠΎΡ‚Π²Π΅Ρ‡Π°Π΅ΠΌ, Ρ‡Ρ‚ΠΎ Π·Π½Π°Ρ‡Π΅Π½ΠΈΠ΅ выраТСния Ρ€Π°Π²Π½ΠΎ Π½ΡƒΠ»ΡŽ:

ΠŸΡ€ΠΈΠΌΠ΅Ρ€ 12. Найти Π·Π½Π°Ρ‡Π΅Π½ΠΈΠ΅ выраТСния

ΠŸΡ€ΠΎΠΈΠ·Π²Π΅Π΄Π΅Π½ΠΈΠ΅ Ρ€Π°Π²Π½ΠΎ Π½ΡƒΠ»ΡŽ, Ссли хотя Π±Ρ‹ ΠΎΠ΄ΠΈΠ½ ΠΈΠ· сомноТитСлСй Ρ€Π°Π²Π΅Π½ Π½ΡƒΠ»ΡŽ.

Π’ нашСм ΠΏΡ€ΠΈΠΌΠ΅Ρ€Π΅ ΠΎΠ΄ΠΈΠ½ ΠΈΠ· сомноТитСлСй Ρ€Π°Π²Π΅Π½ Π½ΡƒΠ»ΡŽ, поэтому Π½Π΅ тСряя Π²Ρ€Π΅ΠΌΠ΅Π½ΠΈ ΠΎΡ‚Π²Π΅Ρ‡Π°Π΅ΠΌ, Ρ‡Ρ‚ΠΎ Π·Π½Π°Ρ‡Π΅Π½ΠΈΠ΅ выраТСния Ρ€Π°Π²Π½ΠΎ Π½ΡƒΠ»ΡŽ:

ΠŸΡ€ΠΈΠΌΠ΅Ρ€ 13. Найти Π·Π½Π°Ρ‡Π΅Π½ΠΈΠ΅ выраТСния

МоТно Π²ΠΎΡΠΏΠΎΠ»ΡŒΠ·ΠΎΠ²Π°Ρ‚ΡŒΡΡ порядком дСйствий ΠΈ сначала Π²Ρ‹Ρ‡ΠΈΡΠ»ΠΈΡ‚ΡŒ Π²Ρ‹Ρ€Π°ΠΆΠ΅Π½ΠΈΠ΅ Π² скобках ΠΈ ΠΏΠΎΠ»ΡƒΡ‡Π΅Π½Π½Ρ‹ΠΉ ΠΎΡ‚Π²Π΅Ρ‚ ΠΏΠ΅Ρ€Π΅ΠΌΠ½ΠΎΠΆΠΈΡ‚ΡŒ с Π΄Ρ€ΠΎΠ±ΡŒΡŽ .

Π•Ρ‰Ρ‘ ΠΌΠΎΠΆΠ½ΠΎ Π²ΠΎΡΠΏΠΎΠ»ΡŒΠ·ΠΎΠ²Π°Ρ‚ΡŒΡΡ Ρ€Π°ΡΠΏΡ€Π΅Π΄Π΅Π»ΠΈΡ‚Π΅Π»ΡŒΠ½Ρ‹ΠΌ Π·Π°ΠΊΠΎΠ½ΠΎΠΌ умноТСния β€” ΡƒΠΌΠ½ΠΎΠΆΠΈΡ‚ΡŒ ΠΊΠ°ΠΆΠ΄ΠΎΠ΅ слагаСмоС суммы Π½Π° Π΄Ρ€ΠΎΠ±ΡŒ ΠΈ ΠΏΠΎΠ»ΡƒΡ‡Π΅Π½Π½Ρ‹Π΅ Ρ€Π΅Π·ΡƒΠ»ΡŒΡ‚Π°Ρ‚Ρ‹ ΡΠ»ΠΎΠΆΠΈΡ‚ΡŒ. Π­Ρ‚ΠΈΠΌ способом ΠΈ Π²ΠΎΡΠΏΠΎΠ»ΡŒΠ·ΡƒΠ΅ΠΌΡΡ.

Богласно порядку дСйствий, Ссли Π² Π²Ρ‹Ρ€Π°ΠΆΠ΅Π½ΠΈΠΈ присутствуСт слоТСниС ΠΈ ΡƒΠΌΠ½ΠΎΠΆΠ΅Π½ΠΈΠ΅, Ρ‚ΠΎ Π² ΠΏΠ΅Ρ€Π²ΡƒΡŽ ΠΎΡ‡Π΅Ρ€Π΅Π΄ΡŒ Π½ΡƒΠΆΠ½ΠΎ Π²Ρ‹ΠΏΠΎΠ»Π½ΡΡ‚ΡŒ ΡƒΠΌΠ½ΠΎΠΆΠ΅Π½ΠΈΠ΅. ΠŸΠΎΡΡ‚ΠΎΠΌΡƒ Π² ΠΏΠΎΠ»ΡƒΡ‡ΠΈΠ²ΡˆΠ΅ΠΌΡΡ Π½ΠΎΠ²ΠΎΠΌ Π²Ρ‹Ρ€Π°ΠΆΠ΅Π½ΠΈΠΈ Π²ΠΎΠ·ΡŒΠΌΡ‘ΠΌ Π² скобки Ρ‚Π΅ ΠΏΠ°Ρ€Π°ΠΌΠ΅Ρ‚Ρ€Ρ‹, ΠΊΠΎΡ‚ΠΎΡ€Ρ‹Π΅ Π΄ΠΎΠ»ΠΆΠ½Ρ‹ Π±Ρ‹Ρ‚ΡŒ ΠΏΠ΅Ρ€Π΅ΠΌΠ½ΠΎΠΆΠ΅Π½Ρ‹. Π’Π°ΠΊ ΠΌΡ‹ Ρ…ΠΎΡ€ΠΎΡˆΠΎ ΡƒΠ²ΠΈΠ΄ΠΈΠΌ, ΠΊΠ°ΠΊΠΈΠ΅ дСйствия Π²Ρ‹ΠΏΠΎΠ»Π½ΠΈΡ‚ΡŒ Ρ€Π°Π½ΡŒΡˆΠ΅, Π° ΠΊΠ°ΠΊΠΈΠ΅ ΠΏΠΎΠ·ΠΆΠ΅:

Π’Ρ€Π΅Ρ‚ΡŒΠ΅ дСйствиС:

ΠžΡ‚Π²Π΅Ρ‚: Π·Π½Π°Ρ‡Π΅Π½ΠΈΠ΅ выраТСния Ρ€Π°Π²Π½ΠΎ

РСшСниС для Π΄Π°Π½Π½ΠΎΠ³ΠΎ ΠΏΡ€ΠΈΠΌΠ΅Ρ€Π° ΠΌΠΎΠΆΠ½ΠΎ Π·Π°ΠΏΠΈΡΠ°Ρ‚ΡŒ Π·Π½Π°Ρ‡ΠΈΡ‚Π΅Π»ΡŒΠ½ΠΎ ΠΊΠΎΡ€ΠΎΡ‡Π΅. Π’Ρ‹Π³Π»ΡΠ΄Π΅Ρ‚ΡŒ ΠΎΠ½ΠΎ Π±ΡƒΠ΄Π΅Ρ‚ ΡΠ»Π΅Π΄ΡƒΡŽΡ‰ΠΈΠΌ ΠΎΠ±Ρ€Π°Π·ΠΎΠΌ:

Π’ΠΈΠ΄Π½ΠΎ, Ρ‡Ρ‚ΠΎ Π΄Π°Π½Π½Ρ‹ΠΉ ΠΏΡ€ΠΈΠΌΠ΅Ρ€ ΠΌΠΎΠΆΠ½ΠΎ Π±Ρ‹Π»ΠΎ Ρ€Π΅ΡˆΠΈΡ‚ΡŒ Π΄Π°ΠΆΠ΅ Π² ΡƒΠΌΠ΅. ΠŸΠΎΡΡ‚ΠΎΠΌΡƒ слСдуСт Ρ€Π°Π·Π²ΠΈΠ²Π°Ρ‚ΡŒ Π² сСбС Π½Π°Π²Ρ‹ΠΊ Π°Π½Π°Π»ΠΈΠ·Π° выраТСния Π΄ΠΎ Π½Π°Ρ‡Π°Π»Π° Π΅Π³ΠΎ Ρ€Π΅ΡˆΠ΅Π½ΠΈΡ. Π’ΠΏΠΎΠ»Π½Π΅ вСроятно, Ρ‡Ρ‚ΠΎ Π΅Π³ΠΎ ΠΌΠΎΠΆΠ½ΠΎ Ρ€Π΅ΡˆΠΈΡ‚ΡŒ Π² ΡƒΠΌΠ΅ ΠΈ ΡΡΠΊΠΎΠ½ΠΎΠΌΠΈΡ‚ΡŒ ΠΌΠ½ΠΎΠ³ΠΎ Π²Ρ€Π΅ΠΌΠ΅Π½ΠΈ ΠΈ Π½Π΅Ρ€Π²ΠΎΠ². А Π½Π° ΠΊΠΎΠ½Ρ‚Ρ€ΠΎΠ»ΡŒΠ½Ρ‹Ρ… ΠΈ экзамСнах, ΠΊΠ°ΠΊ извСстно врСмя ΠΎΡ‡Π΅Π½ΡŒ Π΄ΠΎΡ€ΠΎΠ³ΠΎ стоит.

ΠŸΡ€ΠΈΠΌΠ΅Ρ€ 14. Найти Π·Π½Π°Ρ‡Π΅Π½ΠΈΠ΅ выраТСния βˆ’4,2 Γ— 3,2

Π­Ρ‚ΠΎ ΡƒΠΌΠ½ΠΎΠΆΠ΅Π½ΠΈΠ΅ Ρ€Π°Ρ†ΠΈΠΎΠ½Π°Π»ΡŒΠ½Ρ‹Ρ… чисСл с Ρ€Π°Π·Π½Ρ‹ΠΌΠΈ Π·Π½Π°ΠΊΠ°ΠΌΠΈ. ΠŸΠ΅Ρ€Π΅ΠΌΠ½ΠΎΠΆΠΈΠΌ ΠΌΠΎΠ΄ΡƒΠ»ΠΈ этих чисСл ΠΈ ΠΏΠ΅Ρ€Π΅Π΄ ΠΏΠΎΠ»ΡƒΡ‡Π΅Π½Π½Ρ‹ΠΌ ΠΎΡ‚Π²Π΅Ρ‚ΠΎΠΌ поставим минус

ΠžΠ±Ρ€Π°Ρ‚ΠΈΡ‚Π΅ Π²Π½ΠΈΠΌΠ°Π½ΠΈΠ΅, ΠΊΠ°ΠΊ ΠΏΠ΅Ρ€Π΅ΠΌΠ½ΠΎΠΆΠ°Π»ΠΈΡΡŒ ΠΌΠΎΠ΄ΡƒΠ»ΠΈ Ρ€Π°Ρ†ΠΈΠΎΠ½Π°Π»ΡŒΠ½Ρ‹Ρ… чисСл. Π’ Π΄Π°Π½Π½ΠΎΠΌ случаС, Ρ‡Ρ‚ΠΎΠ±Ρ‹ ΠΏΠ΅Ρ€Π΅ΠΌΠ½ΠΎΠΆΠΈΡ‚ΡŒ ΠΌΠΎΠ΄ΡƒΠ»ΠΈ Ρ€Π°Ρ†ΠΈΠΎΠ½Π°Π»ΡŒΠ½Ρ‹Ρ… чисСл, ΠΏΠΎΡ‚Ρ€Π΅Π±ΠΎΠ²Π°Π»ΠΎΡΡŒ .

ΠŸΡ€ΠΈΠΌΠ΅Ρ€ 15. Найти Π·Π½Π°Ρ‡Π΅Π½ΠΈΠ΅ выраТСния βˆ’0,15 Γ— 4

Π­Ρ‚ΠΎ ΡƒΠΌΠ½ΠΎΠΆΠ΅Π½ΠΈΠ΅ Ρ€Π°Ρ†ΠΈΠΎΠ½Π°Π»ΡŒΠ½Ρ‹Ρ… чисСл с Ρ€Π°Π·Π½Ρ‹ΠΌΠΈ Π·Π½Π°ΠΊΠ°ΠΌΠΈ. ΠŸΠ΅Ρ€Π΅ΠΌΠ½ΠΎΠΆΠΈΠΌ ΠΌΠΎΠ΄ΡƒΠ»ΠΈ этих чисСл ΠΈ ΠΏΠ΅Ρ€Π΅Π΄ ΠΏΠΎΠ»ΡƒΡ‡Π΅Π½Π½Ρ‹ΠΌ ΠΎΡ‚Π²Π΅Ρ‚ΠΎΠΌ поставим минус

ΠžΠ±Ρ€Π°Ρ‚ΠΈΡ‚Π΅ Π²Π½ΠΈΠΌΠ°Π½ΠΈΠ΅, ΠΊΠ°ΠΊ ΠΏΠ΅Ρ€Π΅ΠΌΠ½ΠΎΠΆΠ°Π»ΠΈΡΡŒ ΠΌΠΎΠ΄ΡƒΠ»ΠΈ Ρ€Π°Ρ†ΠΈΠΎΠ½Π°Π»ΡŒΠ½Ρ‹Ρ… чисСл. Π’ Π΄Π°Π½Π½ΠΎΠΌ случаС, Ρ‡Ρ‚ΠΎΠ±Ρ‹ ΠΏΠ΅Ρ€Π΅ΠΌΠ½ΠΎΠΆΠΈΡ‚ΡŒ ΠΌΠΎΠ΄ΡƒΠ»ΠΈ Ρ€Π°Ρ†ΠΈΠΎΠ½Π°Π»ΡŒΠ½Ρ‹Ρ… чисСл, ΠΏΠΎΡ‚Ρ€Π΅Π±ΠΎΠ²Π°Π»ΠΎΡΡŒ ΡΡƒΠΌΠ΅Ρ‚ΡŒ .

ΠŸΡ€ΠΈΠΌΠ΅Ρ€ 16. Найти Π·Π½Π°Ρ‡Π΅Π½ΠΈΠ΅ выраТСния βˆ’4,2 Γ— (βˆ’7,5)

Π­Ρ‚ΠΎ ΡƒΠΌΠ½ΠΎΠΆΠ΅Π½ΠΈΠ΅ ΠΎΡ‚Ρ€ΠΈΡ†Π°Ρ‚Π΅Π»ΡŒΠ½Ρ‹Ρ… Ρ€Π°Ρ†ΠΈΠΎΠ½Π°Π»ΡŒΠ½Ρ‹Ρ… чисСл. ΠŸΠ΅Ρ€Π΅ΠΌΠ½ΠΎΠΆΠΈΠΌ ΠΌΠΎΠ΄ΡƒΠ»ΠΈ этих чисСл ΠΈ ΠΏΠ΅Ρ€Π΅Π΄ ΠΏΠΎΠ»ΡƒΡ‡Π΅Π½Π½Ρ‹ΠΌ ΠΎΡ‚Π²Π΅Ρ‚ΠΎΠΌ поставим плюс

Π”Π΅Π»Π΅Π½ΠΈΠ΅ Ρ€Π°Ρ†ΠΈΠΎΠ½Π°Π»ΡŒΠ½Ρ‹Ρ… чисСл

ΠŸΡ€Π°Π²ΠΈΠ»Π° дСлСния Ρ†Π΅Π»Ρ‹Ρ… чисСл справСдливы ΠΈ для Ρ€Π°Ρ†ΠΈΠΎΠ½Π°Π»ΡŒΠ½Ρ‹Ρ… чисСл. Π˜Π½Ρ‹ΠΌΠΈ словами, Ρ‡Ρ‚ΠΎΠ±Ρ‹ ΡƒΠΌΠ΅Ρ‚ΡŒ Π΄Π΅Π»ΠΈΡ‚ΡŒ Ρ€Π°Ρ†ΠΈΠΎΠ½Π°Π»ΡŒΠ½Ρ‹Π΅ числа, Π½ΡƒΠΆΠ½ΠΎ ΡƒΠΌΠ΅Ρ‚ΡŒ

Π’ ΠΎΡΡ‚Π°Π»ΡŒΠ½ΠΎΠΌ ΠΆΠ΅ ΠΏΡ€ΠΈΠΌΠ΅Π½ΡΡŽΡ‚ΡΡ Ρ‚Π΅ ΠΆΠ΅ ΠΌΠ΅Ρ‚ΠΎΠ΄Ρ‹ дСлСния ΠΎΠ±Ρ‹ΠΊΠ½ΠΎΠ²Π΅Π½Π½Ρ‹Ρ… ΠΈ дСсятичных Π΄Ρ€ΠΎΠ±Π΅ΠΉ. Π§Ρ‚ΠΎΠ±Ρ‹ Ρ€Π°Π·Π΄Π΅Π»ΠΈΡ‚ΡŒ ΠΎΠ±Ρ‹ΠΊΠ½ΠΎΠ²Π΅Π½Π½ΡƒΡŽ Π΄Ρ€ΠΎΠ±ΡŒ Π½Π° Π΄Ρ€ΡƒΠ³ΡƒΡŽ Π΄Ρ€ΠΎΠ±ΡŒ, Π½ΡƒΠΆΠ½ΠΎ ΠΏΠ΅Ρ€Π²ΡƒΡŽ Π΄Ρ€ΠΎΠ±ΡŒ ΡƒΠΌΠ½ΠΎΠΆΠΈΡ‚ΡŒ Π½Π° Π΄Ρ€ΠΎΠ±ΡŒ, ΠΎΠ±Ρ€Π°Ρ‚Π½ΡƒΡŽ Π²Ρ‚ΠΎΡ€ΠΎΠΉ.

А Ρ‡Ρ‚ΠΎΠ±Ρ‹ Ρ€Π°Π·Π΄Π΅Π»ΠΈΡ‚ΡŒ Π΄Π΅ΡΡΡ‚ΠΈΡ‡Π½ΡƒΡŽ Π΄Ρ€ΠΎΠ±ΡŒ Π½Π° Π΄Ρ€ΡƒΠ³ΡƒΡŽ Π΄Π΅ΡΡΡ‚ΠΈΡ‡Π½ΡƒΡŽ Π΄Ρ€ΠΎΠ±ΡŒ, Π½ΡƒΠΆΠ½ΠΎ Π² Π΄Π΅Π»ΠΈΠΌΠΎΠΌ ΠΈ Π² Π΄Π΅Π»ΠΈΡ‚Π΅Π»Π΅ пСрСнСсти Π·Π°ΠΏΡΡ‚ΡƒΡŽ Π²ΠΏΡ€Π°Π²ΠΎ Π½Π° ΡΡ‚ΠΎΠ»ΡŒΠΊΠΎ Ρ†ΠΈΡ„Ρ€, сколько ΠΈΡ… послС запятой Π² Π΄Π΅Π»ΠΈΡ‚Π΅Π»Π΅, Π·Π°Ρ‚Π΅ΠΌ Π²Ρ‹ΠΏΠΎΠ»Π½ΠΈΡ‚ΡŒ Π΄Π΅Π»Π΅Π½ΠΈΠ΅, ΠΊΠ°ΠΊ Π½Π° ΠΎΠ±Ρ‹Ρ‡Π½ΠΎΠ΅ число.

ΠŸΡ€ΠΈΠΌΠ΅Ρ€ 1. Найти Π·Π½Π°Ρ‡Π΅Π½ΠΈΠ΅ выраТСния:

Π­Ρ‚ΠΎ Π΄Π΅Π»Π΅Π½ΠΈΠ΅ Ρ€Π°Ρ†ΠΈΠΎΠ½Π°Π»ΡŒΠ½Ρ‹Ρ… чисСл с Ρ€Π°Π·Π½Ρ‹ΠΌΠΈ Π·Π½Π°ΠΊΠ°ΠΌΠΈ. Π§Ρ‚ΠΎΠ±Ρ‹ Π²Ρ‹Ρ‡ΠΈΡΠ»ΠΈΡ‚ΡŒ Ρ‚Π°ΠΊΠΎΠ΅ Π²Ρ‹Ρ€Π°ΠΆΠ΅Π½ΠΈΠ΅, Π½ΡƒΠΆΠ½ΠΎ ΠΏΠ΅Ρ€Π²ΡƒΡŽ Π΄Ρ€ΠΎΠ±ΡŒ ΡƒΠΌΠ½ΠΎΠΆΠΈΡ‚ΡŒ Π½Π° Π΄Ρ€ΠΎΠ±ΡŒ, ΠΎΠ±Ρ€Π°Ρ‚Π½ΡƒΡŽ Π²Ρ‚ΠΎΡ€ΠΎΠΉ.

Π˜Ρ‚Π°ΠΊ, ΡƒΠΌΠ½ΠΎΠΆΠΈΠΌ ΠΏΠ΅Ρ€Π²ΡƒΡŽ Π΄Ρ€ΠΎΠ±ΡŒ Π½Π° Π΄Ρ€ΠΎΠ±ΡŒ ΠΎΠ±Ρ€Π°Ρ‚Π½ΡƒΡŽ Π²Ρ‚ΠΎΡ€ΠΎΠΉ.

ΠŸΠΎΠ»ΡƒΡ‡ΠΈΠ»ΠΈ ΡƒΠΌΠ½ΠΎΠΆΠ΅Π½ΠΈΠ΅ Ρ€Π°Ρ†ΠΈΠΎΠ½Π°Π»ΡŒΠ½Ρ‹Ρ… чисСл с Ρ€Π°Π·Π½Ρ‹ΠΌΠΈ Π·Π½Π°ΠΊΠ°ΠΌΠΈ. А ΠΊΠ°ΠΊ Π²Ρ‹Ρ‡ΠΈΡΠ»ΡΡ‚ΡŒ Ρ‚Π°ΠΊΠΈΠ΅ выраТСния ΠΌΡ‹ ΡƒΠΆΠ΅ Π·Π½Π°Π΅ΠΌ. Для этого Π½ΡƒΠΆΠ½ΠΎ ΠΏΠ΅Ρ€Π΅ΠΌΠ½ΠΎΠΆΠΈΡ‚ΡŒ ΠΌΠΎΠ΄ΡƒΠ»ΠΈ этих Ρ€Π°Ρ†ΠΈΠΎΠ½Π°Π»ΡŒΠ½Ρ‹Ρ… чисСл ΠΈ ΠΏΠ΅Ρ€Π΅Π΄ ΠΏΠΎΠ»ΡƒΡ‡Π΅Π½Π½Ρ‹ΠΌ ΠΎΡ‚Π²Π΅Ρ‚ΠΎΠΌ ΠΏΠΎΡΡ‚Π°Π²ΠΈΡ‚ΡŒ минус.

Π”ΠΎΡ€Π΅ΡˆΠ°Π΅ΠΌ Π΄Π°Π½Π½Ρ‹ΠΉ ΠΏΡ€ΠΈΠΌΠ΅Ρ€ Π΄ΠΎ ΠΊΠΎΠ½Ρ†Π°. Π—Π°ΠΏΠΈΡΡŒ с модулями ΠΌΠΎΠΆΠ½ΠΎ ΠΏΡ€ΠΎΠΏΡƒΡΡ‚ΠΈΡ‚ΡŒ, Ρ‡Ρ‚ΠΎΠ±Ρ‹ Π½Π΅ Π·Π°Π³Ρ€ΠΎΠΌΠΎΠΆΠ΄Π°Ρ‚ΡŒ Π²Ρ‹Ρ€Π°ΠΆΠ΅Π½ΠΈΠ΅

Π’Π°ΠΊΠΈΠΌ ΠΎΠ±Ρ€Π°Π·ΠΎΠΌ, Π·Π½Π°Ρ‡Π΅Π½ΠΈΠ΅ выраТСния Ρ€Π°Π²Π½ΠΎ

ΠŸΠΎΠ΄Ρ€ΠΎΠ±Π½ΠΎΠ΅ Ρ€Π΅ΡˆΠ΅Π½ΠΈΠ΅ выглядит ΡΠ»Π΅Π΄ΡƒΡŽΡ‰ΠΈΠΌ ΠΎΠ±Ρ€Π°Π·ΠΎΠΌ:

ΠšΠΎΡ€ΠΎΡ‚ΠΊΠΎΠ΅ Ρ€Π΅ΡˆΠ΅Π½ΠΈΠ΅ Π±ΡƒΠ΄Π΅Ρ‚ Π²Ρ‹Π³Π»ΡΠ΄Π΅Ρ‚ΡŒ Ρ‚Π°ΠΊ:

ΠŸΡ€ΠΈΠΌΠ΅Ρ€ 2. Найти Π·Π½Π°Ρ‡Π΅Π½ΠΈΠ΅ выраТСния

Π­Ρ‚ΠΎ Π΄Π΅Π»Π΅Π½ΠΈΠ΅ Ρ€Π°Ρ†ΠΈΠΎΠ½Π°Π»ΡŒΠ½Ρ‹Ρ… чисСл с Ρ€Π°Π·Π½Ρ‹ΠΌΠΈ Π·Π½Π°ΠΊΠ°ΠΌΠΈ. Π§Ρ‚ΠΎΠ±Ρ‹ Π²Ρ‹Ρ‡ΠΈΡΠ»ΠΈΡ‚ΡŒ Π΄Π°Π½Π½ΠΎΠ΅ Π²Ρ‹Ρ€Π°ΠΆΠ΅Π½ΠΈΠ΅, Π½ΡƒΠΆΠ½ΠΎ ΠΏΠ΅Ρ€Π²ΡƒΡŽ Π΄Ρ€ΠΎΠ±ΡŒ ΡƒΠΌΠ½ΠΎΠΆΠΈΡ‚ΡŒ Π½Π° Π΄Ρ€ΠΎΠ±ΡŒ, ΠΎΠ±Ρ€Π°Ρ‚Π½ΡƒΡŽ Π²Ρ‚ΠΎΡ€ΠΎΠΉ.

ΠžΠ±Ρ€Π°Ρ‚Π½Π°Ρ для Π²Ρ‚ΠΎΡ€ΠΎΠΉ Π΄Ρ€ΠΎΠ±ΠΈ это Π΄Ρ€ΠΎΠ±ΡŒ . На Π½Π΅Ρ‘ ΠΈ ΡƒΠΌΠ½ΠΎΠΆΠΈΠΌ ΠΏΠ΅Ρ€Π²ΡƒΡŽ Π΄Ρ€ΠΎΠ±ΡŒ:

ΠšΠΎΡ€ΠΎΡ‚ΠΊΠΎΠ΅ Ρ€Π΅ΡˆΠ΅Π½ΠΈΠ΅ Π±ΡƒΠ΄Π΅Ρ‚ Π²Ρ‹Π³Π»ΡΠ΄Π΅Ρ‚ΡŒ ΡΠ»Π΅Π΄ΡƒΡŽΡ‰ΠΈΠΌ ΠΎΠ±Ρ€Π°Π·ΠΎΠΌ:

ΠŸΡ€ΠΈΠΌΠ΅Ρ€ 3. Найти Π·Π½Π°Ρ‡Π΅Π½ΠΈΠ΅ выраТСния

Π­Ρ‚ΠΎ Π΄Π΅Π»Π΅Π½ΠΈΠ΅ ΠΎΡ‚Ρ€ΠΈΡ†Π°Ρ‚Π΅Π»ΡŒΠ½Ρ‹Ρ… Ρ€Π°Ρ†ΠΈΠΎΠ½Π°Π»ΡŒΠ½Ρ‹Ρ… чисСл. Π§Ρ‚ΠΎΠ±Ρ‹ Π²Ρ‹Ρ‡ΠΈΡΠ»ΠΈΡ‚ΡŒ Π΄Π°Π½Π½ΠΎΠ΅ Π²Ρ‹Ρ€Π°ΠΆΠ΅Π½ΠΈΠ΅, ΠΎΠΏΡΡ‚ΡŒ ΠΆΠ΅ Π½ΡƒΠΆΠ½ΠΎ ΠΏΠ΅Ρ€Π²ΡƒΡŽ Π΄Ρ€ΠΎΠ±ΡŒ ΡƒΠΌΠ½ΠΎΠΆΠΈΡ‚ΡŒ Π½Π° Π΄Ρ€ΠΎΠ±ΡŒ ΠΎΠ±Ρ€Π°Ρ‚Π½ΡƒΡŽ Π²Ρ‚ΠΎΡ€ΠΎΠΉ.

ΠžΠ±Ρ€Π°Ρ‚Π½Π°Ρ для Π²Ρ‚ΠΎΡ€ΠΎΠΉ Π΄Ρ€ΠΎΠ±ΠΈ это Π΄Ρ€ΠΎΠ±ΡŒ . На Π½Π΅Ρ‘ ΠΈ ΡƒΠΌΠ½ΠΎΠΆΠΈΠΌ ΠΏΠ΅Ρ€Π²ΡƒΡŽ Π΄Ρ€ΠΎΠ±ΡŒ:

ΠŸΠΎΠ»ΡƒΡ‡ΠΈΠ»ΠΈ ΡƒΠΌΠ½ΠΎΠΆΠ΅Π½ΠΈΠ΅ ΠΎΡ‚Ρ€ΠΈΡ†Π°Ρ‚Π΅Π»ΡŒΠ½Ρ‹Ρ… Ρ€Π°Ρ†ΠΈΠΎΠ½Π°Π»ΡŒΠ½Ρ‹Ρ… чисСл. Как вычисляСтся ΠΏΠΎΠ΄ΠΎΠ±Π½ΠΎΠ΅ Π²Ρ‹Ρ€Π°ΠΆΠ΅Π½ΠΈΠ΅ ΠΌΡ‹ ΡƒΠΆΠ΅ Π·Π½Π°Π΅ΠΌ. НуТно ΠΏΠ΅Ρ€Π΅ΠΌΠ½ΠΎΠΆΠΈΡ‚ΡŒ ΠΌΠΎΠ΄ΡƒΠ»ΠΈ Ρ€Π°Ρ†ΠΈΠΎΠ½Π°Π»ΡŒΠ½Ρ‹Ρ… чисСл ΠΈ ΠΏΠ΅Ρ€Π΅Π΄ ΠΏΠΎΠ»ΡƒΡ‡Π΅Π½Π½Ρ‹ΠΌ ΠΎΡ‚Π²Π΅Ρ‚ΠΎΠΌ ΠΏΠΎΡΡ‚Π°Π²ΠΈΡ‚ΡŒ плюс.

Π”ΠΎΡ€Π΅ΡˆΠ°Π΅ΠΌ этот ΠΏΡ€ΠΈΠΌΠ΅Ρ€ Π΄ΠΎ ΠΊΠΎΠ½Ρ†Π°. Π—Π°ΠΏΠΈΡΡŒ с модулями ΠΌΠΎΠΆΠ½ΠΎ ΠΏΡ€ΠΎΠΏΡƒΡΡ‚ΠΈΡ‚ΡŒ, Ρ‡Ρ‚ΠΎΠ±Ρ‹ Π½Π΅ Π·Π°Π³Ρ€ΠΎΠΌΠΎΠΆΠ΄Π°Ρ‚ΡŒ Π²Ρ‹Ρ€Π°ΠΆΠ΅Π½ΠΈΠ΅:

ΠŸΡ€ΠΈΠΌΠ΅Ρ€ 4. Найти Π·Π½Π°Ρ‡Π΅Π½ΠΈΠ΅ выраТСния

Π§Ρ‚ΠΎΠ±Ρ‹ Π²Ρ‹Ρ‡ΠΈΡΠ»ΠΈΡ‚ΡŒ Π΄Π°Π½Π½ΠΎΠ΅ Π²Ρ‹Ρ€Π°ΠΆΠ΅Π½ΠΈΠ΅, Π½ΡƒΠΆΠ½ΠΎ ΠΏΠ΅Ρ€Π²ΠΎΠ΅ число βˆ’3 ΡƒΠΌΠ½ΠΎΠΆΠΈΡ‚ΡŒ Π½Π° Π΄Ρ€ΠΎΠ±ΡŒ, ΠΎΠ±Ρ€Π°Ρ‚Π½ΡƒΡŽ Π΄Ρ€ΠΎΠ±ΠΈ .

ΠžΠ±Ρ€Π°Ρ‚Π½Π°Ρ для Π΄Ρ€ΠΎΠ±ΠΈ это Π΄Ρ€ΠΎΠ±ΡŒ . На Π½Π΅Ρ‘ ΠΈ ΡƒΠΌΠ½ΠΎΠΆΠΈΠΌ ΠΏΠ΅Ρ€Π²ΠΎΠ΅ число βˆ’3

ΠŸΡ€ΠΈΠΌΠ΅Ρ€ 6. Найти Π·Π½Π°Ρ‡Π΅Π½ΠΈΠ΅ выраТСния

Π§Ρ‚ΠΎΠ±Ρ‹ Π²Ρ‹Ρ‡ΠΈΡΠ»ΠΈΡ‚ΡŒ Π΄Π°Π½Π½ΠΎΠ΅ Π²Ρ‹Ρ€Π°ΠΆΠ΅Π½ΠΈΠ΅, Π½ΡƒΠΆΠ½ΠΎ ΠΏΠ΅Ρ€Π²ΡƒΡŽ Π΄Ρ€ΠΎΠ±ΡŒ ΡƒΠΌΠ½ΠΎΠΆΠΈΡ‚ΡŒ Π½Π° число, ΠΎΠ±Ρ€Π°Ρ‚Π½ΠΎΠ΅ числу 4.

ΠžΠ±Ρ€Π°Ρ‚Π½ΠΎΠ΅ для числа 4 это Π΄Ρ€ΠΎΠ±ΡŒ . На Π½Π΅Ρ‘ ΠΈ ΡƒΠΌΠ½ΠΎΠΆΠΈΠΌ ΠΏΠ΅Ρ€Π²ΡƒΡŽ Π΄Ρ€ΠΎΠ±ΡŒ

ΠŸΡ€ΠΈΠΌΠ΅Ρ€ 5. Найти Π·Π½Π°Ρ‡Π΅Π½ΠΈΠ΅ выраТСния

Π§Ρ‚ΠΎΠ±Ρ‹ Π²Ρ‹Ρ‡ΠΈΡΠ»ΠΈΡ‚ΡŒ Π΄Π°Π½Π½ΠΎΠ΅ Π²Ρ‹Ρ€Π°ΠΆΠ΅Π½ΠΈΠ΅, Π½ΡƒΠΆΠ½ΠΎ ΠΏΠ΅Ρ€Π²ΡƒΡŽ Π΄Ρ€ΠΎΠ±ΡŒ ΡƒΠΌΠ½ΠΎΠΆΠΈΡ‚ΡŒ Π½Π° число, ΠΎΠ±Ρ€Π°Ρ‚Π½ΠΎΠ΅ числу βˆ’3

ΠžΠ±Ρ€Π°Ρ‚Π½ΠΎΠ΅ для числа βˆ’3 это Π΄Ρ€ΠΎΠ±ΡŒ . На Π½Π΅Ρ‘ ΠΈ ΡƒΠΌΠ½ΠΎΠΆΠΈΠΌ ΠΏΠ΅Ρ€Π²ΡƒΡŽ Π΄Ρ€ΠΎΠ±ΡŒ:

ΠŸΡ€ΠΈΠΌΠ΅Ρ€ 6. Найти Π·Π½Π°Ρ‡Π΅Π½ΠΈΠ΅ Π²Ρ‹Ρ€Π°ΠΆΠ΅Π½ΠΈΠ΅ βˆ’14,4: 1,8

Π­Ρ‚ΠΎ Π΄Π΅Π»Π΅Π½ΠΈΠ΅ Ρ€Π°Ρ†ΠΈΠΎΠ½Π°Π»ΡŒΠ½Ρ‹Ρ… чисСл с Ρ€Π°Π·Π½Ρ‹ΠΌΠΈ Π·Π½Π°ΠΊΠ°ΠΌΠΈ. Π§Ρ‚ΠΎΠ±Ρ‹ Π²Ρ‹Ρ‡ΠΈΡΠ»ΠΈΡ‚ΡŒ Π΄Π°Π½Π½ΠΎΠ΅ Π²Ρ‹Ρ€Π°ΠΆΠ΅Π½ΠΈΠ΅, Π½ΡƒΠΆΠ½ΠΎ ΠΌΠΎΠ΄ΡƒΠ»ΡŒ Π΄Π΅Π»ΠΈΠΌΠΎΠ³ΠΎ Ρ€Π°Π·Π΄Π΅Π»ΠΈΡ‚ΡŒ Π½Π° ΠΌΠΎΠ΄ΡƒΠ»ΡŒ дСлитСля ΠΈ ΠΏΠ΅Ρ€Π΅Π΄ ΠΏΠΎΠ»ΡƒΡ‡Π΅Π½Π½Ρ‹ΠΌ ΠΎΡ‚Π²Π΅Ρ‚ΠΎΠΌ ΠΏΠΎΡΡ‚Π°Π²ΠΈΡ‚ΡŒ минус

ΠžΠ±Ρ€Π°Ρ‚ΠΈΡ‚Π΅ Π²Π½ΠΈΠΌΠ°Π½ΠΈΠ΅, ΠΊΠ°ΠΊ ΠΌΠΎΠ΄ΡƒΠ»ΡŒ Π΄Π΅Π»ΠΈΠΌΠΎΠ³ΠΎ Π±Ρ‹Π» Ρ€Π°Π·Π΄Π΅Π»Ρ‘Π½ Π½Π° ΠΌΠΎΠ΄ΡƒΠ»ΡŒ дСлитСля. Π’ Π΄Π°Π½Π½ΠΎΠΌ случаС, Ρ‡Ρ‚ΠΎΠ±Ρ‹ ΡΠ΄Π΅Π»Π°Ρ‚ΡŒ это ΠΏΡ€Π°Π²ΠΈΠ»ΡŒΠ½ΠΎ, ΠΏΠΎΡ‚Ρ€Π΅Π±ΠΎΠ²Π°Π»ΠΎΡΡŒ ΡΡƒΠΌΠ΅Ρ‚ΡŒ .

Если Π½Π΅Ρ‚ ТСлания Π²ΠΎΠ·ΠΈΡ‚ΡŒΡΡ с дСсятичными дробями (Π° это Π±Ρ‹Π²Π°Π΅Ρ‚ часто), Ρ‚ΠΎ эти , Π·Π°Ρ‚Π΅ΠΌ пСрСвСсти эти ΡΠΌΠ΅ΡˆΠ°Π½Π½Ρ‹Π΅ числа Π² Π½Π΅ΠΏΡ€Π°Π²ΠΈΠ»ΡŒΠ½Ρ‹Π΅ Π΄Ρ€ΠΎΠ±ΠΈ, Π° Π·Π°Ρ‚Π΅ΠΌ Π·Π°Π½ΡΡ‚ΡŒΡΡ нСпосрСдствСнно Π΄Π΅Π»Π΅Π½ΠΈΠ΅ΠΌ.

Вычислим ΠΏΡ€Π΅Π΄Ρ‹Π΄ΡƒΡ‰Π΅Π΅ Π²Ρ‹Ρ€Π°ΠΆΠ΅Π½ΠΈΠ΅ βˆ’14,4: 1,8 этим способом. ΠŸΠ΅Ρ€Π΅Π²Π΅Π΄Ρ‘ΠΌ дСсятичныС Π΄Ρ€ΠΎΠ±ΠΈ Π² ΡΠΌΠ΅ΡˆΠ°Π½Π½Ρ‹Π΅ числа:

Π’Π΅ΠΏΠ΅Ρ€ΡŒ ΠΏΠ΅Ρ€Π΅Π²Π΅Π΄Ρ‘ΠΌ ΠΏΠΎΠ»ΡƒΡ‡Π΅Π½Π½Ρ‹Π΅ ΡΠΌΠ΅ΡˆΠ°Π½Π½Ρ‹Π΅ числа Π² Π½Π΅ΠΏΡ€Π°Π²ΠΈΠ»ΡŒΠ½Ρ‹Π΅ Π΄Ρ€ΠΎΠ±ΠΈ:

Π’Π΅ΠΏΠ΅Ρ€ΡŒ ΠΌΠΎΠΆΠ½ΠΎ Π·Π°Π½ΡΡ‚ΡŒΡΡ нСпосрСдствСнно Π΄Π΅Π»Π΅Π½ΠΈΠ΅ΠΌ, Π° ΠΈΠΌΠ΅Π½Π½ΠΎ Ρ€Π°Π·Π΄Π΅Π»ΠΈΡ‚ΡŒ Π΄Ρ€ΠΎΠ±ΡŒ Π½Π° Π΄Ρ€ΠΎΠ±ΡŒ . Для этого Π½ΡƒΠΆΠ½ΠΎ ΠΏΠ΅Ρ€Π²ΡƒΡŽ Π΄Ρ€ΠΎΠ±ΡŒ ΡƒΠΌΠ½ΠΎΠΆΠΈΡ‚ΡŒ Π½Π° Π΄Ρ€ΠΎΠ±ΡŒ, ΠΎΠ±Ρ€Π°Ρ‚Π½ΡƒΡŽ Π²Ρ‚ΠΎΡ€ΠΎΠΉ:

ΠŸΡ€ΠΈΠΌΠ΅Ρ€ 7. Найти Π·Π½Π°Ρ‡Π΅Π½ΠΈΠ΅ выраТСния

ΠŸΠ΅Ρ€Π΅Π²Π΅Π΄Ρ‘ΠΌ Π΄Π΅ΡΡΡ‚ΠΈΡ‡Π½ΡƒΡŽ Π΄Ρ€ΠΎΠ±ΡŒ βˆ’2,06 Π² Π½Π΅ΠΏΡ€Π°Π²ΠΈΠ»ΡŒΠ½ΡƒΡŽ Π΄Ρ€ΠΎΠ±ΡŒ, ΠΈ ΡƒΠΌΠ½ΠΎΠΆΠΈΠΌ эту Π΄Ρ€ΠΎΠ±ΡŒ Π½Π° Π΄Ρ€ΠΎΠ±ΡŒ, ΠΎΠ±Ρ€Π°Ρ‚Π½ΡƒΡŽ Π²Ρ‚ΠΎΡ€ΠΎΠΉ:

ΠœΠ½ΠΎΠ³ΠΎΡΡ‚Π°ΠΆΠ½Ρ‹Π΅ Π΄Ρ€ΠΎΠ±ΠΈ

Часто ΠΌΠΎΠΆΠ½ΠΎ Π²ΡΡ‚Ρ€Π΅Ρ‚ΠΈΡ‚ΡŒ Π²Ρ‹Ρ€Π°ΠΆΠ΅Π½ΠΈΠ΅, Π² ΠΊΠΎΡ‚ΠΎΡ€ΠΎΠΌ Π΄Π΅Π»Π΅Π½ΠΈΠ΅ Π΄Ρ€ΠΎΠ±Π΅ΠΉ записано с ΠΏΠΎΠΌΠΎΡ‰ΡŒΡŽ Π΄Ρ€ΠΎΠ±Π½ΠΎΠΉ Ρ‡Π΅Ρ€Ρ‚Ρ‹. НапримСр, Π²Ρ‹Ρ€Π°ΠΆΠ΅Π½ΠΈΠ΅ ΠΌΠΎΠΆΠ΅Ρ‚ Π±Ρ‹Ρ‚ΡŒ записано ΡΠ»Π΅Π΄ΡƒΡŽΡ‰ΠΈΠΌ ΠΎΠ±Ρ€Π°Π·ΠΎΠΌ:

Π’ Ρ‡Ρ‘ΠΌ ΠΆΠ΅ Ρ€Π°Π·Π½ΠΈΡ†Π° ΠΌΠ΅ΠΆΠ΄Ρƒ выраТСниями ΠΈ ? На самом Π΄Π΅Π»Π΅ Ρ€Π°Π·Π½ΠΈΡ†Ρ‹ Π½ΠΈΠΊΠ°ΠΊΠΎΠΉ. Π­Ρ‚ΠΈ Π΄Π²Π° выраТСния нСсут ΠΎΠ΄Π½ΠΎ ΠΈ Ρ‚ΠΎ ΠΆΠ΅ Π·Π½Π°Ρ‡Π΅Π½ΠΈΠ΅ ΠΈ ΠΌΠ΅ΠΆΠ΄Ρƒ Π½ΠΈΠΌΠΈ ΠΌΠΎΠΆΠ½ΠΎ ΠΏΠΎΡΡ‚Π°Π²ΠΈΡ‚ΡŒ Π·Π½Π°ΠΊ равСнства:

Π’ ΠΏΠ΅Ρ€Π²ΠΎΠΌ случаС Π·Π½Π°ΠΊ дСлСния прСдставляСт собой Π΄Π²ΠΎΠ΅Ρ‚ΠΎΡ‡ΠΈΠ΅ ΠΈ Π²Ρ‹Ρ€Π°ΠΆΠ΅Π½ΠΈΠ΅ записано Π² ΠΎΠ΄Π½Ρƒ строку. Π’ΠΎ Π²Ρ‚ΠΎΡ€ΠΎΠΌ случаС Π΄Π΅Π»Π΅Π½ΠΈΠ΅ Π΄Ρ€ΠΎΠ±Π΅ΠΉ записано с ΠΏΠΎΠΌΠΎΡ‰ΡŒΡŽ Π΄Ρ€ΠΎΠ±Π½ΠΎΠΉ Ρ‡Π΅Ρ€Ρ‚Ρ‹. Π’ Ρ€Π΅Π·ΡƒΠ»ΡŒΡ‚Π°Ρ‚Π΅ получаСтся Π΄Ρ€ΠΎΠ±ΡŒ, ΠΊΠΎΡ‚ΠΎΡ€ΡƒΡŽ Π² Π½Π°Ρ€ΠΎΠ΄Π΅ Π΄ΠΎΠ³ΠΎΠ²ΠΎΡ€ΠΈΠ»ΠΈΡΡŒ Π½Π°Π·Ρ‹Π²Π°Ρ‚ΡŒ многоэтаТной .

ΠŸΡ€ΠΈ встрСчС с Ρ‚Π°ΠΊΠΈΠΌΠΈ многоэтаТными выраТСниями, Π½ΡƒΠΆΠ½ΠΎ ΠΏΡ€ΠΈΠΌΠ΅Π½ΡΡ‚ΡŒ Ρ‚Π΅ ΠΆΠ΅ ΠΏΡ€Π°Π²ΠΈΠ»Π° дСлСния ΠΎΠ±Ρ‹ΠΊΠ½ΠΎΠ²Π΅Π½Π½Ρ‹Ρ… Π΄Ρ€ΠΎΠ±Π΅ΠΉ. ΠŸΠ΅Ρ€Π²ΡƒΡŽ Π΄Ρ€ΠΎΠ±ΡŒ Π½Π΅ΠΎΠ±Ρ…ΠΎΠ΄ΠΈΠΌΠΎ ΡƒΠΌΠ½ΠΎΠΆΠ°Ρ‚ΡŒ Π½Π° Π΄Ρ€ΠΎΠ±ΡŒ, ΠΎΠ±Ρ€Π°Ρ‚Π½ΡƒΡŽ Π²Ρ‚ΠΎΡ€ΠΎΠΉ.

Π˜ΡΠΏΠΎΠ»ΡŒΠ·ΠΎΠ²Π°Ρ‚ΡŒ Π² Ρ€Π΅ΡˆΠ΅Π½ΠΈΠΈ ΠΏΠΎΠ΄ΠΎΠ±Π½Ρ‹Π΅ Π΄Ρ€ΠΎΠ±ΠΈ ΠΊΡ€Π°ΠΉΠ½Π΅ Π½Π΅ΡƒΠ΄ΠΎΠ±Π½ΠΎ, поэтому ΠΌΠΎΠΆΠ½ΠΎ Π·Π°ΠΏΠΈΡΠ°Ρ‚ΡŒ ΠΈΡ… Π² понятном Π²ΠΈΠ΄Π΅, ΠΈΡΠΏΠΎΠ»ΡŒΠ·ΡƒΡ Π² качСствС Π·Π½Π°ΠΊΠ° дСлСния Π½Π΅ Π΄Ρ€ΠΎΠ±Π½ΡƒΡŽ Ρ‡Π΅Ρ€Ρ‚Ρƒ, Π° Π΄Π²ΠΎΠ΅Ρ‚ΠΎΡ‡ΠΈΠ΅.

НапримСр, запишСм ΠΌΠ½ΠΎΠ³ΠΎΡΡ‚Π°ΠΆΠ½ΡƒΡŽ Π΄Ρ€ΠΎΠ±ΡŒ Π² понятном Π²ΠΈΠ΄Π΅. Для этого сначала Π½ΡƒΠΆΠ½ΠΎ Ρ€Π°Π·ΠΎΠ±Ρ€Π°Ρ‚ΡŒΡΡ, Π³Π΄Π΅ пСрвая Π΄Ρ€ΠΎΠ±ΡŒ ΠΈ Π³Π΄Π΅ вторая, ΠΏΠΎΡ‚ΠΎΠΌΡƒ Ρ‡Ρ‚ΠΎ ΡΠ΄Π΅Π»Π°Ρ‚ΡŒ это ΠΏΡ€Π°Π²ΠΈΠ»ΡŒΠ½ΠΎ удаётся Π½Π΅ всСгда. Π’ многоэтаТных дробях имССтся нСсколько Π΄Ρ€ΠΎΠ±Π½Ρ‹Ρ… Ρ‡Π΅Ρ€Ρ‚, ΠΊΠΎΡ‚ΠΎΡ€Ρ‹Π΅ ΠΌΠΎΠ³ΡƒΡ‚ Π·Π°ΠΏΡƒΡ‚Π°Ρ‚ΡŒ. Главная дробная Ρ‡Π΅Ρ€Ρ‚Π°, которая отдСляСт ΠΏΠ΅Ρ€Π²ΡƒΡŽ Π΄Ρ€ΠΎΠ±ΡŒ ΠΎΡ‚ Π²Ρ‚ΠΎΡ€ΠΎΠΉ, ΠΎΠ±Ρ‹Ρ‡Π½ΠΎ Π±Ρ‹Π²Π°Π΅Ρ‚ Π΄Π»ΠΈΠ½Π½Π΅Π΅ ΠΎΡΡ‚Π°Π»ΡŒΠ½Ρ‹Ρ….

ПослС опрСдСлСния Π³Π»Π°Π²Π½ΠΎΠΉ Π΄Ρ€ΠΎΠ±Π½ΠΎΠΉ Ρ‡Π΅Ρ€Ρ‚Ρ‹ ΠΌΠΎΠΆΠ½ΠΎ Π±Π΅Π· Ρ‚Ρ€ΡƒΠ΄Π° ΠΏΠΎΠ½ΡΡ‚ΡŒ, Π³Π΄Π΅ пСрвая Π΄Ρ€ΠΎΠ±ΡŒ ΠΈ Π³Π΄Π΅ вторая:

ΠŸΡ€ΠΈΠΌΠ΅Ρ€ 2.

Находим Π³Π»Π°Π²Π½ΡƒΡŽ Π΄Ρ€ΠΎΠ±Π½ΡƒΡŽ Ρ‡Π΅Ρ€Ρ‚Ρƒ (ΠΎΠ½Π° самая длинная) ΠΈ Π²ΠΈΠ΄ΠΈΠΌ, Ρ‡Ρ‚ΠΎ осущСствляСтся Π΄Π΅Π»Π΅Π½ΠΈΠ΅ Ρ†Π΅Π»ΠΎΠ³ΠΎ числа βˆ’3 Π½Π° ΠΎΠ±Ρ‹ΠΊΠ½ΠΎΠ²Π΅Π½Π½ΡƒΡŽ Π΄Ρ€ΠΎΠ±ΡŒ

А Ссли Π±Ρ‹ ΠΌΡ‹ ΠΏΠΎ ошибкС приняли Π²Ρ‚ΠΎΡ€ΡƒΡŽ Π΄Ρ€ΠΎΠ±Π½ΡƒΡŽ Ρ‡Π΅Ρ€Ρ‚Ρƒ Π·Π° Π³Π»Π°Π²Π½ΡƒΡŽ (Ρ‚Ρƒ, Ρ‡Ρ‚ΠΎ ΠΊΠΎΡ€ΠΎΡ‡Π΅), Ρ‚ΠΎ ΠΏΠΎΠ»ΡƒΡ‡ΠΈΠ»ΠΎΡΡŒ Π±Ρ‹, Ρ‡Ρ‚ΠΎ ΠΌΡ‹ Π΄Π΅Π»ΠΈΠΌ Π΄Ρ€ΠΎΠ±ΡŒ Π½Π° Ρ†Π΅Π»ΠΎΠ΅ число 5Π’ этом случаС, Π΄Π°ΠΆΠ΅ Ссли это Π²Ρ‹Ρ€Π°ΠΆΠ΅Π½ΠΈΠ΅ Π²Ρ‹Ρ‡ΠΈΡΠ»ΠΈΡ‚ΡŒ Π²Π΅Ρ€Π½ΠΎ, Π·Π°Π΄Π°Ρ‡Π° Π±ΡƒΠ΄Π΅Ρ‚ Ρ€Π΅ΡˆΠ΅Π½Π° Π½Π΅ΠΏΡ€Π°Π²ΠΈΠ»ΡŒΠ½ΠΎ, ΠΏΠΎΡΠΊΠΎΠ»ΡŒΠΊΡƒ Π΄Π΅Π»ΠΈΠΌΡ‹ΠΌ Π² Π΄Π°Π½Π½ΠΎΠΌ случаС являСтся число βˆ’3, Π° Π΄Π΅Π»ΠΈΡ‚Π΅Π»Π΅ΠΌ β€” Π΄Ρ€ΠΎΠ±ΡŒ .

ΠŸΡ€ΠΈΠΌΠ΅Ρ€ 3. Π—Π°ΠΏΠΈΡˆΠ΅ΠΌ Π² понятном Π²ΠΈΠ΄Π΅ ΠΌΠ½ΠΎΠ³ΠΎΡΡ‚Π°ΠΆΠ½ΡƒΡŽ Π΄Ρ€ΠΎΠ±ΡŒ

Находим Π³Π»Π°Π²Π½ΡƒΡŽ Π΄Ρ€ΠΎΠ±Π½ΡƒΡŽ Ρ‡Π΅Ρ€Ρ‚Ρƒ (ΠΎΠ½Π° самая длинная) ΠΈ Π²ΠΈΠ΄ΠΈΠΌ, Ρ‡Ρ‚ΠΎ осущСствляСтся Π΄Π΅Π»Π΅Π½ΠΈΠ΅ Π΄Ρ€ΠΎΠ±ΠΈ Π½Π° Ρ†Π΅Π»ΠΎΠ΅ число 2

А Ссли Π±Ρ‹ ΠΌΡ‹ ΠΏΠΎ ошибкС приняли ΠΏΠ΅Ρ€Π²ΡƒΡŽ Π΄Ρ€ΠΎΠ±Π½ΡƒΡŽ Ρ‡Π΅Ρ€Ρ‚Ρƒ Π·Π° Π³Π»Π°Π²Π½ΡƒΡŽ (Ρ‚Ρƒ, Ρ‡Ρ‚ΠΎ ΠΊΠΎΡ€ΠΎΡ‡Π΅), Ρ‚ΠΎ ΠΏΠΎΠ»ΡƒΡ‡ΠΈΠ»ΠΎΡΡŒ Π±Ρ‹, Ρ‡Ρ‚ΠΎ ΠΌΡ‹ Π΄Π΅Π»ΠΈΠΌ Ρ†Π΅Π»ΠΎΠ΅ число βˆ’5 Π½Π° Π΄Ρ€ΠΎΠ±ΡŒ Π’ этом случаС, Π΄Π°ΠΆΠ΅ Ссли это Π²Ρ‹Ρ€Π°ΠΆΠ΅Π½ΠΈΠ΅ Π²Ρ‹Ρ‡ΠΈΡΠ»ΠΈΡ‚ΡŒ Π²Π΅Ρ€Π½ΠΎ, Π·Π°Π΄Π°Ρ‡Π° Π±ΡƒΠ΄Π΅Ρ‚ Ρ€Π΅ΡˆΠ΅Π½Π° Π½Π΅ΠΏΡ€Π°Π²ΠΈΠ»ΡŒΠ½ΠΎ, ΠΏΠΎΡΠΊΠΎΠ»ΡŒΠΊΡƒ Π΄Π΅Π»ΠΈΠΌΡ‹ΠΌ Π² Π΄Π°Π½Π½ΠΎΠΌ случаС являСтся Π΄Ρ€ΠΎΠ±ΡŒ , Π° Π΄Π΅Π»ΠΈΡ‚Π΅Π»Π΅ΠΌ β€” Ρ†Π΅Π»ΠΎΠ΅ число 2.

НСсмотря Π½Π° Ρ‚ΠΎ, Ρ‡Ρ‚ΠΎ многоэтаТныС Π΄Ρ€ΠΎΠ±ΠΈ Π½Π΅ΡƒΠ΄ΠΎΠ±Π½Ρ‹ Π² Ρ€Π°Π±ΠΎΡ‚Π΅, ΡΡ‚Π°Π»ΠΊΠΈΠ²Π°Ρ‚ΡŒΡΡ ΠΌΡ‹ с Π½ΠΈΠΌΠΈ Π±ΡƒΠ΄Π΅ΠΌ ΠΎΡ‡Π΅Π½ΡŒ часто, особСнно ΠΏΡ€ΠΈ ΠΈΠ·ΡƒΡ‡Π΅Π½ΠΈΠΈ Π²Ρ‹ΡΡˆΠ΅ΠΉ ΠΌΠ°Ρ‚Π΅ΠΌΠ°Ρ‚ΠΈΠΊΠΈ.

ЕстСствСнно, Π½Π° ΠΏΠ΅Ρ€Π΅Π²ΠΎΠ΄ многоэтаТной Π΄Ρ€ΠΎΠ±ΠΈ Π² понятный Π²ΠΈΠ΄ ΡƒΡ…ΠΎΠ΄ΠΈΡ‚ Π΄ΠΎΠΏΠΎΠ»Π½ΠΈΡ‚Π΅Π»ΡŒΠ½ΠΎΠ΅ врСмя ΠΈ мСсто. ΠŸΠΎΡΡ‚ΠΎΠΌΡƒ ΠΌΠΎΠΆΠ½ΠΎ Π²ΠΎΡΠΏΠΎΠ»ΡŒΠ·ΠΎΠ²Π°Ρ‚ΡŒΡΡ Π±ΠΎΠ»Π΅Π΅ быстрым ΠΌΠ΅Ρ‚ΠΎΠ΄ΠΎΠΌ. Π”Π°Π½Π½Ρ‹ΠΉ ΠΌΠ΅Ρ‚ΠΎΠ΄ ΡƒΠ΄ΠΎΠ±Π΅Π½ ΠΈ Π½Π° Π²Ρ‹Ρ…ΠΎΠ΄Π΅ позволяСт ΠΏΠΎΠ»ΡƒΡ‡ΠΈΡ‚ΡŒ Π³ΠΎΡ‚ΠΎΠ²ΠΎΠ΅ Π²Ρ‹Ρ€Π°ΠΆΠ΅Π½ΠΈΠ΅, Π² ΠΊΠΎΡ‚ΠΎΡ€ΠΎΠΌ пСрвая Π΄Ρ€ΠΎΠ±ΡŒ ΡƒΠΆΠ΅ ΡƒΠΌΠ½ΠΎΠΆΠ΅Π½Π° Π½Π° Π΄Ρ€ΠΎΠ±ΡŒ, ΠΎΠ±Ρ€Π°Ρ‚Π½ΡƒΡŽ Π²Ρ‚ΠΎΡ€ΠΎΠΉ.

РСализуСтся этот ΠΌΠ΅Ρ‚ΠΎΠ΄ ΡΠ»Π΅Π΄ΡƒΡŽΡ‰ΠΈΠΌ ΠΎΠ±Ρ€Π°Π·ΠΎΠΌ:

Если Π΄Ρ€ΠΎΠ±ΡŒ чСтырСхэтаТная, Π½Π°ΠΏΡ€ΠΈΠΌΠ΅Ρ€ ΠΊΠ°ΠΊ , Ρ‚ΠΎ Ρ†ΠΈΡ„Ρ€Ρƒ Π½Π°Ρ…ΠΎΠ΄ΡΡ‰ΡƒΡŽΡΡ Π½Π° ΠΏΠ΅Ρ€Π²ΠΎΠΌ этаТС ΠΏΠΎΠ΄Π½ΠΈΠΌΠ°ΡŽΡ‚ Π½Π° самый Π²Π΅Ρ€Ρ…Π½ΠΈΠΉ этаТ. А Ρ†ΠΈΡ„Ρ€Ρƒ, Π½Π°Ρ…ΠΎΠ΄ΡΡ‰ΡƒΡŽΡΡ Π½Π° Π²Ρ‚ΠΎΡ€ΠΎΠΌ этаТС ΠΏΠΎΠ΄Π½ΠΈΠΌΠ°ΡŽΡ‚ Π½Π° Ρ‚Ρ€Π΅Ρ‚ΠΈΠΉ этаТ. ΠŸΠΎΠ»ΡƒΡ‡Π΅Π½Π½Ρ‹Π΅ Ρ†ΠΈΡ„Ρ€Ρ‹ Π½ΡƒΠΆΠ½ΠΎ ΡΠΎΠ΅Π΄ΠΈΠ½ΠΈΡ‚ΡŒ Π·Π½Π°Ρ‡ΠΊΠ°ΠΌΠΈ умноТСния (Γ—)

Π’ Ρ€Π΅Π·ΡƒΠ»ΡŒΡ‚Π°Ρ‚Π΅, минуя ΠΏΡ€ΠΎΠΌΠ΅ΠΆΡƒΡ‚ΠΎΡ‡Π½ΡƒΡŽ запись ΠΌΡ‹ ΠΏΠΎΠ»ΡƒΡ‡Π°Π΅ΠΌ Π½ΠΎΠ²ΠΎΠ΅ Π²Ρ‹Ρ€Π°ΠΆΠ΅Π½ΠΈΠ΅ , Π² ΠΊΠΎΡ‚ΠΎΡ€ΠΎΠΌ пСрвая Π΄Ρ€ΠΎΠ±ΡŒ ΡƒΠΆΠ΅ ΡƒΠΌΠ½ΠΎΠΆΠ΅Π½Π° Π½Π° Π΄Ρ€ΠΎΠ±ΡŒ, ΠΎΠ±Ρ€Π°Ρ‚Π½ΡƒΡŽ Π²Ρ‚ΠΎΡ€ΠΎΠΉ. Удобство Π΄Π° ΠΈ Ρ‚ΠΎΠ»ΡŒΠΊΠΎ!

Π§Ρ‚ΠΎΠ±Ρ‹ Π½Π΅ Π΄ΠΎΠΏΡƒΡΠΊΠ°Ρ‚ΡŒ ошибок ΠΏΡ€ΠΈ использовании Π΄Π°Π½Π½ΠΎΠ³ΠΎ ΠΌΠ΅Ρ‚ΠΎΠ΄Π°, ΠΌΠΎΠΆΠ½ΠΎ Ρ€ΡƒΠΊΠΎΠ²ΠΎΠ΄ΡΡ‚Π²ΠΎΠ²Π°Ρ‚ΡŒΡΡ ΡΠ»Π΅Π΄ΡƒΡŽΡ‰ΠΈΠΌ ΠΏΡ€Π°Π²ΠΈΠ»ΠΎΠΌ:

Π‘ ΠΏΠ΅Ρ€Π²ΠΎΠ³ΠΎ Π½Π° Ρ‡Π΅Ρ‚Π²Ρ‘Ρ€Ρ‚Ρ‹ΠΉ. Π‘ΠΎ Π²Ρ‚ΠΎΡ€ΠΎΠ³ΠΎ Π½Π° Ρ‚Ρ€Π΅Ρ‚ΠΈΠΉ.

Π’ ΠΏΡ€Π°Π²ΠΈΠ»Π΅ Ρ€Π΅Ρ‡ΡŒ ΠΈΠ΄Π΅Ρ‚ ΠΎΠ± этаТах. Π¦ΠΈΡ„Ρ€Ρƒ с ΠΏΠ΅Ρ€Π²ΠΎΠ³ΠΎ этаТа Π½ΡƒΠΆΠ½ΠΎ ΠΏΠΎΠ΄Π½ΠΈΠΌΠ°Ρ‚ΡŒ Π½Π° Ρ‡Π΅Ρ‚Π²Π΅Ρ€Ρ‚Ρ‹ΠΉ этаТ. А Ρ†ΠΈΡ„Ρ€Ρƒ со Π²Ρ‚ΠΎΡ€ΠΎΠ³ΠΎ этаТа Π½ΡƒΠΆΠ½ΠΎ ΠΏΠΎΠ΄Π½ΠΈΠΌΠ°Ρ‚ΡŒ Π½Π° Ρ‚Ρ€Π΅Ρ‚ΠΈΠΉ этаТ.

ΠŸΠΎΠΏΡ€ΠΎΠ±ΡƒΠ΅ΠΌ Π²Ρ‹Ρ‡ΠΈΡΠ»ΠΈΡ‚ΡŒ ΠΌΠ½ΠΎΠ³ΠΎΡΡ‚Π°ΠΆΠ½ΡƒΡŽ Π΄Ρ€ΠΎΠ±ΡŒ ΠΏΠΎΠ»ΡŒΠ·ΡƒΡΡΡŒ Π²Ρ‹ΡˆΠ΅ΠΏΡ€ΠΈΠ²Π΅Π΄Ρ‘Π½Π½Ρ‹ΠΌ ΠΏΡ€Π°Π²ΠΈΠ»ΠΎΠΌ.

Π˜Ρ‚Π°ΠΊ, Ρ†ΠΈΡ„Ρ€Ρƒ Π½Π°Ρ…ΠΎΠ΄ΡΡ‰ΡƒΡŽΡΡ Π½Π° ΠΏΠ΅Ρ€Π²ΠΎΠΌ этаТС ΠΏΠΎΠ΄Π½ΠΈΠΌΠ°Π΅ΠΌ Π½Π° Ρ‡Π΅Ρ‚Π²Ρ‘Ρ€Ρ‚Ρ‹ΠΉ этаТ, Π° Ρ†ΠΈΡ„Ρ€Ρƒ Π½Π°Ρ…ΠΎΠ΄ΡΡ‰ΡƒΡŽΡΡ Π½Π° Π²Ρ‚ΠΎΡ€ΠΎΠΌ этаТС ΠΏΠΎΠ΄Π½ΠΈΠΌΠ°Π΅ΠΌ Π½Π° Ρ‚Ρ€Π΅Ρ‚ΠΈΠΉ этаТ

Π’ Ρ€Π΅Π·ΡƒΠ»ΡŒΡ‚Π°Ρ‚Π΅, минуя ΠΏΡ€ΠΎΠΌΠ΅ΠΆΡƒΡ‚ΠΎΡ‡Π½ΡƒΡŽ запись ΠΌΡ‹ ΠΏΠΎΠ»ΡƒΡ‡Π°Π΅ΠΌ Π½ΠΎΠ²ΠΎΠ΅ Π²Ρ‹Ρ€Π°ΠΆΠ΅Π½ΠΈΠ΅ , Π² ΠΊΠΎΡ‚ΠΎΡ€ΠΎΠΌ пСрвая Π΄Ρ€ΠΎΠ±ΡŒ ΡƒΠΆΠ΅ ΡƒΠΌΠ½ΠΎΠΆΠ΅Π½Π° Π½Π° Π΄Ρ€ΠΎΠ±ΡŒ, ΠΎΠ±Ρ€Π°Ρ‚Π½ΠΎΠΉ Π²Ρ‚ΠΎΡ€ΠΎΠΉ. Π”Π°Π»Π΅Π΅ ΠΌΠΎΠΆΠ½ΠΎ Π²ΠΎΡΠΏΠΎΠ»ΡŒΠ·ΠΎΠ²Π°Ρ‚ΡŒΡΡ ΠΈΠΌΠ΅ΡŽΡ‰ΠΈΠΌΠΈΡΡ знаниями:

ΠŸΠΎΠΏΡ€ΠΎΠ±ΡƒΠ΅ΠΌ Π²Ρ‹Ρ‡ΠΈΡΠ»ΠΈΡ‚ΡŒ ΠΌΠ½ΠΎΠ³ΠΎΡΡ‚Π°ΠΆΠ½ΡƒΡŽ Π΄Ρ€ΠΎΠ±ΡŒ ΠΏΠΎΠ»ΡŒΠ·ΡƒΡΡΡŒ Π½ΠΎΠ²ΠΎΠΉ схСмой.

Π—Π΄Π΅ΡΡŒ имССтся Ρ‚ΠΎΠ»ΡŒΠΊΠΎ ΠΏΠ΅Ρ€Π²Ρ‹ΠΉ, Π²Ρ‚ΠΎΡ€ΠΎΠΉ ΠΈ Ρ‡Π΅Ρ‚Π²Ρ‘Ρ€Ρ‚Ρ‹ΠΉ этаТи. Π’Ρ€Π΅Ρ‚ΠΈΠΉ этаТ отсутствуСт. Но ΠΌΡ‹ Π½Π΅ ΠΎΡ‚Ρ…ΠΎΠ΄ΠΈΠΌ ΠΎΡ‚ основной схСмы: Ρ†ΠΈΡ„Ρ€Ρƒ с ΠΏΠ΅Ρ€Π²ΠΎΠ³ΠΎ этаТа ΠΏΠΎΠ΄Π½ΠΈΠΌΠ°Π΅ΠΌ Π½Π° Ρ‡Π΅Ρ‚Π²Ρ‘Ρ€Ρ‚Ρ‹ΠΉ этаТ. А ΠΏΠΎΡΠΊΠΎΠ»ΡŒΠΊΡƒ Ρ‚Ρ€Π΅Ρ‚ΠΈΠΉ этаТ отсутствуСт, Ρ‚ΠΎ Ρ†ΠΈΡ„Ρ€Ρƒ Π½Π°Ρ…ΠΎΠ΄ΡΡ‰ΡƒΡŽΡΡ Π½Π° Π²Ρ‚ΠΎΡ€ΠΎΠΌ этаТС оставляСм, ΠΊΠ°ΠΊ Π΅ΡΡ‚ΡŒ

Π’ Ρ€Π΅Π·ΡƒΠ»ΡŒΡ‚Π°Ρ‚Π΅, минуя ΠΏΡ€ΠΎΠΌΠ΅ΠΆΡƒΡ‚ΠΎΡ‡Π½ΡƒΡŽ запись ΠΌΡ‹ ΠΏΠΎΠ»ΡƒΡ‡ΠΈΠ»ΠΈ Π½ΠΎΠ²ΠΎΠ΅ Π²Ρ‹Ρ€Π°ΠΆΠ΅Π½ΠΈΠ΅ , Π² ΠΊΠΎΡ‚ΠΎΡ€ΠΎΠΌ ΠΏΠ΅Ρ€Π²ΠΎΠ΅ число βˆ’3 ΡƒΠΆΠ΅ ΡƒΠΌΠ½ΠΎΠΆΠ΅Π½ΠΎ Π½Π° Π΄Ρ€ΠΎΠ±ΡŒ, ΠΎΠ±Ρ€Π°Ρ‚Π½ΡƒΡŽ Π²Ρ‚ΠΎΡ€ΠΎΠΉ. Π”Π°Π»Π΅Π΅ ΠΌΠΎΠΆΠ½ΠΎ Π²ΠΎΡΠΏΠΎΠ»ΡŒΠ·ΠΎΠ²Π°Ρ‚ΡŒΡΡ ΠΈΠΌΠ΅ΡŽΡ‰ΠΈΠΌΠΈΡΡ знаниями:

ΠŸΠΎΠΏΡ€ΠΎΠ±ΡƒΠ΅ΠΌ Π²Ρ‹Ρ‡ΠΈΡΠ»ΠΈΡ‚ΡŒ ΠΌΠ½ΠΎΠ³ΠΎΡΡ‚Π°ΠΆΠ½ΡƒΡŽ Π΄Ρ€ΠΎΠ±ΡŒ , ΠΏΠΎΠ»ΡŒΠ·ΡƒΡΡΡŒ Π½ΠΎΠ²ΠΎΠΉ схСмой.

Π—Π΄Π΅ΡΡŒ имССтся Ρ‚ΠΎΠ»ΡŒΠΊΠΎ Π²Ρ‚ΠΎΡ€ΠΎΠΉ, Ρ‚Ρ€Π΅Ρ‚ΠΈΠΉ ΠΈ Ρ‡Π΅Ρ‚Π²Ρ‘Ρ€Ρ‚Ρ‹ΠΉ этаТи. ΠŸΠ΅Ρ€Π²Ρ‹ΠΉ этаТ отсутствуСт. ΠŸΠΎΡΠΊΠΎΠ»ΡŒΠΊΡƒ ΠΏΠ΅Ρ€Π²Ρ‹ΠΉ этаТ отсутствуСт, ΠΏΠΎΠ΄Π½ΠΈΠΌΠ°Ρ‚ΡŒΡΡ Π½Π° Ρ‡Π΅Ρ‚Π²Ρ‘Ρ€Ρ‚Ρ‹ΠΉ этаТ Π½Π΅Ρ‡Π΅ΠΌΡƒ, Π½ΠΎ Π·Π°Ρ‚ΠΎ ΠΌΡ‹ ΠΌΠΎΠΆΠ΅ΠΌ ΠΏΠΎΠ΄Π½ΡΡ‚ΡŒ Ρ†ΠΈΡ„Ρ€Ρƒ со Π²Ρ‚ΠΎΡ€ΠΎΠ³ΠΎ этаТа Π½Π° Ρ‚Ρ€Π΅Ρ‚ΠΈΠΉ:

Π’ Ρ€Π΅Π·ΡƒΠ»ΡŒΡ‚Π°Ρ‚Π΅, минуя ΠΏΡ€ΠΎΠΌΠ΅ΠΆΡƒΡ‚ΠΎΡ‡Π½ΡƒΡŽ запись ΠΌΡ‹ ΠΏΠΎΠ»ΡƒΡ‡ΠΈΠ»ΠΈ Π½ΠΎΠ²ΠΎΠ΅ Π²Ρ‹Ρ€Π°ΠΆΠ΅Π½ΠΈΠ΅ , Π² ΠΊΠΎΡ‚ΠΎΡ€ΠΎΠΌ пСрвая Π΄Ρ€ΠΎΠ±ΡŒ ΡƒΠΆΠ΅ ΡƒΠΌΠ½ΠΎΠΆΠ΅Π½Π° Π½Π° число, ΠΎΠ±Ρ€Π°Ρ‚Π½ΠΎΠ΅ Π΄Π΅Π»ΠΈΡ‚Π΅Π»ΡŽ. Π”Π°Π»Π΅Π΅ ΠΌΠΎΠΆΠ½ΠΎ Π²ΠΎΡΠΏΠΎΠ»ΡŒΠ·ΠΎΠ²Π°Ρ‚ΡŒΡΡ ΠΈΠΌΠ΅ΡŽΡ‰ΠΈΠΌΠΈΡΡ знаниями:

ИспользованиС ΠΏΠ΅Ρ€Π΅ΠΌΠ΅Π½Π½Ρ‹Ρ…

Если Π²Ρ‹Ρ€Π°ΠΆΠ΅Π½ΠΈΠ΅ слоТноС ΠΈ Π²Π°ΠΌ каТСтся, Ρ‡Ρ‚ΠΎ ΠΎΠ½ΠΎ Π·Π°ΠΏΡƒΡ‚Π°Π΅Ρ‚ вас Π² процСссС Ρ€Π΅ΡˆΠ΅Π½ΠΈΡ Π·Π°Π΄Π°Ρ‡ΠΈ, Ρ‚ΠΎ Ρ‡Π°ΡΡ‚ΡŒ выраТСния ΠΌΠΎΠΆΠ½ΠΎ занСсти Π² ΠΏΠ΅Ρ€Π΅ΠΌΠ΅Π½Π½ΡƒΡŽ ΠΈ Π΄Π°Π»Π΅Π΅ Ρ€Π°Π±ΠΎΡ‚Π°Ρ‚ΡŒ с этой ΠΏΠ΅Ρ€Π΅ΠΌΠ΅Π½Π½ΠΎΠΉ.

ΠœΠ°Ρ‚Π΅ΠΌΠ°Ρ‚ΠΈΠΊΠΈ часто Ρ‚Π°ΠΊ ΠΈ Π΄Π΅Π»Π°ΡŽΡ‚. Π‘Π»ΠΎΠΆΠ½ΡƒΡŽ Π·Π°Π΄Π°Ρ‡Ρƒ Ρ€Π°Π·Π±ΠΈΠ²Π°ΡŽΡ‚ Π½Π° Π±ΠΎΠ»Π΅Π΅ Π»Ρ‘Π³ΠΊΠΈΠ΅ ΠΏΠΎΠ΄Π·Π°Π΄Π°Ρ‡ΠΈ ΠΈ Ρ€Π΅ΡˆΠ°ΡŽΡ‚ ΠΈΡ…. Π—Π°Ρ‚Π΅ΠΌ ΡΠΎΠ±ΠΈΡ€Π°ΡŽΡ‚ Ρ€Π΅ΡˆΡ‘Π½Π½Ρ‹Π΅ ΠΏΠΎΠ΄Π·Π°Π΄Π°Ρ‡ΠΈ Π² ΠΎΠ΄Π½ΠΎ Π΅Π΄ΠΈΠ½ΠΎΠ΅ Ρ†Π΅Π»ΠΎΠ΅. Π­Ρ‚ΠΎ творчСский процСсс ΠΈ этому учатся Π³ΠΎΠ΄Π°ΠΌΠΈ, ΡƒΠΏΠΎΡ€Π½ΠΎ Ρ‚Ρ€Π΅Π½ΠΈΡ€ΡƒΡΡΡŒ.

ИспользованиС ΠΏΠ΅Ρ€Π΅ΠΌΠ΅Π½Π½Ρ‹Ρ… ΠΎΠΏΡ€Π°Π²Π΄Π°Π½ΠΎ, ΠΏΡ€ΠΈ Ρ€Π°Π±ΠΎΡ‚Π΅ с многоэтаТными дробями. НапримСр:

Найти Π·Π½Π°Ρ‡Π΅Π½ΠΈΠ΅ выраТСния

Π˜Ρ‚Π°ΠΊ, имССтся Π΄Ρ€ΠΎΠ±Π½ΠΎΠ΅ Π²Ρ‹Ρ€Π°ΠΆΠ΅Π½ΠΈΠ΅ Π² числитСлС ΠΈ Π² Π·Π½Π°ΠΌΠ΅Π½Π°Ρ‚Π΅Π»Π΅ ΠΊΠΎΡ‚ΠΎΡ€ΠΎΠΌ Π΄Ρ€ΠΎΠ±Π½Ρ‹Π΅ выраТСния. Π”Ρ€ΡƒΠ³ΠΈΠΌΠΈ словами, ΠΏΠ΅Ρ€Π΅Π΄ Π½Π°ΠΌΠΈ снова многоэтаТная Π΄Ρ€ΠΎΠ±ΡŒ, ΠΊΠΎΡ‚ΠΎΡ€ΡƒΡŽ ΠΌΡ‹ Ρ‚Π°ΠΊ Π½Π΅ любим.

Π’Ρ‹Ρ€Π°ΠΆΠ΅Π½ΠΈΠ΅, находящССся Π² числитСлС ΠΌΠΎΠΆΠ½ΠΎ занСсти Π² ΠΏΠ΅Ρ€Π΅ΠΌΠ΅Π½Π½ΡƒΡŽ с Π»ΡŽΠ±Ρ‹ΠΌ Π½Π°Π·Π²Π°Π½ΠΈΠ΅ΠΌ, Π½Π°ΠΏΡ€ΠΈΠΌΠ΅Ρ€:

Но Π² ΠΌΠ°Ρ‚Π΅ΠΌΠ°Ρ‚ΠΈΠΊΠ΅ Π² ΠΏΠΎΠ΄ΠΎΠ±Π½ΠΎΠΌ случаС ΠΏΠ΅Ρ€Π΅ΠΌΠ΅Π½Π½Ρ‹ΠΌ принято Π΄Π°Π²Π°Ρ‚ΡŒ Π½Π°Π·Π²Π°Π½ΠΈΠ΅ ΠΈΠ· Π±ΠΎΠ»ΡŒΡˆΠΈΡ… латинских Π±ΡƒΠΊΠ². Π”Π°Π²Π°ΠΉΡ‚Π΅ Π½Π΅ Π±ΡƒΠ΄Π΅ΠΌ Π½Π°Ρ€ΡƒΡˆΠ°Ρ‚ΡŒ эту Ρ‚Ρ€Π°Π΄ΠΈΡ†ΠΈΡŽ, ΠΈ ΠΎΠ±ΠΎΠ·Π½Π°Ρ‡ΠΈΠΌ ΠΏΠ΅Ρ€Π²ΠΎΠ΅ Π²Ρ‹Ρ€Π°ΠΆΠ΅Π½ΠΈΠ΅ Ρ‡Π΅Ρ€Π΅Π· Π±ΠΎΠ»ΡŒΡˆΡƒΡŽ Π»Π°Ρ‚ΠΈΠ½ΡΠΊΡƒΡŽ Π±ΡƒΠΊΠ²Ρƒ A

А Π²Ρ‹Ρ€Π°ΠΆΠ΅Π½ΠΈΠ΅, находящССся Π² Π·Π½Π°ΠΌΠ΅Π½Π°Ρ‚Π΅Π»Π΅ ΠΌΠΎΠΆΠ½ΠΎ ΠΎΠ±ΠΎΠ·Π½Π°Ρ‡ΠΈΡ‚ΡŒ Ρ‡Π΅Ρ€Π΅Π· Π±ΠΎΠ»ΡŒΡˆΡƒΡŽ Π»Π°Ρ‚ΠΈΠ½ΡΠΊΡƒΡŽ Π±ΡƒΠΊΠ²Ρƒ B

Π’Π΅ΠΏΠ΅Ρ€ΡŒ нашС ΠΈΠ·Π½Π°Ρ‡Π°Π»ΡŒΠ½ΠΎΠ΅ Π²Ρ‹Ρ€Π°ΠΆΠ΅Π½ΠΈΠ΅ ΠΏΡ€ΠΈΠ½ΠΈΠΌΠ°Π΅Ρ‚ Π²ΠΈΠ΄ . Π’ΠΎ Π΅ΡΡ‚ΡŒ, ΠΌΡ‹ сдСлали Π·Π°ΠΌΠ΅Π½Ρƒ числового выраТСния Π½Π° Π±ΡƒΠΊΠ²Π΅Π½Π½ΠΎΠ΅, ΠΏΡ€Π΅Π΄Π²Π°Ρ€ΠΈΡ‚Π΅Π»ΡŒΠ½ΠΎ занСся Ρ‡ΠΈΡΠ»ΠΈΡ‚Π΅Π»ΡŒ ΠΈ Π·Π½Π°ΠΌΠ΅Π½Π°Ρ‚Π΅Π»ΡŒ Π² ΠΏΠ΅Ρ€Π΅ΠΌΠ΅Π½Π½Ρ‹Π΅ A ΠΈ B.

Π’Π΅ΠΏΠ΅Ρ€ΡŒ ΠΌΡ‹ ΠΌΠΎΠΆΠ΅ΠΌ ΠΎΡ‚Π΄Π΅Π»ΡŒΠ½ΠΎ Π²Ρ‹Ρ‡ΠΈΡΠ»ΠΈΡ‚ΡŒ значСния ΠΏΠ΅Ρ€Π΅ΠΌΠ΅Π½Π½ΠΎΠΉ A ΠΈ Π·Π½Π°Ρ‡Π΅Π½ΠΈΠ΅ ΠΏΠ΅Ρ€Π΅ΠΌΠ΅Π½Π½ΠΎΠΉ B. Π“ΠΎΡ‚ΠΎΠ²Ρ‹Π΅ значСния ΠΌΡ‹ вставим Π² Π²Ρ‹Ρ€Π°ΠΆΠ΅Π½ΠΈΠ΅ .

Найдём Π·Π½Π°Ρ‡Π΅Π½ΠΈΠ΅ ΠΏΠ΅Ρ€Π΅ΠΌΠ΅Π½Π½ΠΎΠΉ A

Найдём Π·Π½Π°Ρ‡Π΅Π½ΠΈΠ΅ ΠΏΠ΅Ρ€Π΅ΠΌΠ΅Π½Π½ΠΎΠΉ B

Π’Π΅ΠΏΠ΅Ρ€ΡŒ подставим Π² Π³Π»Π°Π²Π½ΠΎΠ΅ выраТСния вмСсто ΠΏΠ΅Ρ€Π΅ΠΌΠ΅Π½Π½Ρ‹Ρ… A ΠΈ B ΠΈΡ… значСния:

ΠœΡ‹ ΠΏΠΎΠ»ΡƒΡ‡ΠΈΠ»ΠΈ ΠΌΠ½ΠΎΠ³ΠΎΡΡ‚Π°ΠΆΠ½ΡƒΡŽ Π΄Ρ€ΠΎΠ±ΡŒ Π² ΠΊΠΎΡ‚ΠΎΡ€ΠΎΠΉ ΠΌΠΎΠΆΠ½ΠΎ Π²ΠΎΡΠΏΠΎΠ»ΡŒΠ·ΠΎΠ²Π°Ρ‚ΡŒΡΡ схСмой «с ΠΏΠ΅Ρ€Π²ΠΎΠ³ΠΎ Π½Π° Ρ‡Π΅Ρ‚Π²Ρ‘Ρ€Ρ‚Ρ‹ΠΉ, со Π²Ρ‚ΠΎΡ€ΠΎΠ³ΠΎ Π½Π° Ρ‚Ρ€Π΅Ρ‚ΠΈΠΉΒ», Ρ‚ΠΎ Π΅ΡΡ‚ΡŒ Ρ†ΠΈΡ„Ρ€Ρƒ Π½Π°Ρ…ΠΎΠ΄ΡΡ‰ΡƒΡŽΡΡ Π½Π° ΠΏΠ΅Ρ€Π²ΠΎΠΌ этаТС ΠΏΠΎΠ΄Π½ΡΡ‚ΡŒ Π½Π° Ρ‡Π΅Ρ‚Π²Ρ‘Ρ€Ρ‚Ρ‹ΠΉ этаТ, Π° Ρ†ΠΈΡ„Ρ€Ρƒ Π½Π°Ρ…ΠΎΠ΄ΡΡ‰ΡƒΡŽΡΡ Π½Π° Π²Ρ‚ΠΎΡ€ΠΎΠΌ этаТС ΠΏΠΎΠ΄Π½ΡΡ‚ΡŒ Π½Π° Ρ‚Ρ€Π΅Ρ‚ΠΈΠΉ этаТ. Π”Π°Π»ΡŒΠ½Π΅ΠΉΡˆΠ΅Π΅ вычислСниС Π½Π΅ составит особого Ρ‚Ρ€ΡƒΠ΄Π°:

Π’Π°ΠΊΠΈΠΌ ΠΎΠ±Ρ€Π°Π·ΠΎΠΌ, Π·Π½Π°Ρ‡Π΅Π½ΠΈΠ΅ выраТСния Ρ€Π°Π²Π½ΠΎ βˆ’1.

ΠšΠΎΠ½Π΅Ρ‡Π½ΠΎ, ΠΌΡ‹ рассмотрСли ΠΏΡ€ΠΎΡΡ‚Π΅ΠΉΡˆΠΈΠΉ ΠΏΡ€ΠΈΠΌΠ΅Ρ€, Π½ΠΎ нашСй Ρ†Π΅Π»ΡŒΡŽ Π±Ρ‹Π»ΠΎ ΡƒΠ·Π½Π°Ρ‚ΡŒ, ΠΊΠ°ΠΊ ΠΌΠΎΠΆΠ½ΠΎ ΠΈΡΠΏΠΎΠ»ΡŒΠ·ΠΎΠ²Π°Ρ‚ΡŒ ΠΏΠ΅Ρ€Π΅ΠΌΠ΅Π½Π½Ρ‹Π΅ для облСгчСния сСбС Π·Π°Π΄Π°Ρ‡ΠΈ, Ρ‡Ρ‚ΠΎΠ±Ρ‹ свСсти ΠΊ ΠΌΠΈΠ½ΠΈΠΌΡƒΠΌΡƒ Π΄ΠΎΠΏΡƒΡ‰Π΅Π½ΠΈΠ΅ ошибок.

ΠžΡ‚ΠΌΠ΅Ρ‚ΠΈΠΌ Ρ‚Π°ΠΊΠΆΠ΅, Ρ‡Ρ‚ΠΎ Ρ€Π΅ΡˆΠ΅Π½ΠΈΠ΅ для Π΄Π°Π½Π½ΠΎΠ³ΠΎ ΠΏΡ€ΠΈΠΌΠ΅Ρ€Π° ΠΌΠΎΠΆΠ½ΠΎ Π·Π°ΠΏΠΈΡΠ°Ρ‚ΡŒ Π½Π΅ примСняя ΠΏΠ΅Ρ€Π΅ΠΌΠ΅Π½Π½Ρ‹Π΅. Π’Ρ‹Π³Π»ΡΠ΄Π΅Ρ‚ΡŒ ΠΎΠ½ΠΎ Π±ΡƒΠ΄Π΅Ρ‚ ΠΊΠ°ΠΊ

Π­Ρ‚ΠΎ Ρ€Π΅ΡˆΠ΅Π½ΠΈΠ΅ Π±ΠΎΠ»Π΅Π΅ быстроС ΠΈ ΠΊΠΎΡ€ΠΎΡ‚ΠΊΠΎΠ΅ ΠΈ Π² Π΄Π°Π½Π½ΠΎΠΌ случаС Π΅Π³ΠΎ цСлСсообразнСС Ρ‚Π°ΠΊ ΠΈ Π·Π°ΠΏΠΈΡΠ°Ρ‚ΡŒ, Π½ΠΎ Ссли Π²Ρ‹Ρ€Π°ΠΆΠ΅Π½ΠΈΠ΅ окаТСтся слоТным, состоящим ΠΈΠ· Π½Π΅ΡΠΊΠΎΠ»ΡŒΠΊΠΈΡ… ΠΏΠ°Ρ€Π°ΠΌΠ΅Ρ‚Ρ€ΠΎΠ², скобок, ΠΊΠΎΡ€Π½Π΅ΠΉ ΠΈ стСпСнСй, Ρ‚ΠΎ ΠΆΠ΅Π»Π°Ρ‚Π΅Π»ΡŒΠ½ΠΎ Π²Ρ‹Ρ‡ΠΈΡΠ»ΡΡ‚ΡŒ Π΅Π³ΠΎ Π² нСсколько этапов, занося Ρ‡Π°ΡΡ‚ΡŒ Π΅Π³ΠΎ Π²Ρ‹Ρ€Π°ΠΆΠ΅Π½ΠΈΠΉ Π² ΠΏΠ΅Ρ€Π΅ΠΌΠ΅Π½Π½Ρ‹Π΅.

ΠŸΠΎΠ½Ρ€Π°Π²ΠΈΠ»ΡΡ ΡƒΡ€ΠΎΠΊ?
Вступай Π² Π½Π°ΡˆΡƒ Π½ΠΎΠ²ΡƒΡŽ Π³Ρ€ΡƒΠΏΠΏΡƒ Π’ΠΊΠΎΠ½Ρ‚Π°ΠΊΡ‚Π΅ ΠΈ Π½Π°Ρ‡Π½ΠΈ ΠΏΠΎΠ»ΡƒΡ‡Π°Ρ‚ΡŒ увСдомлСния ΠΎ Π½ΠΎΠ²Ρ‹Ρ… ΡƒΡ€ΠΎΠΊΠ°Ρ…

Π’ этой ΡΡ‚Π°Ρ‚ΡŒΠ΅ ΠΌΡ‹ разбСрСмся с ΡƒΠΌΠ½ΠΎΠΆΠ΅Π½ΠΈΠ΅ΠΌ чисСл с Ρ€Π°Π·Π½Ρ‹ΠΌΠΈ Π·Π½Π°ΠΊΠ°ΠΌΠΈ . Π—Π΄Π΅ΡΡŒ ΠΌΡ‹ сначала сформулируСм ΠΏΡ€Π°Π²ΠΈΠ»ΠΎ умноТСния ΠΏΠΎΠ»ΠΎΠΆΠΈΡ‚Π΅Π»ΡŒΠ½ΠΎΠ³ΠΎ ΠΈ ΠΎΡ‚Ρ€ΠΈΡ†Π°Ρ‚Π΅Π»ΡŒΠ½ΠΎΠ³ΠΎ числа, обоснуСм Π΅Π³ΠΎ, Π° послС этого рассмотрим ΠΏΡ€ΠΈΠΌΠ΅Π½Π΅Π½ΠΈΠ΅ Π΄Π°Π½Π½ΠΎΠ³ΠΎ ΠΏΡ€Π°Π²ΠΈΠ»Π° ΠΏΡ€ΠΈ Ρ€Π΅ΡˆΠ΅Π½ΠΈΠΈ ΠΏΡ€ΠΈΠΌΠ΅Ρ€ΠΎΠ².

Навигация ΠΏΠΎ страницС.

ΠŸΡ€Π°Π²ΠΈΠ»ΠΎ умноТСния чисСл с Ρ€Π°Π·Π½Ρ‹ΠΌΠΈ Π·Π½Π°ΠΊΠ°ΠΌΠΈ

Π£ΠΌΠ½ΠΎΠΆΠ΅Π½ΠΈΠ΅ ΠΏΠΎΠ»ΠΎΠΆΠΈΡ‚Π΅Π»ΡŒΠ½ΠΎΠ³ΠΎ числа Π½Π° ΠΎΡ‚Ρ€ΠΈΡ†Π°Ρ‚Π΅Π»ΡŒΠ½ΠΎΠ΅, Π° Ρ‚Π°ΠΊΠΆΠ΅ ΠΎΡ‚Ρ€ΠΈΡ†Π°Ρ‚Π΅Π»ΡŒΠ½ΠΎΠ³ΠΎ Π½Π° ΠΏΠΎΠ»ΠΎΠΆΠΈΡ‚Π΅Π»ΡŒΠ½ΠΎΠ΅, проводится ΠΏΠΎ ΡΠ»Π΅Π΄ΡƒΡŽΡ‰Π΅ΠΌΡƒ ΠΏΡ€Π°Π²ΠΈΠ»Ρƒ умноТСния чисСл с Ρ€Π°Π·Π½Ρ‹ΠΌΠΈ Π·Π½Π°ΠΊΠ°ΠΌΠΈ : Ρ‡Ρ‚ΠΎΠ±Ρ‹ ΡƒΠΌΠ½ΠΎΠΆΠΈΡ‚ΡŒ числа с Ρ€Π°Π·Π½Ρ‹ΠΌΠΈ Π·Π½Π°ΠΊΠ°ΠΌΠΈ, Π½Π°Π΄ΠΎ ΡƒΠΌΠ½ΠΎΠΆΠΈΡ‚ΡŒ , ΠΈ ΠΏΠ΅Ρ€Π΅Π΄ ΠΏΠΎΠ»ΡƒΡ‡Π΅Π½Π½Ρ‹ΠΌ ΠΏΡ€ΠΎΠΈΠ·Π²Π΅Π΄Π΅Π½ΠΈΠ΅ΠΌ ΠΏΠΎΡΡ‚Π°Π²ΠΈΡ‚ΡŒ Π·Π½Π°ΠΊ минус.

Π—Π°ΠΏΠΈΡˆΠ΅ΠΌ Π΄Π°Π½Π½ΠΎΠ΅ ΠΏΡ€Π°Π²ΠΈΠ»ΠΎ Π² Π±ΡƒΠΊΠ²Π΅Π½Π½ΠΎΠΌ Π²ΠΈΠ΄Π΅. Для любого ΠΏΠΎΠ»ΠΎΠΆΠΈΡ‚Π΅Π»ΡŒΠ½ΠΎΠ³ΠΎ Π΄Π΅ΠΉΡΡ‚Π²ΠΈΡ‚Π΅Π»ΡŒΠ½ΠΎΠ³ΠΎ числа a ΠΈ Π΄Π΅ΠΉΡΡ‚Π²ΠΈΡ‚Π΅Π»ΡŒΠ½ΠΎΠ³ΠΎ ΠΎΡ‚Ρ€ΠΈΡ†Π°Ρ‚Π΅Π»ΡŒΠ½ΠΎΠ³ΠΎ числа βˆ’b справСдливо равСнство aΒ·(βˆ’b)=βˆ’(|a|Β·|b|) , Π° Ρ‚Π°ΠΊΠΆΠ΅ для ΠΎΡ‚Ρ€ΠΈΡ†Π°Ρ‚Π΅Π»ΡŒΠ½ΠΎΠ³ΠΎ числа βˆ’a ΠΈ ΠΏΠΎΠ»ΠΎΠΆΠΈΡ‚Π΅Π»ΡŒΠ½ΠΎΠ³ΠΎ числа b справСдливо равСнство (βˆ’a)Β·b=βˆ’(|a|Β·|b|) .

ΠŸΡ€Π°Π²ΠΈΠ»ΠΎ умноТСния чисСл с Ρ€Π°Π·Π½Ρ‹ΠΌΠΈ Π·Π½Π°ΠΊΠ°ΠΌΠΈ ΠΏΠΎΠ»Π½ΠΎΡΡ‚ΡŒΡŽ согласуСтся со свойствами дСйствий с Π΄Π΅ΠΉΡΡ‚Π²ΠΈΡ‚Π΅Π»ΡŒΠ½Ρ‹ΠΌΠΈ числами . Π”Π΅ΠΉΡΡ‚Π²ΠΈΡ‚Π΅Π»ΡŒΠ½ΠΎ, Π½Π° ΠΈΡ… основС нСслоТно ΠΏΠΎΠΊΠ°Π·Π°Ρ‚ΡŒ, Ρ‡Ρ‚ΠΎ для Π΄Π΅ΠΉΡΡ‚Π²ΠΈΡ‚Π΅Π»ΡŒΠ½Ρ‹Ρ… ΠΈ ΠΏΠΎΠ»ΠΎΠΆΠΈΡ‚Π΅Π»ΡŒΠ½Ρ‹Ρ… чисСл a ΠΈ b справСдлива Ρ†Π΅ΠΏΠΎΡ‡ΠΊΠ° равСнств Π²ΠΈΠ΄Π° aΒ·(βˆ’b)+aΒ·b=aΒ·((βˆ’b)+b)=aΒ·0=0 , которая Π΄ΠΎΠΊΠ°Π·Ρ‹Π²Π°Π΅Ρ‚, Ρ‡Ρ‚ΠΎ aΒ·(βˆ’b) ΠΈ aΒ·b – ΠΏΡ€ΠΎΡ‚ΠΈΠ²ΠΎΠΏΠΎΠ»ΠΎΠΆΠ½Ρ‹Π΅ числа, ΠΎΡ‚ΠΊΡƒΠ΄Π° слСдуСт равСнство aΒ·(βˆ’b)=βˆ’(aΒ·b) . А ΠΈΠ· Π½Π΅Π³ΠΎ слСдуСт ΡΠΏΡ€Π°Π²Π΅Π΄Π»ΠΈΠ²ΠΎΡΡ‚ΡŒ рассматриваСмого ΠΏΡ€Π°Π²ΠΈΠ»Π° умноТСния.

Π‘Π»Π΅Π΄ΡƒΠ΅Ρ‚ ΠΎΡ‚ΠΌΠ΅Ρ‚ΠΈΡ‚ΡŒ, Ρ‡Ρ‚ΠΎ ΠΎΠ·Π²ΡƒΡ‡Π΅Π½Π½ΠΎΠ΅ ΠΏΡ€Π°Π²ΠΈΠ»ΠΎ умноТСния чисСл с Ρ€Π°Π·Π½Ρ‹ΠΌΠΈ Π·Π½Π°ΠΊΠ°ΠΌΠΈ справСдливо ΠΊΠ°ΠΊ для Π΄Π΅ΠΉΡΡ‚Π²ΠΈΡ‚Π΅Π»ΡŒΠ½Ρ‹Ρ… чисСл, Ρ‚Π°ΠΊ ΠΈ для Ρ€Π°Ρ†ΠΈΠΎΠ½Π°Π»ΡŒΠ½Ρ‹Ρ… чисСл ΠΈ для Ρ†Π΅Π»Ρ‹Ρ… чисСл . Π­Ρ‚ΠΎ слСдуСт ΠΈΠ· Ρ‚ΠΎΠ³ΠΎ, Ρ‡Ρ‚ΠΎ дСйствия с Ρ€Π°Ρ†ΠΈΠΎΠ½Π°Π»ΡŒΠ½Ρ‹ΠΌΠΈ ΠΈ Ρ†Π΅Π»Ρ‹ΠΌΠΈ числами ΠΎΠ±Π»Π°Π΄Π°ΡŽΡ‚ Ρ‚Π΅ΠΌΠΈ ΠΆΠ΅ свойствами, ΠΊΠΎΡ‚ΠΎΡ€Ρ‹Π΅ использовались ΠΏΡ€ΠΈ Π΄ΠΎΠΊΠ°Π·Π°Ρ‚Π΅Π»ΡŒΡΡ‚Π²Π΅ Π²Ρ‹ΡˆΠ΅.

ΠŸΠΎΠ½ΡΡ‚Π½ΠΎ, Ρ‡Ρ‚ΠΎ ΡƒΠΌΠ½ΠΎΠΆΠ΅Π½ΠΈΠ΅ чисСл с Ρ€Π°Π·Π½Ρ‹ΠΌΠΈ Π·Π½Π°ΠΊΠ°ΠΌΠΈ ΠΏΠΎ ΠΏΠΎΠ»ΡƒΡ‡Π΅Π½Π½ΠΎΠΌΡƒ ΠΏΡ€Π°Π²ΠΈΠ»Ρƒ сводится ΠΊ ΡƒΠΌΠ½ΠΎΠΆΠ΅Π½ΠΈΡŽ ΠΏΠΎΠ»ΠΎΠΆΠΈΡ‚Π΅Π»ΡŒΠ½Ρ‹Ρ… чисСл.

ΠžΡΡ‚Π°Π»ΠΎΡΡŒ лишь Ρ€Π°ΡΡΠΌΠΎΡ‚Ρ€Π΅Ρ‚ΡŒ ΠΏΡ€ΠΈΠΌΠ΅Ρ€Ρ‹ примСнСния Ρ€Π°Π·ΠΎΠ±Ρ€Π°Π½Π½ΠΎΠ³ΠΎ ΠΏΡ€Π°Π²ΠΈΠ»Π° умноТСния ΠΏΡ€ΠΈ ΡƒΠΌΠ½ΠΎΠΆΠ΅Π½ΠΈΠΈ чисСл с Ρ€Π°Π·Π½Ρ‹ΠΌΠΈ Π·Π½Π°ΠΊΠ°ΠΌΠΈ.

ΠŸΡ€ΠΈΠΌΠ΅Ρ€Ρ‹ умноТСния чисСл с Ρ€Π°Π·Π½Ρ‹ΠΌΠΈ Π·Π½Π°ΠΊΠ°ΠΌΠΈ

Π Π°Π·Π±Π΅Ρ€Π΅ΠΌ Ρ€Π΅ΡˆΠ΅Π½ΠΈΡ Π½Π΅ΡΠΊΠΎΠ»ΡŒΠΊΠΈΡ… ΠΏΡ€ΠΈΠΌΠ΅Ρ€ΠΎΠ² умноТСния чисСл с Ρ€Π°Π·Π½Ρ‹ΠΌΠΈ Π·Π½Π°ΠΊΠ°ΠΌΠΈ . НачнСм с простого случая, Ρ‡Ρ‚ΠΎΠ±Ρ‹ ΡΠΎΡΡ€Π΅Π΄ΠΎΡ‚ΠΎΡ‡ΠΈΡ‚ΡŒΡΡ Π½Π° ΡˆΠ°Π³Π°Ρ… ΠΏΡ€Π°Π²ΠΈΠ»Π°, Π° Π½Π΅ Π½Π° Π²Ρ‹Ρ‡ΠΈΡΠ»ΠΈΡ‚Π΅Π»ΡŒΠ½Ρ‹Ρ… слоТностях.

ΠŸΡ€ΠΈΠΌΠ΅Ρ€.

Π’Ρ‹ΠΏΠΎΠ»Π½ΠΈΡ‚Π΅ ΡƒΠΌΠ½ΠΎΠΆΠ΅Π½ΠΈΠ΅ ΠΎΡ‚Ρ€ΠΈΡ†Π°Ρ‚Π΅Π»ΡŒΠ½ΠΎΠ³ΠΎ числа βˆ’4 Π½Π° ΠΏΠΎΠ»ΠΎΠΆΠΈΡ‚Π΅Π»ΡŒΠ½ΠΎΠ΅ число 5 .

РСшСниС.

По ΠΏΡ€Π°Π²ΠΈΠ»Ρƒ умноТСния чисСл с Ρ€Π°Π·Π½Ρ‹ΠΌΠΈ Π·Π½Π°ΠΊΠ°ΠΌΠΈ Π½Π°ΠΌ сначала Π½ΡƒΠΆΠ½ΠΎ ΠΏΠ΅Ρ€Π΅ΠΌΠ½ΠΎΠΆΠΈΡ‚ΡŒ ΠΌΠΎΠ΄ΡƒΠ»ΠΈ исходных ΠΌΠ½ΠΎΠΆΠΈΡ‚Π΅Π»Π΅ΠΉ. ΠœΠΎΠ΄ΡƒΠ»ΡŒ βˆ’4 Ρ€Π°Π²Π΅Π½ 4 , Π° ΠΌΠΎΠ΄ΡƒΠ»ΡŒ 5 Ρ€Π°Π²Π΅Π½ 5 , Π° ΡƒΠΌΠ½ΠΎΠΆΠ΅Π½ΠΈΠ΅ Π½Π°Ρ‚ΡƒΡ€Π°Π»ΡŒΠ½Ρ‹Ρ… чисСл 4 ΠΈ 5 Π΄Π°Π΅Ρ‚ 20 . НаконСц, ΠΎΡΡ‚Π°Π»ΠΎΡΡŒ ΠΏΠΎΡΡ‚Π°Π²ΠΈΡ‚ΡŒ Π·Π½Π°ΠΊ минус ΠΏΠ΅Ρ€Π΅Π΄ ΠΏΠΎΠ»ΡƒΡ‡Π΅Π½Π½Ρ‹ΠΌ числом, ΠΈΠΌΠ΅Π΅ΠΌ βˆ’20 . На этом ΡƒΠΌΠ½ΠΎΠΆΠ΅Π½ΠΈΠ΅ Π·Π°Π²Π΅Ρ€ΡˆΠ΅Π½ΠΎ.

ΠšΡ€Π°Ρ‚ΠΊΠΎ Ρ€Π΅ΡˆΠ΅Π½ΠΈΠ΅ ΠΌΠΎΠΆΠ½ΠΎ Π·Π°ΠΏΠΈΡΠ°Ρ‚ΡŒ Ρ‚Π°ΠΊ: (βˆ’4)Β·5=βˆ’(4Β·5)=βˆ’20 .

ΠžΡ‚Π²Π΅Ρ‚:

(βˆ’4)Β·5=βˆ’20 .

ΠŸΡ€ΠΈ ΡƒΠΌΠ½ΠΎΠΆΠ΅Π½ΠΈΠΈ Π΄Ρ€ΠΎΠ±Π½Ρ‹Ρ… чисСл с Ρ€Π°Π·Π½Ρ‹ΠΌΠΈ Π·Π½Π°ΠΊΠ°ΠΌΠΈ Π½ΡƒΠΆΠ½ΠΎ ΡƒΠΌΠ΅Ρ‚ΡŒ Π²Ρ‹ΠΏΠΎΠ»Π½ΡΡ‚ΡŒ ΡƒΠΌΠ½ΠΎΠΆΠ΅Π½ΠΈΠ΅ ΠΎΠ±Ρ‹ΠΊΠ½ΠΎΠ²Π΅Π½Π½Ρ‹Ρ… Π΄Ρ€ΠΎΠ±Π΅ΠΉ , ΡƒΠΌΠ½ΠΎΠΆΠ΅Π½ΠΈΠ΅ дСсятичных Π΄Ρ€ΠΎΠ±Π΅ΠΉ ΠΈ ΠΈΡ… ΠΊΠΎΠΌΠ±ΠΈΠ½Π°Ρ†ΠΈΠΉ с Π½Π°Ρ‚ΡƒΡ€Π°Π»ΡŒΠ½Ρ‹ΠΌΠΈ ΠΈ ΡΠΌΠ΅ΡˆΠ°Π½Π½Ρ‹ΠΌΠΈ числами.

ΠŸΡ€ΠΈΠΌΠ΅Ρ€.

ΠŸΡ€ΠΎΠ²Π΅Π΄ΠΈΡ‚Π΅ ΡƒΠΌΠ½ΠΎΠΆΠ΅Π½ΠΈΠ΅ чисСл с Ρ€Π°Π·Π½Ρ‹ΠΌΠΈ Π·Π½Π°ΠΊΠ°ΠΌΠΈ 0,(2) ΠΈ .

РСшСниС.

Π’Ρ‹ΠΏΠΎΠ»Π½ΠΈΠ² ΠΏΠ΅Ρ€Π΅Π²ΠΎΠ΄ пСриодичСской дСсятичной Π΄Ρ€ΠΎΠ±ΠΈ Π² ΠΎΠ±Ρ‹ΠΊΠ½ΠΎΠ²Π΅Π½Π½ΡƒΡŽ Π΄Ρ€ΠΎΠ±ΡŒ , Π° Ρ‚Π°ΠΊΠΆΠ΅ Π²Ρ‹ΠΏΠΎΠ»Π½ΠΈΠ² ΠΏΠ΅Ρ€Π΅Ρ…ΠΎΠ΄ ΠΎΡ‚ смСшанного числа ΠΊ Π½Π΅ΠΏΡ€Π°Π²ΠΈΠ»ΡŒΠ½ΠΎΠΉ Π΄Ρ€ΠΎΠ±ΠΈ , ΠΎΡ‚ исходного произвСдСния ΠΌΡ‹ ΠΏΡ€ΠΈΠ΄Π΅ΠΌ ΠΊ ΠΏΡ€ΠΎΠΈΠ·Π²Π΅Π΄Π΅Π½ΠΈΡŽ ΠΎΠ±Ρ‹ΠΊΠ½ΠΎΠ²Π΅Π½Π½Ρ‹Ρ… Π΄Ρ€ΠΎΠ±Π΅ΠΉ с Ρ€Π°Π·Π½Ρ‹ΠΌΠΈ Π·Π½Π°ΠΊΠ°ΠΌΠΈ Π²ΠΈΠ΄Π° . Π­Ρ‚ΠΎ ΠΏΡ€ΠΎΠΈΠ·Π²Π΅Π΄Π΅Π½ΠΈΠ΅ ΠΏΠΎ ΠΏΡ€Π°Π²ΠΈΠ»Ρƒ умноТСния чисСл с Ρ€Π°Π·Π½Ρ‹ΠΌΠΈ Π·Π½Π°ΠΊΠ°ΠΌΠΈ Ρ€Π°Π²Π½ΠΎ . ΠžΡΡ‚Π°Π»ΠΎΡΡŒ лишь ΠΏΠ΅Ρ€Π΅ΠΌΠ½ΠΎΠΆΠΈΡ‚ΡŒ ΠΎΠ±Ρ‹ΠΊΠ½ΠΎΠ²Π΅Π½Π½Ρ‹Π΅ Π΄Ρ€ΠΎΠ±ΠΈ Π² скобках, ΠΈΠΌΠ΅Π΅ΠΌ .

Π‘Π»ΠΎΠΆΠ΅Π½ΠΈΠ΅ ΠΈ Π²Ρ‹Ρ‡ΠΈΡ‚Π°Π½ΠΈΠ΅ ΠΎΡ‚Ρ€ΠΈΡ†Π°Ρ‚Π΅Π»ΡŒΠ½Ρ‹Ρ… Π΄Ρ€ΠΎΠ±Π΅ΠΉ.

  • ΠΠ»ΡŒΡ„Π°ΡˆΠΊΠΎΠ»Π°
  • Π‘Ρ‚Π°Ρ‚ΡŒΠΈ
  • Π‘Π»ΠΎΠΆΠ΅Π½ΠΈΠ΅ ΠΈ Π²Ρ‹Ρ‡ΠΈΡ‚Π°Π½ΠΈΠ΅ ΠΎΡ‚Ρ€ΠΈΡ†Π°Ρ‚Π΅Π»ΡŒΠ½Ρ‹Ρ… Π΄Ρ€ΠΎΠ±Π΅ΠΉ

ΠžΡ‚Ρ€ΠΈΡ†Π°Ρ‚Π΅Π»ΡŒΠ½Ρ‹Π΅ Π΄Ρ€ΠΎΠ±ΠΈ ΡΠΊΠ»Π°Π΄Ρ‹Π²Π°ΡŽΡ‚ΡΡ ΠΈ Π²Ρ‹Ρ‡ΠΈΡ‚Π°ΡŽΡ‚ΡΡ Ρ‚Π°ΠΊΠΆΠ΅ ΠΊΠ°ΠΊ ΠΈ ΠΎΡ‚Ρ€ΠΈΡ†Π°Ρ‚Π΅Π»ΡŒΠ½Ρ‹Π΅ числа, Ρ‚ΠΎΠ»ΡŒΠΊΠΎ ΠΏΠΎ ΠΏΡ€Π°Π²ΠΈΠ»Π°ΠΌ слоТСния Π΄Ρ€ΠΎΠ±Π΅ΠΉ. ΠœΡ‹ ΠΌΠΎΠΆΠ΅Ρ‚Π΅ Π΄ΠΎΠ±Π°Π²Π»ΡΡ‚ΡŒ Ρ‚ΠΎΠ»ΡŒΠΊΠΎ ΠΏΠΎΠ»ΠΎΠ²ΠΈΠ½ΠΊΠΈ ΠΊ ΠΏΠΎΠ»ΠΎΠ²ΠΈΠ½ΠΊΠ°ΠΌ, Ρ‡Π΅Ρ‚Π²Π΅Ρ€Ρ‚ΠΈ ΠΊ чСтвСртям ΠΈΠ»ΠΈ дСсятыС ΠΊ дСсятым ΠΈ Ρ‚Π°ΠΊ Π΄Π°Π»Π΅Π΅. Π’Ρ‹Ρ‡ΠΈΡ‚Π°Π½ΠΈΠ΅ ΠΎΡ‚Ρ€ΠΈΡ†Π°Ρ‚Π΅Π»ΡŒΠ½Ρ‹Ρ… Π΄Ρ€ΠΎΠ±Π΅ΠΉ слСдуСт Ρ‚Π΅ΠΌ ΠΆΠ΅ ΠΏΡ€Π°Π²ΠΈΠ»Π°ΠΌ. Для Ρ‚ΠΎΠ³ΠΎ Ρ‡Ρ‚ΠΎΠ±Ρ‹ ΡΠ»ΠΎΠΆΠΈΡ‚ΡŒ ΠΈΠ»ΠΈ Π²Ρ‹Ρ‡Π΅ΡΡ‚ΡŒ Π΄Π²Π΅ Π΄Ρ€ΠΎΠ±ΠΈ Π½Π°ΠΌ Π½ΡƒΠΆΠ½ΠΎ привСсти ΠΈΡ… ΠΊ ΠΎΠ±Ρ‰Π΅ΠΌΡƒ Π·Π½Π°ΠΌΠ΅Π½Π°Ρ‚Π΅Π»ΡŽ. Если Π΄Ρ€ΠΎΠ±ΡŒ ΠΎΡ‚Ρ€ΠΈΡ†Π°Ρ‚Π΅Π»ΡŒΠ½Π°Ρ ΠΌΡ‹ ΠΌΠΎΠΆΠ΅ΠΌ Π·Π½Π°ΠΊ минус ΠΏΠΎΡΡ‚Π°Π²ΠΈΡ‚ΡŒ Π² Ρ‡ΠΈΡΠ»ΠΈΡ‚Π΅Π»ΡŒ ΠΈ Π½Π°ΠΎΠ±ΠΎΡ€ΠΎΡ‚. НиТС расписано слоТСниС \(-\frac{7}{6}+\frac{5}{7}\):

ΠŸΡ€ΠΈ слоТСнии Π΄Π²ΡƒΡ… ΠΎΡ‚Ρ€ΠΈΡ†Π°Ρ‚Π΅Π»ΡŒΠ½Ρ‹Ρ… Π΄Ρ€ΠΎΠ±Π΅ΠΉ Ρ€Π΅Π·ΡƒΠ»ΡŒΡ‚Π°Ρ‚ Π±ΡƒΠ΄Π΅Ρ‚ ΠΎΡ‚Ρ€ΠΈΡ†Π°Ρ‚Π΅Π»ΡŒΠ½Ρ‹ΠΌ. Когда ΠΌΡ‹ Π²Ρ‹Ρ‡ΠΈΡ‚Π°Π΅ΠΌ Π΄Π²Π΅ ΠΎΡ‚Ρ€ΠΈΡ†Π°Ρ‚Π΅Π»ΡŒΠ½Ρ‹Π΅ Π΄Ρ€ΠΎΠ±ΠΈ, Ρ‚ΠΎ ΠΌΡ‹ ΠΊ ΠΏΠ΅Ρ€Π²ΠΎΠΉ ΠΎΡ‚Ρ€ΠΈΡ†Π°Ρ‚Π΅Π»ΡŒΠ½ΠΎΠΉ Π΄Ρ€ΠΎΠ±ΠΈ прибавляСм ΠΏΠΎΠ»ΠΎΠΆΠΈΡ‚Π΅Π»ΡŒΠ½ΡƒΡŽ Π²Ρ‚ΠΎΡ€ΡƒΡŽ, Ρ‚Π°ΠΊ ΠΊΠ°ΠΊ минус Π½Π° минус Π΄Π°Π΅Ρ‚ плюс.

Β 

Β 

Рассмотрим слоТСниС ΠΈ Π²Ρ‹Ρ‡ΠΈΡ‚Π°Π½ΠΈΠ΅ ΡΠ»Π΅Π΄ΡƒΡŽΡ‰ΠΈΡ… ΠΎΡ‚Ρ€ΠΈΡ†Π°Ρ‚Π΅Π»ΡŒΠ½Ρ‹Ρ… Π΄Ρ€ΠΎΠ±Π΅ΠΉ.

Π—Π°Π΄Π°Ρ‡Π° 1. ВычислитС \(\frac{1}{4}+(-\frac{3}{10})-(-\frac{1}{2}).\).

РСшСниС. ΠŸΡ€ΠΈΠ²ΠΎΠ΄ΠΈΠΌ ΠΊ Π½Π°ΠΈΠΌΠ΅Π½ΡŒΡˆΠ΅ΠΌΡƒ ΠΎΠ±Ρ‰Π΅ΠΌΡƒ Π·Π½Π°ΠΌΠ΅Π½Π°Ρ‚Π΅Π»ΡŽ:

\(\frac{1}{4}+(-\frac{3}{10})-(-\frac{1}{2})=\frac{5-3*2+10}{20}=\frac{9}{20}\)

ΠžΡ‚Π²Π΅Ρ‚: \(\frac{9}{20}\).

Π—Π°Π΄Π°Ρ‡Π° 2. ВычислитС \(\frac{1}{7}-(-\frac{5}{6})-(-\frac{1}{3})\).

РСшСниС. ΠŸΡ€ΠΈΠ²ΠΎΠ΄ΠΈΠΌ ΠΊ Π½Π°ΠΈΠΌΠ΅Π½ΡŒΡˆΠ΅ΠΌΡƒ ΠΎΠ±Ρ‰Π΅ΠΌΡƒ Π·Π½Π°ΠΌΠ΅Π½Π°Ρ‚Π΅Π»ΡŽ:

\(\frac{6-35+14}{42}=-\frac{15}{42}=-\frac{5}{14}\)

ΠžΡ‚Π²Π΅Ρ‚: \(-\frac{5}{14}\).

Π—Π°Π΄Π°Ρ‡Π° 3. ВычислитС \(\frac{1}{4}+\frac{5}{3}-(-\frac{1}{12})\).

РСшСниС. ΠŸΡ€ΠΈΠ²ΠΎΠ΄ΠΈΠΌ ΠΊ Π½Π°ΠΈΠΌΠ΅Π½ΡŒΡˆΠ΅ΠΌΡƒ ΠΎΠ±Ρ‰Π΅ΠΌΡƒ Π·Π½Π°ΠΌΠ΅Π½Π°Ρ‚Π΅Π»ΡŽ:

\(\frac{3+20+1}{12}=\frac{24}{12}=2.\)

ΠžΡ‚Π²Π΅Ρ‚: \(2.\).

Π—Π°ΠΏΠΈΡˆΠΈΡΡŒ Π½Π° бСсплатный ΠΏΡ€ΠΎΠ±Π½Ρ‹ΠΉ ΡƒΡ€ΠΎΠΊΒ Ρ‚ΡƒΡ‚Β ΠΈ Ρ€Π°Π·Π±Π΅Ρ€ΠΈΡΡŒ с Ρ‚Π΅ΠΌ, Ρ‡Ρ‚ΠΎ Ρ‚Π΅Π±Π΅ нСпонятно.

Β 

Β 

Β 

Β 

Β 

Β 

Β 

Β 

Β 

Β 

Π‘ΠΎΠ»ΡŒΡˆΠ΅ ΡƒΡ€ΠΎΠΊΠΎΠ² ΠΈ Π·Π°Π΄Π°Π½ΠΈΠΉ ΠΏΠΎ ΠΌΠ°Ρ‚Π΅ΠΌΠ°Ρ‚ΠΈΠΊΠ΅ вмСстС с прСподаватСлями нашСй ΠΎΠ½Π»Π°ΠΉΠ½-ΡˆΠΊΠΎΠ»Ρ‹ «ΠΠ»ΡŒΡ„Π°». Π—Π°ΠΏΠΈΡˆΠΈΡ‚Π΅ΡΡŒ Π½Π° ΠΏΡ€ΠΎΠ±Π½ΠΎΠ΅ занятиС ΡƒΠΆΠ΅ сСйчас!

Π—Π°ΠΏΠΈΡˆΠΈΡ‚Π΅ΡΡŒ Π½Π° бСсплатноС тСстированиС Π·Π½Π°Π½ΠΈΠΉ!

НаТимая ΠΊΠ½ΠΎΠΏΠΊΡƒ «Π—Π°ΠΏΠΈΡΠ°Ρ‚ΡŒΡΡ» ΠΏΡ€ΠΈΠ½ΠΈΠΌΠ°ΡŽ условия ΠŸΠΎΠ»ΡŒΠ·ΠΎΠ²Π°Ρ‚Π΅Π»ΡŒΡΠΊΠΎΠ³ΠΎ соглашСния ΠΈ ΠŸΠΎΠ»ΠΈΡ‚ΠΈΠΊΠΈ ΠΊΠΎΠ½Ρ„ΠΈΠ΄Π΅Π½Ρ†ΠΈΠ°Π»ΡŒΠ½ΠΎΡΡ‚ΠΈ

Наши ΠΏΡ€Π΅ΠΏΠΎΠ΄Π°Π²Π°Ρ‚Π΅Π»ΠΈ

Анна АлСксандровна Ваструхина

Π Π΅ΠΏΠ΅Ρ‚ΠΈΡ‚ΠΎΡ€ ΠΏΠΎ ΠΌΠ°Ρ‚Π΅ΠΌΠ°Ρ‚ΠΈΠΊΠ΅

Π‘Ρ‚Π°ΠΆ (Π»Π΅Ρ‚)

ΠžΠ±Ρ€Π°Π·ΠΎΠ²Π°Π½ΠΈΠ΅:

ΠšΡ€Π°ΡΠ½ΠΎΠ΄Π°Ρ€ΡΠΊΠΈΠΉ государствСнный унивСрситСт ΠΊΡƒΠ»ΡŒΡ‚ΡƒΡ€Ρ‹ ΠΈ искусств

ΠŸΡ€ΠΎΠ²Π΅Π΄Π΅Π½Π½Ρ‹Ρ… занятий:

Π€ΠΎΡ€ΠΌΠ° обучСния:

Дистанционно (Π‘ΠΊΠ°ΠΉΠΏ)

Π’Π΅Π΄Ρƒ занятия ΠΏΠΎ ΠΎΠ±Ρ‰Π΅ΡΡ‚Π²ΠΎΠ·Π½Π°Π½ΠΈΡŽ с ΡƒΡ‡Π΅Π½ΠΈΠΊΠ°ΠΌΠΈ 6-11 классов, Π³ΠΎΡ‚ΠΎΠ²Π»ΡŽ ΠΊ ΡƒΡΠΏΠ΅ΡˆΠ½ΠΎΠΉ сдачС Π’ΠŸΠ /ΠžΠ“Π­/Π•Π“Π­. Π’ Ρ€Π°Π±ΠΎΡ‚Π΅ ΠΈΡΠΏΠΎΠ»ΡŒΠ·ΡƒΡŽΡ‚ΡΡ ΠΎΠ΄ΠΎΠ±Ρ€Π΅Π½Π½Ρ‹Π΅ Π€Π“ΠžΠ‘ ΠΌΠ°Ρ‚Π΅Ρ€ΠΈΠ°Π»Ρ‹: ΠΊΠ°ΠΊ ΡƒΡ‡Π΅Π±Π½ΠΈΠΊΠΈ, Ρ‚Π°ΠΊ ΠΈ ΠΈΠ½Ρ‚Π΅Ρ€Π½Π΅Ρ‚ рСсурсы.

Виктория АлСксандровна Π‘ΠΈΠΌΠ°Π³ΠΎ

Π Π΅ΠΏΠ΅Ρ‚ΠΈΡ‚ΠΎΡ€ ΠΏΠΎ ΠΌΠ°Ρ‚Π΅ΠΌΠ°Ρ‚ΠΈΠΊΠ΅

Π‘Ρ‚Π°ΠΆ (Π»Π΅Ρ‚)

ΠžΠ±Ρ€Π°Π·ΠΎΠ²Π°Π½ΠΈΠ΅:

ГроднСнский государствСнный унивСрситСт ΠΈΠΌ. Π―Π½ΠΊΠΈ ΠšΡƒΠΏΠ°Π»Ρ‹

ΠŸΡ€ΠΎΠ²Π΅Π΄Π΅Π½Π½Ρ‹Ρ… занятий:

Π€ΠΎΡ€ΠΌΠ° обучСния:

Дистанционно (Π‘ΠΊΠ°ΠΉΠΏ)

Π Π΅ΠΏΠ΅Ρ‚ΠΈΡ‚ΠΎΡ€ 1-6 классов. Π’ ΠΌΠ°Ρ‚Π΅ΠΌΠ°Ρ‚ΠΈΠΊΠ΅ Π΅ΡΡ‚ΡŒ Π½Π΅ Ρ‚ΠΎΠ»ΡŒΠΊΠΎ истина, Π½ΠΎ ΠΈ Π½Π°ΠΈΠ²Ρ‹ΡΡˆΠ°Ρ красота! ΠŸΠΎΠΌΠΎΠ³Ρƒ Π’Π°ΠΌ ΠΏΠΎΠ·Π½Π°Ρ‚ΡŒ эту красоту ΠΈ простоту. ΠŸΠΎΠ½ΡΡ‚ΡŒ ΠΌΠ°Ρ‚Π΅ΠΌΠ°Ρ‚ΠΈΠΊΡƒ Π½Π΅ слоТно, эта Π½Π°ΡƒΠΊΠ° — Ρ„Π°ΠΊΡ‚ΠΎΠ². И ΠΊΠ°ΠΊ Π½ΠΈ странно это Π·Π²ΡƒΡ‡ΠΈΡ‚, Π»Π΅Π³Ρ‡Π΅ ΠΏΠΎΠ½ΠΈΠΌΠ°ΡŽΡ‚ этот язык Ρ„Π°ΠΊΡ‚ΠΎΠ² ΠΈΠΌΠ΅Π½Π½ΠΎ Π΄Π΅Ρ‚ΠΈ. И Ρ‡Π΅ΠΌ Ρ€Π°Π½ΡŒΡˆΠ΅ ΠΌΡ‹ Π½Π°Ρ‡ΠΈΠ½Π°Π΅ΠΌ ΡƒΠ΄Π΅Π»ΡΡ‚ΡŒ этому врСмя, Ρ‚Π΅ΠΌ Π±ΠΎΠ»ΡŒΡˆΠΈΡ… высот ΠΌΡ‹ достигаСм! ΠœΡ‹ Π±ΡƒΠ΄Π΅ΠΌ Ρ€Π°Π·Π²ΠΈΠ²Π°Ρ‚ΡŒ ΠΌΠΎΠ·Π³, ΠΎΠ½ Ρ‚Π°ΠΊΠΆΠ΅ ΠΊΠ°ΠΊ мускулы, развиваСтся, ΠΊΠΎΠ³Π΄Π° Π΅Π³ΠΎ Ρ‚Ρ€Π΅Π½ΠΈΡ€ΡƒΠ΅ΡˆΡŒ! Π’ процСссС ΡƒΡ‡Π΅Π±Ρ‹ Π±ΡƒΠ΄Π΅Ρ‚ Ρ€Π°Π·Π²ΠΈΠ²Π°Ρ‚ΡŒΡΡ Π²ΠΎΠΎΠ±Ρ€Π°ΠΆΠ΅Π½ΠΈΠ΅ ΠΈ логичСскоС ΠΌΡ‹ΡˆΠ»Π΅Π½ΠΈΠ΅ ΠΈ самоС Π³Π»Π°Π²Π½ΠΎΠ΅ — ΡƒΠ²Π΅Ρ€Π΅Π½Π½ΠΎΡΡ‚ΡŒ Π² сСбС. Π’Ρ‹ Ρ‚ΠΎΡ‡Π½ΠΎ смоТСшь ΠΎΠ²Π»Π°Π΄Π΅Ρ‚ΡŒ матСматичСскими знаниями, умСниями ΠΈ Π½Π°Π²Ρ‹ΠΊΠ°ΠΌΠΈ Ρ‚Π°ΠΊ Π½Π΅ΠΎΠ±Ρ…ΠΎΠ΄ΠΈΠΌΡ‹ΠΌΠΈ Π² ΠΆΠΈΠ·Π½ΠΈ. Π”ΠΎΠ²Π΅Ρ€ΡŒΡ‚Π΅ΡΡŒ ΠΌΠ½Π΅ ΠΈ ΠΌΠΎΠ΅ΠΌΡƒ ΠΎΠΏΡ‹Ρ‚Ρƒ ΠΈ всС получится!

Π•Π»ΠΈΠ·Π°Π²Π΅Ρ‚Π° Π‘ΠΈΠΌΠ±Π΅Ρ‚ΠΎΠ²Π½Π° ВулСмисова

Π Π΅ΠΏΠ΅Ρ‚ΠΈΡ‚ΠΎΡ€ ΠΏΠΎ ΠΌΠ°Ρ‚Π΅ΠΌΠ°Ρ‚ΠΈΠΊΠ΅

Π‘Ρ‚Π°ΠΆ (Π»Π΅Ρ‚)

ΠžΠ±Ρ€Π°Π·ΠΎΠ²Π°Π½ΠΈΠ΅:

Астраханский государствСнный пСдагогичСский институт

ΠŸΡ€ΠΎΠ²Π΅Π΄Π΅Π½Π½Ρ‹Ρ… занятий:

Π€ΠΎΡ€ΠΌΠ° обучСния:

Дистанционно (Π‘ΠΊΠ°ΠΉΠΏ)

Π‘Π΅Ρ€Π΄Π΅Ρ‡Π½ΠΎ всСх ΠΏΡ€ΠΈΠ²Π΅Ρ‚ΡΡ‚Π²ΡƒΡŽ! МСня Π·ΠΎΠ²ΡƒΡ‚ Π•Π»ΠΈΠ·Π°Π²Π΅Ρ‚Π° Π‘ΠΈΠΌΠ±Π΅Ρ‚ΠΎΠ²Π½Π°, ΡƒΡ‡ΠΈΡ‚Π΅Π»ΡŒ Π²Ρ‹ΡΡˆΠ΅ΠΉ ΠΊΠ°Ρ‚Π΅Π³ΠΎΡ€ΠΈΠΈ, ΠŸΠΎΡ‡Π΅Ρ‚Π½Ρ‹ΠΉ Ρ€Π°Π±ΠΎΡ‚Π½ΠΈΠΊ воспитания ΠΈ образования Π Π€, 2020 Π³. . ИмСю большой ΠΎΠΏΡ‹Ρ‚ обучСния иностранным языкам (Π½Π΅ΠΌΠ΅Ρ†ΠΊΠΈΠΉ ΠΈ английский), послСдниС 18 Π»Π΅Ρ‚ Ρ€Π°Π±ΠΎΡ‚Π°Π»Π° ΡƒΡ‡ΠΈΡ‚Π΅Π»Π΅ΠΌ Π² лингвистичСской Π³ΠΈΠΌΠ½Π°Π·ΠΈΠΈ. Π•ΡΡ‚ΡŒ Π±ΠΎΠ³Π°Ρ‚Ρ‹ΠΉ ΠΎΠΏΡ‹Ρ‚ Ρ€Π°Π±ΠΎΡ‚Ρ‹ с ΡƒΡ‡Π΅Π±Π½Ρ‹ΠΌΠΈ пособиями иностранных ΠΈΠ·Π΄Π°Ρ‚Π΅Π»ΡŒΡΡ‚Π² Hueber, Oxford, Cambridge, ΠΏΠΎΠ΄Π³ΠΎΡ‚ΠΎΠ²ΠΊΠΈ ΡƒΡ‡Π΅Π½ΠΈΠΊΠΎΠ² ΠΊ ΠΌΠ΅ΠΆΠ΄ΡƒΠ½Π°Ρ€ΠΎΠ΄Π½Ρ‹ΠΌ экзамСнам. Π’Π»Π°Π΄Π΅ΡŽ ΠΊΠΎΠΌΠΌΡƒΠ½ΠΈΠΊΠ°Ρ‚ΠΈΠ²Π½ΠΎΠΉ ΠΌΠ΅Ρ‚ΠΎΠ΄ΠΈΠΊΠΎΠΉ обучСния ΠΈ ΠΎΡ€ΠΈΠ΅Π½Ρ‚ΠΈΡ€ΡƒΡŽΡΡŒ Π½Π° Π»ΠΈΡ‡Π½ΠΎΡΡ‚ΡŒ ΡƒΡ‡Π΅Π½ΠΈΠΊΠ°, Π΅Π³ΠΎ поТСлания, Ρ†Π΅Π»ΠΈ, ΡƒΡ€ΠΎΠ²Π΅Π½ΡŒ владСния иностранным языком. Π“ΠΎΡ‚ΠΎΠ²Π»ΡŽ ΠΊ Π•Π“Π­, ΠžΠ“Π­, Π’ΠŸΠ , Ρ€Π΅Π·ΡƒΠ»ΡŒΡ‚Π°Ρ‚Ρ‹ ΠΎΡ‚ 65 Π΄ΠΎ 100 Π±Π°Π»Π»ΠΎΠ². Выпускники ΡƒΡΠΏΠ΅ΡˆΠ½ΠΎ ΠΎΠ±ΡƒΡ‡Π°ΡŽΡ‚ΡΡ Π² Π²Π΅Π΄ΡƒΡ‰ΠΈΡ… Π²ΡƒΠ·Π°Ρ… страны, Π° Ρ‚Π°ΠΊΠΆΠ΅ Π·Π° Ρ€ΡƒΠ±Π΅ΠΆΠΎΠΌ. Π‘ΡƒΠ΄Ρƒ Ρ€Π°Π΄Π° сотрудничСству! Π”ΠΎ встрСчи!

ΠŸΠΎΡ…ΠΎΠΆΠΈΠ΅ ΡΡ‚Π°Ρ‚ΡŒΠΈ

  • Π–ΠΈΠ·Π½ΠΈ ΠΌΠ°Ρ‚Π΅ΠΌΠ°Ρ‚ΠΈΠΊΠΎΠ² (Ρ‡Π°ΡΡ‚ΡŒ 2)
  • Π‘Ρ€Π΅Π΄Π½Π΅Π΅ арифмСтичСскоС
  • РСшСниС нСравСнств с ΠΌΠΎΠ΄ΡƒΠ»Π΅ΠΌ
  • Ѐинансовый УнивСрситСт: ΠŸΡ€ΠΈΠΊΠ»Π°Π΄Π½Π°Ρ ΠœΠ°Ρ‚Π΅ΠΌΠ°Ρ‚ΠΈΠΊΠ° ΠΈ Π˜Π½Ρ„ΠΎΡ€ΠΌΠ°Ρ‚ΠΈΠΊΠ°
  • ΠŸΡ€Π΅ΠΎΠ±Ρ€Π°Π·ΠΎΠ²Π°Π½ΠΈΠ΅ алгСбраичСских Π²Ρ‹Ρ€Π°ΠΆΠ΅Π½ΠΈΠΉ
  • Как ΡˆΠΊΠΎΠ»ΡŒΠ½ΠΈΠΊΡƒ Π²Ρ‹ΡΡ‚Ρ€ΠΎΠΈΡ‚ΡŒ ΠΈΠ΄Π΅Π°Π»ΡŒΠ½Ρ‹Π΅ ΠΎΡ‚Π½ΠΎΡˆΠ΅Π½ΠΈΡ с родитСлями
  • Как Π²Ρ‹Π±Ρ€Π°Ρ‚ΡŒ Ρ€Π΅Π±Π΅Π½ΠΊΡƒ коньки ΠΈ Π½Π°ΡƒΡ‡ΠΈΡ‚ΡŒ ΠΊΠ°Ρ‚Π°Ρ‚ΡŒΡΡ
  • Π˜ΡΡ‚ΠΎΡ€ΠΈΡ происхоТдСния ΠΈ смысл пословиц ΠΈ ΠΏΠΎΠ³ΠΎΠ²ΠΎΡ€ΠΎΠΊ

НаТимая ΠΊΠ½ΠΎΠΏΠΊΡƒ «Π—Π°ΠΏΠΈΡΠ°Ρ‚ΡŒΡΡ» ΠΏΡ€ΠΈΠ½ΠΈΠΌΠ°ΡŽ условия ΠŸΠΎΠ»ΡŒΠ·ΠΎΠ²Π°Ρ‚Π΅Π»ΡŒΡΠΊΠΎΠ³ΠΎ соглашСния ΠΈ ΠŸΠΎΠ»ΠΈΡ‚ΠΈΠΊΠΈ ΠΊΠΎΠ½Ρ„ΠΈΠ΄Π΅Π½Ρ†ΠΈΠ°Π»ΡŒΠ½ΠΎΡΡ‚ΠΈ

Π£ΠΌΠ½ΠΎΠΆΠ΅Π½ΠΈΠ΅ ΠΎΡ‚Ρ€ΠΈΡ†Π°Ρ‚Π΅Π»ΡŒΠ½Ρ‹Ρ… чисСл ΠΊΡ€Π°Ρ‚ΠΊΠΎ АрифмСтика

Π£ΠΌΠ½ΠΎΠΆΠ΅Π½ΠΈΠ΅ ΠΎΡ‚Ρ€ΠΈΡ†Π°Ρ‚Π΅Π»ΡŒΠ½Ρ‹Ρ… чисСл ΠΊΡ€Π°Ρ‚ΠΊΠΎ АрифмСтика

ΠŸΡ€ΠΈΠ²Π΅Ρ‚, ΠΌΠΎΠΉ Π΄Ρ€ΡƒΠ³, Ρ‚Π΅Π±Π΅ интСрСсно ΡƒΠ·Π½Π°Ρ‚ΡŒ всС ΠΏΡ€ΠΎ ΡƒΠΌΠ½ΠΎΠΆΠ΅Π½ΠΈΠ΅ ΠΎΡ‚Ρ€ΠΈΡ†Π°Ρ‚Π΅Π»ΡŒΠ½Ρ‹Ρ… чисСл, Ρ‚ΠΎΠ³Π΄Π° с Π²Π΄ΠΎΡ…Π½ΠΎΠ²Π΅Π½ΠΈΠ΅ΠΌ ΠΏΡ€ΠΎΡ‡Ρ‚ΠΈ Π΄ΠΎ ΠΊΠΎΠ½Ρ†Π°. Для Ρ‚ΠΎΠ³ΠΎ Ρ‡Ρ‚ΠΎΠ±Ρ‹ Π»ΡƒΡ‡ΡˆΠ΅ ΠΏΠΎΠ½ΠΈΠΌΠ°Ρ‚ΡŒ Ρ‡Ρ‚ΠΎ Ρ‚Π°ΠΊΠΎΠ΅ ΡƒΠΌΠ½ΠΎΠΆΠ΅Π½ΠΈΠ΅ ΠΎΡ‚Ρ€ΠΈΡ†Π°Ρ‚Π΅Π»ΡŒΠ½Ρ‹Ρ… чисСл , Π½Π°ΡΡ‚ΠΎΡΡ‚Π΅Π»ΡŒΠ½ΠΎ Ρ€Π΅ΠΊΠΎΠΌΠ΅Π½Π΄ΡƒΡŽ ΠΏΡ€ΠΎΡ‡ΠΈΡ‚Π°Ρ‚ΡŒ всС ΠΈΠ· ΠΊΠ°Ρ‚Π΅Π³ΠΎΡ€ΠΈΠΈ АрифмСтика.

Π˜ΡΠΏΠΎΠ»ΡŒΠ·ΡƒΡ понятиС модуля числа, сформулируСм ΠΏΡ€Π°Π²ΠΈΠ»Π° умноТСния ΠΏΠΎΠ»ΠΎΠΆΠΈΡ‚Π΅Π»ΡŒΠ½Ρ‹Ρ… ΠΈ ΠΎΡ‚Ρ€ΠΈΡ†Π°Ρ‚Π΅Π»ΡŒΠ½Ρ‹Ρ… чисСл.

Π£ΠΌΠ½ΠΎΠΆΠ΅Π½ΠΈΠ΅ чисСл с ΠΎΠ΄ΠΈΠ½Π°ΠΊΠΎΠ²Ρ‹ΠΌΠΈ Π·Π½Π°ΠΊΠ°ΠΌΠΈ

ΠŸΠ΅Ρ€Π²Ρ‹ΠΉ случай, ΠΊΠΎΡ‚ΠΎΡ€Ρ‹ΠΉ ΠΌΠΎΠΆΠ΅Ρ‚ Π²Π°ΠΌ Π²ΡΡ‚Ρ€Π΅Ρ‚ΠΈΡ‚ΡŒΡΡ — это ΡƒΠΌΠ½ΠΎΠΆΠ΅Π½ΠΈΠ΅ чисСл с ΠΎΠ΄ΠΈΠ½Π°ΠΊΠΎΠ²Ρ‹ΠΌΠΈ Π·Π½Π°ΠΊΠ°ΠΌΠΈ.

Π§Ρ‚ΠΎΠ±Ρ‹ ΡƒΠΌΠ½ΠΎΠΆΠΈΡ‚ΡŒ Π΄Π²Π° числа с ΠΎΠ΄ΠΈΠ½Π°ΠΊΠΎΠ²Ρ‹ΠΌΠΈ Π·Π½Π°ΠΊΠ°ΠΌΠΈ Π½Π°Π΄ΠΎ:

  • ΠΏΠ΅Ρ€Π΅ΠΌΠ½ΠΎΠΆΠΈΡ‚ΡŒ ΠΌΠΎΠ΄ΡƒΠ»ΠΈ чисСл;
  • ΠΏΠ΅Ρ€Π΅Π΄ ΠΏΠΎΠ»ΡƒΡ‡Π΅Π½Π½Ρ‹ΠΌ ΠΏΡ€ΠΎΠΈΠ·Π²Π΅Π΄Π΅Π½ΠΈΠ΅ΠΌ ΠΏΠΎΡΡ‚Π°Π²ΠΈΡ‚ΡŒ Π·Π½Π°ΠΊ Β«+Β» (ΠΏΡ€ΠΈ записи ΠΎΡ‚Π²Π΅Ρ‚Π° Π·Π½Π°ΠΊ «плюс» ΠΏΠ΅Ρ€Π΅Π΄ ΠΏΠ΅Ρ€Π²Ρ‹ΠΌ числом слСва ΠΌΠΎΠΆΠ½ΠΎ ΠΎΠΏΡƒΡΠΊΠ°Ρ‚ΡŒ).

ΠŸΡ€ΠΈΠΌΠ΅Ρ€Ρ‹ умноТСния ΠΎΡ‚Ρ€ΠΈΡ†Π°Ρ‚Π΅Π»ΡŒΠ½Ρ‹Ρ… ΠΈ ΠΏΠΎΠ»ΠΎΠΆΠΈΡ‚Π΅Π»ΡŒΠ½Ρ‹Ρ… чисСл.

  • (- 3) β€’ (- 6) = + 18 = 18
  • 2 β€’ 3 = 6

Π£ΠΌΠ½ΠΎΠΆΠ΅Π½ΠΈΠ΅ чисСл с Ρ€Π°Π·Π½Ρ‹ΠΌΠΈ Π·Π½Π°ΠΊΠ°ΠΌΠΈ

Π’Ρ‚ΠΎΡ€ΠΎΠΉ Π²ΠΎΠ·ΠΌΠΎΠΆΠ½Ρ‹ΠΉ случай — это ΡƒΠΌΠ½ΠΎΠΆΠ΅Π½ΠΈΠ΅ чисСл с Ρ€Π°Π·Π½Ρ‹ΠΌΠΈ Π·Π½Π°ΠΊΠ°ΠΌΠΈ.

Π§Ρ‚ΠΎΠ±Ρ‹ ΡƒΠΌΠ½ΠΎΠΆΠΈΡ‚ΡŒ Π΄Π²Π° числа с Ρ€Π°Π·Π½Ρ‹ΠΌΠΈ Π·Π½Π°ΠΊΠ°ΠΌΠΈ, Π½Π°Π΄ΠΎ:

  • ΠΏΠ΅Ρ€Π΅ΠΌΠ½ΠΎΠΆΠΈΡ‚ΡŒ ΠΌΠΎΠ΄ΡƒΠ»ΠΈ чисСл;
  • ΠΏΠ΅Ρ€Π΅Π΄ ΠΏΠΎΠ»ΡƒΡ‡Π΅Π½Π½Ρ‹ΠΌ ΠΏΡ€ΠΎΠΈΠ·Π²Π΅Π΄Π΅Π½ΠΈΠ΅ΠΌ ΠΏΠΎΡΡ‚Π°Π²ΠΈΡ‚ΡŒ Π·Π½Π°ΠΊ Β«-Β».

ΠŸΡ€ΠΈΠΌΠ΅Ρ€Ρ‹ умноТСния ΠΎΡ‚Ρ€ΠΈΡ†Π°Ρ‚Π΅Π»ΡŒΠ½Ρ‹Ρ… ΠΈ ΠΏΠΎΠ»ΠΎΠΆΠΈΡ‚Π΅Π»ΡŒΠ½Ρ‹Ρ… чисСл.

  • (- 0,3) β€’ 0,5 = — 1,5
  • 1,2 β€’ (- 7) = — 8,4

ΠŸΡ€Π°Π²ΠΈΠ»Π° Π·Π½Π°ΠΊΠΎΠ² для умноТСния

Π—Π°ΠΏΠΎΠΌΠ½ΠΈΡ‚ΡŒ ΠΏΡ€Π°Π²ΠΈΠ»ΠΎ Π·Π½Π°ΠΊΠΎΠ² для умноТСния ΠΎΡ‡Π΅Π½ΡŒ просто . Об этом Π³ΠΎΠ²ΠΎΡ€ΠΈΡ‚ сайт https://intellect.icu . Π”Π°Π½Π½ΠΎΠ΅ ΠΏΡ€Π°Π²ΠΈΠ»ΠΎ совпадаСт с ΠΏΡ€Π°Π²ΠΈΠ»ΠΎΠΌ раскрытия скобок.

ΠœΠΈΠ½ΡƒΡ Π½Π° минус Π΄Π°Π΅Ρ‚ плюс,

Плюс Π½Π° минус Π΄Π°Π΅Ρ‚ минус.

+ β€’ (+) = + + β€’ (-) = —
— β€’ (-) = + — β€’ (+) = —

Π’ Β«Π΄Π»ΠΈΠ½Π½Ρ‹Ρ…Β» ΠΏΡ€ΠΈΠΌΠ΅Ρ€Π°Ρ…, Π² ΠΊΠΎΡ‚ΠΎΡ€Ρ‹Ρ… Π΅ΡΡ‚ΡŒ Ρ‚ΠΎΠ»ΡŒΠΊΠΎ дСйствиС ΡƒΠΌΠ½ΠΎΠΆΠ΅Π½ΠΈΠ΅, Π·Π½Π°ΠΊ произвСдСния ΠΌΠΎΠΆΠ½ΠΎ ΠΎΠΏΡ€Π΅Π΄Π΅Π»ΡΡ‚ΡŒ ΠΏΠΎ количСству ΠΎΡ‚Ρ€ΠΈΡ†Π°Ρ‚Π΅Π»ΡŒΠ½Ρ‹Ρ… ΠΌΠ½ΠΎΠΆΠΈΡ‚Π΅Π»Π΅ΠΉ.

ΠŸΡ€ΠΈ Ρ‡Π΅Ρ‚Π½ΠΎΠΌ числС ΠΎΡ‚Ρ€ΠΈΡ†Π°Ρ‚Π΅Π»ΡŒΠ½Ρ‹Ρ… ΠΌΠ½ΠΎΠΆΠΈΡ‚Π΅Π»Π΅ΠΉ Ρ€Π΅Π·ΡƒΠ»ΡŒΡ‚Π°Ρ‚ Π±ΡƒΠ΄Π΅Ρ‚ ΠΏΠΎΠ»ΠΎΠΆΠΈΡ‚Π΅Π»ΡŒΠ½Ρ‹ΠΌ, Π° ΠΏΡ€ΠΈ Π½Π΅Ρ‡Π΅Ρ‚Π½ΠΎΠΌ количСствС — ΠΎΡ‚Ρ€ΠΈΡ†Π°Ρ‚Π΅Π»ΡŒΠ½Ρ‹ΠΌ.

ΠŸΡ€ΠΈΠΌΠ΅Ρ€.

(- 6) β€’ (- 3) β€’ (- 4) β€’ (- 2) β€’ 12 β€’ (- 1) =

Π’ ΠΏΡ€ΠΈΠΌΠ΅Ρ€Π΅ ΠΏΡΡ‚ΡŒ ΠΎΡ‚Ρ€ΠΈΡ†Π°Ρ‚Π΅Π»ΡŒΠ½Ρ‹Ρ… ΠΌΠ½ΠΎΠΆΠΈΡ‚Π΅Π»Π΅ΠΉ. Π—Π½Π°Ρ‡ΠΈΡ‚, Π·Π½Π°ΠΊ Ρ€Π΅Π·ΡƒΠ»ΡŒΡ‚Π°Ρ‚Π° Π±ΡƒΠ΄Π΅Ρ‚ «минус».

Π’Π΅ΠΏΠ΅Ρ€ΡŒ вычислим ΠΏΡ€ΠΎΠΈΠ·Π²Π΅Π΄Π΅Π½ΠΈΠ΅ ΠΌΠΎΠ΄ΡƒΠ»Π΅ΠΉ, Π½Π΅ обращая Π²Π½ΠΈΠΌΠ°Π½ΠΈΠ΅ Π½Π° Π·Π½Π°ΠΊΠΈ.

6 β€’ 3 β€’ 4 β€’ 2 β€’ 12 β€’ 1 = 1728

ΠšΠΎΠ½Π΅Ρ‡Π½Ρ‹ΠΉ Ρ€Π΅Π·ΡƒΠ»ΡŒΡ‚Π°Ρ‚ умноТСния исходных чисСл Π±ΡƒΠ΄Π΅Ρ‚:

(- 6) β€’ (- 3) β€’ (- 4) β€’ (- 2) β€’ 12 β€’ (- 1) = — 1728

Π£ΠΌΠ½ΠΎΠΆΠ΅Π½ΠΈΠ΅ Π½Π° ноль ΠΈ Π΅Π΄ΠΈΠ½ΠΈΡ†Ρƒ

Если срСди ΠΌΠ½ΠΎΠΆΠΈΡ‚Π΅Π»Π΅ΠΉ Π΅ΡΡ‚ΡŒ число ноль ΠΈΠ»ΠΈ ΠΏΠΎΠ»ΠΎΠΆΠΈΡ‚Π΅Π»ΡŒΠ½Π°Ρ Π΅Π΄ΠΈΠ½ΠΈΡ†Π°, Ρ‚ΠΎ ΡƒΠΌΠ½ΠΎΠΆΠ΅Π½ΠΈΠ΅ выполняСтся ΠΏΠΎ извСстным ΠΏΡ€Π°Π²ΠΈΠ»Π°ΠΌ.

  • 0 β€’ a = 0
  • a β€’ 0 = 0
  • a β€’ 1 = a

ΠŸΡ€ΠΈΠΌΠ΅Ρ€Ρ‹:

  • 0 β€’ (- 3) = 0
  • 0,4 β€’ 1 = 0,4

ΠžΡΠΎΠ±ΡƒΡŽ Ρ€ΠΎΠ»ΡŒ ΠΏΡ€ΠΈ ΡƒΠΌΠ½ΠΎΠΆΠ΅Π½ΠΈΠΈ Ρ€Π°Ρ†ΠΈΠΎΠ½Π°Π»ΡŒΠ½Ρ‹Ρ… чисСл ΠΈΠ³Ρ€Π°Π΅Ρ‚ ΠΎΡ‚Ρ€ΠΈΡ†Π°Ρ‚Π΅Π»ΡŒΠ½Π°Ρ Π΅Π΄ΠΈΠ½ΠΈΡ†Π° (- 1).

ΠŸΡ€ΠΈ ΡƒΠΌΠ½ΠΎΠΆΠ΅Π½ΠΈΠΈ Π½Π° (- 1) число мСняСтся Π½Π° ΠΏΡ€ΠΎΡ‚ΠΈΠ²ΠΎΠΏΠΎΠ»ΠΎΠΆΠ½ΠΎΠ΅.

Π’ Π±ΡƒΠΊΠ²Π΅Π½Π½ΠΎΠΌ Π²Ρ‹Ρ€Π°ΠΆΠ΅Π½ΠΈΠΈ это свойство ΠΌΠΎΠΆΠ½ΠΎ Π·Π°ΠΏΠΈΡΠ°Ρ‚ΡŒ:

a β€’ (- 1) = (- 1) β€’ a = — a

ΠŸΡ€ΠΈ совмСстном Π²Ρ‹ΠΏΠΎΠ»Π½Π΅Π½ΠΈΠΈ слоТСния, вычитания ΠΈ умноТСния Ρ€Π°Ρ†ΠΈΠΎΠ½Π°Π»ΡŒΠ½Ρ‹Ρ… чисСл сохраняСтся порядок дСйствий, установлСнный для ΠΏΠΎΠ»ΠΎΠΆΠΈΡ‚Π΅Π»ΡŒΠ½Ρ‹Ρ… чисСл ΠΈ нуля.

ΠŸΡ€ΠΈΠΌΠ΅Ρ€ умноТСния ΠΎΡ‚Ρ€ΠΈΡ†Π°Ρ‚Π΅Π»ΡŒΠ½Ρ‹Ρ… ΠΈ ΠΏΠΎΠ»ΠΎΠΆΠΈΡ‚Π΅Π»ΡŒΠ½Ρ‹Ρ… чисСл.

Π‘ΠΌ. Ρ‚Π°ΠΊΠΆΠ΅

  • слоТСниС ΠΎΡ‚Ρ€ΠΈΡ†Π°Ρ‚Π΅Π»ΡŒΠ½Ρ‹Ρ… чисСл ,
  • Π΄Π΅Π»Π΅Π½ΠΈΠ΅ ΠΎΡ‚Ρ€ΠΈΡ†Π°Ρ‚Π΅Π»ΡŒΠ½Ρ‹Ρ… чисСл ,
  • ΠΌΠ΅Ρ‚ΠΎΠ΄Ρ‹ Π΄Π΅ΠΊΠΎΠΌΠΏΠΎΠ·ΠΈΡ†ΠΈΠΈ ячССк ,
  • дСсятичныС Π΄Ρ€ΠΎΠ±ΠΈ ,
  • ΡƒΠΌΠ½ΠΎΠΆΠ΅Π½ΠΈΠ΅ Π² столбик ,
  • ΠΊΠ°ΠΊ Ρ‡ΠΈΡ‚Π°Ρ‚ΡŒ дСсятичныС Π΄Ρ€ΠΎΠ±ΠΈ ,
  • ΠΏΠ΅Ρ€Π΅Π²ΠΎΠ΄ ΠΎΠ±Ρ‹ΠΊΠ½ΠΎΠ²Π΅Π½Π½ΠΎΠΉ Π΄Ρ€ΠΎΠ±ΠΈ Π² Π΄Π΅ΡΡΡ‚ΠΈΡ‡Π½ΡƒΡŽ ,
  • Π½Π°Ρ…ΠΎΠΆΠ΄Π΅Π½ΠΈΠ΅ ΠΎΠ±Ρ‹ΠΊΠ½ΠΎΠ²Π΅Π½Π½ΠΎΠΉ Π΄Ρ€ΠΎΠ±ΠΈ ΠΎΡ‚ числа ,
  • Π΄Π΅Π»Π΅Π½ΠΈΠ΅ ΠΎΠ±Ρ‹ΠΊΠ½ΠΎΠ²Π΅Π½Π½Ρ‹Ρ… Π΄Ρ€ΠΎΠ±Π΅ΠΉ ,
  • ΡƒΠΌΠ½ΠΎΠΆΠ΅Π½ΠΈΠ΅ ΠΎΠ±Ρ‹ΠΊΠ½ΠΎΠ²Π΅Π½Π½Ρ‹Ρ… Π΄Ρ€ΠΎΠ±Π΅ΠΉ ,
  • Π²Ρ‹Ρ‡ΠΈΡ‚Π°Π½ΠΈΠ΅ ΠΎΠ±Ρ‹ΠΊΠ½ΠΎΠ²Π΅Π½Π½Ρ‹Ρ… Π΄Ρ€ΠΎΠ±Π΅ΠΉ ,
  • Π²Π·Π°ΠΈΠΌΠ½ΠΎ ΠΎΠ±Ρ€Π°Ρ‚Π½Ρ‹Π΅ числа , Π²Π·Π°ΠΈΠΌΠ½ΠΎ ΠΎΠ±Ρ€Π°Ρ‚Π½Ρ‹Π΅ Π΄Ρ€ΠΎΠ±ΠΈ ,
  • сравнСниС ΠΎΠ±Ρ‹ΠΊΠ½ΠΎΠ²Π΅Π½Π½Ρ‹Ρ… Π΄Ρ€ΠΎΠ±Π΅ΠΉ ,
  • пСриодичСская Π΄Ρ€ΠΎΠ±ΡŒ ,
  • слоТСниС ΠΎΠ±Ρ‹ΠΊΠ½ΠΎΠ²Π΅Π½Π½Ρ‹Ρ… Π΄Ρ€ΠΎΠ±Π΅ΠΉ , ΠΎΠ±Ρ‰ΠΈΠΉ Π·Π½Π°ΠΌΠ΅Π½Π°Ρ‚Π΅Π»ΡŒ ,
  • сокращСниС ΠΎΠ±Ρ‹ΠΊΠ½ΠΎΠ²Π΅Π½Π½Ρ‹Ρ… Π΄Ρ€ΠΎΠ±Π΅ΠΉ ,
  • ΡΠΌΠ΅ΡˆΠ°Π½Π½Ρ‹Π΅ числа , Π²Ρ‹Π΄Π΅Π»Π΅Π½ΠΈΠ΅ Ρ†Π΅Π»ΠΎΠΉ части ΠΎΠ±Ρ‹ΠΊΠ½ΠΎΠ²Π΅Π½Π½ΠΎΠΉ Π΄Ρ€ΠΎΠ±ΠΈ ,
  • свойства умноТСния , свойства дСлСния ,

Как Ρ‚Ρ‹ ΡΡ‡ΠΈΡ‚Π°Π΅Π΅ΡˆΡŒ, Π±ΡƒΠ΄Π΅Ρ‚ Π»ΠΈ тСория ΠΏΡ€ΠΎ ΡƒΠΌΠ½ΠΎΠΆΠ΅Π½ΠΈΠ΅ ΠΎΡ‚Ρ€ΠΈΡ†Π°Ρ‚Π΅Π»ΡŒΠ½Ρ‹Ρ… чисСл ΡƒΠ»ΡƒΡ‡ΡˆΠ΅Π½Π° Π² ΠΎΠ±ΠΎΠ·Ρ€ΠΈΠΌΠΎΠΌ Π±ΡƒΠ΄ΡƒΡŽΡ‰Π΅ΠΌ? НадСюсь, Ρ‡Ρ‚ΠΎ Ρ‚Π΅ΠΏΠ΅Ρ€ΡŒ Ρ‚Ρ‹ понял Ρ‡Ρ‚ΠΎ Ρ‚Π°ΠΊΠΎΠ΅ ΡƒΠΌΠ½ΠΎΠΆΠ΅Π½ΠΈΠ΅ ΠΎΡ‚Ρ€ΠΈΡ†Π°Ρ‚Π΅Π»ΡŒΠ½Ρ‹Ρ… чисСл ΠΈ для Ρ‡Π΅Π³ΠΎ всС это Π½ΡƒΠΆΠ½ΠΎ, Π° Ссли Π½Π΅ понял, ΠΈΠ»ΠΈ Π΅ΡΡ‚ΡŒ замСчания, Ρ‚ΠΎ нСстСсняся пиши ΠΈΠ»ΠΈ ΡΠΏΡ€Π°ΡˆΠΈΠ²Π°ΠΉ Π² коммСнтариях, с ΡƒΠ΄ΠΎΠ²ΠΎΠ»ΡŒΡΡ‚Π²ΠΈΠ΅ΠΌ ΠΎΡ‚Π²Π΅Ρ‡Ρƒ. Для Ρ‚ΠΎΠ³ΠΎ Ρ‡Ρ‚ΠΎΠ±Ρ‹ Π³Π»ΡƒΠ±ΠΆΠ΅ ΠΏΠΎΠ½ΡΡ‚ΡŒ Π½Π°ΡΡ‚ΠΎΡΡ‚Π΅Π»ΡŒΠ½ΠΎ Ρ€Π΅ΠΊΠΎΠΌΠ΅Π½Π΄ΡƒΡŽ ΠΈΠ·ΡƒΡ‡ΠΈΡ‚ΡŒ всю ΠΈΠ½Ρ„ΠΎΡ€ΠΌΠ°Ρ†ΠΈΡŽ ΠΈΠ· ΠΊΠ°Ρ‚Π΅Π³ΠΎΡ€ΠΈΠΈ АрифмСтика

Из ΡΡ‚Π°Ρ‚ΡŒΠΈ ΠΌΡ‹ ΡƒΠ·Π½Π°Π»ΠΈ ΠΊΡ€Π°Ρ‚ΠΊΠΎ, Π½ΠΎ Π΅ΠΌΠΊΠΎ ΠΏΡ€ΠΎ ΡƒΠΌΠ½ΠΎΠΆΠ΅Π½ΠΈΠ΅ ΠΎΡ‚Ρ€ΠΈΡ†Π°Ρ‚Π΅Π»ΡŒΠ½Ρ‹Ρ… чисСл

Π§Ρ‚ΠΎ Π±ΡƒΠ΄Π΅Ρ‚, Ссли Π΄Ρ€ΠΎΠ±ΡŒ ΡƒΠΌΠ½ΠΎΠΆΠΈΡ‚ΡŒ Π½Π° минус 1? – ΠžΠ±Π·ΠΎΡ€Ρ‹ Π’ΠΈΠΊΠΈ

Если ΠΎΠ±Π΅ Π΄Ρ€ΠΎΠ±ΠΈ ΠΈΠΌΠ΅ΡŽΡ‚ ΠΎΠ΄ΠΈΠ½Π°ΠΊΠΎΠ²Ρ‹Π΅ Π·Π½Π°ΠΊΠΈ, Π»ΠΈΠ±ΠΎ ΠΎΠ±Π΅ ΠΏΠΎΠ»ΠΎΠΆΠΈΡ‚Π΅Π»ΡŒΠ½Ρ‹Π΅, Π»ΠΈΠ±ΠΎ ΠΎΠ±Π΅ ΠΎΡ‚Ρ€ΠΈΡ†Π°Ρ‚Π΅Π»ΡŒΠ½Ρ‹Π΅, ΠΎΡ‚Π²Π΅Ρ‚ Π±ΡƒΠ΄Π΅Ρ‚ ΠΏΠΎΠ»ΠΎΠΆΠΈΡ‚Π΅Π»ΡŒΠ½Ρ‹ΠΌ. Вис ΠΎΠ±Π΅ Π΄Ρ€ΠΎΠ±ΠΈ ΠΈΠΌΠ΅ΡŽΡ‚ Ρ€Π°Π·Π½Ρ‹Π΅ Π·Π½Π°ΠΊΠΈ, ΠΎΠ΄ΠΈΠ½ ΠΏΠΎΠ»ΠΎΠΆΠΈΡ‚Π΅Π»ΡŒΠ½Ρ‹ΠΉ ΠΈ ΠΎΠ΄ΠΈΠ½ ΠΎΡ‚Ρ€ΠΈΡ†Π°Ρ‚Π΅Π»ΡŒΠ½Ρ‹ΠΉ, ΠΎΡ‚Π²Π΅Ρ‚ Π±ΡƒΠ΄Π΅Ρ‚ ΠΎΡ‚Ρ€ΠΈΡ†Π°Ρ‚Π΅Π»ΡŒΠ½Ρ‹ΠΌ.

Π”ΠΎΠΏΠΎΠ»Π½ΠΈΡ‚Π΅Π»ΡŒΠ½ΠΎ Π§Ρ‚ΠΎ происходит, ΠΊΠΎΠ³Π΄Π° Π²Ρ‹ ΡƒΠΌΠ½ΠΎΠΆΠ°Π΅Ρ‚Π΅ ΠΎΡ‚Ρ€ΠΈΡ†Π°Ρ‚Π΅Π»ΡŒΠ½ΠΎΠ΅ Π·Π½Π°Ρ‡Π΅Π½ΠΈΠ΅ Π½Π° ΠΎΡ‚Ρ€ΠΈΡ†Π°Ρ‚Π΅Π»ΡŒΠ½ΠΎΠ΅? ΠžΡ‚Π²Π΅Ρ‚: Когда Π²Ρ‹ ΡƒΠΌΠ½ΠΎΠΆΠ°Π΅Ρ‚Π΅ ΠΎΡ‚Ρ€ΠΈΡ†Π°Ρ‚Π΅Π»ΡŒΠ½ΠΎΠ΅ Π·Π½Π°Ρ‡Π΅Π½ΠΈΠ΅ Π½Π° ΠΎΡ‚Ρ€ΠΈΡ†Π°Ρ‚Π΅Π»ΡŒΠ½ΠΎΠ΅, Π²Ρ‹ ΠΏΠΎΠ»ΡƒΡ‡ΠΈΡ‚ΡŒ ΠΏΠΎΠ»ΠΎΠΆΠΈΡ‚Π΅Π»ΡŒΠ½Ρ‹ΠΉ Ρ€Π΅Π·ΡƒΠ»ΡŒΡ‚Π°Ρ‚, ΠΏΠΎΡ‚ΠΎΠΌΡƒ Ρ‡Ρ‚ΠΎ Π΄Π²Π° ΠΎΡ‚Ρ€ΠΈΡ†Π°Ρ‚Π΅Π»ΡŒΠ½Ρ‹Ρ… Π·Π½Π°ΠΊΠ° ΡΠΎΠΊΡ€Π°Ρ‰Π°ΡŽΡ‚ΡΡ. Π’Π°ΠΊΠΈΠΌ ΠΎΠ±Ρ€Π°Π·ΠΎΠΌ, минус, ΡƒΠΌΠ½ΠΎΠΆΠ΅Π½Π½Ρ‹ΠΉ Π½Π° минус, всСгда Π±ΡƒΠ΄Π΅Ρ‚ плюсом.

Как ΡƒΠΌΠ½ΠΎΠΆΠΈΡ‚ΡŒ ΠΎΡ‚Ρ€ΠΈΡ†Π°Ρ‚Π΅Π»ΡŒΠ½ΡƒΡŽ Π΄Ρ€ΠΎΠ±ΡŒ Π½Π° смСшанноС число?

Как Π΄Π΅Π»ΠΈΡ‚ΡŒ Π½Π° ΠΎΡ‚Ρ€ΠΈΡ†Π°Ρ‚Π΅Π»ΡŒΠ½ΠΎΠ΅ число? ΠŸΡ€ΠΈ Π΄Π΅Π»Π΅Π½ΠΈΠΈ Π΄Π²ΡƒΡ… ΠΎΡ‚Ρ€ΠΈΡ†Π°Ρ‚Π΅Π»ΡŒΠ½Ρ‹Ρ… чисСл ΠΎΡ‚Π²Π΅Ρ‚ Π±ΡƒΠ΄Π΅Ρ‚ всСгда ΠΏΠΎΠ»ΠΎΠΆΠΈΡ‚Π΅Π»ΡŒΠ½ΠΎΠ΅ число. НапримСр, -4, Ρ€Π°Π·Π΄Π΅Π»Π΅Π½Π½ΠΎΠ΅ Π½Π° -2, Ρ€Π°Π²Π½ΠΎ 2. Когда ΠΎΠ±Π° числа ΠΎΡ‚Ρ€ΠΈΡ†Π°Ρ‚Π΅Π»ΡŒΠ½Ρ‹, ΠΎΡ‚Ρ€ΠΈΡ†Π°Ρ‚Π΅Π»ΡŒΠ½Ρ‹Π΅ значСния ΠΊΠΎΠΌΠΏΠ΅Π½ΡΠΈΡ€ΡƒΡŽΡ‚ΡΡ, Π² Ρ€Π΅Π·ΡƒΠ»ΡŒΡ‚Π°Ρ‚Π΅ Ρ‡Π΅Π³ΠΎ ΠΎΡ‚Π²Π΅Ρ‚ всСгда Π±ΡƒΠ΄Π΅Ρ‚ ΠΏΠΎΠ»ΠΎΠΆΠΈΡ‚Π΅Π»ΡŒΠ½Ρ‹ΠΌ числом.

МоТно Π»ΠΈ Π΄Π΅Π»ΠΈΡ‚ΡŒ Π½Π° ΠΎΡ‚Ρ€ΠΈΡ†Π°Ρ‚Π΅Π»ΡŒΠ½ΠΎΠ΅ число? Π£ΠΌΠ½ΠΎΠΆΠ΅Π½ΠΈΠ΅ ΠΈ Π΄Π΅Π»Π΅Π½ΠΈΠ΅ с ΠΎΡ‚Ρ€ΠΈΡ†Π°Ρ‚Π΅Π»ΡŒΠ½Ρ‹ΠΌΠΈ числами практичСски Ρ‚Π°ΠΊ ΠΆΠ΅, ΠΊΠ°ΠΊ ΠΈ с ΠΏΠΎΠ»ΠΎΠΆΠΈΡ‚Π΅Π»ΡŒΠ½Ρ‹ΠΌΠΈ числами. … Вопрос Ρ‚ΠΎΠ»ΡŒΠΊΠΎ Π² Ρ‚ΠΎΠΌ, ΠΏΠΎΠ»ΠΎΠΆΠΈΡ‚Π΅Π»ΡŒΠ½Ρ‹ΠΉ это Π·Π½Π°ΠΊ ΠΈΠ»ΠΈ ΠΎΡ‚Ρ€ΠΈΡ†Π°Ρ‚Π΅Π»ΡŒΠ½Ρ‹ΠΉ. ΠŸΡ€ΠΎΡΡ‚ΠΎ ΠΏΠΎΠΌΠ½ΠΈΡ‚Π΅, Ρ‡Ρ‚ΠΎ ΠΊΠΎΠ³Π΄Π° Π²Ρ‹ ΡƒΠΌΠ½ΠΎΠΆΠ°Π΅Ρ‚Π΅ ΠΈΠ»ΠΈ Π΄Π΅Π»ΠΈΡ‚Π΅ Π΄Π²Π° числа, Ссли числа ΠΈΠΌΠ΅ΡŽΡ‚ ΠΎΠ΄ΠΈΠ½Π°ΠΊΠΎΠ²Ρ‹ΠΉ Π·Π½Π°ΠΊ, Ρ€Π΅Π·ΡƒΠ»ΡŒΡ‚Π°Ρ‚ всСгда ΠΏΠΎΠ»ΠΎΠΆΠΈΡ‚Π΅Π»ΡŒΠ½Ρ‹ΠΉ.

Π§Ρ‚ΠΎ Ρ‚Π°ΠΊΠΎΠ΅ ΠΎΡ‚Ρ€ΠΈΡ†Π°Ρ‚Π΅Π»ΡŒΠ½ΠΎΠ΅ 1 Π² Π²ΠΈΠ΄Π΅ Π΄Ρ€ΠΎΠ±ΠΈ?

Π’Π°ΠΊΠΆΠ΅ Π§Ρ‚ΠΎ происходит, ΠΊΠΎΠ³Π΄Π° Π²Ρ‹ ΡƒΠΌΠ½ΠΎΠΆΠ°Π΅Ρ‚Π΅ ΠΎΡ‚Ρ€ΠΈΡ†Π°Ρ‚Π΅Π»ΡŒΠ½ΠΎΠ΅ Π½Π° ΠΏΠΎΠ»ΠΎΠΆΠΈΡ‚Π΅Π»ΡŒΠ½ΠΎΠ΅? Когда Π²Ρ‹ ΡƒΠΌΠ½ΠΎΠΆΠ°Π΅Ρ‚Π΅ ΠΎΡ‚Ρ€ΠΈΡ†Π°Ρ‚Π΅Π»ΡŒΠ½ΠΎΠ΅ число Π½Π° ΠΏΠΎΠ»ΠΎΠΆΠΈΡ‚Π΅Π»ΡŒΠ½ΠΎΠ΅ число, Ρ‚ΠΎ ΠΏΡ€ΠΎΠ΄ΡƒΠΊΡ‚ всСгда ΠΎΡ‚Ρ€ΠΈΡ†Π°Ρ‚Π΅Π»ΡŒΠ½Ρ‹ΠΉ. Когда Π²Ρ‹ ΡƒΠΌΠ½ΠΎΠΆΠ°Π΅Ρ‚Π΅ Π΄Π²Π° ΠΎΡ‚Ρ€ΠΈΡ†Π°Ρ‚Π΅Π»ΡŒΠ½Ρ‹Ρ… числа ΠΈΠ»ΠΈ Π΄Π²Π° ΠΏΠΎΠ»ΠΎΠΆΠΈΡ‚Π΅Π»ΡŒΠ½Ρ‹Ρ… числа, ΠΏΡ€ΠΎΠΈΠ·Π²Π΅Π΄Π΅Π½ΠΈΠ΅ всСгда ΠΏΠΎΠ»ΠΎΠΆΠΈΡ‚Π΅Π»ΡŒΠ½ΠΎ. … Π’Π΅ΠΏΠ΅Ρ€ΡŒ Ρƒ нас Π΅ΡΡ‚ΡŒ Π΄Π²Π° ΠΎΡ‚Ρ€ΠΈΡ†Π°Ρ‚Π΅Π»ΡŒΠ½Ρ‹Ρ… числа, поэтому Ρ€Π΅Π·ΡƒΠ»ΡŒΡ‚Π°Ρ‚ ΠΏΠΎΠ»ΠΎΠΆΠΈΡ‚Π΅Π»ΡŒΠ½Ρ‹ΠΉ.

ΠŸΠΎΡ‡Π΅ΠΌΡƒ 2 минуса Π΄Π°ΡŽΡ‚ плюс?

Π‘ ΠΊΠ°ΠΆΠ΄Ρ‹ΠΌ числом связано Β«Π°Π΄Π΄ΠΈΡ‚ΠΈΠ²Π½ΠΎΠ΅ ΠΎΠ±Ρ€Π°Ρ‚Π½ΠΎΠ΅Β» (своСго Ρ€ΠΎΠ΄Π° Β«ΠΏΡ€ΠΎΡ‚ΠΈΠ²ΠΎΠΏΠΎΠ»ΠΎΠΆΠ½ΠΎΠ΅Β» число), ΠΊΠΎΡ‚ΠΎΡ€ΠΎΠ΅ ΠΏΡ€ΠΈ Π΄ΠΎΠ±Π°Π²Π»Π΅Π½ΠΈΠΈ ΠΊ исходному числу Π΄Π°Π΅Ρ‚ ноль. … Π’ΠΎΡ‚ Ρ„Π°ΠΊΡ‚, Ρ‡Ρ‚ΠΎ ΠΏΡ€ΠΎΠΈΠ·Π²Π΅Π΄Π΅Π½ΠΈΠ΅ Π΄Π²ΡƒΡ… Π½Π΅Π³Π°Ρ‚ΠΈΠ²ΠΎΠ² являСтся ΠΏΠΎΠ»ΠΎΠΆΠΈΡ‚Π΅Π»ΡŒΠ½Ρ‹ΠΌ, Π‘Π»Π΅Π΄ΠΎΠ²Π°Ρ‚Π΅Π»ΡŒΠ½ΠΎ, это связано с Ρ‚Π΅ΠΌ, Ρ‡Ρ‚ΠΎ ΠΎΠ±Ρ€Π°Ρ‚Π½ΠΎΠ΅ ΠΊ ΠΏΠΎΠ»ΠΎΠΆΠΈΡ‚Π΅Π»ΡŒΠ½ΠΎΠΌΡƒ числу являСтся ΠΎΠ±Ρ€Π°Ρ‚Π½Ρ‹ΠΌ ΠΏΠΎΠ»ΠΎΠΆΠΈΡ‚Π΅Π»ΡŒΠ½Ρ‹ΠΌ числом..

Как ΡƒΠ·Π½Π°Ρ‚ΡŒ, ΠΊΠΎΠ³Π΄Π° Π½ΡƒΠΆΠ½ΠΎ ΡΠ»ΠΎΠΆΠΈΡ‚ΡŒ, Π²Ρ‹Ρ‡Π΅ΡΡ‚ΡŒ, ΡƒΠΌΠ½ΠΎΠΆΠΈΡ‚ΡŒ ΠΈΠ»ΠΈ Ρ€Π°Π·Π΄Π΅Π»ΠΈΡ‚ΡŒ?

Π‘ΠΎ Π²Ρ€Π΅ΠΌΠ΅Π½Π΅ΠΌ ΠΌΠ°Ρ‚Π΅ΠΌΠ°Ρ‚ΠΈΠΊΠΈ ΠΏΡ€ΠΈΡˆΠ»ΠΈ ΠΊ соглашСнию ΠΎ Π½Π°Π±ΠΎΡ€Π΅ ΠΏΡ€Π°Π²ΠΈΠ», Π½Π°Π·Ρ‹Π²Π°Π΅ΠΌΡ‹Ρ… порядок ΠΎΠΏΠ΅Ρ€Π°Ρ†ΠΈΠΉ Ρ‡Ρ‚ΠΎΠ±Ρ‹ ΠΎΠΏΡ€Π΅Π΄Π΅Π»ΠΈΡ‚ΡŒ, ΠΊΠ°ΠΊΡƒΡŽ ΠΎΠΏΠ΅Ρ€Π°Ρ†ΠΈΡŽ Π²Ρ‹ΠΏΠΎΠ»Π½ΠΈΡ‚ΡŒ Π² ΠΏΠ΅Ρ€Π²ΡƒΡŽ ΠΎΡ‡Π΅Ρ€Π΅Π΄ΡŒ. Когда Π²Ρ‹Ρ€Π°ΠΆΠ΅Π½ΠΈΠ΅ Π²ΠΊΠ»ΡŽΡ‡Π°Π΅Ρ‚ Ρ‚ΠΎΠ»ΡŒΠΊΠΎ Ρ‡Π΅Ρ‚Ρ‹Ρ€Π΅ основных ΠΎΠΏΠ΅Ρ€Π°Ρ†ΠΈΠΈ, Π²ΠΎΡ‚ ΠΏΡ€Π°Π²ΠΈΠ»Π°: Π£ΠΌΠ½ΠΎΠΆΠ΅Π½ΠΈΠ΅ ΠΈ Π΄Π΅Π»Π΅Π½ΠΈΠ΅ слСва Π½Π°ΠΏΡ€Π°Π²ΠΎ. Π‘Π»ΠΎΠΆΠΈΡ‚Π΅ ΠΈ Π²Ρ‹Ρ‡Ρ‚ΠΈΡ‚Π΅ слСва Π½Π°ΠΏΡ€Π°Π²ΠΎ.

Как ΠΊ ΡΠΌΠ΅ΡˆΠ°Π½Π½ΠΎΠΌΡƒ числу Π΄ΠΎΠ±Π°Π²ΠΈΡ‚ΡŒ ΠΎΡ‚Ρ€ΠΈΡ†Π°Ρ‚Π΅Π»ΡŒΠ½ΡƒΡŽ Π΄Ρ€ΠΎΠ±ΡŒ?

Как ΡƒΠΌΠ½ΠΎΠΆΠ°Ρ‚ΡŒ Π΄Ρ€ΠΎΠ±ΠΈ? ΠŸΠ΅Ρ€Π²Ρ‹ΠΉ шаг ΠΏΡ€ΠΈ ΡƒΠΌΠ½ΠΎΠΆΠ΅Π½ΠΈΠΈ Π΄Ρ€ΠΎΠ±Π΅ΠΉ ΡƒΠΌΠ½ΠΎΠΆΠΈΡ‚ΡŒ Π΄Π²Π° числитСля. Π’Ρ‚ΠΎΡ€ΠΎΠΉ шаг — ΡƒΠΌΠ½ΠΎΠΆΠΈΡ‚ΡŒ Π΄Π²Π° знамСнатСля. НаконСц, упроститС Π½ΠΎΠ²Ρ‹Π΅ Π΄Ρ€ΠΎΠ±ΠΈ. ΠŸΠ΅Ρ€Π΅Π΄ ΡƒΠΌΠ½ΠΎΠΆΠ΅Π½ΠΈΠ΅ΠΌ Π΄Ρ€ΠΎΠ±ΠΈ Ρ‚Π°ΠΊΠΆΠ΅ ΠΌΠΎΠΆΠ½ΠΎ ΡƒΠΏΡ€ΠΎΡΡ‚ΠΈΡ‚ΡŒ, Π²Ρ‹Ρ‡Π»Π΅Π½ΠΈΠ² ΠΎΠ±Ρ‰ΠΈΠ΅ ΠΌΠ½ΠΎΠΆΠΈΡ‚Π΅Π»ΠΈ Π² числитСлС ΠΈ Π·Π½Π°ΠΌΠ΅Π½Π°Ρ‚Π΅Π»Π΅.

ΠŸΠΎΡ‡Π΅ΠΌΡƒ ΠΏΠΎΠ»ΠΎΠΆΠΈΡ‚Π΅Π»ΡŒΠ½ΠΎΠ΅ дСлится Π½Π° ΠΎΡ‚Ρ€ΠΈΡ†Π°Ρ‚Π΅Π»ΡŒΠ½ΠΎΠ΅ ΠΎΡ‚Ρ€ΠΈΡ†Π°Ρ‚Π΅Π»ΡŒΠ½ΠΎΠ΅? ΠŸΠΎΡ‚ΠΎΠΌΡƒ Ρ‡Ρ‚ΠΎ ΠΏΠΎΠ·ΠΈΡ‚ΠΈΠ² прСдставляСт Ρ‚ΠΎΠ»ΡŒΠΊΠΎ количСство Ρ€Π°Π·, ΠΊΠΎΡ‚ΠΎΡ€ΠΎΠ΅ Π΄Π΅Π»ΠΈΡ‚Π΅Π»ΡŒ вписываСтся Π² Ρ€Π°Π·Π΄Π΅Π»Π΅Π½Π½ΠΎΠ΅. По Ρ‚ΠΎΠΉ ΠΆΠ΅ ΠΏΡ€ΠΈΡ‡ΠΈΠ½Π΅, Ρ‡Ρ‚ΠΎ ΠΎΡ‚Ρ€ΠΈΡ†Π°Ρ‚Π΅Π»ΡŒΠ½ΠΎΠ΅, ΡƒΠΌΠ½ΠΎΠΆΠ΅Π½Π½ΠΎΠ΅ Π½Π° ΠΎΡ‚Ρ€ΠΈΡ†Π°Ρ‚Π΅Π»ΡŒΠ½ΠΎΠ΅, являСтся ΠΏΠΎΠ»ΠΎΠΆΠΈΡ‚Π΅Π»ΡŒΠ½Ρ‹ΠΌ. ΠžΠ±Ρ€Π°Ρ‚ΠΈΡ‚Π΅ Π²Π½ΠΈΠΌΠ°Π½ΠΈΠ΅, Ρ‡Ρ‚ΠΎ ΠΌΡƒΠ»ΡŒΡ‚ΠΈΠΏΠ»ΠΈΠΊΠ°Ρ‚ΠΈΠ²Π½Π°Ρ инвСрсия -1 Ρ€Π°Π²Π½Π° -1.

Π§Ρ‚ΠΎ Ρ‚Π°ΠΊΠΎΠ΅ ΠΏΠΎΠ»ΠΎΠ²ΠΈΠ½Π° ΠΎΡ‚Ρ€ΠΈΡ†Π°Ρ‚Π΅Π»ΡŒΠ½ΠΎΠ³ΠΎ числа?

Половина ΠΎΡ‚Ρ€ΠΈΡ†Π°Ρ‚Π΅Π»ΡŒΠ½ΠΎΠ³ΠΎ числа Ρ€Π°Π²Π½Π° Ρ‚ΠΎ ΠΆΠ΅, Ρ‡Ρ‚ΠΎ Π΄Π΅Π»Π΅Π½ΠΈΠ΅ ΠΏΠΎΠ»ΠΎΠΆΠΈΡ‚Π΅Π»ΡŒΠ½ΠΎΠ³ΠΎ числа ΠΏΠΎΠΏΠΎΠ»Π°ΠΌ. Π’Ρ‹ ΠΌΠΎΠΆΠ΅Ρ‚Π΅ ΡΠΌΠΎΡ‚Ρ€Π΅Ρ‚ΡŒ Π½Π° это ΠΊΠ°ΠΊ Π½Π° Π°Π±ΡΠΎΠ»ΡŽΡ‚Π½ΠΎΠ΅ Π·Π½Π°Ρ‡Π΅Π½ΠΈΠ΅. Π‘ΠΊΠ°ΠΆΠ΅ΠΌ, Ρƒ нас Π΅ΡΡ‚ΡŒ ΠΏΠΎΠ»ΠΎΠΆΠΈΡ‚Π΅Π»ΡŒΠ½ΠΎΠ΅ число Ρ…, Ρ… это Ρ… ΠΎΡ‚ нуля. Половина этого расстояния Ρ€Π°Π²Π½Π° ΠΏΠΎΠ»ΠΎΠ²ΠΈΠ½Π΅ Ρ… ΠΈΠ»ΠΈ Ρ…/2.

КакоС число дСлится Π½Π° минус 1?

ВсС Ρ†Π΅Π»Ρ‹Π΅ числа дСлятся Π½Π° -1. Когда Π²Ρ‹ Π΄Π΅Π»ΠΈΡ‚Π΅ Π½Π° минус 1, Π²Ρ‹ мСняСтС число с ΠΏΠΎΠ»ΠΎΠΆΠΈΡ‚Π΅Π»ΡŒΠ½ΠΎΠ³ΠΎ Π½Π° ΠΎΡ‚Ρ€ΠΈΡ†Π°Ρ‚Π΅Π»ΡŒΠ½Ρ‹ΠΉ ΠΈΠ»ΠΈ ΠΎΡ‚Ρ€ΠΈΡ†Π°Ρ‚Π΅Π»ΡŒΠ½ΠΎΠ΅ Π½Π° ΠΏΠΎΠ»ΠΎΠΆΠΈΡ‚Π΅Π»ΡŒΠ½ΠΎΠ΅. 50 Γ· -1 = -50. -2 Γ· -1 = 2.

ΠŸΠΎΡ‡Π΅ΠΌΡƒ ΠΎΡ‚Ρ€ΠΈΡ†Π°Ρ‚Π΅Π»ΡŒΠ½ΠΎΠ΅, Π΄Π΅Π»Π΅Π½Π½ΠΎΠ΅ Π½Π° ΠΎΡ‚Ρ€ΠΈΡ†Π°Ρ‚Π΅Π»ΡŒΠ½ΠΎΠ΅, Ρ€Π°Π²Π½ΠΎ ΠΏΠΎΠ»ΠΎΠΆΠΈΡ‚Π΅Π»ΡŒΠ½ΠΎΠΌΡƒ? ΠŸΠ΅Ρ€Π²ΠΎΠ½Π°Ρ‡Π°Π»ΡŒΠ½Ρ‹ΠΉ ΠΎΡ‚Π²Π΅Ρ‚: ΠŸΠΎΡ‡Π΅ΠΌΡƒ Π΄Π΅Π»Π΅Π½ΠΈΠ΅ ΠΎΡ‚Ρ€ΠΈΡ†Π°Ρ‚Π΅Π»ΡŒΠ½ΠΎΠ³ΠΎ Π½Π° ΠΎΡ‚Ρ€ΠΈΡ†Π°Ρ‚Π΅Π»ΡŒΠ½ΠΎΠ΅ Ρ€Π°Π²Π½ΠΎ ΠΏΠΎΠ»ΠΎΠΆΠΈΡ‚Π΅Π»ΡŒΠ½ΠΎΠΌΡƒ? ΠŸΠΎΡΠΊΠΎΠ»ΡŒΠΊΡƒ ΡƒΠΌΠ½ΠΎΠΆΠ΅Π½ΠΈΠ΅ ΠΎΡ‚Ρ€ΠΈΡ†Π°Ρ‚Π΅Π»ΡŒΠ½ΠΎΠ³ΠΎ Π½Π° ΠΎΡ‚Ρ€ΠΈΡ†Π°Ρ‚Π΅Π»ΡŒΠ½ΠΎΠ΅ Π΄Π°Π΅Ρ‚ ΠΏΠΎΠ»ΠΎΠΆΠΈΡ‚Π΅Π»ΡŒΠ½ΠΎΠ΅ Π·Π½Π°Ρ‡Π΅Π½ΠΈΠ΅, ΠΎΠ½ сохраняСт ΡΠΎΡΡ‚ΠΎΡΡ‰ΡƒΡŽ ΠΌΠ°Ρ‚Π΅ΠΌΠ°Ρ‚ΠΈΠΊΡƒ.

Π―Π²Π»ΡΡŽΡ‚ΡΡ Π»ΠΈ ΠΎΡ‚Ρ€ΠΈΡ†Π°Ρ‚Π΅Π»ΡŒΠ½Ρ‹Π΅ Π΄Ρ€ΠΎΠ±ΠΈ ΠΏΡ€Π°Π²ΠΈΠ»ΡŒΠ½Ρ‹ΠΌΠΈ дробями?

A ΠΎΡ‚Ρ€ΠΈΡ†Π°Ρ‚Π΅Π»ΡŒΠ½Π°Ρ ΠΏΡ€Π°Π²ΠΈΠ»ΡŒΠ½Π°Ρ Π΄Ρ€ΠΎΠ±ΡŒ Π±ΡƒΠ΄Π΅Ρ‚ ΠΈΠΌΠ΅Ρ‚ΡŒ ΠΎΡ‚Ρ€ΠΈΡ†Π°Ρ‚Π΅Π»ΡŒΠ½Ρ‹ΠΉ Π·Π½Π°ΠΊ Π»ΠΈΠ±ΠΎ Π² числитСлС, Π»ΠΈΠ±ΠΎ Π² Π·Π½Π°ΠΌΠ΅Π½Π°Ρ‚Π΅Π»Π΅. ΠŸΠΎΠ»ΠΎΠΆΠΈΡ‚Π΅Π»ΡŒΠ½Π°Ρ ΠΏΡ€Π°Π²ΠΈΠ»ΡŒΠ½Π°Ρ Π΄Ρ€ΠΎΠ±ΡŒ: ΠΏΠΎΠ»ΠΎΠΆΠΈΡ‚Π΅Π»ΡŒΠ½Π°Ρ ΠΏΡ€Π°Π²ΠΈΠ»ΡŒΠ½Π°Ρ Π΄Ρ€ΠΎΠ±ΡŒ Π±ΡƒΠ΄Π΅Ρ‚ ΠΈΠΌΠ΅Ρ‚ΡŒ ΠΏΠΎΠ»ΠΎΠΆΠΈΡ‚Π΅Π»ΡŒΠ½Ρ‹ΠΉ Π·Π½Π°ΠΊ ΠΊΠ°ΠΊ Π² числитСлС, Ρ‚Π°ΠΊ ΠΈ Π² Π·Π½Π°ΠΌΠ΅Π½Π°Ρ‚Π΅Π»Π΅. … Π”Ρ€ΠΎΠ±ΠΈ, Ρƒ ΠΊΠΎΡ‚ΠΎΡ€Ρ‹Ρ… Ρ‡ΠΈΡΠ»ΠΈΡ‚Π΅Π»ΡŒ (Π²Π΅Ρ€Ρ…Π½Π΅Π΅ число) большС ΠΈΠ»ΠΈ Ρ€Π°Π²Π΅Π½ Π·Π½Π°ΠΌΠ΅Π½Π°Ρ‚Π΅Π»ΡŽ (Π½ΠΈΠΆΠ½Π΅ΠΌΡƒ числу).

ΠœΠΎΠΆΠ΅Ρ‚ Π»ΠΈ Π±Ρ‹Ρ‚ΡŒ ΠΎΡ‚Ρ€ΠΈΡ†Π°Ρ‚Π΅Π»ΡŒΠ½Ρ‹ΠΉ Ρ‡ΠΈΡΠ»ΠΈΡ‚Π΅Π»ΡŒ? ΠžΠ±Ρ‹Ρ‡Π½ΠΎ ΠΏΠ΅Ρ€Π΅Π΄ Π΄Ρ€ΠΎΠ±ΡŒΡŽ ставится Π·Π½Π°ΠΊ «минус», Π½ΠΎ ΠΈΠ½ΠΎΠ³Π΄Π° Π²ΡΡ‚Ρ€Π΅Ρ‡Π°ΡŽΡ‚ΡΡ Π΄Ρ€ΠΎΠ±ΠΈ с ΠΎΡ‚Ρ€ΠΈΡ†Π°Ρ‚Π΅Π»ΡŒΠ½Ρ‹ΠΌ числитСлСм ΠΈΠ»ΠΈ Π·Π½Π°ΠΌΠ΅Π½Π°Ρ‚Π΅Π»Π΅ΠΌ. … Если ΠΈ Ρ‡ΠΈΡΠ»ΠΈΡ‚Π΅Π»ΡŒ, ΠΈ Π·Π½Π°ΠΌΠ΅Π½Π°Ρ‚Π΅Π»ΡŒ ΠΎΡ‚Ρ€ΠΈΡ†Π°Ρ‚Π΅Π»ΡŒΠ½Ρ‹, Ρ‚ΠΎ сама Π΄Ρ€ΠΎΠ±ΡŒ ΠΏΠΎΠ»ΠΎΠΆΠΈΡ‚Π΅Π»ΡŒΠ½Π°, ΠΏΠΎΡ‚ΠΎΠΌΡƒ Ρ‡Ρ‚ΠΎ ΠΌΡ‹ Π΄Π΅Π»ΠΈΠΌ ΠΎΡ‚Ρ€ΠΈΡ†Π°Ρ‚Π΅Π»ΡŒΠ½ΠΎΠ΅ Π½Π° ΠΎΡ‚Ρ€ΠΈΡ†Π°Ρ‚Π΅Π»ΡŒΠ½ΠΎΠ΅.

ЯвляСтся Π»ΠΈ ΠΎΡ‚Ρ€ΠΈΡ†Π°Ρ‚Π΅Π»ΡŒΠ½Π°Ρ Π΄Ρ€ΠΎΠ±ΡŒ Π΄Π΅ΠΉΡΡ‚Π²ΠΈΡ‚Π΅Π»ΡŒΠ½Ρ‹ΠΌ числом?

Π”Π΅ΠΉΡΡ‚Π²ΠΈΡ‚Π΅Π»ΡŒΠ½Ρ‹Π΅ числа Π²ΠΊΠ»ΡŽΡ‡Π°ΡŽΡ‚ Π² сСбя ΠΏΠΎΠ»ΠΎΠΆΠΈΡ‚Π΅Π»ΡŒΠ½Ρ‹Π΅ ΠΈ ΠΎΡ‚Ρ€ΠΈΡ†Π°Ρ‚Π΅Π»ΡŒΠ½Ρ‹Π΅ Ρ†Π΅Π»Ρ‹Π΅ числа ΠΈ Π΄Ρ€ΠΎΠ±ΠΈ (ΠΈΠ»ΠΈ Ρ€Π°Ρ†ΠΈΠΎΠ½Π°Π»ΡŒΠ½Ρ‹Π΅ числа), Π° Ρ‚Π°ΠΊΠΆΠ΅ ΠΈΡ€Ρ€Π°Ρ†ΠΈΠΎΠ½Π°Π»ΡŒΠ½Ρ‹ΠΉ Π½ΠΎΠΌΠ΅Ρ€Π°.

Как Π½Π°ΠΉΡ‚ΠΈ ΠΎΠ±Ρ€Π°Ρ‚Π½ΡƒΡŽ Π΄Ρ€ΠΎΠ±ΡŒ

Как Π½Π°ΠΉΡ‚ΠΈ ΠΎΠ±Ρ€Π°Ρ‚Π½ΡƒΡŽ Π΄Ρ€ΠΎΠ±ΡŒ — SAT Math

—>

  • Π’ΠΎΠΉΡ‚ΠΈ
  • Π‘ΠΈΠΎΠ³Ρ€Π°Ρ„ΠΈΠΈ Ρ€Π΅ΠΏΠ΅Ρ‚ΠΈΡ‚ΠΎΡ€Π°
  • ΠŸΠΎΠ΄Π³ΠΎΡ‚ΠΎΠ²ΠΊΠ° ΠΊ тСсту
    БРЕДНЯЯ Π¨ΠšΠžΠ›Π
    • ACT РСпСтиторство
    • SAT РСпСтиторство
    • РСпСтиторство PSAT
    • ASPIRE РСпСтиторство
    • ШБАВ РСпСтиторство
    • РСпСтиторство STAAR
    ВЫБШАЯ Π¨ΠšΠžΠ›Π
    • РСпСтиторство MCAT
    • РСпСтиторство GRE
    • РСпСтиторство ΠΏΠΎ LSAT
    • РСпСтиторство ΠΏΠΎ GMAT
    К-8
    • РСпСтиторство AIMS
    • РСпСтиторство ΠΏΠΎ HSPT
    • РСпСтиторство ISEE
    • РСпСтиторство ISAT
    • РСпСтиторство ΠΏΠΎ SSAT
    • РСпСтиторство STAAR
    Поиск 50+ тСстов
  • АкадСмичСскоС ΠΎΠ±ΡƒΡ‡Π΅Π½ΠΈΠ΅
    рСпСтиторство ΠΏΠΎ ΠΌΠ°Ρ‚Π΅ΠΌΠ°Ρ‚ΠΈΠΊΠ΅
    • АлгСбра
    • Π˜ΡΡ‡ΠΈΡΠ»Π΅Π½ΠΈΠ΅
    • ЭлСмСнтарная ΠΌΠ°Ρ‚Π΅ΠΌΠ°Ρ‚ΠΈΠΊΠ°
    • ГСомСтрия
    • ΠŸΡ€Π΅Π΄Π²Π°Ρ€ΠΈΡ‚Π΅Π»ΡŒΠ½Ρ‹ΠΉ расчСт
    • Бтатистика
    • ВригономСтрия
    РСпСтиторство ΠΏΠΎ СстСствСнным Π½Π°ΡƒΠΊΠ°ΠΌ
    • Анатомия
    • Биология
    • Π₯имия
    • Π€ΠΈΠ·ΠΈΠΊΠ°
    • Ѐизиология
    иностранныС языки
    • французский
    • Π½Π΅ΠΌΠ΅Ρ†ΠΊΠΈΠΉ
    • Латинский
    • ΠšΠΈΡ‚Π°ΠΉΡΠΊΠΈΠΉ ΠΌΠ°Π½Π΄Π°Ρ€ΠΈΠ½
    • Испанский
    Π½Π°Ρ‡Π°Π»ΡŒΠ½ΠΎΠ΅ ΠΎΠ±ΡƒΡ‡Π΅Π½ΠΈΠ΅
    • Π§Ρ‚Π΅Π½ΠΈΠ΅
    • Акустика
    • ЭлСмСнтарная ΠΌΠ°Ρ‚Π΅ΠΌΠ°Ρ‚ΠΈΠΊΠ°
    ΠΏΡ€ΠΎΡ‡ΠΈΠ΅
    • БухгалтСрский ΡƒΡ‡Π΅Ρ‚
    • Π˜Π½Ρ„ΠΎΡ€ΠΌΠ°Ρ‚ΠΈΠΊΠ°
    • Π­ΠΊΠΎΠ½ΠΎΠΌΠΈΠΊΠ°
    • Английский
    • Ѐинансы
    • Π˜ΡΡ‚ΠΎΡ€ΠΈΡ
    • Письмо
    • Π›Π΅Ρ‚ΠΎ
    Поиск ΠΏΠΎ 350+ Ρ‚Π΅ΠΌΠ°ΠΌ
  • О
    • ΠžΠ±Π·ΠΎΡ€ Π²ΠΈΠ΄Π΅ΠΎ
    • ΠŸΡ€ΠΎΡ†Π΅ΡΡ Π²Ρ‹Π±ΠΎΡ€Π° наставника
    • Онлайн-рСпСтиторство
    • МобильноС ΠΎΠ±ΡƒΡ‡Π΅Π½ΠΈΠ΅
    • МгновСнноС ΠΎΠ±ΡƒΡ‡Π΅Π½ΠΈΠ΅
    • Как ΠΌΡ‹ Ρ€Π°Π±ΠΎΡ‚Π°Π΅ΠΌ
    • Наша гарантия
    • ВлияниС рСпСтиторства
    • ΠΎΠ±Π·ΠΎΡ€ΠΎΠ² ΠΈ ΠΎΡ‚Π·Ρ‹Π²ΠΎΠ²
    • ΠžΡΠ²Π΅Ρ‰Π΅Π½ΠΈΠ΅ Π² БМИ
    • О прСподаватСлях унивСрситСта

Π—Π²ΠΎΠ½ΠΈΡ‚Π΅ прямо сСйчас, Ρ‡Ρ‚ΠΎΠ±Ρ‹ Π·Π°ΠΏΠΈΡΠ°Ρ‚ΡŒΡΡ Π½Π° ΠΎΠ±ΡƒΡ‡Π΅Π½ΠΈΠ΅:

(888) 888-0446

ВсС матСматичСскиС рСсурсы SAT

16 диагностичСских тСстов 660 практичСских тСстов Вопрос дня ΠšΠ°Ρ€Ρ‚ΠΎΡ‡ΠΊΠΈ Learn by Concept

SAT Math Help Β» АрифмСтика Β» Π€Ρ€Π°ΠΊΡ†ΠΈΠΈ Β» Π’Π·Π°ΠΈΠΌΠ½Ρ‹Π΅ ΠΏΠ»Π°Ρ‚Π΅ΠΆΠΈ Β» Как Π½Π°ΠΉΡ‚ΠΈ ΠΎΠ±Ρ€Π°Ρ‚Π½ΡƒΡŽ Π΄Ρ€ΠΎΠ±ΡŒ

Π§Ρ‚ΠΎ являСтся Π²Π·Π°ΠΈΠΌΠ½Ρ‹ΠΌ ΠΈΠ· ΡΠ»Π΅Π΄ΡƒΡŽΡ‰Π΅ΠΉ Ρ„Ρ€Π°ΠΊΡ†ΠΈΠΈ: 18/27

Π’ΠΎΠ·ΠΌΠΎΠΆΠ½Ρ‹Π΅ ΠΎΡ‚Π²Π΅Ρ‚Ρ‹:

-18/27

27/18

9

9/27

ΠŸΡ€Π°Π²ΠΈΠ»ΡŒΠ½Ρ‹ΠΉ ΠΎΡ‚Π²Π΅Ρ‚:

27 27 27 27 27 27 27 27 27 27 27 27 27 27 27 27 27 27 27 27

. /18

ПояснСниС:

Π”Ρ€ΠΎΠ±ΡŒ, умноТСнная Π½Π° ΠΎΠ±Ρ€Π°Ρ‚Π½ΡƒΡŽ, Π±ΡƒΠ΄Π΅Ρ‚ Ρ€Π°Π²Π½Π° 1. Π§Ρ‚ΠΎΠ±Ρ‹ Π½Π°ΠΉΡ‚ΠΈ ΠΎΠ±Ρ€Π°Ρ‚Π½ΡƒΡŽ Π΄Ρ€ΠΎΠ±ΡŒ, помСняйтС мСстами Π·Π½Π°ΠΌΠ΅Π½Π°Ρ‚Π΅Π»ΡŒ ΠΈ Ρ‡ΠΈΡΠ»ΠΈΡ‚Π΅Π»ΡŒ. ΠžΠ±Ρ€Π°Ρ‚Π½ΠΎΠ΅ число 18/27 Ρ€Π°Π²Π½ΠΎ 27/18.

Π‘ΠΎΠΎΠ±Ρ‰ΠΈΡ‚ΡŒ ΠΎΠ± ошибкС

Π§Π΅ΠΌΡƒ Ρ€Π°Π²Π½Π° Π΄Ρ€ΠΎΠ±ΡŒ, обратная ΠΏΡ€ΠΈΠ²Π΅Π΄Π΅Π½Π½ΠΎΠΉ Π½ΠΈΠΆΠ΅?

Π’ΠΎΠ·ΠΌΠΎΠΆΠ½Ρ‹Π΅ ΠΎΡ‚Π²Π΅Ρ‚Ρ‹:

ΠŸΡ€Π°Π²ΠΈΠ»ΡŒΠ½Ρ‹ΠΉ ΠΎΡ‚Π²Π΅Ρ‚:

ПояснСниС:

ΠžΠ±Ρ€Π°Ρ‚Π½Π°Ρ Π΄Ρ€ΠΎΠ±ΡŒ ΠΌΠΎΠΆΠ΅Ρ‚ Π±Ρ‹Ρ‚ΡŒ ΠΏΠΎΠ»ΡƒΡ‡Π΅Π½Π° ΠΏΡƒΡ‚Π΅ΠΌ Π·Π°ΠΌΠ΅Π½Ρ‹ числитСля ΠΈ знамСнатСля.

Β 

Π§ΠΈΡΠ»ΠΈΡ‚Π΅Π»ΡŒ Π² Π΄Π°Π½Π½ΠΎΠΌ случаС Ρ‚Π°ΠΊΠΎΠ², Ρ‡Ρ‚ΠΎ ΠΎΠ½ станСт Π·Π½Π°ΠΌΠ΅Π½Π°Ρ‚Π΅Π»Π΅ΠΌ.

Π—Π½Π°ΠΌΠ΅Π½Π°Ρ‚Π΅Π»ΡŒ Π² этом случаС Ρ€Π°Π²Π΅Π½ Β , поэтому ΠΎΠ½ станСт числитСлСм.

Π‘Π»Π΅Π΄ΠΎΠ²Π°Ρ‚Π΅Π»ΡŒΠ½ΠΎ, ΠΎΠ±Ρ€Π°Ρ‚Π½ΠΎΠ΅ Π·Π½Π°Ρ‡Π΅Π½ΠΈΠ΅ Ρ€Π°Π²Π½ΠΎΒ .

Π‘ΠΎΠΎΠ±Ρ‰ΠΈΡ‚ΡŒ ΠΎΠ± ошибкС

Π§Ρ‚ΠΎ являСтся ΠΎΠ±Ρ€Π°Ρ‚Π½Ρ‹ΠΌ Π·Π½Π°Ρ‡Π΅Π½ΠΈΠ΅ΠΌΒ ?

Π’ΠΎΠ·ΠΌΠΎΠΆΠ½Ρ‹Π΅ ΠΎΡ‚Π²Π΅Ρ‚Ρ‹:

ΠŸΡ€Π°Π²ΠΈΠ»ΡŒΠ½Ρ‹ΠΉ ΠΎΡ‚Π²Π΅Ρ‚:

ОбъяснСниС:

ΠžΠ±Ρ€Π°Ρ‚Π½Π°Ρ Π²Π΅Π»ΠΈΡ‡ΠΈΠ½Π° ΠΌΠΎΠΆΠ΅Ρ‚ Π±Ρ‹Ρ‚ΡŒ ΠΎΠΏΡ€Π΅Π΄Π΅Π»Π΅Π½Π° ΠΏΡƒΡ‚Π΅ΠΌ дСлСния всСго количСства Π½Π° Π΅Π΄ΠΈΠ½ΠΈΡ†Ρƒ.

Π­Ρ‚ΠΎ просто пСрСстановка числитСля ΠΈ знамСнатСля.

ΠžΡ‚Π²Π΅Ρ‚Β .

Π‘ΠΎΠΎΠ±Ρ‰ΠΈΡ‚ΡŒ ΠΎΠ± ошибкС

Π§Ρ‚ΠΎ являСтся ΠΎΠ±Ρ€Π°Ρ‚Π½Ρ‹ΠΌ Π·Π½Π°Ρ‡Π΅Π½ΠΈΠ΅ΠΌΒ ?

Π’ΠΎΠ·ΠΌΠΎΠΆΠ½Ρ‹Π΅ ΠΎΡ‚Π²Π΅Ρ‚Ρ‹:

ΠŸΡ€Π°Π²ΠΈΠ»ΡŒΠ½Ρ‹ΠΉ ΠΎΡ‚Π²Π΅Ρ‚:

ПояснСниС:

Π§Ρ‚ΠΎΠ±Ρ‹ Π½Π°ΠΉΡ‚ΠΈ ΠΎΠ±Ρ€Π°Ρ‚Π½ΡƒΡŽ, Π½ΡƒΠΆΠ½ΠΎ Π²Π·ΡΡ‚ΡŒ 1 ΠΈ Ρ€Π°Π·Π΄Π΅Π»ΠΈΡ‚ΡŒ Π½Π° всС количСство Π΄Ρ€ΠΎΠ±ΠΈ. ΠžΠ±Ρ€Π°Ρ‚Π½ΠΎΠ΅ просто мСняСт мСстами Ρ‡ΠΈΡΠ»ΠΈΡ‚Π΅Π»ΡŒ ΠΈ Π·Π½Π°ΠΌΠ΅Π½Π°Ρ‚Π΅Π»ΡŒ.

Π‘ΠΎΠΎΠ±Ρ‰ΠΈΡ‚ΡŒ ΠΎΠ± ошибкС

Π§Π΅ΠΌΡƒ Ρ€Π°Π²Π½ΠΎ ΠΎΡ‚Ρ€ΠΈΡ†Π°Ρ‚Π΅Π»ΡŒΠ½ΠΎΠ΅ ΠΎΠ±Ρ€Π°Ρ‚Π½ΠΎΠ΅ Π·Π½Π°Ρ‡Π΅Π½ΠΈΠ΅Β ?

Π’ΠΎΠ·ΠΌΠΎΠΆΠ½Ρ‹Π΅ ΠΎΡ‚Π²Π΅Ρ‚Ρ‹:

ΠŸΡ€Π°Π²ΠΈΠ»ΡŒΠ½Ρ‹ΠΉ ΠΎΡ‚Π²Π΅Ρ‚:

ОбъяснСниС:

ΠžΡ‚Ρ€ΠΈΡ†Π°Ρ‚Π΅Π»ΡŒΠ½Π°Ρ обратная Π²Π΅Π»ΠΈΡ‡ΠΈΠ½Π° числа состоит Π² Ρ‚ΠΎΠΌ, Ρ‡Ρ‚ΠΎΠ±Ρ‹ Π²Π·ΡΡ‚ΡŒ ΠΎΡ‚Ρ€ΠΈΡ†Π°Ρ‚Π΅Π»ΡŒΠ½ΡƒΡŽ Π΅Π΄ΠΈΠ½ΠΈΡ†Ρƒ ΠΈ Ρ€Π°Π·Π΄Π΅Π»ΠΈΡ‚ΡŒ Π½Π° Π·Π½Π°Ρ‡Π΅Π½ΠΈΠ΅. ΠŸΡ€ΠΎΡΡ‚ΠΎ помСняйтС мСстами Ρ‡ΠΈΡΠ»ΠΈΡ‚Π΅Π»ΡŒ ΠΈ Π·Π½Π°ΠΌΠ΅Π½Π°Ρ‚Π΅Π»ΡŒ ΠΈ Π΄ΠΎΠ±Π°Π²ΡŒΡ‚Π΅ Π·Π½Π°ΠΊ минус.

Напомним, Ρ‡Ρ‚ΠΎ Π΄Π΅Π»Π΅Π½ΠΈΠ΅ Π½Π° Π΄Ρ€ΠΎΠ±ΡŒ Ρ€Π°Π²Π½ΠΎΡΠΈΠ»ΡŒΠ½ΠΎ ΡƒΠΌΠ½ΠΎΠΆΠ΅Π½ΠΈΡŽ Π½Π° ΠΎΠ±Ρ€Π°Ρ‚Π½ΠΎΠ΅.

ΠŸΡ€Π°Π²ΠΈΠ»ΡŒΠ½Ρ‹ΠΉ ΠΎΡ‚Π²Π΅Ρ‚:

Π‘ΠΎΠΎΠ±Ρ‰ΠΈΡ‚ΡŒ ΠΎΠ± ошибкС0192

ОбъяснСниС:

ΠžΠ±Ρ€Π°Ρ‚Π½Π°Ρ Ρ‡Π°ΡΡ‚ΡŒ Π΄Ρ€ΠΎΠ±ΠΈ Π½Π° Π΅Π΄ΠΈΠ½ΠΈΡ†Ρƒ большС количСства Π΄Ρ€ΠΎΠ±ΠΈ.

ΠŸΡ€ΠΎΡΡ‚ΠΎ помСняйтС мСстами Ρ‡Π»Π΅Π½Ρ‹ числитСля ΠΈ знамСнатСля.

ΠžΡ‚Π²Π΅Ρ‚Β .

Π‘ΠΎΠΎΠ±Ρ‰ΠΈΡ‚ΡŒ ΠΎΠ± ошибкС

ΠžΠ±Ρ€Π°Ρ‚Π½Π°Ρ Π²Π΅Π»ΠΈΡ‡ΠΈΠ½Π°Β Β Ρ€Π°Π²Π½Π°Β .

Π§Ρ‚ΠΎ являСтся ΠΎΠ±Ρ€Π°Ρ‚Π½Ρ‹ΠΌ Π·Π½Π°Ρ‡Π΅Π½ΠΈΠ΅ΠΌΒ ?

Π’ΠΎΠ·ΠΌΠΎΠΆΠ½Ρ‹Π΅ ΠΎΡ‚Π²Π΅Ρ‚Ρ‹:

ΠŸΡ€Π°Π²ΠΈΠ»ΡŒΠ½Ρ‹ΠΉ ΠΎΡ‚Π²Π΅Ρ‚:

ОбъяснСниС:

ΠžΠ±Ρ€Π°Ρ‚Π½Π°Ρ Π²Π΅Π»ΠΈΡ‡ΠΈΠ½Π° Ρ€Π°Π²Π½Π°Β , поэтому  – Π²Π΅Π»ΠΈΡ‡ΠΈΠ½Π°, обратная Π²Π΅Π»ΠΈΡ‡ΠΈΠ½Π΅Β Β , являСтся ΠΎΠ±Ρ€Π°Ρ‚Π½ΠΎΠΉ Π²Π΅Π»ΠΈΡ‡ΠΈΠ½ΠΎΠΉΒ . , так  являСтся обратным этому, ΠΈΠ»ΠΈΒ .

Π’Π°ΠΊ ΠΊΠ°ΠΊΒ ,Β 

ΠžΠ±Ρ€Π°Ρ‚Π½Π°Ρ Π²Π΅Π»ΠΈΡ‡ΠΈΠ½Π° Ρ€Π°Π²Π½Π°Β .

Π‘ΠΎΠΎΠ±Ρ‰ΠΈΡ‚ΡŒ ΠΎΠ± ошибкС.

Π£Π²Π΅Π΄ΠΎΠΌΠ»Π΅Π½ΠΈΠ΅ ΠΎΠ± авторских ΠΏΡ€Π°Π²Π°Ρ….

ΠŸΠΎΡΠΌΠΎΡ‚Ρ€Π΅Ρ‚ΡŒ Π Π΅ΠΏΠ΅Ρ‚ΠΈΡ‚ΠΎΡ€Ρ‹ ΠΏΠΎ ΠΌΠ°Ρ‚Π΅ΠΌΠ°Ρ‚ΠΈΠΊΠ΅ SAT

Π­Ρ€ΠΈΠΊ
Π‘Π΅Ρ€Ρ‚ΠΈΡ„ΠΈΡ†ΠΈΡ€ΠΎΠ²Π°Π½Π½Ρ‹ΠΉ ΠΏΡ€Π΅ΠΏΠΎΠ΄Π°Π²Π°Ρ‚Π΅Π»ΡŒ

УнивСрситСт Иллинойса Π² Π£Ρ€Π±Π°Π½Π°-Π¨Π°ΠΌΠΏΠ΅ΠΉΠ½, Π±Π°ΠΊΠ°Π»Π°Π²Ρ€ Π½Π°ΡƒΠΊ, Ρ„ΠΈΠ·ΠΈΠΊΠ°. ΠšΠ°Π»ΠΈΡ„ΠΎΡ€Π½ΠΈΠΉΡΠΊΠΈΠΉ унивСрситСт Π² Π‘Π°Π½-Π”ΠΈΠ΅Π³ΠΎ, магистр Π½Π°ΡƒΠΊ…

View SAT Π Π΅ΠΏΠ΅Ρ‚ΠΈΡ‚ΠΎΡ€Ρ‹ ΠΏΠΎ ΠΌΠ°Ρ‚Π΅ΠΌΠ°Ρ‚ΠΈΠΊΠ΅

Π Π°ΠΉΠ°Π½
Π‘Π΅Ρ€Ρ‚ΠΈΡ„ΠΈΡ†ΠΈΡ€ΠΎΠ²Π°Π½Π½Ρ‹ΠΉ Ρ€Π΅ΠΏΠ΅Ρ‚ΠΈΡ‚ΠΎΡ€

УнивСрситСт ΠœΠΈΡΡΡƒΡ€ΠΈ-ΠšΠΎΠ»ΡƒΠΌΠ±ΠΈΡ, Π±Π°ΠΊΠ°Π»Π°Π²Ρ€ Π½Π°ΡƒΠΊ, ΠΎΠ±Ρ€Π°Π·ΠΎΠ²Π°Π½ΠΈΠ΅. Π’Π°ΡˆΠΈΠ½Π³Ρ‚ΠΎΠ½ΡΠΊΠΈΠΉ унивСрситСт Π² Π‘Π΅Π½Ρ‚-ЛуисС, Π΄ΠΎΠΊΡ‚ΠΎΡ€ ΡŽΡ€ΠΈΠ΄ΠΈΡ‡Π΅ΡΠΊΠΈΡ… Π½Π°ΡƒΠΊ, ΡŽΡ€ΠΈΠ΄ΠΈΡ‡Π΅ΡΠΊΠΈΠΉ…

ВсС матСматичСскиС рСсурсы SAT

16 диагностичСских тСстов 660 практичСских тСстов Вопрос дня ΠšΠ°Ρ€Ρ‚ΠΎΡ‡ΠΊΠΈ Π£Ρ‡ΠΈΡ‚Π΅ΡΡŒ ΠΏΠΎ ΠΊΠΎΠ½Ρ†Π΅ΠΏΡ†ΠΈΠΈ

ΠšΠ°Π»ΡŒΠΊΡƒΠ»ΡΡ‚ΠΎΡ€ Π΄Ρ€ΠΎΠ±Π΅ΠΉ


Π­Ρ‚ΠΎΡ‚ ΠΊΠ°Π»ΡŒΠΊΡƒΠ»ΡΡ‚ΠΎΡ€ выполняСт основныС ΠΈ Ρ€Π°ΡΡˆΠΈΡ€Π΅Π½Π½Ρ‹Π΅ ΠΎΠΏΠ΅Ρ€Π°Ρ†ΠΈΠΈ с дробями, выраТСния с дробями Π² сочСтании с Ρ†Π΅Π»Ρ‹ΠΌΠΈ, дСсятичными ΠΈ ΡΠΌΠ΅ΡˆΠ°Π½Π½Ρ‹ΠΌΠΈ числами. Он Ρ‚Π°ΠΊΠΆΠ΅ ΠΏΠΎΠΊΠ°Π·Ρ‹Π²Π°Π΅Ρ‚ ΠΏΠΎΠ΄Ρ€ΠΎΠ±Π½ΡƒΡŽ ΠΏΠΎΡˆΠ°Π³ΠΎΠ²ΡƒΡŽ ΠΈΠ½Ρ„ΠΎΡ€ΠΌΠ°Ρ†ΠΈΡŽ ΠΎ ΠΏΡ€ΠΎΡ†Π΅Π΄ΡƒΡ€Π΅ расчСта Π΄Ρ€ΠΎΠ±ΠΈ. ΠšΠ°Π»ΡŒΠΊΡƒΠ»ΡΡ‚ΠΎΡ€ ΠΏΠΎΠΌΠΎΠ³Π°Π΅Ρ‚ Π½Π°ΠΉΡ‚ΠΈ Π·Π½Π°Ρ‡Π΅Π½ΠΈΠ΅ ΠΈΠ· ΠΎΠΏΠ΅Ρ€Π°Ρ†ΠΈΠΉ с нСсколькими дробями. Π Π΅ΡˆΠ°ΠΉΡ‚Π΅ Π·Π°Π΄Π°Ρ‡ΠΈ с двумя, трСмя ΠΈ Π±ΠΎΠ»Π΅Π΅ дробями ΠΈ числами Π² ΠΎΠ΄Π½ΠΎΠΌ Π²Ρ‹Ρ€Π°ΠΆΠ΅Π½ΠΈΠΈ.

ΠŸΡ€Π°Π²ΠΈΠ»Π° выраТСния с дробями:

Π”Ρ€ΠΎΠ±ΠΈ — ΠΈΡΠΏΠΎΠ»ΡŒΠ·ΡƒΠΉΡ‚Π΅ ΠΊΠΎΡΡƒΡŽ Ρ‡Π΅Ρ€Ρ‚Ρƒ для дСлСния числитСля Π½Π° Π·Π½Π°ΠΌΠ΅Π½Π°Ρ‚Π΅Π»ΡŒ, Ρ‚. Π΅. для пятисотых Π²Π²Π΅Π΄ΠΈΡ‚Π΅ 5/100 . Если Π²Ρ‹ ΠΈΡΠΏΠΎΠ»ΡŒΠ·ΡƒΠ΅Ρ‚Π΅ ΡΠΌΠ΅ΡˆΠ°Π½Π½Ρ‹Π΅ числа, ΠΎΡΡ‚Π°Π²ΡŒΡ‚Π΅ ΠΏΡ€ΠΎΠ±Π΅Π» ΠΌΠ΅ΠΆΠ΄Ρƒ Ρ†Π΅Π»ΠΎΠΉ ΠΈ Π΄Ρ€ΠΎΠ±Π½ΠΎΠΉ частями.

Π‘ΠΌΠ΅ΡˆΠ°Π½Π½Ρ‹Π΅ числа (ΡΠΌΠ΅ΡˆΠ°Π½Π½Ρ‹Π΅ числа ΠΈΠ»ΠΈ Π΄Ρ€ΠΎΠ±ΠΈ) ΡΠΎΡ…Ρ€Π°Π½ΡΡŽΡ‚ ΠΎΠ΄ΠΈΠ½ ΠΏΡ€ΠΎΠ±Π΅Π» ΠΌΠ΅ΠΆΠ΄Ρƒ Ρ†Π΅Π»Ρ‹ΠΌ числом ΠΈ Π΄Ρ€ΠΎΠ±ΡŒΡŽ
ΠΈ ΠΈΡΠΏΠΎΠ»ΡŒΠ·ΡƒΡŽΡ‚ ΠΊΠΎΡΡƒΡŽ Ρ‡Π΅Ρ€Ρ‚Ρƒ для Π²Π²ΠΎΠ΄Π° Π΄Ρ€ΠΎΠ±Π΅ΠΉ, Π½Π°ΠΏΡ€ΠΈΠΌΠ΅Ρ€, 1Β 2/3 . ΠŸΡ€ΠΈΠΌΠ΅Ρ€ ΠΎΡ‚Ρ€ΠΈΡ†Π°Ρ‚Π΅Π»ΡŒΠ½ΠΎΠΉ смСшанной Π΄Ρ€ΠΎΠ±ΠΈ: -5 1/2 .
ΠŸΠΎΡΠΊΠΎΠ»ΡŒΠΊΡƒ косая Ρ‡Π΅Ρ€Ρ‚Π° являСтся ΠΎΠ΄Π½ΠΎΠ²Ρ€Π΅ΠΌΠ΅Π½Π½ΠΎ Π·Π½Π°ΠΊΠΎΠΌ Π΄Ρ€ΠΎΠ±Π½ΠΎΠΉ части ΠΈ дСлСния, ΠΈΡΠΏΠΎΠ»ΡŒΠ·ΡƒΠΉΡ‚Π΅ Π΄Π²ΠΎΠ΅Ρ‚ΠΎΡ‡ΠΈΠ΅ (:) Π² качСствС ΠΎΠΏΠ΅Ρ€Π°Ρ‚ΠΎΡ€Π° дСлСния Π΄Ρ€ΠΎΠ±Π΅ΠΉ, Ρ‚. Π΅. 1/2 : 1/3 .
Decimals (дСсятичныС числа) вводятся с дСсятичной Ρ‚ΠΎΡ‡ΠΊΠΎΠΉ . ΠΈ ΠΎΠ½ΠΈ автоматичСски ΠΏΡ€Π΅ΠΎΠ±Ρ€Π°Π·ΡƒΡŽΡ‚ΡΡ Π² Π΄Ρ€ΠΎΠ±ΠΈ — Ρ‚.Π΅. 1,45 .

Math Symbols


Symbol Symbol name Symbol Meaning Example
+ plus sign addition 1/2 + 1/3
Π·Π½Π°ΠΊ минус Π²Ρ‹Ρ‡ΠΈΡ‚Π°Π½ΠΈΠ΅ 1 1/2 — 2/3
* asterisk multiplication 2/3 * 3/4 ​​
Γ— times sign multiplication 2 /3 Γ— 5/6
: division sign division 1/2 : 3
/ division slash division 1/3 / 5

/2
β€’ слоТСниС Π΄Ρ€ΠΎΠ±Π΅ΠΉ ΠΈ ΡΠΌΠ΅ΡˆΠ°Π½Π½Ρ‹Ρ… чисСл: 8/5 + 6 2/7
β€’ Π΄Π΅Π»Π΅Π½ΠΈΠ΅ Ρ†Π΅Π»Ρ‹Ρ… чисСл ΠΈ Π΄Ρ€ΠΎΠ±Π΅ΠΉ: 5 Γ· 1/2
β€’ слоТныС Π΄Ρ€ΠΎΠ±ΠΈ: 5/8 : 2 2/3
β€’ дСсятичная Π΄Ρ€ΠΎΠ±ΡŒ: 0,625
β€’ ΠŸΡ€Π΅ΠΎΠ±Ρ€Π°Π·ΠΎΠ²Π°Π½ΠΈΠ΅ Π΄Ρ€ΠΎΠ±ΠΈ Π² Π΄Π΅ΡΡΡ‚ΠΈΡ‡Π½ΡƒΡŽ: 1/4
β€’ ΠŸΡ€Π΅ΠΎΠ±Ρ€Π°Π·ΠΎΠ²Π°Π½ΠΈΠ΅ Π΄Ρ€ΠΎΠ±ΠΈ Π² ΠΏΡ€ΠΎΡ†Π΅Π½Ρ‚: 1/8 %
β€’ сравнСниС Π΄Ρ€ΠΎΠ±Π΅ΠΉ: 1/4 2/3
β€’ ΡƒΠΌΠ½ΠΎΠΆΠ΅Π½ΠΈΠ΅ Π΄Ρ€ΠΎΠ±ΠΈ Π½Π° Ρ†Π΅Π»ΠΎΠ΅ число: 6 * 3/4 ​​
β€’ ΠΊΠ²Π°Π΄Ρ€Π°Ρ‚Π½Ρ‹ΠΉ ΠΊΠΎΡ€Π΅Π½ΡŒ Π΄Ρ€ΠΎΠ±ΠΈ: sqrt(1/16)
β€’ ΡƒΠΌΠ΅Π½ΡŒΡˆΠ΅Π½ΠΈΠ΅ ΠΈΠ»ΠΈ ΡƒΠΏΡ€ΠΎΡ‰Π΅Π½ΠΈΠ΅ Π΄Ρ€ΠΎΠ±ΠΈ (ΡƒΠΏΡ€ΠΎΡ‰Π΅Π½ΠΈΠ΅) — Π΄Π΅Π»Π΅Π½ΠΈΠ΅ числитСля ΠΈ знамСнатСля Π΄Ρ€ΠΎΠ±ΠΈ Π½Π° ΠΎΠ΄Π½ΠΎ ΠΈ Ρ‚ΠΎ ΠΆΠ΅ Π½Π΅Π½ΡƒΠ»Π΅Π²ΠΎΠ΅ число — эквивалСнтная Π΄Ρ€ΠΎΠ±ΡŒ: 4/22
β€’ Π²Ρ‹Ρ€Π°ΠΆΠ΅Π½ΠΈΠ΅ со скобками: 1/3 * (1/2 — 3 3/8)
β€’ составная Π΄Ρ€ΠΎΠ±ΡŒ: 3/4 ΠΎΡ‚ 5/7
β€’ ΠΊΡ€Π°Ρ‚Π½Ρ‹Π΅ Π΄Ρ€ΠΎΠ±ΠΈ: 2/3 ΠΎΡ‚ 3/5
β€’ Ρ€Π°Π·Π΄Π΅Π»ΠΈΡ‚ΡŒ, Ρ‡Ρ‚ΠΎΠ±Ρ‹ Π½Π°ΠΉΡ‚ΠΈ частноС: 3/5 Γ· 2/3

ΠšΠ°Π»ΡŒΠΊΡƒΠ»ΡΡ‚ΠΎΡ€ слСдуСт извСстным ΠΏΡ€Π°Π²ΠΈΠ»Π°ΠΌ для порядка ΠΎΠΏΠ΅Ρ€Π°Ρ†ΠΈΠΉ . НаиболСС распространСнныС ΠΌΠ½Π΅ΠΌΠΎΠ½ΠΈΠΊΠΈ для запоминания этого порядка ΠΎΠΏΠ΅Ρ€Π°Ρ†ΠΈΠΉ:
PEMDAS β€” Π‘ΠΊΠΎΠ±ΠΊΠΈ, ЭкспонСнты, Π£ΠΌΠ½ΠΎΠΆΠ΅Π½ΠΈΠ΅, Π”Π΅Π»Π΅Π½ΠΈΠ΅, Π‘Π»ΠΎΠΆΠ΅Π½ΠΈΠ΅, Π’Ρ‹Ρ‡ΠΈΡ‚Π°Π½ΠΈΠ΅.
BEDMAS — Π‘ΠΊΠΎΠ±ΠΊΠΈ, ЭкспонСнты, Π”Π΅Π»Π΅Π½ΠΈΠ΅, Π£ΠΌΠ½ΠΎΠΆΠ΅Π½ΠΈΠ΅, Π‘Π»ΠΎΠΆΠ΅Π½ΠΈΠ΅, Π’Ρ‹Ρ‡ΠΈΡ‚Π°Π½ΠΈΠ΅
BODMAS — Π‘ΠΊΠΎΠ±ΠΊΠΈ, ΠŸΠΎΡ€ΡΠ΄ΠΎΠΊ, Π”Π΅Π»Π΅Π½ΠΈΠ΅, Π£ΠΌΠ½ΠΎΠΆΠ΅Π½ΠΈΠ΅, Π‘Π»ΠΎΠΆΠ΅Π½ΠΈΠ΅, Π’Ρ‹Ρ‡ΠΈΡ‚Π°Π½ΠΈΠ΅.
GEMDAS β€” символы Π³Ρ€ΡƒΠΏΠΏΠΈΡ€ΠΎΠ²ΠΊΠΈ β€” скобки (){}, ΠΏΠΎΠΊΠ°Π·Π°Ρ‚Π΅Π»ΠΈ стСпСни, ΡƒΠΌΠ½ΠΎΠΆΠ΅Π½ΠΈΠ΅, Π΄Π΅Π»Π΅Π½ΠΈΠ΅, слоТСниС, Π²Ρ‹Ρ‡ΠΈΡ‚Π°Π½ΠΈΠ΅.
MDAS β€” Π£ΠΌΠ½ΠΎΠΆΠ΅Π½ΠΈΠ΅ ΠΈ Π΄Π΅Π»Π΅Π½ΠΈΠ΅ ΠΈΠΌΠ΅ΡŽΡ‚ Ρ‚ΠΎΡ‚ ΠΆΠ΅ ΠΏΡ€ΠΈΠΎΡ€ΠΈΡ‚Π΅Ρ‚, Ρ‡Ρ‚ΠΎ ΠΈ слоТСниС ΠΈ Π²Ρ‹Ρ‡ΠΈΡ‚Π°Π½ΠΈΠ΅. ΠŸΡ€Π°Π²ΠΈΠ»ΠΎ MDAS являСтся Ρ‡Π°ΡΡ‚ΡŒΡŽ порядка ΠΎΠΏΠ΅Ρ€Π°Ρ†ΠΈΠΉ ΠΏΡ€Π°Π²ΠΈΠ»Π° PEMDAS.
Π‘ΡƒΠ΄ΡŒ остороТСн; всСгда выполняйтС ΡƒΠΌΠ½ΠΎΠΆΠ΅Π½ΠΈΠ΅ ΠΈ Π΄Π΅Π»Π΅Π½ΠΈΠ΅ ΠΏΠ΅Ρ€Π΅Π΄ слоТСниСм ΠΈ Π²Ρ‹Ρ‡ΠΈΡ‚Π°Π½ΠΈΠ΅ΠΌ . НСкоторыС ΠΎΠΏΠ΅Ρ€Π°Ρ‚ΠΎΡ€Ρ‹ (+ ΠΈ -) ΠΈ (* ΠΈ /) ΠΈΠΌΠ΅ΡŽΡ‚ ΠΎΠ΄ΠΈΠ½Π°ΠΊΠΎΠ²Ρ‹ΠΉ ΠΏΡ€ΠΈΠΎΡ€ΠΈΡ‚Π΅Ρ‚ ΠΈ Π΄ΠΎΠ»ΠΆΠ½Ρ‹ ΠΎΡ†Π΅Π½ΠΈΠ²Π°Ρ‚ΡŒΡΡ слСва Π½Π°ΠΏΡ€Π°Π²ΠΎ.

  • Π’ столовой
    Π’ классной ΠΊΠΎΠΌΠ½Π°Ρ‚Π΅ Π”ΠΆΠ΅ΠΉΠΊΠΎΠ±Π° 18 ΡƒΡ‡Π΅Π½ΠΈΠΊΠΎΠ². Π¨Π΅ΡΡ‚ΡŒ ΡƒΡ‡Π΅Π½ΠΈΠΊΠΎΠ² приносят ΠΎΠ±Π΅Π΄ Π² ΡˆΠΊΠΎΠ»Ρƒ. ΠžΡΡ‚Π°Π»ΡŒΠ½Ρ‹Π΅ ΠΎΠ±Π΅Π΄Π°ΡŽΡ‚ Π² столовой. ΠŸΡ€ΠΎΡ‰Π΅ говоря, какая Ρ‡Π°ΡΡ‚ΡŒ студСнтов ΠΎΠ±Π΅Π΄Π°Π΅Ρ‚ Π² столовой?
  • Π£ Макса 2
    Π£ Макса 13 ΠΏΠ°Ρ€ носков. ΠžΡ‚ΡΡŽΠ΄Π° ΡˆΠ΅ΡΡ‚ΡŒ ΠΏΠ°Ρ€ синих, Ρ‚Ρ€ΠΈ ΠΏΠ°Ρ€Ρ‹ ΠΊΠΎΡ€ΠΈΡ‡Π½Π΅Π²Ρ‹Ρ…, Π΄Π²Π΅ Ρ‡Π΅Ρ€Π½Ρ‹Ρ… ΠΈ Π΄Π²Π΅ Π±Π΅Π»Ρ‹Ρ…. Какая Ρ‡Π°ΡΡ‚ΡŒ носков Макса ΠΊΠΎΡ€ΠΈΡ‡Π½Π΅Π²ΠΎΠ³ΠΎ ΠΈΠ»ΠΈ Ρ‡Π΅Ρ€Π½ΠΎΠ³ΠΎ Ρ†Π²Π΅Ρ‚Π°?
  • Π—Π½Π°Ρ‡Π΅Π½ΠΈΠ΅ Z
    ΠŸΡ€ΠΈ x = -9, ΠΊΠ°ΠΊΠΎΠ²ΠΎ Π·Π½Π°Ρ‡Π΅Π½ΠΈΠ΅ Z, Π³Π΄Π΅ Z Ρ€Π°Π²Π½ΠΎ Ρ‡ΠΈΡΠ»ΠΈΡ‚Π΅Π»ΡŽ Π΄Ρ€ΠΎΠ±ΠΈ x минус 17 Π² Π·Π½Π°ΠΌΠ΅Π½Π°Ρ‚Π΅Π»Π΅ 6,5 ΠΊΠΎΠ½Π΅Ρ† Π΄Ρ€ΠΎΠ±ΠΈ Π”Π°ΠΉΡ‚Π΅ ΠΎΡ‚Π²Π΅Ρ‚ с Ρ‚ΠΎΡ‡Π½ΠΎΡΡ‚ΡŒΡŽ Π΄ΠΎ 2 Π·Π½Π°ΠΊΠΎΠ² послС запятой.
  • ΠšΠΎΡ€Π·ΠΈΠ½Π° с Ρ„Ρ€ΡƒΠΊΡ‚Π°ΠΌΠΈ
    Если Π² ΠΊΠΎΡ€Π·ΠΈΠ½Π΅ 7 яблок ΠΈ 5 апСльсинов, Ρ‚ΠΎ какая доля апСльсинов Π² ΠΊΠΎΡ€Π·ΠΈΠ½Π΅ с Ρ„Ρ€ΡƒΠΊΡ‚Π°ΠΌΠΈ?
  • Π”Ρ€ΠΎΠ±ΡŒ ΠΈ дСсятичная Π΄Ρ€ΠΎΠ±ΡŒ
    ΠŸΠΈΡˆΠΈΡ‚Π΅ Π² Π²ΠΈΠ΄Π΅ Π΄Ρ€ΠΎΠ±ΠΈ ΠΈ дСсятичной Π΄Ρ€ΠΎΠ±ΠΈ. Один ΠΈ Π΄Π²Π° плюс Ρ‚Ρ€ΠΈ ΠΈ ΠΏΡΡ‚ΡŒ сотых
  • Π”Ρ€ΠΎΠ±ΡŒ Π΄ΠΎ дСсятичной
    Π—Π°ΠΏΠΈΡˆΠΈΡ‚Π΅ Π΄Ρ€ΠΎΠ±ΡŒ 3/22 Π² Π²ΠΈΠ΄Π΅ дСсятичной Π΄Ρ€ΠΎΠ±ΠΈ.
  • Компания
    Компания ΠΈΠΌΠ΅Π΅Ρ‚ 860 сотрудников, ΠΈΠ· ΠΊΠΎΡ‚ΠΎΡ€Ρ‹Ρ… 500 ΠΆΠ΅Π½Ρ‰ΠΈΠ½. ΠΠ°ΠΏΠΈΡˆΠΈΡ‚Π΅ Π΄Ρ€ΠΎΠ±ΡŒ, ΠΎΠ±ΠΎΠ·Π½Π°Ρ‡Π°ΡŽΡ‰ΡƒΡŽ сотрудниц ΠΊΠΎΠΌΠΏΠ°Π½ΠΈΠΈ.
  • ΠšΡ‚ΠΎ-Ρ‚ΠΎ
    ΠšΡ‚ΠΎ-Ρ‚ΠΎ съСл 1/10 Ρ‚ΠΎΡ€Ρ‚Π°, ΠΎΡΡ‚Π°Π»ΠΎΡΡŒ Ρ‚ΠΎΠ»ΡŒΠΊΠΎ 9/10. Если Π²Ρ‹ ΡΡŠΠ΅Π΄ΠΈΡ‚Π΅ 2/3 ΠΎΡΡ‚Π°Π²ΡˆΠ΅Π³ΠΎΡΡ Ρ‚ΠΎΡ€Ρ‚Π°, сколько всСго Ρ‚ΠΎΡ€Ρ‚Π° Π²Ρ‹ ΡΡŠΠ΅Π΄ΠΈΡ‚Π΅?
  • Дробями
    ΠœΡƒΡ€Π°Π²Π΅ΠΉ поднимаСтся Π½Π° 2/5 ΡˆΠ΅ΡΡ‚Π° Π·Π° ΠΏΠ΅Ρ€Π²Ρ‹ΠΉ час ΠΈ Π½Π° 1/4 ΡˆΠ΅ΡΡ‚Π° Π·Π° ΡΠ»Π΅Π΄ΡƒΡŽΡ‰ΠΈΠΉ час. ΠšΠ°ΠΊΡƒΡŽ Ρ‡Π°ΡΡ‚ΡŒ ΡˆΠ΅ΡΡ‚Π° ΠΏΡ€Π΅ΠΎΠ΄ΠΎΠ»Π΅Π²Π°Π΅Ρ‚ ΠΌΡƒΡ€Π°Π²Π΅ΠΉ Π·Π° Π΄Π²Π° часа?
  • ΠœΡΡ‚ΡŒΡŽ
    Π£ ΠœΡΡ‚ΡŒΡŽ восСмь ΠΊΠ°Ρ€Π°Π½Π΄Π°ΡˆΠ΅ΠΉ. Π£ Ρ‚Ρ€Π΅Ρ… ΠΈΠ· Π½ΠΈΡ… Π½Π΅Ρ‚ ластика Π½Π° ΠΊΠΎΠ½Ρ†Π΅. Какая Ρ‡Π°ΡΡ‚ΡŒ ΠΊΠ°Ρ€Π°Π½Π΄Π°ΡˆΠ΅ΠΉ Π½Π΅ ΠΈΠΌΠ΅Π΅Ρ‚ ластика Π½Π° ΠΊΠΎΠ½Ρ†Π΅?
  • a Quarter
    a Quarter ΠΎΡ‚ числа 72 IS:

ΠŸΠΎΠ΄Ρ€ΠΎΠ±Π½Π΅Π΅ Π·Π°Π΄Π°Ρ‡ΠΈ ΠΏΠΎ ΠΌΠ°Ρ‚Π΅ΠΌΠ°Ρ‚ΠΈΠΊΠ΅ Β»

  • ДСсятиц
  • Π€Ρ€Π°ΠΊΡ†ΠΈΠΈ
  • Π’Ρ€Π΅ΡƒΠ³ΠΎΠ»ΡŒΠ½Ρ‹Π΅
  • ΠΠžΠ”
  • Π–ΠšΠ˜
  • ΠšΠΎΠΌΠ±ΠΈΠ½Π°Ρ‚ΠΎΡ€ΠΈΠΊΠ°
  • уравнСния
  • статистика
  • … всС матСматичСскиС ΠΊΠ°Π»ΡŒΠΊΡƒΠ»ΡΡ‚ΠΎΡ€Ρ‹

Π£ΠΌΠ½ΠΎΠΆΠ΅Π½ΠΈΠ΅ ΠΈ Π΄Π΅Π»Π΅Π½ΠΈΠ΅ ΠΎΡ‚Ρ€ΠΈΡ†Π°Ρ‚Π΅Π»ΡŒΠ½Ρ‹Ρ… чисСл

Π’Π²Π΅Π΄Π΅Π½ΠΈΠ΅Π‘Π»ΠΎΠΆΠ΅Π½ΠΈΠ΅ ΠΈ вычитаниСЭкспонСнты

Purplemath На самом Π΄Π΅Π»Π΅, ΠΌΡ‹ ΡƒΠΆΠ΅ рассмотрСли ΡΠ»ΠΎΠΆΠ½ΡƒΡŽ Ρ‡Π°ΡΡ‚ΡŒ: Π²Ρ‹ ΡƒΠΆΠ΅ Π·Π½Π°Π΅Ρ‚Π΅ ΠΏΡ€Π°Π²ΠΈΠ»Π° Β«Π·Π½Π°ΠΊΠ°Β»:

плюс ΡƒΠΌΠ½ΠΎΠΆΠΈΡ‚ΡŒ Π½Π° плюс это плюс
(Π΄ΠΎΠ±Π°Π²Π»Π΅Π½ΠΈΠ΅ большого количСства горячих ΠΊΡƒΠ±ΠΈΠΊΠΎΠ² ΠΏΠΎΠ²Ρ‹ΡˆΠ°Π΅Ρ‚ Ρ‚Π΅ΠΌΠΏΠ΅Ρ€Π°Ρ‚ΡƒΡ€Ρƒ)

минус ΡƒΠΌΠ½ΠΎΠΆΠΈΡ‚ΡŒ плюс минус
(ΡƒΠ΄Π°Π»Π΅Π½ΠΈΠ΅ большого количСства горячих ΠΊΡƒΠ±ΠΈΠΊΠΎΠ² сниТаСт Ρ‚Π΅ΠΌΠΏΠ΅Ρ€Π°Ρ‚ΡƒΡ€Ρƒ)

плюс Ρ€Π°Π· минус минус
(Π΄ΠΎΠ±Π°Π²Π»Π΅Π½ΠΈΠ΅ большого количСства Ρ…ΠΎΠ»ΠΎΠ΄Π½Ρ‹Ρ… ΠΊΡƒΠ±ΠΈΠΊΠΎΠ² сниТаСт Ρ‚Π΅ΠΌΠΏΠ΅Ρ€Π°Ρ‚ΡƒΡ€Ρƒ)

минус Ρ€Π°Π· минус плюс
(ΡƒΠ΄Π°Π»Π΅Π½ΠΈΠ΅ большого количСства ΠΊΡƒΠ±ΠΈΠΊΠΎΠ² Ρ…ΠΎΠ»ΠΎΠ΄Π° ΠΏΠΎΠ²Ρ‹ΡˆΠ°Π΅Ρ‚ Ρ‚Π΅ΠΌΠΏΠ΅Ρ€Π°Ρ‚ΡƒΡ€Ρƒ)

Π‘ΠΎΠ΄Π΅Ρ€ΠΆΠ°Π½ΠΈΠ΅ продолТаСтся Π½ΠΈΠΆΠ΅

MathHelp.

com

Π£ΠΌΠ½ΠΎΠΆΠ΅Π½ΠΈΠ΅ ΠΈ Π΄Π΅Π»Π΅Π½ΠΈΠ΅ Ρ†Π΅Π»Ρ‹Ρ… чисСл

ΠŸΡ€Π°Π²ΠΈΠ»Π° Π·Π½Π°ΠΊΠΎΠ² Ρ€Π°Π±ΠΎΡ‚Π°ΡŽΡ‚ Ρ‚Π°ΠΊ ΠΆΠ΅ для дСлСния; просто Π·Π°ΠΌΠ΅Π½ΠΈΡ‚Π΅ Β«Ρ€Π°Π·Β» Π½Π° «дСлится Π½Π°Β». Π’ΠΎΡ‚ ΠΏΡ€ΠΈΠΌΠ΅Ρ€ ΠΏΡ€Π°Π²ΠΈΠ» Π² Π΄ΠΈΠ²ΠΈΠ·ΠΈΠΎΠ½Π΅:

(ΠŸΠΎΠΌΠ½ΠΈΡ‚Π΅, Ρ‡Ρ‚ΠΎ Π΄Ρ€ΠΎΠ±ΠΈ β€” это Π΅Ρ‰Π΅ ΠΎΠ΄Π½Π° Ρ„ΠΎΡ€ΠΌΠ° дСлСния! Β«Π”Ρ€ΠΎΠ±ΠΈ β€” это Π΄Π΅Π»Π΅Π½ΠΈΠ΅Β»!)


НСкоторым людям нравится Π΄ΡƒΠΌΠ°Ρ‚ΡŒ ΠΎΠ± ΠΎΡ‚Ρ€ΠΈΡ†Π°Ρ‚Π΅Π»ΡŒΠ½Ρ‹Ρ… числах Π² Ρ‚Π΅Ρ€ΠΌΠΈΠ½Π°Ρ… Π΄ΠΎΠ»Π³ΠΎΠ². Π’Π°ΠΊ, Π½Π°ΠΏΡ€ΠΈΠΌΠ΅Ρ€, Ссли Π²Ρ‹ Π΄ΠΎΠ»ΠΆΠ½Ρ‹ 10 Π΄ΠΎΠ»Π»Π°Ρ€ΠΎΠ² ΡˆΠ΅ΡΡ‚ΠΈ людям, ваш ΠΎΠ±Ρ‰ΠΈΠΉ Π΄ΠΎΠ»Π³ Π±ΡƒΠ΄Π΅Ρ‚ 6 Γ— 10 Π΄ΠΎΠ»Π»Π°Ρ€ΠΎΠ² = 60 Π΄ΠΎΠ»Π»Π°Ρ€ΠΎΠ². Π’ этом контСкстС ΠΏΠΎΠ»ΡƒΡ‡Π΅Π½ΠΈΠ΅ ΠΎΡ‚Ρ€ΠΈΡ†Π°Ρ‚Π΅Π»ΡŒΠ½ΠΎΠ³ΠΎ ΠΎΡ‚Π²Π΅Ρ‚Π° ΠΈΠΌΠ΅Π΅Ρ‚ смысл. Но Π² ΠΊΠ°ΠΊΠΎΠΌ контСкстС ΠΌΠΎΠΆΠ΅Ρ‚ ΠΈΠΌΠ΅Ρ‚ΡŒ смысл Π΄Π΅Π»Π΅Π½ΠΈΠ΅ ΠΎΡ‚Ρ€ΠΈΡ†Π°Ρ‚Π΅Π»ΡŒΠ½ΠΎΠ³ΠΎ Π½Π° ΠΎΡ‚Ρ€ΠΈΡ†Π°Ρ‚Π΅Π»ΡŒΠ½ΠΎΠ΅ (ΠΈ ΠΏΠΎΠ»ΡƒΡ‡Π΅Π½ΠΈΠ΅ ΠΏΠΎΠ»ΠΎΠΆΠΈΡ‚Π΅Π»ΡŒΠ½ΠΎΠ³ΠΎ)?

ΠŸΠΎΠ΄ΡƒΠΌΠ°ΠΉΡ‚Π΅ ΠΎ пСрСкусС Π² ΠΊΠ°Ρ„Π΅. Когда Π²Ρ‹ ΠΈΠ΄Π΅Ρ‚Π΅ ΠΏΠ»Π°Ρ‚ΠΈΡ‚ΡŒ, Ρƒ Ρ€Π΅Π±Π΅Π½ΠΊΠ° Π²ΠΎΠ·Π½ΠΈΠΊΠ°ΡŽΡ‚ ΠΏΡ€ΠΎΠ±Π»Π΅ΠΌΡ‹ с вашСй Π΄Π΅Π±Π΅Ρ‚ΠΎΠ²ΠΎΠΉ ΠΊΠ°Ρ€Ρ‚ΠΎΠΉ. Он ΠΏΡ€ΠΎΠ²ΠΎΠ΄ΠΈΡ‚ Сю ΡˆΠ΅ΡΡ‚ΡŒ Ρ€Π°Π·, ΠΏΡ€Π΅ΠΆΠ΄Π΅ Ρ‡Π΅ΠΌ, Π½Π°ΠΊΠΎΠ½Π΅Ρ†, Π²Π΅Ρ€Π½ΡƒΡ‚ΡŒ ΠΊΠ°Ρ€Ρ‚Ρƒ Π²Π°ΠΌ. Π’Π΅Ρ€Π½ΡƒΠ²ΡˆΠΈΡΡŒ Π΄ΠΎΠΌΠΎΠΉ, Π²Ρ‹ провСряСтС свой банковский счСт ΠΎΠ½Π»Π°ΠΉΠ½. Π’Ρ‹ ΠΌΠΎΠΆΠ΅Ρ‚Π΅ ΡΠΊΠ°Π·Π°Ρ‚ΡŒ ΠΏΠΎ суммС, Ρ‡Ρ‚ΠΎ Π΄Π°, ΠΎΠ½ Π΄Π΅ΠΉΡΡ‚Π²ΠΈΡ‚Π΅Π»ΡŒΠ½ΠΎ взял с вас ΠΏΡƒΡ‚ΡŒ Π±ΠΎΠ»Π΅Π΅ ΠΎΠ΄Π½ΠΎΠ³ΠΎ Ρ€Π°Π·Π°. НСкоторая Ρ‡Π°ΡΡ‚ΡŒ этого ΠΎΠ±Ρ‰Π΅Π³ΠΎ Π΄Π΅Π±Π΅Ρ‚Π° (ΠΎΡ‚Ρ€ΠΈΡ†Π°Ρ‚Π΅Π»ΡŒΠ½Π°Ρ сумма Π½Π° вашСм счСту) Π½Π΅Π²Π΅Ρ€Π½Π°.

ΠŸΡ€Π΅ΠΆΠ΄Π΅ Ρ‡Π΅ΠΌ Π·Π²ΠΎΠ½ΠΈΡ‚ΡŒ Π² свой Π±Π°Π½ΠΊ, Ρ‡Ρ‚ΠΎΠ±Ρ‹ ΠΈΡΠΏΡ€Π°Π²ΠΈΡ‚ΡŒ ΡΠΈΡ‚ΡƒΠ°Ρ†ΠΈΡŽ, Π²Ρ‹ Ρ…ΠΎΡ‚ΠΈΡ‚Π΅ ΠΏΠΎΠ΄Ρ‚Π²Π΅Ρ€Π΄ΠΈΡ‚ΡŒ количСство пСрСрасходов. Как Π²Ρ‹ ΠΌΠΎΠΆΠ΅Ρ‚Π΅ ΠΏΠΎΠ½ΡΡ‚ΡŒ это? Π’Ρ‹ ΠΌΠΎΠΆΠ΅Ρ‚Π΅ Ρ€Π°Π·Π΄Π΅Π»ΠΈΡ‚ΡŒ всю сумму (скаТСм, 76,02 Π΄ΠΎΠ»Π»Π°Ρ€Π° БША) Π½Π° сумму, ΡƒΠΊΠ°Π·Π°Π½Π½ΡƒΡŽ Π² ΠΊΠ²ΠΈΡ‚Π°Π½Ρ†ΠΈΠΈ (Π½Π°ΠΏΡ€ΠΈΠΌΠ΅Ρ€, 12,67 Π΄ΠΎΠ»Π»Π°Ρ€Π° БША), которая являСтся суммой ΠΎΠ΄Π½ΠΎΠ³ΠΎ ΠΏΠ»Π°Ρ‚Π΅ΠΆΠ°. КаТдоС списаниС β€” это минус Π½Π° вашСм счСтС, поэтому ΠΌΠ°Ρ‚Π΅ΠΌΠ°Ρ‚ΠΈΠΊΠ° Ρ‚Π°ΠΊΠΎΠ²Π°:

(- 76,02 Π΄ΠΎΠ»Π»Π°Ρ€Π° БША) Γ· (- 12,67 Π΄ΠΎΠ»Π»Π°Ρ€Π° БША) = 6

Π’Π°ΠΊΠΈΠΌ ΠΎΠ±Ρ€Π°Π·ΠΎΠΌ, всСго Π±Ρ‹Π»ΠΎ Π΄Π΅ΠΉΡΡ‚Π²ΠΈΡ‚Π΅Π»ΡŒΠ½ΠΎ ΡˆΠ΅ΡΡ‚ΡŒ списаний. ΠšΠΎΠ»ΠΈΡ‡Π΅ΡΡ‚Π²ΠΎ зарядов, 6, ΠΏΠΎ подсчСту количСства событий, Π΄ΠΎΠ»ΠΆΠ½ΠΎ Π±Ρ‹Ρ‚ΡŒ ΠΏΠΎΠ»ΠΎΠΆΠΈΡ‚Π΅Π»ΡŒΠ½Ρ‹ΠΌ . Π’ этом Ρ€Π΅Π°Π»ΡŒΠ½ΠΎΠΌ контСкстС Π΄Π΅Π»Π΅Π½ΠΈΠ΅ минуса Π½Π° минус ΠΈ ΠΏΠΎΠ»ΡƒΡ‡Π΅Π½ΠΈΠ΅ плюса ΠΈΠΌΠ΅Π΅Ρ‚ смысл. И Ρ‚Π΅ΠΏΠ΅Ρ€ΡŒ Π²Ρ‹ Π·Π½Π°Π΅Ρ‚Π΅, Ρ‡Ρ‚ΠΎ ΠΏΠΎΡ€ΡƒΡ‡ΠΈΡ‚ΡŒ слуТбС ΠΏΠΎΠ΄Π΄Π΅Ρ€ΠΆΠΊΠΈ ΠΎΡ‚ΠΌΠ΅Π½ΠΈΡ‚ΡŒ Ρ€ΠΎΠ²Π½ΠΎ ΠΏΡΡ‚ΡŒ ΠΏΠ»Π°Ρ‚Π΅ΠΆΠ΅ΠΉ.


Π’Ρ‹ ΠΌΠΎΠΆΠ΅Ρ‚Π΅ Π·Π°ΠΌΠ΅Ρ‚ΠΈΡ‚ΡŒ, Ρ‡Ρ‚ΠΎ люди Β«ΠΎΡ‚ΠΌΠ΅Π½ΡΡŽΡ‚Β» Π·Π½Π°ΠΊΠΈ минус. Они ΠΏΠΎΠ»ΡŒΠ·ΡƒΡŽΡ‚ΡΡ Ρ‚Π΅ΠΌ Ρ„Π°ΠΊΡ‚ΠΎΠΌ, Ρ‡Ρ‚ΠΎ «минус ΡƒΠΌΠ½ΠΎΠΆΠΈΡ‚ΡŒ Π½Π° минус β€” это плюс». НапримСр, ΠΏΡ€Π΅Π΄ΠΏΠΎΠ»ΠΎΠΆΠΈΠΌ, Ρ‡Ρ‚ΠΎ Ρƒ вас Π΅ΡΡ‚ΡŒ (βˆ’2)(βˆ’3)(βˆ’4). Π›ΡŽΠ±Ρ‹Π΅ Π΄Π²Π° ΠΎΡ‚Ρ€ΠΈΡ†Π°Ρ‚Π΅Π»ΡŒΠ½Ρ‹Ρ… числа, ΠΏΠ΅Ρ€Π΅ΠΌΠ½ΠΎΠΆΠ΅Π½Π½Ρ‹Π΅ вмСстС, становятся ΠΎΠ΄Π½ΠΈΠΌ ΠΏΠΎΠ»ΠΎΠΆΠΈΡ‚Π΅Π»ΡŒΠ½Ρ‹ΠΌ. Π˜Ρ‚Π°ΠΊ, Π²Ρ‹Π±Π΅Ρ€ΠΈΡ‚Π΅ Π»ΡŽΠ±Ρ‹Π΅ Π΄Π²Π° ΡƒΠΌΠ½ΠΎΠΆΠ΅Π½Π½Ρ‹Ρ… (ΠΈΠ»ΠΈ Ρ€Π°Π·Π΄Π΅Π»Π΅Π½Π½Ρ‹Ρ…) отрицания ΠΈ Β«ΠΎΡ‚ΠΌΠ΅Π½ΠΈΡ‚Π΅Β» ΠΈΡ… Π·Π½Π°ΠΊΠΈ:

Π― Π½Π°Ρ‡Π½Ρƒ с Ρ‚ΠΎΠ³ΠΎ, Ρ‡Ρ‚ΠΎ ΡƒΠ±Π΅Ρ€Ρƒ ΠΎΠ΄Π½Ρƒ ΠΏΠ°Ρ€Ρƒ Π·Π½Π°ΠΊΠΎΠ² «ΠΌΠΈΠ½ΡƒΡ». ΠŸΠΎΡ‚ΠΎΠΌ ΡƒΠΌΠ½ΠΎΠΆΡƒ ΠΊΠ°ΠΊ ΠΎΠ±Ρ‹Ρ‡Π½ΠΎ.

(-2)(-3)(-4)

= (-2)(-3) (-4)

= (+6) (-4)

= -24

Если Π²Π°ΠΌ Π΄Π°Π½ΠΎ Π΄Π»ΠΈΠ½Π½ΠΎΠ΅ ΡƒΠΌΠ½ΠΎΠΆΠ΅Π½ΠΈΠ΅ с ΠΎΡ‚Ρ€ΠΈΡ†Π°Ρ‚Π΅Π»ΡŒΠ½Ρ‹ΠΌΠΈ числами, просто сократитС Π·Π½Π°ΠΊΠΈ «минус» ΠΏΠΎΠΏΠ°Ρ€Π½ΠΎ:

ΠŸΠ΅Ρ€Π²ΠΎΠ΅, Ρ‡Ρ‚ΠΎ я сдСлаю, это ΠΏΠΎΡΡ‡ΠΈΡ‚Π°ΡŽ Π·Π½Π°ΠΊΠΈ «минус». Один Π΄Π²Π° Ρ‚Ρ€ΠΈ Ρ‡Π΅Ρ‚Ρ‹Ρ€Π΅ ΠΏΡΡ‚ΡŒ ΡˆΠ΅ΡΡ‚ΡŒ сСмь. Π˜Ρ‚Π°ΠΊ, Π΅ΡΡ‚ΡŒ Ρ‚Ρ€ΠΈ ΠΏΠ°Ρ€Ρ‹, ΠΊΠΎΡ‚ΠΎΡ€Ρ‹Π΅ я ΠΌΠΎΠ³Ρƒ ΠΎΡ‚ΠΌΠ΅Π½ΠΈΡ‚ΡŒ, ΠΈ ΠΎΠ΄Π½Π° останСтся. Π’ Ρ€Π΅Π·ΡƒΠ»ΡŒΡ‚Π°Ρ‚Π΅ ΠΌΠΎΠΉ ΠΎΠΊΠΎΠ½Ρ‡Π°Ρ‚Π΅Π»ΡŒΠ½Ρ‹ΠΉ ΠΎΡ‚Π²Π΅Ρ‚ Π΄ΠΎΠ»ΠΆΠ΅Π½ Π±Ρ‹Ρ‚ΡŒ ΠΎΡ‚Ρ€ΠΈΡ†Π°Ρ‚Π΅Π»ΡŒΠ½Ρ‹ΠΌ. Если я ΠΏΠΎΠ»ΡƒΡ‡Ρƒ ΠΏΠΎΠ»ΠΎΠΆΠΈΡ‚Π΅Π»ΡŒΠ½Ρ‹ΠΉ Ρ€Π΅Π·ΡƒΠ»ΡŒΡ‚Π°Ρ‚, я Π±ΡƒΠ΄Ρƒ Π·Π½Π°Ρ‚ΡŒ, Ρ‡Ρ‚ΠΎ сдСлал Ρ‡Ρ‚ΠΎ-Ρ‚ΠΎ Π½Π΅ Ρ‚Π°ΠΊ.

(-1)(-2)(-1)(-3)(-4)(-2)(-1)

= (-1)(-2) (-1)(-3 )(βˆ’4)(βˆ’2)(βˆ’1)

= (+1)(+2) (βˆ’1)(βˆ’3)(βˆ’4)(βˆ’2)(βˆ’1)

= ( 1)(2) (-1)(-3) (-4)(-2)(-1)

= (1)(2) (+1)(+3) (-4) (βˆ’2)(βˆ’1)

= (1)(2)(1)(3) (βˆ’4)(βˆ’2) (βˆ’1)

= (1)(2)(1)( 3) (+4)(+2) (-1)

= (1)(2)(1)(3)(4)(2)(-1)

= (2)(3) (4)(2)(-1)

= 48(-1)

= -48

Π― ΠΏΠΎΠ»ΡƒΡ‡ΠΈΠ» ΠΎΡ‚Ρ€ΠΈΡ†Π°Ρ‚Π΅Π»ΡŒΠ½Ρ‹ΠΉ ΠΎΡ‚Π²Π΅Ρ‚, поэтому я знаю, Ρ‡Ρ‚ΠΎ ΠΌΠΎΠΉ Π·Π½Π°ΠΊ ΠΏΡ€Π°Π²ΠΈΠ»ΡŒΠ½Ρ‹ΠΉ.

Π’ΠΎΡ‚ Π΅Ρ‰Π΅ ΠΎΠ΄ΠΈΠ½ ΠΏΡ€ΠΈΠΌΠ΅Ρ€, ΠΏΠΎΠΊΠ°Π·Ρ‹Π²Π°ΡŽΡ‰ΠΈΠΉ Ρ‚ΠΎΡ‚ ΠΆΠ΅ процСсс ΠΎΡ‚ΠΌΠ΅Π½Ρ‹ Π² контСкстС дСлСния:


ΠžΡ‚Ρ€ΠΈΡ†Π°Ρ‚Π΅Π»ΡŒΠ½Ρ‹Π΅ числа Ρ‡Π΅Ρ€Π΅Π· ΠΊΡ€ΡƒΠ³Π»Ρ‹Π΅ скобки

Основная Ρ‚Ρ€ΡƒΠ΄Π½ΠΎΡΡ‚ΡŒ, с ΠΊΠΎΡ‚ΠΎΡ€ΠΎΠΉ люди ΡΡ‚Π°Π»ΠΊΠΈΠ²Π°ΡŽΡ‚ΡΡ с отрицаниями, Π·Π°ΠΊΠ»ΡŽΡ‡Π°Π΅Ρ‚ΡΡ Π² Ρ€Π°Π±ΠΎΡ‚Π΅ со скобками; особСнно ΠΏΡ€ΠΈ взятии отрицания Ρ‡Π΅Ρ€Π΅Π· ΠΊΡ€ΡƒΠ³Π»Ρ‹Π΅ скобки. ΠžΠ±Ρ‹Ρ‡Π½Π°Ρ ситуация ΠΏΡ€ΠΈΠΌΠ΅Ρ€Π½ΠΎ такая:

βˆ’3( x + 4)

Если Π±Ρ‹ Ρƒ вас Π±Ρ‹Π»ΠΎ Β«3 ( x + 4)Β», Π²Ρ‹ Π±Ρ‹ Π·Π½Π°Π»ΠΈ, Ρ‡Ρ‚ΠΎ Π½ΡƒΠΆΠ½ΠΎ Β«Ρ€Π°ΡΠΏΡ€Π΅Π΄Π΅Π»ΠΈΡ‚ΡŒΒ» 3 Β«ΠΏΠΎΒ» скобкам:

3( x + 4) = 3( x ) + 3(4) = 3 x + 12

Π’Π΅ ΠΆΠ΅ ΠΏΡ€Π°Π²ΠΈΠ»Π° ΠΏΡ€ΠΈΠΌΠ΅Π½ΡΡŽΡ‚ΡΡ ΠΏΡ€ΠΈ Ρ€Π°Π±ΠΎΡ‚Π΅ с отрицаниями. Если Ρƒ вас Π²ΠΎΠ·Π½ΠΈΠΊΠ»ΠΈ ΠΏΡ€ΠΎΠ±Π»Π΅ΠΌΡ‹ с отслСТиваниСм, ΠΈΡΠΏΠΎΠ»ΡŒΠ·ΡƒΠΉΡ‚Π΅ малСнькиС стрСлки:

← пролистнитС , Ρ‡Ρ‚ΠΎΠ±Ρ‹ ΠΏΡ€ΠΎΡΠΌΠΎΡ‚Ρ€Π΅Ρ‚ΡŒ ΠΏΠΎΠ»Π½ΠΎΠ΅ ΠΈΠ·ΠΎΠ±Ρ€Π°ΠΆΠ΅Π½ΠΈΠ΅ β†’


  • Π£ΠΏΡ€ΠΎΡΡ‚ΠΈΡ‚ΡŒ 3 (
    x βˆ’ 5).

МнС Π½ΡƒΠΆΠ½ΠΎ Π²Π·ΡΡ‚ΡŒ 3 Ρ‡Π΅Ρ€Π΅Π· ΠΊΡ€ΡƒΠ³Π»Ρ‹Π΅ скобки:

3( Ρ… — 5) = 3( Ρ… ) + 3(-5) = 3 Ρ… — 15

  • Π£ΠΏΡ€ΠΎΡΡ‚ΠΈΡ‚ΡŒ βˆ’2 (
    x βˆ’ 3).

Π—Π΄Π΅ΡΡŒ я Π²ΠΎΠ·ΡŒΠΌΡƒ «ΠΌΠΈΠ½ΡƒΡ» Ρ‡Π΅Ρ€Π΅Π· ΠΊΡ€ΡƒΠ³Π»Ρ‹Π΅ скобки; Π― Π±ΡƒΠ΄Ρƒ Ρ€Π°ΡΠΏΡ€Π΅Π΄Π΅Π»ΡΡ‚ΡŒ -2 Π½Π° x ΠΈ минус 3.

-2( Ρ… — 3) = -2( Ρ… ) — 2(-3) = -2 Ρ… + 2(+3) = -2 Ρ… + 6

ΠžΠ±Ρ€Π°Ρ‚ΠΈΡ‚Π΅ Π²Π½ΠΈΠΌΠ°Π½ΠΈΠ΅, ΠΊΠ°ΠΊ я Ρ‚Ρ‰Π°Ρ‚Π΅Π»ΡŒΠ½ΠΎ слСдил Π·Π° Π·Π½Π°ΠΊΠ°ΠΌΠΈ Π² скобках. Β«ΠœΠΈΠ½ΡƒΡΒ» Π±Ρ‹Π» сохранСн с 3 Π·Π° счСт использования Π΄Ρ€ΡƒΠ³ΠΎΠ³ΠΎ Π½Π°Π±ΠΎΡ€Π° скобок. НС ΡΡ‚Π΅ΡΠ½ΡΠΉΡ‚Π΅ΡΡŒ ΠΈΡΠΏΠΎΠ»ΡŒΠ·ΠΎΠ²Π°Ρ‚ΡŒ Π³Ρ€ΡƒΠΏΠΏΠΈΡ€ΡƒΡŽΡ‰ΠΈΠ΅ символы, Ρ‡Ρ‚ΠΎΠ±Ρ‹ ΡΠ΄Π΅Π»Π°Ρ‚ΡŒ вашС ΠΏΡ€Π΅Π΄ΠΏΠΎΠ»Π°Π³Π°Π΅ΠΌΠΎΠ΅ Π·Π½Π°Ρ‡Π΅Π½ΠΈΠ΅ ясным ΠΊΠ°ΠΊ для ΠΎΡ†Π΅Π½Ρ‰ΠΈΠΊΠ°, Ρ‚Π°ΠΊ ΠΈ для вас самих.


Другая ΠΏΡ€ΠΎΠ±Π»Π΅ΠΌΠ°, связанная с ΠΏΡ€Π΅Π΄Ρ‹Π΄ΡƒΡ‰Π΅ΠΉ, связана с Π²Ρ‹Ρ‡ΠΈΡ‚Π°Π½ΠΈΠ΅ΠΌ скобок. Π’Ρ‹ ΠΌΠΎΠΆΠ΅Ρ‚Π΅ ΠΎΡ‚ΡΠ»Π΅ΠΆΠΈΠ²Π°Ρ‚ΡŒ Π·Π½Π°ΠΊ вычитания, прСобразуя Π²Ρ‹Ρ‡ΠΈΡ‚Π°Π½ΠΈΠ΅ Π² ΡƒΠΌΠ½ΠΎΠΆΠ΅Π½ΠΈΠ΅ Π½Π° минус:

Π― Π½Π°Ρ‡Π½Ρƒ с Ρ‚ΠΎΠ³ΠΎ, Ρ‡Ρ‚ΠΎ Π½Π°ΠΏΠΈΡˆΡƒ ΠΌΠ°Π»Π΅Π½ΡŒΠΊΡƒΡŽ «1» ΠΏΠ΅Ρ€Π΅Π΄ скобками. Π—Π°Ρ‚Π΅ΠΌ я Π½Π°Ρ€ΠΈΡΡƒΡŽ стрСлки ΠΎΡ‚ этой 1 ΠΊ Ρ‚Π΅Ρ€ΠΌΠΈΠ½Π°ΠΌ Π² скобках, Ρ‡Ρ‚ΠΎΠ±Ρ‹ Π½Π°ΠΏΠΎΠΌΠ½ΠΈΡ‚ΡŒ сСбС, Ρ‡Ρ‚ΠΎ ΠΌΠ½Π΅ Π½ΡƒΠΆΠ½ΠΎ ΡΠ΄Π΅Π»Π°Ρ‚ΡŒ.

← пролистнитС для просмотра ΠΏΠΎΠ»Π½ΠΎΠ³ΠΎ изобраТСния β†’

НС Π±ΠΎΠΉΡ‚Π΅ΡΡŒ ΠΏΠΈΡΠ°Ρ‚ΡŒ Π² этой малСнькой «1» ΠΈ Ρ€ΠΈΡΠΎΠ²Π°Ρ‚ΡŒ эти малСнькиС стрСлки. Π’Ρ‹ Π΄ΠΎΠ»ΠΆΠ½Ρ‹ Π΄Π΅Π»Π°Ρ‚ΡŒ всС, Ρ‡Ρ‚ΠΎ Π²Π°ΠΌ Π½ΡƒΠΆΠ½ΠΎ, Ρ‡Ρ‚ΠΎΠ±Ρ‹ ваша Ρ€Π°Π±ΠΎΡ‚Π° Π±Ρ‹Π»Π° ΠΏΡ€Π°Π²ΠΈΠ»ΡŒΠ½ΠΎΠΉ, ΠΈ Π²Ρ‹ постоянно ΠΏΠΎΠ»ΡƒΡ‡Π°Π»ΠΈ ΠΏΡ€Π°Π²ΠΈΠ»ΡŒΠ½Ρ‹ΠΉ ΠΎΡ‚Π²Π΅Ρ‚.

Π― Π±ΡƒΠ΄Ρƒ Ρ€Π°Π±ΠΎΡ‚Π°Ρ‚ΡŒ ΠΈΠ·Π½ΡƒΡ‚Ρ€ΠΈ, упрощая сначала Π²Π½ΡƒΡ‚Ρ€ΠΈ Π²Π½ΡƒΡ‚Ρ€Π΅Π½Π½ΠΈΠ΅ символы Π³Ρ€ΡƒΠΏΠΏΠΈΡ€ΠΎΠ²ΠΊΠΈ, Π² соотвСтствии с ΠŸΠΎΡ€ΡΠ΄ΠΊΠΎΠΌ ΠΎΠΏΠ΅Ρ€Π°Ρ†ΠΈΠΉ. Π˜Ρ‚Π°ΠΊ, ΠΏΠ΅Ρ€Π²ΠΎΠ΅, Ρ‡Ρ‚ΠΎ я сдСлаю, это пронСсу βˆ’4 сквозь скобки. Π’ΠΎΠ³Π΄Π° я ΡƒΠΏΡ€ΠΎΡ‰Ρƒ; Π― ΠΏΡ€ΠΎΠ΄ΠΎΠ»ΠΆΡƒ, поставив 1 ΠΏΠ΅Ρ€Π΅Π΄ ΠΊΡ€ΡƒΠ³Π»Ρ‹ΠΌΠΈ скобками, ΠΈ, Ρ‡Ρ‚ΠΎΠ±Ρ‹ ΠΏΠΎΠΌΠΎΡ‡ΡŒ ΠΌΠ½Π΅ ΠΎΡ‚ΡΠ»Π΅ΠΆΠΈΠ²Π°Ρ‚ΡŒ это -1, ΠΊΠΎΡ‚ΠΎΡ€ΠΎΠ΅ я Π±ΡƒΠ΄Ρƒ Ρ€Π°ΡΠΏΡ€Π΅Π΄Π΅Π»ΡΡ‚ΡŒ, я Π½Π°Ρ€ΠΈΡΡƒΡŽ свои малСнькиС стрСлки.

← пролистнитС для просмотра ΠΏΠΎΠ»Π½ΠΎΠ³ΠΎ изобраТСния β†’



Π­Ρ‚ΠΎ слоТно. Они Π·Π°ΡΡ‚Π°Π²Π»ΡΡŽΡ‚ мСня Π²Ρ‹Ρ‡Π΅ΡΡ‚ΡŒ Π΄Ρ€ΠΎΠ±ΡŒ. МнС Π½ΡƒΠΆΠ½ΠΎ ΡΠ»ΠΎΠΆΠΈΡ‚ΡŒ Π΄Ρ€ΠΎΠ±ΠΈ, Π° Π·Π½Π°Ρ‡ΠΈΡ‚ ΡΠ»ΠΎΠΆΠΈΡ‚ΡŒ числитСли. Π§Ρ‚ΠΎΠ±Ρ‹ ΡƒΠ±Π΅Π΄ΠΈΡ‚ΡŒΡΡ, Ρ‡Ρ‚ΠΎ я Π½Π΅ упустил ΠΈΠ· Π²ΠΈΠ΄Ρƒ, Ρ‡Ρ‚ΠΎ ΠΈΠΌΠ΅Π½Π½ΠΎ ΠΎΠ·Π½Π°Ρ‡Π°Π΅Ρ‚ этот «минус» (Π° ΠΈΠΌΠ΅Π½Π½ΠΎ, Ρ‡Ρ‚ΠΎ я ΠΌΠΈΠ½ΡƒΡΡƒΡŽ вСсь Ρ‡ΠΈΡΠ»ΠΈΡ‚Π΅Π»ΡŒ Π²Ρ‚ΠΎΡ€ΠΎΠΉ Π΄Ρ€ΠΎΠ±ΠΈ, Π° Π½Π΅ Ρ‚ΠΎΠ»ΡŒΠΊΠΎ x ), я ΠΏΡ€Π΅ΠΎΠ±Ρ€Π°Π·ΡƒΡŽ минус плюс -1:

← пролистнитС , Ρ‡Ρ‚ΠΎΠ±Ρ‹ ΠΏΡ€ΠΎΡΠΌΠΎΡ‚Ρ€Π΅Ρ‚ΡŒ ΠΏΠΎΠ»Π½ΠΎΠ΅ ΠΈΠ·ΠΎΠ±Ρ€Π°ΠΆΠ΅Π½ΠΈΠ΅ β†’

ΠžΠ±Ρ€Π°Ρ‚ΠΈΡ‚Π΅ Π²Π½ΠΈΠΌΠ°Π½ΠΈΠ΅, Ρ‡Ρ‚ΠΎ я ΠΏΠ΅Ρ€Π΅ΡˆΠ΅Π» ΠΎΡ‚ вычитания Π΄Ρ€ΠΎΠ±ΠΈ ΠΊ добавлСнию ΠΎΡ‚Ρ€ΠΈΡ†Π°Ρ‚Π΅Π»ΡŒΠ½ΠΎΠΉ Π΅Π΄ΠΈΠ½ΠΈΡ†Ρ‹, ΡƒΠΌΠ½ΠΎΠΆΠ΅Π½Π½ΠΎΠΉ Π½Π° Π΄Ρ€ΠΎΠ±ΡŒ. ΠžΡ‡Π΅Π½ΡŒ Π»Π΅Π³ΠΊΠΎ Β«ΠΏΠΎΡ‚Π΅Ρ€ΡΡ‚ΡŒΒ» минус, ΠΊΠΎΠ³Π΄Π° Π²Ρ‹ складываСтС Ρ‚Π°ΠΊΠΈΠ΅ бСспорядочныС ΠΏΠΎΠ»ΠΈΠ½ΠΎΠΌΠΈΠ°Π»ΡŒΠ½Ρ‹Π΅ Π΄Ρ€ΠΎΠ±ΠΈ. Бамая распространСнная ошибка β€” ΠΏΠΎΡΡ‚Π°Π²ΠΈΡ‚ΡŒ минус Π½Π° x ΠΈ Π·Π°Π±Ρ‹Ρ‚ΡŒ довСсти Π΄ΠΎ βˆ’2. Π‘ΡƒΠ΄ΡŒΡ‚Π΅ особСнно остороТны с дробями!

Для Π΄ΠΎΠΏΠΎΠ»Π½ΠΈΡ‚Π΅Π»ΡŒΠ½ΠΎΠΉ ΠΏΡ€Π°ΠΊΡ‚ΠΈΠΊΠΈ со скобками ΠΏΠΎΠΏΡ€ΠΎΠ±ΡƒΠΉΡ‚Π΅ здСсь.


URL: https://www.purplemath.com/modules/negative3.htm

Π‘Ρ‚Ρ€Π°Π½ΠΈΡ†Π° 1Π‘Ρ‚Ρ€Π°Π½ΠΈΡ†Π° 2Π‘Ρ‚Ρ€Π°Π½ΠΈΡ†Π° 4

ΠŸΠ΅Ρ€Π΅ΠΊΡ€Π΅ΡΡ‚Π½ΠΎΠ΅ ΡƒΠΌΠ½ΠΎΠΆΠ΅Π½ΠΈΠ΅ Π΄Ρ€ΠΎΠ±Π΅ΠΉ (Π²ΠΈΠ΄Π΅ΠΎ ΠΈ практичСскиС вопросы)